You are on page 1of 225

www.mheducation.co.

in
ISBN-13: 978-93-5260-529-3
ISBN-10: 93-5260-529-2

Inorganic chem _IIT JEE


09 March 2017 12:48:15 PM
McGraw Hill Education (India) Private Limited
It gives me immense pleasure to present the first edition of this book for JEE aspirants. This is an outcome of teaching
experience gained through years of interaction with students preparing for JEE.
The objective of this book is to provide proper guidance and relevant material to the JEE aspirants. The topics and
problems of this book are framed in a way that they touch the required level of depth for each topic.
All the chapters of this book have key concepts, solved examples, three levels of problems and previous years’
questions to provide a quick revision to the aspirants.
The details of the salient points are given below:
y KEY CONCEPTS – Efforts have been made to highlight the important theories in short form.
y SOLVED EXAMPLES – Improve the problem-solving capacity of the aspirants in a short span of time.
y LEVEL-I– are the problems based on basic concepts useful for JEE Main Exam.
y LEVEL-II– are the conceptual problems with wide application of topics which are useful for JEE Main Exam.
y LEVEL-III– are the problems based on comprehension (passage), integer answer types, column matching type and
one or more than one correct answer types to make the students familiar with JEE ADVANCED pattern.
y PREVIOUS YEARS’ QUESTIONS FOR JEE (Main & Advanced) –covers previous years’ questions asked in IIT-
JEE, AIEEE and JEE Main Exam.
I have tried my best to keep this book free from errors. Last but not the least, constructive criticism and valuable
suggestions from the readers will be highly appreciated to make this book more precise, accurate and useful.
– Author
Prefacev

Chapter 1 Periodic Table 1.1 – 1.31

Chapter 2 Chemical Bonding 2.1 – 2.36

Chapter 3 Coordination Compounds 3.1 – 3.32


viii

Chapter 4 Metallurgy 4.1 – 4.18

Chapter 5 Hydrogen and Its Compounds and S-Block elements 5.1 – 5.24

Chapter 6 d-and f-block elements 6.1 – 6.22

Chapter 7 p-Block Elements 7.1 – 7.26

Chapter 8 Salt Analysis 8.1 – 8.26


Chapter

Key Concepts

20. Total number of gaseous elements : 11(H, N, O,


in the periodic table F, Cl, He, Ne,
Ar, Kr, Xe,
Rn)
1. Lowest electronegativity : Cs 21. Total number of liquid elements in : 6 (Ga, Br, Cs,
2. Highest electronegativity : F the periodic table Hg, Fr, Unb)
3. Highest ionization potential : He 22. Smallest anion : F –
4. Lowest ionization potential : Cs 23. Liquid element of radioactive nature : Fr
5. Lowest electron affinity : Noble gases 24. Total number of radioactive : 25
6. Highest electron affinity : Chlorine elements in the periodic table
7. Least electropositive element : F 25. Volatile d-block elements : Zn, Cd, Hg,
Unb
8. Lowest melting point metal : Hg
26. Element containing no neutron : H
9. Highest melting point and : W (Tungsten)
27. Most abundant element in earth’s : Oxygen
boiling point metal
crust
10. Lowest melting point and
28. Rarest element on earth : At (astatine)
boiling point non-metal : He
29. Most abundant metal in crust earth : Al
11. Notorious element : Hydrogen
30. Element having maximum tendency : Carbon
12. Lightest element : Hydrogen
for catenation in periodic table
13. Smallest atomic size : H
31. Non-metal having highest melting : Carbon
14. Largest atomic size : Cs point, boiling point (diamond)
15. Largest anionic size : I– 32. Metals showing highest oxidation : Os (+8)
16. Smallest cation : H+ state
17. Most electropositive element : Cs 33. Most electrovalent compound : CsF
18. Element with electronegativity : Oxygen 34. Most stable carbonate : Cs2CO3
next to Fluorine 35. Strongest base : CsOH
19. Group containing maximum number : Zero 36. Strongest basic oxide : Cs2O
of gaseous elements in the group(18th) 37. Best electricity conductor among : Ag
periodic table metals
1.2

38. Best electricity conductor among : Graphite 66. Most abundant element in the : Hydrogen
non-metals universe
39. Most poisonous element : Pu
(Plutonium)
40. Liquid non-metal : Br
41. Element kept in water : Phosphorous i. It was proposed by Henry Moseley.
42. Elements kept in kerosene : IA group
ii. Modern periodic table is based on atomic number.
element
(except Li) iii. Moseley did an experiment in which he
43. Elements sublime on heating : I2 bombarded high speed electron on different metal
surfaces and obtained X-rays. He found out that
44. Noble metals : Au, Pt etc.
v µ Z where v = frequency of X-rays
45. Amphoteric metal : Be, Zn, Al,
From this experiment, Moseley concluded that
Sn, Pb
the physical and chemical properties of the
46. Amphoteric metalloid : Si elements are periodic function of their atomic
47. Metalloids elements : Si, As, Te, number. It means that when the elements are
At, Ge, Sb arranged in the increasing order of their atomic
48. Non-metals having metallic lusture : Graphite, number elements having similar properties after a
Iodine regular interval. This is also known as ‘Modern
periodic Law’.
49. Heaviest naturally occurring element : Uranium
Modern periodic Law – The physical and
iv.
50. Poorest conductor of electricity : Diamond
chemical properties of elements are a periodic
51. Hardest naturally occurring element : Diamond function of the atomic number.
52. Lightest solid metal : Li
53. Amphoteric oxides : BeO, Al2O3,
ZnO, PbO,
SnO, SnO2,
Sb2O3, i. It consist of 7 horizontal periods and 18 vertical
As2O3, etc. columns (groups)
54. Neutral oxides of non metals : NO, CO, ii. According to IUPAC 18 vertical columns are
H2O, N2O named as 1st to 18th group.
55. Dry bleacher : H2O2 iii. The co-relation between the groups in long form
56. Dry ice : Solid CO2 of periodic table and in modern form of periodic
57. First man-made element : 43Te table are given below.
(Technicium) IA IIA IIIB IVB VB VIB VIIB VIII IB IIB IIIA IVA VA VIA VIIA 0
58. Smallest period : Ist 1 2 3 4 5 6 7 8 9 10 11 12 13 14 15 16 17 18

(2 elements) iv. Elements belonging to same group having same


th number of electrons in the outermost shell so their
59. Largest period in periodic table : 6
(32 elements) properties are similar.
60. Largest group in periodic table : IIIB
(32 elements)
61. Most abundant d-block metal : Fe
No.of
62. Most abundant s-block metal : Ca Period n Sub-shell Element Name of period
elements
63. Highest density (metals) : Os, Ir 1 1 1s 2 1H, 2He Shortest
64. Highest density (non-metals) : Boron 2 2 2s, 2p 8 3Li-10Ne I Short
65. Most abundant gas in atmosphere : Nitrogen 3 3 3s, 3p 8 11Na-18Ar II Short
1.3

4 4 4s, 3d, 4p 18 19K-36Kr I Long (iii) In 6th period all types of elements are included (s, p, d
5 5 5s, 4d, 5p 18 37Rb-58Xe II Long and f)
Longest (very (iv) No inert gas in 7th period.
6 6 6s, 4f, 5d, 6p 32 55Cs-86Rn long) (v) Normal elements present in all periods.
7 7 7s, 5f, 6d, 7p 26 87Fr-112Unb Incomplete
(vi) Atomic number of last inert gas element is 86.
(vii) Long form modern periodic table can be divided into
four portions:
1. Left portion (IA and IIA) – s-block.
1st/IA/Alkali metals:
General electronic configuration = ns1 2. Right portion (IIIA to VIIA + zero group) –
(n = Number of shell) p-block.
Number of valence shell e- = 1 3. Middle portion (IIIB to VIIB + VIII + IB and IIB)
2nd/IIA/Alkaline earth metals: – d-block.
General electronic configuration = ns2 4. Bottom portion (IIIB) – f-block elements
Number of valence shell e- = 2
13th/IIIA/Boron family:
General electronic configuration = ns2np1
Number of valence shell e- = 3
14th/IVA/Carbon family:
General electronic configuration = ns2 np2 i. The elements in which ultimate orbit is incomplete
Number of valence shell e- = 4 while penultimate orbits are complete are called
15th/VA/Nitrogen Family/Pricogens: (Used in fertilizer as as normal elements.
urea) ii. Their general electronic configuration is:
General electronic configuration = ns2 np3
Number of valence shell e- = 5 IA   IIA  IIIA  IVA  VA  VIA  VIIA
1 2  2 1 2 2 2 3 2 4 
16th/VIA/Oxygen family/Chalcogens: (Ore forming) ns   ns ns np  ns np  ns np  ns np ns2np5
General electronic configuration = ns2 np4
Number of valence shell e- = 6 ns1-2 ns2 np1-5
17th/VIIA/Halogen family/Halogens: (Salt forming)
General electronic configuration = ns2 np5
Number of valence shell e- = 7
18th/Zero group/Inert gases/Noble gases:
The elements in which both ultimate (n) as well
General electronic configuration = ns2 np6 (except penultimate shells (n-1) are incomplete either in atomic
He) state or in some oxidation state are called as transition
Number of valence shell e- = 8 elements.
Note: According to this concept Zn, Cd, Hg and Unb
are not transition elements because they do not have
incomplete penultimate shell either in atomic state or in
(i) 2nd period elements (Li, Be, B) show diagonal some oxidation state.
relationship with 3rd period elements (Mg, Al, Si). Group number: IIIB to VIIB + VIII + IB and IIB
Due to almost similar ionic potential (Ionic potential
= charge/Radius) value they show similarily in Periods: 4th to 7th
properties. i. General electronic configuration is (n-1)d1-10
Li Be B ns1 or 2
   
  ii. Total number of d-block elements = 40
Na Mg Al Si Total number of transition elements = 36 (Except
(ii) 3rd period elements (Na, Mg, Al, Si, P, S, Cl) are called Zn, Cd, Hg and Unb)
typical elements because they represent the properties Note: All transition elements are d-block but all

of other element of their respective group. d-block elements are not transition elements.
1.4

2. Metallic radius (rm): Such type of radius is


determined if atoms are bonded with metallic bond.

The elements in which all the three shells, i.e. ultimate (n),
penultimate (n-1) and pre or antipenultimate (n-2) shells,
are incomplete are called as inner-transition elements.
i. General electronic configuration is: da-a
rm= ––––
d a-a 2
(n-2)f1-14 (n-1)d0 or 1 ns2
3. van der Waal’s radius (rv): Such type of radius is
ii. These are 28 in number. determined if molecules are bonded with van der
iii. Group – IIIB Waal’s force of attraction.
iv. Period – 6thand 7th     A2.      A2
v. Inner transition elements are divided into two
series:
(a)
Lanthanoid series/Rare earth elements/
Lanthenones (Ce58–Lu71 14 elements)
(b) Actinoid series/Man-made elements/Actinones d A2–A2
(Th90–Lw103 14 elements) dA2 - A2

rv = –––––––
2
rv > rm > rc

Note: Noble gases are monoatomic molecules bonded


with van der Waal’s force of attraction hence, for noble
gases, van der Waal’s radius is considered.
1. Atomic radius of an element cannot be determined
because atoms never exist in their free state and 4. Ionic radius (radius of ions):
position of their outermost e- is uncertain. a. A cation is always smaller than its parent atom
because it has greater Zeff than its parent atom.
2. Atomic radius is determined in bonded state
As positive oxidation state increases, radius
decreases.
Mn > Mn+2 > Mn+7
1. Covalent Radius (rc) b. An anion is always larger than its parent atom
a. Such type of radius is determined if a single because the anion has lower Zeff than its parent
covalent bond is present between two similar atom. As negative oxidation state increases,
atoms. radius increases.
O < O- < O-2
da – a
r = ––––––
c 2 1. Species (atoms, molecules or ions) having same
number of electrons are known as isoelectronic.
Internuclear distance(da-a) e.g Si N 2 CO CN- NO+
14e 14e 14e- 14e- 14e-
- -
b. If a single bond is present between two different 2. Order of radius in monoatomic isoelectronic species:
atoms,
N-3 > O-2 > F- > Na+ > Mg+2 > Al+3
da – b = ra + rb - 0.09 (DEN), Å In isoelectronic species, as atomic number increases,
(Bond length) radius decreases. It is due to increment in Zeff.
ra = covalent radius of A
rb = covalent radius of B 1. In periods:
DEN = difference in electronegativity of A and B
a. As we move left to right in a period, when Zeff
1.5

increase in the atomic radius decrease except in b. In p-block:


noble gases. In ~ Tl
Li > Be > B > C > N > O > F < < Ne Sn ~ Pb
Na > Mg > Al > Si > P > S > Cl < < Ar
b. Order of radius in 3d- series:
Sc > Ti > V > Cr < Mn > Fe ~ Co ~ Ni < Cu < Zn
(a) It is the energy required to remove an e- from
2. In groups:
outermost shell of isolated (free) gaseous atom.
a. As we move top to bottom in a group, when
number of shell increase the atomic radius (b) This process is endothermic.
increases M (g) + IE of M " M+(g) + e-; DH = I E
Li < Na < K < Rb < Cs
F < Cl < Br < I
b. Exception (in d-block): Zeff: IE µ Zeff
(i)
4d series ~ 5d series (due to lanthanoid (ii) Atomic size: IE µ 1
contraction) At. Size
c. Exception(in p-block): Al > Ga (iii)
Penetration power of orbitals: s > p > d > f
Note: Radius mainly depends on number of (iv)
Electronic configuration of outermost subshell:
shells. a. Elements having fully filled or half filled
Some exceptions are: outermost subshell have greater IE than
Li+ > Mg+2 expected.
0.76Å 0.72Å
(b) Such elements in a period have greater IE
H- > F-
than adjacent elements.
1.40Å 1.33Å
1 2 13 14 15 16 17 18
ns1 ns2 np1 np2 np3 np4 np5 np6
1. In lanthanoid series, as atomic number increases, (fully filled) (half filled) (fully filled)
atomic and ionic radius gradually decreases. It is
called as lanthanoid contraction. 1 < 2 > 13 < 14 < 15 > 16 < 17 < 18
2. Cause: As me move from Ce to Lu, nuclear 1 < 13 < 2 < 14 < 16 < 15 < 17 < 18
charge (Z) increases and 1e- is successively added Order of IE in second period:
into inner 4f-subshell. Since f-e- produces almost Li < B < Be < C < O < N < F < Ne
negligible screening effect hence, screening
Order of IE in third period:
coefficient (s) remains almost constant and Zeff
increases thus, radius decreases. (due to poor Na < Al < Mg < Si < S < P < Cl < Ar
screening of 4f-e- on outer e-) Periodicity in IE
3. Effect of lanthanoid contraction is also present 1. In periods: as we move from left to right, in
from 72Hf to 82Pb. It is also called as post general, IE increases. (except for fullyfilled and
lanthanoid contraction or lanthanoid contraction. half filled elements)
Due to this, these element have greater Zeff than 2. In groups: as we move top to bottom in a group,
expected (its due to poor screening by 14e- present in general, IE decreases. (it is due to increase in
in 4f-subshell). atomic size)
Order of radius (along the group) Exception: (a) due to lanthanoid contraction,
a. In d-block: 4d series ~ 5d Series In d-block:
(Zeff high) 4d series < 5d series
Zr ~ Hf Zeff high
Pd ~ Pt Zr < Hf
Y < La (No lanthanoid Pd < Pt
contraction) Y > La (No lanthanoid contraction)
1.6

In p-block:
In < Tl 1. It is the energy released when an e- is added to
Sn < Pb outermost shell of an isolated gaseous atom.
(b) Al < Ga
(Zeff high ) 2. This process in generally exothermic. (DH = -ve)
General order of IE is: X(g) + e- " X-(g) + EA of X ; DH = -EA
s-block < f-block < d-block < p-block -
e- gain enthalpy
DH = -EA

1. Successive IE always increases because during -DH = EA
successive removal of e- zeff increases and size negative e- gain enthalpy = EA
decreases.
EA of X = IE of X-
M(g) + IE1 " M+(g) + e-; DH = + IE1
3. (a) Elements having fully filled or half filled
M+(g) + IE2 " M+2(g) + e-; DH = + IE2
outermost sub-shell do not add another e-
M+2(g) + IE3 " M+3(g) + e- ; DH = + IE3
hence, their EA is generally zero.
IE1 < IE2 < IE3 < - - - - - - - (b) If we still add e- to such elements, process
Energy required to remove nth e- = IEn becomes endothermic and formed anion
Energy required to remove ne- = (IE1 + IE2 + ---- becomes unstable.

--------- IEn) Y(g) + e- " Y(g) – EA; DH = + EA
2. IE2 of M = IE1 of M+ (group 2,18 and N )
IE3 of M = IE2 of M+ = IE1 of M+2 4. EA1 process is generally exothermic while all
3. Successive IE always increases but if during higher EA processes are always endothermic
successive removal of e- electronic configuration because anions resist addition of another e-.
becomes stable than rate of increment in X(g) + e- " X-(g) + EA1. ; DH = -EA1
successive I.E. is much more than expected. (Except group 2, 18, N)
Mg = (Ne) 3s2 - - -2
X (g) + e " X (g) - EA2. ; DH = EA2
IE1 < IE2 << IE3 (because third electron is removed X-2(g) + e- " X-3(g) - EA3. ; DH = EA3
from fulfilled electronic configuration)
Order of IE2 in second period:
IE2 of M = IE1 of M+ 1.
In periods: In general as we move from left to
Li + + + +
Be B C N O F Ne + + + + right EA increases.
1s2 2s1 2s2 2p1 2p2 2p3 2p4 2p5 In period (2):
fully fully half Ne < Be < N < B < Li < C < O < F
filled filled filled
In period (3):
Be < C < B < N < F < O < Ne < Li Ar < Mg < Al < Na < P < Si < S < Cl
(in a particular period alkali metal has highest IE2 because
2.
In groups: In general as we move from top to
it has very high Zeff)
bottom in a group EA decreases.
Order of IE3 in second period:
Note: Second period elements have lower EA than
IE3 of M = IE1 of M2+ expected. They have exceptionally small size. Hence,
Li+2 Be+2 B+2 C+2 N+2 O+2 F+2 Ne+2 incoming e- feels more repulsion than expected and net
1s1 1s2 2s1 2s2 2p1 2p2 2p3 2p4 attraction becomes less than expected so their EA becomes
fully fully half
less than expected.
filled filled filled
B < N < C < O < Ne < F < Li < Be Order of EA in various groups:
(in a particular period alkaline earth metal has highest IE3 Cl > F > Br > I
because it has very high Zeff) S > Se > Te > O
1.7

N < P < As < Bi < Sb


Si > C > Ge > Sn > Pb 1.
Mulliken’s scale:
Al > Ga > In > Tl > B
Xm = IE + EA (both are in eV/atom)
2
1. Tendency of an atom to attract bonded e- pair
towards itself in a bond is known as EN of that 2.
Pauling’s scale:
atom.
DEN = | XA–XB | = 0.208 (EA-B – (EA-A× EB-B))

2. Noble gases do not form interatomic bond hence
Bond energies in kcal/mol
their EN is assumed as zero.
Factors affecting EN: OR,
1. Zeff: = 0.1017 (EA-B – (EA-A × EB-B))
EN µ Zeff
Bond energies in kJ/mol
2. Atomic size: EN µ 1
atomic size Xp ~ Xm

3. Oxidation state: 2.8
(a) As positive oxidation state increases, EN 3. Allred-Roshow scale:
increases.
A < A+ < A+2 X = 0.359 Zeff + 0.744 (r = covalent radius (in A°))
r2
 +7. +4
KMnO4. > MnO2
(b) As negative oxidation state increases, EN
decreases.
1. Metallic and Non-metallic properties:
B > B- > B-2
-2 -1  +2 Metallic property µ 1
H2O < H2O2 < OF2 EN
4. % s-character: As % s-character increases, EN Non-metallic property µ EN
increases.
N
sp > sp2 > sp3 Non-
O
metals B
  50% 33.33%  25%
Metalloids L
Metals E

G
1. In periods: As we move from left to right in A
a period, Zeff increases hence, EN increases. S
(Except Noble gases)
Li < Be < B < C < N < O < F Metalloids or semi-metals: elements which can
both gain or loose e-.
Na < Mg < Al < Si < P < S < Cl
2. In groups: As we move top to bottom in a group,
H
atomic size increases hence, EN decreases. B C N O F
F > Cl > Br > I Si P S Cl Non-metals
a. Exception: Al < Ga (High Zeff ) Ge As Se Br
Sb Te I
b. Exception: Due to lanthanoid contraction, Po At Metalloids
d-block:
4d-series < 5d-series (high Zeff )
Zr < Hf s-block " Metals
Y > La (No lanthanoid contraction) d and f-block " Metals
p-block:
In < Tl p-block " Non-metals, metalloids, metals and
Sn < Pb noble gases.
1.8

2. Nature of bond: Na2O < MgO < Al2O3 < SiO2 < P4O10 < SO3 < Cl2O7
Nature of interatomic bond depends on DEN. +2 +4 +7
DEN Nature of bond MnO < MnO2 < Mn2O7

0 Pure covalent +1   +2 +3 +5
0.1 – 0.8 Covalent Covalent N2O < NO < N2O3 < N2O5

0.9 – 1.6 Polar Covalent c. Non-metallic oxides are generally acidic. (Some
are neutral)
1.7 50% ionic and 50% covalent
Neutral oxides are these which do not react
1.8 or more Ionic with both acid and base
% ionic character = 16 (DEN) + 3.5 (DEN)2 eg. CO, NO, N2O, H2O
(Henery – Smith formula) d. Metallic oxides are generally basic. (Some are
3.
Nature of hydride: amphoteric)
Hydrides: Binary compounds having one element Amphoteric oxides are those which react
H. with both acid and base.
eg. s-block: BeO
eg. CH4, H2S, HCl etc.
(along the group) d-block: TiO2, VO2, CrO2, Cr2O3, MnO2,
Mn3O4, ZnO etc.
C N O F i. Size of central atom (M) increases
p-block: Al2O3, Ga2O3, SnO, SnO2, PbO,
Si P S Cl ii. Bond length of M-H bond increases
PbO2, As2O3, Sb2O3 etc.
Ge As Se Br iii. H+ loosing tendency increases
Some metallic oxides like CrO3, Mn2O7 etc
Sn Sb Te I iv. Acidic strength increases are acidic in nature.
(along the period)
i. EN of Central atom (M) increases
ii. DEN of M-H bond increases 1. Atomic density:
iii. Bond polarity(or ionic character) of M-H (a) In groups: Down the group both atomic mass
bond increases and atomic volume increases but increment
in mass is much more than volume. Hence,
iv. Tendency to loose H+ in water increases
atomic density increases.
v. acidic strength increases
Exception: Density of Na > K
Order of Acidic Strength: Mg > Ca
HF < HCl < HBr < HI (b) In periods:
CH4 < NH3 < H2O < HF S block < d-block > p-block
CH4 < H2S < HI
4.
Nature of hydroxides and oxides:
a. oxides form hydroxides in water hence, the nature
of oxides and hydroxides of an element is similar.    Lighter metal Heavy metal

b. Acidic strength of oxides and hydroxides µ EN of 2. Melting point and boiling point:
central atom (a) In periods: The general order is,
Order of acidic strength: s-block < d-block > p-block
ClOH > BrOH > IOH (b) In groups: Down the group the general order
MgO > CaO > SrO > BaO is:
s-block d-block groups 13 and 14 groups15 to18
decreases increases decreases increases
1.9

Solved Examples

1. Which of the following is incorrect match? (a) Co+2, Cr+3, V+3


(a) Z = 48, group = IIB , period No. = 5th (b) Mn+2, Fe+3, Cr+
(b) (Xe) 4f7 5d1 6s2, group = IIIB , period = 6th (c) Ni+2, Mn+2, Co+2
(c) (Rn) 6d2 7s2, group = IVB, period = 7th (d) Fe+2, Mn+2, Co+2
(d) Z = 56, group = IIA , period = 6th Sol.(b)
Sol.(c) Element, having Z = 48, is Cd Ion Electronic configuration No. of
It is member of group 12 or IIB and period 6 th unpaired e-
Element having electronic configuration (Xe) 4f7

Co+2 (Ar) 4s0 3d7 3
5d1 6s2 is a lanthanoid. All lanthanoids belong to Cr+3 (Ar) 4s0 3d3 3
group IIIB and period 6th. V+3 (Ar) 4s0 3d2 2
Element having electronic configuration (Rn) 6d2 Mn+2 (Ar) 4s0 3d5 5
7s2 is an actinoid. All actinoids belong to group Fe+3 (Ar) 4s0 3d5 5
IIIB and period 7th. Cr+ (Ar) 4s0 3d5 5
Ni+2 (Ar) 4s0 3d8 2
Element, having Z = 56, is Ba. It is member of Fe+2 (Ar) 4s0 3d6 4
group 2 or IIA and period 6th.
Ions, having similar number of unpaired e- , have
2. Which of the following metals give inflammable same magnetic moment.
gas with both acid and base?
5. The correct order of acidic strength of the
(a) Na and Zn (b) Mg and Al following is:
(c) Mg and Be (d) Zn and Al (a) SO2 > P2O3 > SiO2 > Al2O3
Sol.(d) Amphoteric metals like Be, Zn, Al, Sn, Pb etc (b) P2O3 > SO2 > SiO2 > Al2O3
give H2 gas (inflammable) with both acid and (c) P2O3 > Al2O3 > SO2 > SiO2
base. (d) Al2O3 > SiO2 > P2O3 > SO2
3. Which of the following have an incorrect order of Sol.(a) Acidic strength of oxides depends on
ionization energy: electronegativity of central atom. As
(a) Pb (IE) > Sn (IE) electronegativity of central atom increases acidic
(b) Na+ (IE) > Mg+ (IE) strength also increases.
(c) Li+ (IE) < O+ (IE) Correct order of acidic strength is:
(d) Be+ (IE) < C+ (IE) SO2 > P2O3 > SiO2 > Al2O3
Sol.(c) Due to lanthanoid contraction Pb has greater 6. Ionization energy of F- is 320 kJ mol-1. The
effective nuclear charge (zeff) than Sn hence, electron gain enthalpy of fluorine would be:
Pb (IE) > Sn (IE) (a) – 320 kJ mol-1
Na+ has electronic configuration of noble gas
(b) – 160 kJ mol-1
hence,
Na+ (IE) > Mg+ (IE) (c) + 320 kJ mol-1
Li+ has electronic configuration of noble gas (d) + 160 kJ mol-1
hence, Sol.(a) Ionization energy of F- is 320 kJ mol-1
Li+ (IE) > O+ (IE) F- (g) + 320 kJ mol-1 " F (g) + e-; H = 320 kJ
C+ has greater effective nuclear charge (zeff) than mol-1
Be+ hence,
Be+ (IE) < C+ (IE) Equation for electron gain enthalpy of F is:

4. Which set of ions have same magnetic moment? F(g) + e- " F- (g) + 320 kJ mol-1; H = –320 kJ
mol-1
1.10

7. The value of IE1, IE2, IE3, and IE4 of an atom are For 1 mol, energy needed is 495 kJ
7.5 eV, 25.6 eV, 48.6 eV and 170.6 eV respectively. Hence, for 10-4 mol, energy needed is
The electronic configuration of the atom will be: 495 × 103 × 10-4 J = 49.5 J
(a) 1s2 2s2 2p6 3s1
11. The correct order of the second ionization
(b) 1s2 2s2 2p6 3s2 3p1 potential of carbon, nitrogen, oxygen and fluorine
(c) 1s2 2s2 2p6 3s2 3p3 is
(d) 1s2 2s2 2p6 3s2 (a) C > N > F > O (b) O > N > F > C
Sol.(b) The biggest jump occurs from IE3 to IE4 (c) O > F > N > C (d) F > O > N > C
IE3 < < IE4 Sol.(c) IE2 of neutral element is IE1 of cation having
(IEn) (IEn+1) single positive charge. Hence, for order of IE2,
n(Valence e-) = 3 first put +1 charge to each element then write
electronic configuration.
Hence, the electronic configuration of the atom
will be 1s2 2s2 2p6 3s2 3p1. C+ N+ O+    F+

8. The correct order increasing radii is: 1s2 2s2 2p6 1s2 2s2 2p2 1s2 2s2 2p3  1s2 2s2 2p4
      -
(a) Be2+, Mg2+, Na+ (b) K+, Ca2+, S2- Half filled
(c) O2-, F- , N3- (d) S2-, O2-, As3- From left to right in a period, IE1 increases and
Sol.(a) Correct order of increasing radii are:- fulfilled or half filled elements have greater IE1
(a) Be+2 < Mg+2 < Na+ than adjacent elements. Hence, correct order of
(b) Ca+2 < K+ < S-2 IE2 is: C > N > F > O
(c) F- < O-2 < N-3 12. The electronegativity of the following elements
(d) O-2 < S-2 < As3- increases in the order:

9. What will be the distance between H and Cl atom (a) S < P < N < O (b) P < S < N < O
in HCl. The radius of hydrogen is 0.37 Å and the (c) N < O < P < S (d) N < P < S < O
radius of chlorine is 1.67 Å?
Sol. (b) Group 15 Group 16
(According to the concept of covalent radius)
Period (II) N O
(a) 2.04Å (b) 1.96Å
(c) 2.12Å (d) 1.0Å
Sol.(b) Bond length of single covalent bond = rA + rB – Period (III) P S
0.09 ( EN) Correct order of electro negativity is:
Bond length (dH - cl) = rH + rcl – 0.09 ( EN) P<S<N<O
rH = 0.37Å; rcl = 1.67Å and EN = 3.0 – 2.1 = 0.9 13. The formation of the oxide ion, O2-(g), from
dH – cl = 0.37 + 1.67 – 0.09 (0.9) oxygen atom requires first an exothermic and
or, dH – cl = 2.04 – 0.08 = 1.96Å then an endothermic step as shown below :
O(g) + e– " O-(g); egH = -141 kJmol-1
10. The ionization energy of sodium is 495 kJ mol–1.
How much energy is needed to convert atoms O-(g) + e- " O2-(g); egH = +780 kJmol-1
present in 2.3 mg of sodium into sodium ions? Thus process of formation of O2- in gas phase is
(a) 4.95 J (b) 49.5 J unfavorable even O2- is isoelectronic with neon. It
is due to the fact that :
(c) 495 J (d) 0.495 J
(a) Oxygen is more electronegative.
Sol.(b) Ionization energy of Na = 495 kJ/mol (b) Addition of electron in oxygen results in
No. of moles of Na in 2.3 mg larger size of the ion.
2.3×10 (c) Electron repulsion outweighs the stability
= ––––––– = 10-4 moles gained by achieving noble gas configuration.
23
1.11

(d) O- ion has comparatively smaller size than Sol.(d) In CrCl2, oxidation state of chromium is +2
oxygen atom. (minimum). Thus it will have maximum radius.
Sol.(c) Process of formation of O2- ion in gaseous phase As positive oxidation state increases, radius
is unfavorable because O- ion (anion) resists decreases.
addition of another e- due to repulsion hence, 18. The IP1, IP2, IP3, IP4, and IP5 of an element are
electron repulsion outweighs the stability gained 7.1, 14.3, 34.5, 46.8, 162.2 eV respectively.
by achieving noble gas configuration.
The element is likely to be-
14. Which is the correct in the following -
(a) Na (b) Si
(a) Radius of Cl atom is 0.99 Å, while that of Cl+
ion is 1.54 Å (c) F (d) Ca
(b) Radius of Cl atom is 0.99 Å, while that of Na Sol.(b) The jump in IP values exist in IP5 and thus
atom is 1.54 Å removal of fifth electron occurs from inner
(c) Radius of Cl atom is 0.99 Å, while that of Cl- shell. Thus element contains four electrons in its
ion is 0.81 Å valence shell. It means the element belongs to the
(d) Radius of Na atom is 0.95 Å, while that of group 14.
Na+ ion is 1.54 Å 19. Following are ground state electronic
Sol.(b) The atomic radius decreases along the period. configuration of some neutral atoms:
Also cations are always smaller than their parent (a) 1s2 2s2 2p3 (b) 1s2 2s2 2p5
atom and anions are always larger than their (c) 1s2 2s22p6 3s1 (d) 1s2 2s2 2p6
parent atom.
(i) Which of the following would have lowest
15. Which oxide of ‘N’ is isoelectronic with CO2: IE?
(a) NO2 (b) NO (ii) Arrange them in increasing order of IE
(c) N2O (d) N2O2 Sol. (i) Three electrons in 2p subshell (i.e. half
Sol.(c) N2O is isoelectronic with CO2. Both have 22 filled) indicate for its greater stability while
electrons. 6 electrons in 2p indicate for its maximum
16. Arrange Ce3+, La3+, Pm3 and Yb3+ in increasing stability. Thus electronic configuration
order of their size - (c) having 1 electron in 3s would require
minimum IE
(a) Yb3+ < Pm3+ < Ce3+ < La3+
(ii) c < a < b < d (increasing order of IE)
(b) Ce3+ < Yb3+ < Pm3+ < La3+
(c) Yb3+ < Pm3+ < La3+ < Ce3+ 20. The atomic number of three elements A, B and
C are a, a+1 and a+2, C is an alkali metal. In a
(d) Pm3+ < La3+ < Ce3+ < Yb3+
compound of A and C, the nature of bonding is-
Sol.(a) Lanthanide contraction is observed in these ions,
(a) Co-ordinate (b) Covalent
i.e., ionic radius decreases as atomic number
increases. (c) Ionic (d) Metallic
17. In which of the following compounds chromium Sol.(c) If C is alkali metal, A should be halogen (non-
shows maximum radius:- metal). Between metal and non-metal ionic bond
is present.
(a) K2Cr2O7 (b) CrO2Cl2
(c) Cr2(SO4)3 (d) CrCl2
1.12

Exercise
(C) Transition elements (iii) (n-2) f1-14 (n-1)d0-1 ns2
(D) Inner- transition elements (iv) (n-1) d1-10 ns1 or 2
2- +
1. X is isoelectronic with “O2 ” and has Z + 1
neutron (Z is atomic number of X2-) then: (a) A- i, B-ii,C-iii, D-iv
(a) Mass number of X2- is 27 (b) A-ii, B-i, C-iii, D-iv
(b) Mass number of X2- is 57 (c) A-ii, B-i, C-iv, D-iii
(c) Atomic number of X2- is 28 (d) A-i, B-ii, C-iv, D-iii
(d) Number of proton X2- is 15 8. Which of the following is an incorrect match?
2. Which of the following statements is not correct (a) Z = 65, group = IIIB, period – 6th
regarding hydrogen atom ? (b) Z = 46, group = VIIIB, period – 5th
(a) It resembles halogens in some properties
(c) Z = 108, group = XB, period – 8th
(b) It resembles alkali metals in some properties
(c) It can be placed in 17th group of periodic (d) Z = 57, group = IIIB, period – 6th
table 9.
The element with atomic number 56 is likely to
(d) It can not be placed in first group of periodic have the same outer shell configuration as the
table element with atomic number:
3. If an atom has electronic configuration 1s2 2s2 2p6 (a) 12 (b) 18
3s2 3p6 3d3 4s2, it will be place in:
(c) 14 (d) 24
Second group Third group
10. Electronic configuration of species M2+ is 1s2,
(c) Fifth group (d) Sixth group 2s2, 2p6, 3s2,3p6,3d6 and its atomic weight is 56.
4. Among the following, the element with the lowest The number of neutrons in the nucleus of species
atomic number that has a ground-state electronic M is:
configuration of (n-1) d5 ns1 is located in the: (a) 32 (b) 26
(a) Fifth period (b) Sixth period
(c) 30 (d) 28
(c) Fourth period (d) Third period
5. In species X2+ the mass number is 20 and number 11.
Which is correct order of ionic mobility in
of neutrons are 10 then calculate the number of aqueous medium?
electrons in species X2+: (a) Li+ < Na+ < K+
4 (b) 7 (b) Na+ < Mg2+ < Al3+
(c) Al3+ < Na+ < Mg2+
(c) 6 (d) 8
(d) Li+> Na+ > K+
6. The elements which are characterised by the
outer shell configuration ns1, ns2 and ns2 np1to 12. Which one of the following is not a characteristic
ns2np5are collectively called as: of p-block elements?
(a) Transition elements (a) The last electrons in them enters into a
(b) Representative elements p-orbital
(c) Lanthanides (b) They mostly form acidic oxides
(d) Inner-transition elements (c) Down the group, stability of lower oxidation
7. state increases
Column - I Column – II (d) They mostly form basic oxides
(Type of element) (Outer electronic 13. Which of the following species has a value of
configuration) magnetic moment, m = 35?
(A) Inert gas elements (i) ns1-2 and ns2 np1 to ns2np5 (a) Cr3+ (b) Mn2+
2 2 6
(B) Representative elements (ii) 1s and ns np (c) Fe2+ (d) Cu2+
1.13

14. The paramagnetic species among the following 22. Which of the following has the largest ionic
is: radius?
(a) Na+ (b) Zn2+ (a) Be2+ (b) Mg2+
(c) Cu+ (d) Fe3+ (c) Ca2+ (d) Sr2+
15. All of the following possess complete d-subshells 23. The correct order of the size of C, N, P and S is:
except: (a) N < C < P < S (b) C < N < P < S
+ 2+
(a) Ag (b) Cu (c) N < C < S < P (d) C < N < S < P
3+
(c) Ga (d) Zn2+ 24. Which of the following pair of elements have
16. Calculate the ‘X’ in Mnx+ if µ=3.87 BM almost similar atomic radii?
(a) 2 (b) 3 (a) Zr, Hf (b) Cu, Ag
(c) 4 (d) 5 (c) Sc, Ti (d) Pd, Ni
17. The first element of a group in many ways differs 25. In which of the following compounds, manganese
from the other heavier members of the group. shows maximum radius?
This is due to: (a) MnO2 (b) KMnO4
(a) the small size
(c) MnO (d) K3 (Mn(CN)6)
(b) the high electronegativity and high ionization
potential 26.
Ionization enthalpies tend to decrease going
down any column of main group elements
(c) the unavailability of d-orbitals
because------------ going down the column.
(d) all of the above (a) Nuclear charge increases
18. Be and AI show diagonal relationship hence, both (b) Number of shielding electrons increases
have: (c) Atomic size increases
(a) almost same of electronegativity (d) Effective nuclear charge increases
(b) amphoteric nature of oxides 27. The ionization potential of nitrogen is more than
(c) approximately same polarizing power of that of oxygen because of:
respective cations (a) Greater attraction of electrons by the nucleus
(d) all the properties above (b) Extra stability of the half-filled p-orbitals
19. Which of the following set contains pair of (c) Smaller size of nitrogen
elements that do not belong to same group but (d) More penetration effect
show chemical resemblance? 28. Which of the following transition involve
(a) Hf, Zr (b) K, Rb maximum amount of energy?
(c) Be, Al (d) B, Al (a) M-(g) " M(g) (b) M-(g) " M+(g)
(c) M+(g) " M2+(g) (d) M2+(g) " M3+(g)
20. Which of the following set of magic numbers is
not correct for given group? 29. Which of the following process refers to IE2 of
(a) 18, 18, 32 & IIIB X?
(a) X(g) " X2+(g) (b) X+(g) " X2+(g)
(b) 8, 8, 18, 18, 32 & VIA
(c) 18, 32, 32 & IB (c) X+(aq) " X2+(g) (d) X(g) " X+(g)
(d) 8, 8, 18, 18, 32 & IIA 30. Which of the following statement concerning
ionization energy is not correct?
21. Correct order of ionic radius of following
(a) The IE2 is always more than the first.
isoelectronic species is:
(b) Within a group, there is a gradual increase
(a) Se-2 > Br- > Kr > Rb+ > Sr+2
in ionization energy because nuclear charge
(b) S-2 > Cl- > K+ > Ar > Ca+2 increases.
(c) N-3 > O-2 > Ne > F- >Ca+2 (c) Ionization energy of Be is more than B.
(d) F- > Ne > Na+ > Al+3 > Mg+2 (d) Ionization energy of noble gases are high.
1.14

31. Lanthanide contraction is related with: 40. As we proceed from top to bottom in the periodic
(a) Sharp decrease in atomic size in lanthanide table:
series (a) hydroxides are more basic
(b) Slow or gradual decrease in atomic size in (b) oxyacids are less acidic
lanthanide series (c) neither (a) and (b) of the above
(c) Constancy in atomic size (d) Both (a) and (b) of the above
(d) All the above 41. Among the following oxides, which is least
32. Relation between electron gain enthalpy and acidic?
electron affinity is: (a) Al2O3 (b) B2O3
(a) EA = DHe.g. (b) EA = 2DHe.g. (c) CO2 (d) NO2
(c) EA = –2DHe.g. (d) EA = –DHe.g. 42. Which of the following oxides is neutral?
33. The process requiring absorption of energy is: (a) SiO2 (b) CO
(a) N " N- (b) F " F- (c) ZnO (d) SnO2
(c) Cl " Cl- (d) H " H- 43. What is the nature of Al2O3 and B2O3?
34. Second and successive electron gain enthalpy of (a) Acidic, Acidic
an element (b) Acidic, Amphoteric
(a) is always negative (energy is released) (c) Amphoteric, Amphoteric
(b) is always positive (energy is absorbed) (d) Amphoteric, Acidic
(c) can be positive or negative
44. Correct order of acidic strength is:
(d) is always zero
(a) SiH4 > PH3 > CH4 > HCl
35. Of the following pairs, the one containing (b) HCl > PH3 > CH4 > SiH4
examples of metalloid elements is:
(c) HCl > SiH4 > PH3 > CH4
(a) B and Al (b) Ga and Ge
(d) HCl > PH3 > SiH4 > CH4
(c) Al and Si (d) As and Sb
45. Which of the following oxide is acidic?
36. The group in the periodic table that contains the
elements in all the different physical states at (a) N2O5 (b) Mn2O7
room temperature is: (c) CrO3 (d) All
(a) VA (b) IA
(c) VIIA (d) IVA
37. Elements of which group form anions most
readily? 1. An element X belongs to fourth period and
(a) Oxygen family (b) Nitrogen family fifteenth group of the periodic table. Which one of
the following is true regarding the outer electronic
(c) Halogens (d) Alkali metals
configuration of X? It has:
38. What is the percentage of ionic character in CsF:
A. Partially filled d-orbitals and completely
(according to Henry-Smith formula)
filled s-orbital
{EN of Cs = 0.7 and EN of F = 4.0}
B. Completely filled s-orbital and completely
(a) 100% (b) 10%
filled p-orbitals
(c) 90.9% (d) 99%
C. Completely filled s-orbital and half filled
39. In halogens, which of the following decrease p-orbitals
from iodine to fluorine?
D. Half filled d-orbitals and completely filled
(a) Bond length
s-orbital
(b) Electronegativity
(c) The ionization energy of the element (a) A,B & C (b) Only A & B
(d) Oxidizing power (c) A, B & D (d) Only C
1.15

2. Vishal Thakur went to meet his friend Sumit, (c) General outermost shell e- configuration of
Where he saw that his friend was doing the study d-block element is ns 1-2 (n -1)d1-10
of a particular chemistry book. But he could not (d) All actinoids are man made elements
find the theoretical value of bond length in H-F
9. A°/2 atoms of X(g) are converted into X+(g)
but he found that rH and rF are 0.37 Å and 0.72 Å
by absorbing energy E1. A°/2 ions of X+(g) are
respectively and eletronegativity of F and H are
converted into X-(g) with release of energy E2.
4.0 and 2.1 respectively. What is bond length of
Hence ionization energy and electron affinity of
H-F bond?
X(g) are:
(a) 1.09 (b) 1.784 2E1 2(E1 – E1 )
(a) ,
(c) 0.92 (d) 0.46 A∞ A∞
3. Choose the correct order of ionic radius for the 2E1 2(E 2 – E1 )
following species: (b) ,
A∞ A∞
(a) Cl- > I- > Te2- > Ar+
(b) Te2- > I- > Cl- > Ar+ (E1 – E 2 ) 2E 2
(c) ,
(c) I- > Te2- > Cl- > Ar+ A∞ A∞
(d) I- > Cl- > Ar+ > Te2- (d) None of these

4. Which statement is correct? 10. Which represents correct order of acidic strength?
(a) Tl+ ion is more stable than Tl3+ (a) NH3 > PH3 > AsH3 > SbH3 > BiH3
(b) Pb4+ salts act as good oxidizing agents (b) K2O > ZnO > NO2
(c) Bi5+ salts act as good oxidizing agents (c) NaOH < KOH < RbOH < CsOH
(d) All of these (d) CH4 < NH3 < H2O < HF

5. Among the elements Ca, Mg, P and Cl, the order 11. Which of the following statements is incorrect?
of increasing atomic radii is: (a) Cesium is the most electropositive element
(a) Mg < Ca < Cl < P while F is the most electronegative element
(b) Cl < P < Mg < Ca (b) Cl has the highest -ve electron gain enthalpy
(c) P < Cl < Ca < Mg out of all the elements
(d) Ca < Mg < P < Cl (c) Electron gain enthalpy of N as well as that of
noble gases is positive
6. Element X belongs to 4th period. It contains 18
(d) In any period, the atomic radius of the noble
and 1 electron in the penultimate and ultimate
gas is lowest
orbit. The element X should be:
(a) Normal element 12. Which of the following is correct order of
decreasing acidic character?
(b) Transition element
(i) ClO2 > SO2 > SiO2 > CO2
(c) Inert gas (ii) ClO2 > SO2 > SiO2 > SnO2
(d) Inner-transition element (iii) N2O3 > P2O3 > As2O3 > Bi2O3
7. General electronic configuration of outermost (iv) N2O5 > P2O5 > As2O5 > Bi2O5
and penultimate shell is (n-1)s2 (n -1)p6 (n -1)dx (a) i, ii, iii (b) ii, iii, iv
ns2 . If n = 4 and x = 5, then number of protons in
the nucleus will be : (c) i, iii, iv (d) i, ii, iv

(a) > 25 (b) < 24 13. Which of the following conclusions are correct
regarding the element having atomic number
(c) 25 (d) 30
equal to 113?
8. Select correct statement: (i) It is present in the 8th period of the modern
(a) La and Ac belong to f-block periodic table
(b) An element having atomic number 31 belongs (ii) It is present in the group 13 in the periodic
to 3rd period table
1.16

(iii) It is a p-block element (c) It belongs to 6th period and 3rd group
(iv) Oxidation states of this element may be +1 or (d) None of these
+3. 19. The set representing the correct order of ionic
(a) i, iii, iv (b) ii, iii, iv radius is:
(c) i, ii, iv (d) i, iv (a) Na+ > Mg2+ >Al3+ > Li+ > Be2+
14. Which of the following statement(s) is(are) (b) Na+ > Li+ > Mg2+ > Al3+ > Be2+
correct? (c) Na+ > Mg2+ > Li+ > Al3+ > Be2+
(a) The electronic configuration of Cr is (Ar) 3d4 (d) Na+ > Mg2+ > Li+ > Al3+ > Be2+
4s2 (Atomic number of Cr = 24) 20. In the compound M-O-H, the M-O bond will be
(b) Cr is a representative element. broken in water if:
(c) In silver atom, 23 electrons have a spin of one (a) D (EN) of M and O < D (EN) of O and H
type and 24 of the opposite type. (b) D (EN) of M and O = D (EN) of O and H
(d) The oxidation state of nitrogen in HN3 is –3. (c) D (EN) of M and O > D (EN) of O and H
15. Find the formula of halide of a metal whose (d) Cannot be predicated according D (EN) data
successive ionization energies are x, 2x, 5x, 20x, 21. Consider the following changes:
25x kJ mol-1 respectively. M(s) " M(g) ........................ (i)
(a) MX (b) MX2 2+ -
M(s) " M (g) + 2e ........................ (ii)
(c) MX3 (d) M2X M(g) " M+(g) + e- ........................ (iii)
16. Which is/are true statement(s) about s-block M+(g) " M2+(g) + e- ........................ (iv)
elements? M(g) " M2+(g) + 2e- ........................ (v)
(a) Metals are obtained by the electrolysis of The second ionization energy of M could be
fused chlorides calculated from the energy values associated
(b) Only one type of valency, +1 for IA and +2 with:
for IIA, is shown (a) i+iii+iv (b) ii-i+iii
(c) Oxides are basic except BeO (c) i+v (d) v-iii
(d) all of the above are correct statements 22. Consider the following conversions:
(i) O(g) + e- " O-(g), DH1
17. Which of the following statement(s) is/are
correct? (ii) F(g) + e- " F-(g), DH2
(i) Vander waal’s radius of iodine is more than (iii) Cl(g) + e- " Cl-(g), DH3
its covalent radius (iv) O-(g) + e- " O2-(g), DH4
(ii) All isoelectronic ions belong to the same That according to given information the incorrect
period of the periodic table statement is:
(iii) IE1 of N is higher than that of O while IE2 of (a) DH3 is more negative than DH1 and DH2
O is higher than that of N (b) DH1is less negative than DH2
(iv) he 1st electron gain enthalpy of Cl is negative (c) DH1, DH2 and DH3 are negative whereas DH4
while second is positive is positive
(d) DH1 and DH3 are negative whereas DH2 and
(a) i, ii (b) i, ii, iii
DH4 are positive
(c) i, iii, iv (d) i, ii, iii, iv
23. Ionic radii of:
18. Consider the following electronic configuration (a) 35Cl- > 37Cl- (b) Mn7+ > Ti4+
of an element (P): (c) K+ > Cl- (d) P3+ > P5+
(Xe)4f145d16s2 24. The correct order of relative stability of half filled
Then correct statement about element (P) is: and completely filled sub-shell is:
(a) It belongs to 6th period and 1st group (a) p3 > d5 < d10 < p6 (b) d5 > p3 < d10 < p6
(b) It belongs to 6th period and 2nd group (c) d5< p3 < d10 < p6 (d) p3 > d10 < d5 < p6
1.17

25. The five successive ionization energies of an waal’s radii, since the bonding forces in the
element are 800, 2427, 3658, 25024 and 32824 metallic crystal lattice are much stronger than
kJ Mol-1 respectively. The number of valence the vander waal’s forces.
electron is: (c) Both (a) & (b)
(a) 3 (b) 5 (d) None of these
(c) 1 (d) 2 31. Which represents alkali metals (i.e. 1st group
26. What is the order of ionization energies of the metals) based on IE1 and IE2 values in kJ mol-1?
coinage metal?  IE1 IE2
(a) Ag > Cu > Au (b) Cu > Ag > Au (a) X 500 1000
(c) Cu < Ag < Au (d) Au > Cu > Ag (b) Y 600 2000
(c) Z 550 7500
27. IE2 for an element is invariably higher than IE1
because: (d) M 700 1400
(a) It is difficult to remove electron from cation 32. Match the correct atomic radius with the element:
(b) The size of the cation is smaller than its atoms S.No.   Element  Code  Atomic radius (pm)
(c) Zeff is more for cation (i)     
Be   (p)    74
(d) All the above (ii)    
C   (q)    88
28. Two p-block elements x (outer configuration ns2 (iii)    
O   (r)    111
np3) and z (outer configuration ns2 np4) occupy
neighbouring positions in a period. Using this (iv)   B   (s)    77
information which of the following is correct with (v)    N   (t)    66
respect to their ionization potential Ix and Iz? (a) (i)–r, (ii)–q, (iii)–t, (iv)–s, (v)–p
(a) Ix> Iz (b) (i)–t, (ii)–s, (iii)–r, (iv)–p, (v)–q
(b) Ix< Iz (c) (i)–r, (ii)–s, (iii)–t, (iv)–q, (v)–p
(c) Ix = Iz (d) (i)–t, (ii)–p, (iii)–r, (iv)–s, (v)–q
(d) Relation between Ix and Iz is uncertain 33. Electronic configurations of four element A, B, C
29.
Fluorine has the highest electronegativity among and D are given below:
the group on the pauling scale, but the electron (i) 1s2 2s2 2p6 (ii) 1s2 2s2 2p4
affinity of fluorine is less than that of chlorine
because: (iii) 1s2 2s2 2p6 3s1 (iv) 1s2 2s2 2p5
(a) The atomic number of fluorine is less than Which of the following is the correct order of
that of chlorine increasing tendency to gain electron ?
(b) Fluorine being the first member of the family (a) i < iii < ii < iv (b) i < ii <iii < iv
behaves in an unusual manner (c) iv < ii < iii < i (d) iv < i < ii < iii
(c) Chlorine can accommodate an electron better
34. Which of the following is the wrong statement?
than fluorine by utilising its vacant 3d orbital
(a) All the actinoid elements are radioactive.
(d) Small size, high electron density and an
increased electron repulsion make addition (b) Alkali and alkaline earth metals are s-block
of an electron to fluorine less favourable than elements.
that in the case of chlorine (c) Pnictogens and halogens are p-block elements.
30. Select correct statement about radius of an atom: (d) The first member of the lanthanoid series is
lanthanum
(a) Values of vanderwaal’s radii are larger
than those of covalent radii because the 35. Which is true statement(s)?
vanderwaal’s forces are much weaker than (a) Larger the value ionization enthalpy, easier is
the forces operating between atoms in a the formation of cation.
covalently bonded molecule. (b) Larger the value of electron affinity, easier is
(b) The metallic radii are smaller than the vander the formation of anion.
1.18

(c) Larger the value of ionization energy as well 43. Which has the lowest anion to cation size ratio?
as electron affinity, smaller is the Mulliken (a) LiF (b) NaF
electronegativity of atom.
(c) Csl (d) CsF
(d) Larger the Zeff, larger is the size of atom.
44. Select the incorrect statement:
36. The lithium ion (Li+) and hydride ion (H-) are
isoelectronic ions. Which statement about these (a) Size of H- is larger than F-
systems is true? (b) Rb is more electropositive compared to Ca
(a) Chemical properties of these ions are identical (c) Na+ is more electronegative than the Na
since they are isoelectronic. (d) Cl-is more electronegative than that of F
(b) Li+ is a stronger reducing agent than H- 45. Four elements P, Q, R and S have atomic number
(c) More energy is needed to ionize H- than Li+ Z-1, Z, Z+1 and Z+2 respectively. If Z is 17, then
(d) Radius of H- is larger than that of Li+ bond between which pair of elements will be least
covalent:
37. The correct order of increasing first ionization
energy is: (a) S and Q (b) P and R
(a) Ca < K < Ne < P < F (c) S and R (d) S and P
(b) F < Ca < Ne < P < K
(c) K < Ca < P< F < Ne
(d) Ne < F < P < Ca < K
38. The number of d-electrons in Fe2+ ONE OR MORE THAN ONE OPTIONS CORRECT
(atomic number = 26) is not equal to that of: TYPE
(a) p-electrons in 10Ne 1. Select the correct statement(s):
(b) s-electrons in 12Mg (a) Alkali metals have lowest IE in respective
(c) d-electrons in Fe period.
(d) p-electrons in Cl- (b) Noble gases have highest IE in respective
39. Which of the following transition results in period.
increase in magnetic moment value? (c) EA1 of N < EA1 of O
(a) Mn2+ " Mn4+ (b) Ni2+ " Ni4+ (d) F-is the strongest reducing agent among
halide ions.
(c) Cu2+ " Cu+ (d) Zr " Zr2+
40. The compound of vanadium with chlorine has 2. The electronic configuration of given species (X)
magnetic moment 1.73 BM. The vanadium is 1s2, 2s2, 2p6,3s2,3p6,3d5, 4s1. This can be its:
chloride has the formula: (a) Cationic form X+ (b) Anionic form X-
(a) VCl2 (b) VCl3 (c) Excited state (d) Ground state
(c) VCl4 (d) VCl5
3. In which of the following arrangements, the order
41. Which of the following order of radius is not is according to the property indicated against it?
correct? (a) IE1: O > N > C > B
(a) Yb+3 < Pm+3< Ce+3 < La+3
(b) egH (with – ve sign): Cl > F > Br > I
(b) Mg+2 < Na+ < Al < F-
(c) Metallic radius: Rb > K > Na > Li
(c) K > Ca > Mg > Li
(d) Ionic size: F-> Na+ > Mg2+ > Al3+
(d) O < O-2 < F < F-
4. In which of the following arrangements, the order
42. Correct trend of first ionization energy in group-13
is according to the property indicated against it?
is:
(a) B > Al > Ga > In > Tl (a) Basic strength: SbH3 > AsH3 > PH3 > NH3
(b) B > Al > Ga > Tl > In (b) IE1: N > O > C > B
(c) B > Tl > Ga > Al > In (c) Oxidising power: PbO2 > SnO2 > SiO2 > CO2
(d) B > Ga > Al > In > Tl (d) Acid strength: HI > HBr > HCl > HF
1.19

5. Which of the following orders is (are) correct for COMPREHENSIONS TYPE QUESTIONS
size? Read the following passage carefully and answer the
(a) Al ª Ga 2- - +
(b) Te > I > Cs > Ba 2+
question.
3+
(c) Cr < Cr 6+
(d) Pd ª Pt Comprehension # 1 (Q. 12 to 14)
6. Which of the following statements is/are correct? It is not possible to measure the atomic radius precisely
(a) The second ionization enthalpy of oxygen since the electron cloud surrounding the atom does
element is greater than that of fluorine not have a sharp boundary. One practical approach to
element. estimate the size of an atom of a non-metallic element is
(b) The third ionization enthalpy of phosphorus to measure the distance between two atoms when they are
is greater than that of aluminium. bound together by a single bond in a covalent molecule
(c) The first ionization enthalpy of aluminium is and then dividing by two. For metals we define the term
slightly greater than that of gallium. “metallic radius” which is taken as half the internuclear
distance separating the metal cores in the metallic crystal.
(d) The second ionization enthalpy of copper is
The van der Waal’s radius represents the overall size of
greater than that of zinc.
the atoms which includes its valence shell in a non bonded
7. Which of the following is/are correct order(s)of situation. It is the half of the distance between two similar
electron affinity? atoms in separate molecules in a solid. The atomic radius
(a) N < C < O < F (b) P < Si < S < Cl decreases across a period and increases down the group.
Same trends are observed in case of ionic radius of the
(c) Si < P < S < Cl (d) C < N < O < F
species having same number of electrons depends on the
8. Which of the following is correct order of number of protons in their nuclei. Sometimes, atomic and
electronegativity? ionic radii give unexpected trends due to poor shielding of
(a) Cs > Rb > Na (b) Li < Be < B nuclear charge by d- and f-orbital electrons.
(c) C < N < O (d) Cl > F > Br 12. Which of the following relations is correct, if
9. Poor shielding of nuclear charge by d or f- orbital considered for the same element ?
electrons is responsible for which of the following (a) R Van der Waal > R Covalent > R Metallic
facts? (b) R Covalent > R Metallic > R Van der Waal
(a) Atomic radius of Nb (4d- series) is (c) R Van der Waal > R Metallic > R Covalent
comparable to that of Ta (5d- series). (d) R Metallic > R Covalent > R Van der Waal
(b) The 1st ionization enthalpy of copper is less
13. K+, Cl-, Ca2+, S2- ions are isoelectronic. The
than that of zinc.
decreasing order of their size is:
(c) The value of electron gain enthalpy is more
negative for sulphur than for oxygen. (a) Ca2+ > K+ > Cl- > S2-

(d) The 1st ionization energy for gold is greater (b) S2- > Cl- > K+ > Ca2+
than that of silver. (c) K+ > Cl- > Ca2+ > S2-
10. Which of the following is/are true order(s)? (d) S2- > Cl- > Ca2+ > K+
(a) B+ < B < B- Size
14. Select the INCORRECT option regarding atomic/
(b) I < Br < Cl < F Electron gain enthalpy ionic sizes:
(with negative sign)
(a) Zn > Cu (b) Pb2+ > Pb4+
(c) O2-< O < O+ Zeff
(d) Na < Al < Mg < Si Ionization potential (c) Zr ª Hf (d) N3- < Al3+
11. Select the endothermic step(s): Comprehension # 2 (Q. 15 to 17)
(a) S-(g) + e- " S2-(g) Effective nuclear charge (Zeff) is the net attractive force on
(b) Ne(g) + e- " Ne-(g) electrons under Consideration and is equal to:
(c) N(g) + e- " N-(g) Zeff = Z – s (nuclear charge – screening constant). Zeff or
(d) Al2+(g) " Al3+(g)+ e- s is calculated by Slater’s formula, as given.
1.20

If one electron is present in the outermost orbit, there will (a) C > N > O > F (b) O > N > F > C
be no screening in that orbital. Each electron contribute, (c) O > F > N > C (d) F > N > O > C
0.35 (total electrons minus one electron) present in the
outermost shell. 19. Four elements have the following first ionization
energies in kJ mol-1 : 762, 709, 59 and 558. The
A contribution of 0.85 for each electron is taken in the elements are Ga, Ge, In and Sn (not in order).
(n-1)th shell. Which of these elements has the ionization energy
For all other electrons contribution is 1 for each electron. of 762 kJ mol-1?
15. The screening constant (s) for 4s electron of Mn (a) In (b) Ga
(Z = 25) will be : (c) Sn (d) Ge
(a) 18.00 (b) 4.25 20. Among the following ionization reactions, which
(c) 18.35 (d) 21.40 one will have the maximum value of ionization
16. Which of the following statement is wrong? energy?
(a) IE1 of Ga > Al, due to imperfect shielding of (a) Be " Be+ (b) Be+ " Be2+
3d-orbitals in Ga. (c) Sr " Sr+ (d) Sr+ " Sr2+
(b) IE1 of Ga > Al, due to perfect shielding of Comprehension # 4 (Q. 21 to 23)
3d-orbitals in Ga.
Energy is released when an electron is added to neutral
(c) The atomic size of Ga and Al are almost same isolated gaseous atom in its ground state to give monoanion
because of poor shielding effect of electrons and this is known as EA1 or egH1. Greater is the amount
in d-orbitals as the effective nuclear increases of energy released the greater will be EA. EA is expressed
in Ga. in eVatom-1 or kJ mol-1
(d) IE1 of group 16 elements is less than that of
21. EA values of N and P are exceptionally low,
group 15 elements.
because:
17. Which of the following statement is wrong? (a) Both N and P have half-filled p-orbitals in the
(a) The number of lobes in d-orbitals are 4. valence shell.
(b) IE1 of element increases along the period. (b) The atom is more stable than the
(c) IE1 of the group 3 elements is more than that corresponding anion.
of the group 2 elements (c) The electronic configuration of the anion
(d) IE1, IE2 and IE3 of an element are 9.5, 18.5 N- and P- is relatively more stable than the
and 154.4 eV predict that the element has corresponding atom.
either two s-electrons or two p-electrons in (d) Both (b) and (c).
the valence shell. 22. Select the correct statements (More than one
correct):
(a) EA1 and DegH1 of an atom of element have
The energy required to remove an electron from the same magnitude
outermost shell of an isolated gaseous atom is known as (b) DegH1(-ve) of Al > B
IE1 of that atom. Similarly, the energy required for the (c) DegH1(-ve)of P > N
removal of the electron from the unipositive ion, dipositive
(d) DegH1(-ve) of S > O
ion and tripositive ion are known as IE2, IE3 and IE4
respectively, and are called successive ionization energies. 23. Select the correct statements (More than one
The magnitude of the charge depends on the size of the correct):
orbital of electron. Electrons in smaller orbitals are on (a) DegH1of noble gases have large positive
average close with each other and have more repulsion. values.
Thus for Be (2s2), the IE1 and IE2 are 9.3 and 18.2 eV (b) DegH1 of noble gases have large negative
atom-1, whereas for Ca (4s2), the vales are 6.1 and 11.9 eV. values.
18. The correct order of arrangement of the first (c) DegH1 if helium (He) is the lowest of all the
ionization energies of C, N, O and F (in decreasing noble gases.
values) is: (d) DegH1 of Ar is lower than that of Ne.
1.21

SINGLE AND DOUBLE VALUE INTEGER TYPE


QUESTIONS
24. Most stable oxidation state of thallium is +n.
What is the value of n?
1. The correct order of acidic strength is:
25. Total number of elements which have more
ionization energy as compare to their next higher (a) Cl2O7 > SO2 > P4O10
atomic number elements. Li, Be ,B, C, N, O, F, (b) CO2 > N2O5 > SO3
Ne (c) Na2O > MgO >Al2O3
26. How many elements are more electropositive (d) K2O > CaO > MgO
than Cl? (III-JEE, 2000)
B, N, O, S, P, At, H, Li 2. The correct order of radii is:
27. Total number of elements which have only (a) N < Be < B (b) F-< O2- < N3-
single oxidation state (other than zero) in their (c) Na < Li < K (d) Fe3+ < Fe2+ < Fe4+
corresponding stable compounds: Cs, Ba, F, Zn, (III-JEE, 2000)
Be, Al, Sr, Ga, Pb
3. The set representing the correct order of first
28. How many pairs in their first species have lower ionization potential is:
ionization energy than second species?
(a) K > Na > Li (b) Be > Mg > Ca
(a) N and O (b) Li and Li+
(c) B > C > N (d) Ge > Si > C
(c) O and S (d) Ba and Sr
(III-JEE, 2001)
(e) I and I– (f) Be and B
(g) Br and K 4. Identify the least stable ion amongst the following:
(a) Li- (b) Be-
MATCHING THE COLUMN TYPE QUESTIONS (c) B- (d) C-
29. (III-JEE, 2002)
Column I Column II 5. Identify the correct order of acidic strengths of
a. Na > Mg > Al > B p. Oxidizing nature CO2, CuO, CaO, H2O is:
b. F > N > C > B > Si q. Lowest IE1 (a) CaO < CuO < H2O < CO2
c. F > O > Cl > N r. Metallic character (b) H2O < CuO < CaO < CO2
(c) CaO < H2O < CuO < CO2
d. Out B, C, Al and s. Non-metallic
Si, C have character (d) H2O < CO2 < CaO < CuO
t. Highest IE1 (IIT-JEE, 2002)
6. Statement-1: Pb4+ compounds are stronger
30. oxidizing agents than Sn4+ compounds.

Column I Column II Statement-2: The higher oxidation states for the


group 14 elements are more stable for the heavier
a. N2O p. Normal oxide
members for the group due to inert pair effect.
b. Na2O q Neutral oxide (a) Statement-1 is True, Statement-2 is true,
c. Ga2O3 r. Suboxide Statement-2 is a correct explanation for
d. C3O2 s. Basic oxide Statement-1.
e. Mn3O4 t. Amphoteric oxide (b) Statement-1 is True, Statement-2 is true,
Statement-2 is NOT a correct explanation for
f. SnO2 u. Mixed oxide
Statement-1.
(c) Statement-1 is True, Statement-2 is False
(d) Statement-1 is False, Statement-2 is True
(III-JEE, 2008)
1.22

7. Which of the following represent the correct order 14. Which one of the following sets of ions represents
of increasing IE1 for Ca, Ba, S, Se and Ar? the collection of isoelectronic species?
(a) S < Se < Ca < Ba < Ar (a) K+, Cl-, Mg2+, Sc3+
(b) Ba < Ca < Se < S < Ar (b) Na+, Ca2+, Sc3+, F-
(c) Ca < Ba < S < Se < Ar (c) K+, Ca2+, Sc3+, Cl-
(d) Ca < S < Ba < Se < Ar (d) Na+, Mg2+, Al3+, Cl-
(III-JEE, 2013) (AIEEE, 2004)
8. The correct order of ionic radius is: 15. Which one of the following ions has the highest
(a) Ce > Sm > Tb > Lu (b) Lu > Tb > Sm > Ce value of ionic radius?
(c) Tb > Lu > Sm > Ce (d) Sm > Tb > Lu > Ce (a) O2- (b) B3+
(AIEEE, 2002) (c) Li+ (d) F-
9. Ce3+, La3+, Pm3+ and Yb3+ have ionic radii in the (AIEEE, 2004)
increasing order as: 16. Among Al2O3, SiO2, P2O3 and SO2 the correct
(a) La3+ < Ce3+ < Pm3+ < Yb3+ order of acid strength is:
(b) Yb3+ < Pm3+ < Ce3+ < La3+ (a) Al2O3 < SiO2 < SO2 < P2O3
(c) La3+ = Ce3+ < Pm3+ < Yb3+ (b) SiO2 < SO2 < Al2O3 < P2O3
(d) Yb3+ < Pm3+ < La3+ < Ce3+ (c) SO2 < P2O3 < SiO2 < Al2O3
(AIEEE, 2002) (d) Al2O3 < SiO2 < P2O3 < SO2
10. According to the modern Periodic Law of (AIEEE, 2004)
elements, the variation in properties of elements 17. The formation of the oxide ion requires first
is related to their? an exothermic and then an endothermic step as
(a) Nuclear masses shown below:
(b) Atomic numbers O(g) + e- = O-(g) DH° = –142 kJ mol-1
(c) Nuclear neutron-proton number ratio O-(g) + e- = O2-(g) DH° = 844 kJ mol-1
(d) Atomic masses This is because of:
(AIEEE, 2003) (a) O- ion will tend to resist the addition of
11. The reduction in atomic size with increase in another electron
atomic number is a characteristic of elements of: (b) Oxygen has high electron affinity
(a) d-block (c) Oxygen is more electronegative
(b) f-block (d) O- ion has comparatively larger size than
(c) Radioactive series oxygen atom
(d) High atomic masses (AIEEE, 2004)
(AIEEE, 2003) 18. Which among the following factors is the most
12. Which one of the following groups represents important in making fluorine the strongest
a collection of isoelectronic species? (Atomic oxidizing halogen?
number of Cs is 55 and of Br is 35) (a) Hydration enthalpy
(a) N3-, F-, Na+ (b) Be, Al3+, Cl- (b) Ionization enthalpy
(c) Ca2+, Cs+, Br (d) Na+, Ca2+, Mg2+ (c) Electron affinity
(AIEEE, 2003) (d) Bond dissociation energy
13. The atomic numbers of vanadium (V), chromium (AIEEE, 2004)
(Cr), manganese (Mn) and iron (Fe) respectively 19. Pick out the isoelectronic structure from the
23, 24, 25 and 26. Which one of these may be following:
expected to have the higher second ionization I. +CH3 II. H3O+
enthalpy? III. NH3 IV. CH3-
(a) Cr (b) Mn (a) I and II (b) III and IV
(c) Fe (d) V (c) I and III (d) II, III and IV
(AIEEE, 2003) (AIEEE, 2005)
1.23

20. Which of the following factors may be regarded 26. Lanthanoid contraction is caused due to:
as the main cause of lathanoid contraction? (a) The same effective nuclear charge from Ce to
(a) Poor shielding of one of 4f electron by Lu
another in the subshell. (b) The imperfect shielding on outer electrons
(b) Effective shielding of one of 4f electrons by by 4f electrons from the nuclear charge
another in the subshell. (c) The appreciable shielding on outer electrons
(c) Poorer shielding of 5d electrons by 4f by 4f electrons from the nuclear charge
electrons (d) The appreciable shielding on outer electrons
(d) Greater shielding of 5d electrons by 4f by 5d electrons from the nuclear charge
electrons
(AIEEE, 2006)
(AIEEE, 2005)
27. Following statements regarding the periodic
21. In which of the following arrangements the
trends of chemical reactivity of the alkali metals
order is NOT according to the property indicated
and the halogens are given. Which of these
against it?
statements gives the correct picture?
(a) Al3+ < Mg2+ < Na+ < F- - Increasing ionic size
(a) Chemical reactivity increases with increase
(b) B < C < N < O - Increasing first ionisation in atomic number down the group in both the
enthalpy alkali metals and halogens
(c) I < Br < F < Cl - Increasing electron gain (b) In alkali metals the reactivity increases but
enthalpy (with negetive sign) in the halogens it decreases with increase in
(d) Li < Na < K < Rb - Increasing metallic redius atomic number down the group
(AIEEE, 2005) (c) The reactivity decreases in the alkali metals
22. The lanthanide contraction is responsible for the but increases in the halogens with increase in
fact that: atomic number down the group
(a) Zr and Y have about the same radius. (d) In both alkali metals and the halogens the
(b) Zr and Nb have similar oxidation state. chemical reactivity decreases with increases
in atomic number down the group
(c) Zr and Hf have about the same radius.
(AIEEE, 2006)
(d) Zr and Zn have same oxidation state.
(AIEEE, 2005) 28. The set representing the correct order of ionic
radius is:
23. Which of the following oxides is amphoteric in
(a) Na+ > Li+ > Mg2+ > Be2+
character?
(b) Li+ > Na+> Mg2+ >Be2+
(a) SnO2 (b) SiO2
(c) Mg2+ > Be2+ > Li+ > Na+
(c) CO2 (d) CaO (d) Li+ > Be2+ > Na+ > Mg2+
(AIEEE, 2005)
(AIEEE, 2009)
24. The increasing order of the first ionization
29. The correct sequence which shows decreasing
enthalpies of the elements B, P, S and F (lowest
order of the ionic radii of the elements is:
first) is:
(a) Al3+ > Mg2+ > Na+> F-> O2-
(a) F < S < P < B (b) P < S < B < F
(b) Na+ > Mg2+ > Al3+ > O2- > F-
(c) B < P < S < F (d) B < S < P < F
(c) Na+ > F- > Mg2+ > O2- > Al3+
(AIEEE, 2006)
(d) O2- > F-> Na+> Mg2+ > Al3+
25. Which one of the following sets of ions represents
(AIEEE, 2010)
a collection of isoelectronic species?
(a) N3-, O2-, F-, S2- 30. The outer electronic configuration of Gd (Atomic
number 64) is:
(b) Li+, Na+, Mg2+, Ca2+
(c) K+, Cl-, Ca2+, Sc3+ (a) 4f3 5d5 6s2 (b) 4f8 5d0 6s2
(d) Ba2+, Sr2+, K+, Ca2+ (c) 4f4 5d4 6s2 (d) 4f7 5d1 6s2
(AIEEE, 2006) (AIEEE, 2011)
1.24

31. The correct order of electron gain enthalpy with 38. What is the following represents the correct order
negative sign of F, Cl, Br and I having atomic of increasing first ionization enthalpy for Ca, Ba,
number 9,17, 35 and 53 respectively is: S, Se and Ar?
(a) F > Cl > Br > I (b) Cl > F > Br > I (a) Ca < S < Ba < Se < Ar
(c) Br > Cl > I > F (d) I > Br > Cl > F (b) S < Se < Ca < Ba < Ar
(AIEEE, 2011) (c) Ba < Ca < Se < S < Ar
(d) Ca < Ba < S < Se < Ar
32. Which one of the following orders presents the
correct sequence of the increasing basic nature of (JEE Main, 2013)
the given oxides? 39. The first ionization potential of Na is 5.1 eV. The
(a) Al2O3 < MgO < Na2O < K2O value of electron gain enthalpy of Na+ will be:
(b) MgO < K2O < Al2O3 < Na2O (a) -2.55 eV (b) -5.1 eV
(c) Na2O < K2O < MgO < Al2O3 (c) -10.2 eV (d) +2.55 eV
(d) K2O < Na2O < Al2O3 < MgO (JEE Main, 2013)
(AIEEE, 2011) 40. Similarity in chemical properties of the atoms of
33. The increasing order of the ionic radii of the given elements in a group of the periodic table is most
isoelectronic species is: closely related to:
(a) Cl-, Ca2+, K+, S2- (b) S2-, Cl-, Ca2+, K+ (a) Atomic numbers
(c) Ca2+, K+, Cl-, S2- (d) K+, S2-, Ca2+, Cl- (b) Atomic masses
(AIEEE, 2012) (c) Number of principal energy levels
(d) Number of valence electrons
34. Which of the following presents the correct order
of second ionization enthalpies of C, N, O and F? (JEE Main Online 2014)

(a) O > N > F > C (b) F > O > N > C 41. Which of the following arrangements represents
the increasing order (smallest to largest) of ionic
(c) C > N > O > F (d) O > F > N > C radii of the given species O2-, S2-, N3-, P3-?
(JEE Main Online 2012)
(a) O2- < N3- < S2-< P3- (b) O2-< P3-< N3-< S2-
35. Which among the following elements has the
(c) N3-< O2-< P3-< S2- (d) N3- < S2-< O2-< P3-
highest ionization enthalpy?
(JEE Main Online 2014)
(a) Nitrogen (b) Boron
42. The ionic radii (in Å) of N3- , O2- and F- are
(c) Carbon (d) Oxygen respectively:
(JEE Main Online 2012)
(a) 1.36, 1.40 and 1.71 (b) 1.36, 1.71 and 1.40
36. Electron gain enthalpy with negative sign of
(c) 1.71, 1.40 and 1.36 (d) 1.71, 1.36 and 1.40
fluorine is less than that of chlorine due to:
(JEE Main, 2015)
(a) High ionization enthalpy of fluorine
(b) Smaller size of chlorine atom 43. Which of the following atoms has the highest first
ionization energy?
(c) Smaller size of fluorine atom
(d) Bigger size of 2p orbital of fluorine (a) Na (b) K
(JEE Main Online 2013) (c) Sc (d) Rb
(JEE Main, 2016)
37. The order of increasing sizes of atomic radii
among the elements O, S, Se and As is:
(a) As < S < O < Se (b) Se < S < As < O
(c) O < S < As < Se (d) O < S < Se As
(JEE Main Online 2013)
1.25

Answer Key

1. (a) 2. (d) 3. (c) 4. (c) 5. (d) 6. (b) 7. (c) 8. (c) 9. (a) 10. (c)
11. (a) 12. (d) 13. (b) 14. (d) 15. (b) 16. (c) 17. (d) 18. (d) 19. (c) 20. (b)
21. (a) 22. (d) 23. (c) 24. (a) 25. (c) 26. (c) 27. (b) 28. (d) 29. (b) 30. (b)
31. (b) 32. (d) 33. (a) 34. (b) 35. (d) 36. (c) 37. (c) 38. (c) 39. (a) 40. (d)
41. (a) 42. (b) 43. (d) 44. (d) 45. (d)

1. (d) 2. (c) 3. (b) 4. (d) 5. (b) 6. (b) 7. (c) 8. (c) 9. (b) 10. (d)
11. (d) 12. (d) 13. (b) 14. (c) 15. (c) 16. (d) 17. (c) 18. (c) 19. (b) 20. (c)
21. (d) 22. (d) 23. (d) 24. (c) 25. (a) 26. (d) 27. (d) 28. (a) 29. (d) 30. (c)
31. (c) 32. (c) 33. (a) 34. (d) 35. (b) 36. (d) 37. (c) 38. (d) 39. (b) 40. (c)
41. (d) 42. (c) 43. (d) 44. (d) 45. (a)

1. (a,b,c) 2. (a,d) 3. (b,c,d) 4. (b,c,d) 5. (a,b,d) 6. (a,b,d) 7. (a,b) 8. (b,c)


9. (a,d)   10. (a,c,d) 11. (a,b,c,d) 12. (c) 13. (b) 14. (d) 15. (d) 16. (b)
17. (c) 18. (d) 19. (d) 20. (b) 21. (a) 22. (a,b,c,d) 23. (a,d) 24. (1)
25. (2) 26. (6) 27. (7) 28. (2)
29. a r; b s; c p;d t
30. a q,r; b p,s; c t; d r; e u; f t

1. (a) 2. (b) 3. (b) 4. (b) 5. (a) 6. (c) 7. (b) 8. (a) 9. (b) 10. (b)
11. (b) 12. (a) 13. (a) 14. (c) 15. (a) 16. (d) 17. (a) 18. (a) 19. (d) 20. (c)
21. (b) 22. (c) 23. (a) 24. (d) 25. (c) 26. (b) 27. (b) 28. (a) 29. (d) 30. (d)
31. (b) 32. (a) 33. (c) 34. (d) 35. (a) 36. (c) 37. (d) 38. (c) 39. (b) 40. (d)
41. (a) 42. (c) 43. (c)

Hints and Solutions


3. (c) This element belongs to d-block
Group number of d-block = (ns + (n – 1) d)
1. (a) Number of e- in O2+ = 15 = number of e- in X2-  = 2 + 3 = 5
Atomic number of X2- is 13 (z) 4. (c) Cr belongs to fourth period.
Number of neutrons = Z+1 = 14 5. (d) Mass number (proton + neutron) of X2+ = 20
Mass number of X2- = 13 + 14 = 27 Number of neutrons = 10
2. (d) Hydrogen resembles halogens in some Hence, Number of protons of X2+ =10
properties and also resembles alkali metals in Number of e- in X2+ = 8
some properties. So, it can be placed in first or
6. (b) Elements of group 1, 2 and 13 to 17 are called
17th group.
as representative elements.
1.26

7. (c) Inert gas elements " 1s2 and ns2 np6 almost similar electronegativity and polarizing
Representative elements " ns1-2 and ns2 np1 to power.
ns2 np5 19. (c) Be and Al show diagonal relationship.
Transition elements " (n – 1) d1-10 ns1or2 20. (b) Correct set of magic numbers for group VIA is
Inner – transition elements " (n – 2) f1-14 8,18,18,32.
(n – 1) d0-1 ns2 21. (a) For a given series of isoelectronic species, as
8. (c) Z = 108, group number Viii B , period – 7th atomic number increases, radius decreases.
9. (a) The element, with atomic number 56, belongs Se-2 > Br- > Kr > Rb+ > Sr+2
to group 2 (alkaline earth metal). The element, 22. (d) Order of ionic radius is:
with atomic number 12, also belongs to group Be+2 < Mg2+ < Ca2+ < Sr2+
2. 23. (c) Their relative positions in periodic table,
0. (c) e- configuration of M2+=1s2 2s2 2p6 3s2 3p6 3d6 C N (II period)
e- configuration of M = 1s2 2s2 2p6 3s2 3p6 4s2 P S (III period)
3d6 The correct order of size is:
= atomic number is 26
N < C < S < P
Atomic weight of M = 56
24. (a) Due to lanthanoid contraction Zr and Hf have
Number of neutrons = 56 – 26 = 30 almost similar atomic radii.
1
11. (a) Ionic mobility µ 25. (c) As positive oxidation state increases, radius
Size in aqueousmedium decreases.
Order of size in aqueous medium: +4 +7 +2 +3
Li+ (aq) > Na+(aq) > K+(aq)
MnO2 KMnO4 MnO K3 [Mn(CN)6]
Order of ionic mobility:
26. (c) Down the group, ionization enthalpy decreases.
Li+ (aq) < Na+(aq) < K+(aq)
It is due to increment in atomic size.
12. (d) p-block elements mostly form acidic oxides
27. (b) Due to extra stability of the half filled p-orbitals,
not basic oxides.
N has greater ionization potential than that of
13. (b) Ion e- configuration  Number of unpaired e-  m
O.
Cr+3 (Ar) 4s° 3d3 3 15
28. (d)
Order of ionization energy is:
Mn (Ar) 4s° 3d5
+2
5 35
M- < M+ < M2+
Fe+2 (Ar) 4s°3d6 4 24
+2 9 M+2has smallest size and highest effective
Cu (Ar) 4s°3d 1 3
- +3 5
nuclear charge.
14. (d) e configuration of Fe = (Ar) 4s° 3d
In IE2 process, 1e- is removed from X+(g)
29. (b)
It has unpaired e- hence, it is a paramagnetic
species. 30. (b) Down the group, IE decreases. It is due to
increment in atomic size.
15. (b) Ag+ = (Kr) 5s° 4d10
Cu+2 = (Ar) 4s° 3d9 31. (b) In lanthanide series, as atomic number
increases, atomic radius gradually decreases It
Ga+3 = (Ar) 4s° 3d10 4p°
is called as lanthanide contraction.
Zn+2 = (Ar) 4s° 3d10
32. (d) Electron affinity (EA) = - D e.g. (e- gain
16. (c) m = 3.87 BM
enthalpy)
Hence, number of unpaired e- in Mn x+ = 3
33. (a) Outermost sub shell of N is half filled. In
Mn = (Ar) 4s° 3d5
process, N " N-, absorption of energy takes
Mn4+ = (Ar) 4s° 3d3
place.
17. (d) The first element of a group generally belongs
to second period. It has a small size, high 34. (b) Second and successive electron gain enthalpy
ionization potential and electronegativity. It of an element is always positive because anions
does not have d – orbitals. resist addition of another e‑.
18. (d) Oxides of Be and Al are amphoteric. They have 35. (d) As and Sb are metalloids.
1.27

36. (c) In group VIIA (Halogens), 7. (c) The electronic configuration of element is 3s2
F2 and Cl2 " gas 3p6 3d5 4s2. This element is ‘Mn’.
Br2 " liquid 8. (c) La and Ac belong to d - block
I2 " solid Element having atomic number 31 belongs to
37. (c) Halogens have highest electro negativity. 4th period.
38. (c) percentage ionic character Elements after 92U are man-made elements.
= 16 (DEN) + 3.5 (DEN)2 9. (b) X(g) + E1 " X+(g) + e-
= 16 (3.3) + 3.5 (3.3)2 A
( 0 atoms)
= 90.9 % 2
A0
39. (a) In halogens, bond length decreases from iodine atoms absorbs E1 energy
to fluorine. 2
2E1
40. (d) Acidic strength of µ Electro negativity of So, 1 atom absorbs energy
A0
oxides and hydroxides central atom 2E1
Hence, ionization energy of X(g) is
41. (a) Order of acidic strength: A0
Al2O3 < B2O3 < CO2 < NO2 2E1
X(g) + " X+(g) + e-……………eq (1)
42. (b) Neutral oxides are CO, NO, N2O, H2O A0
43. (d) Al2O3 Amphoteric X+(g) + 2e " X-(g) + E2
A
( 0 ions)
B2O3 Acidic 2
44. (d) In order of acidic strength: A0
ions release E2 energy
HCl > PH3 > SiH4 > CH4 2
2E2
45. (d) They all are acidic. So, 1 ion releases energy
A0
2E2
Hence, X+(g) + 2e-" X-(g) + ….....eq(2)
(1 ion) A0
1. d) The elements ‘X’ is ‘As’. Its electronic Equation for electron affinity of X(g) is,
configuration is (Ar) 4s2 3d10 4p3. X(g) + e-" X-(g) + EA of X(g).
2. (c) Bond length of H – F = rH + rF – 0.09 (D EN) We can get this equation by adding equation
= 0.37 + 0.72 – 0.09 (1.9) (1) and (2),
= 0.92 Å 2(E 2 - E1 )
X(g) + e-"X-(g) +
3. (b) Order of ionic radius: A0
I- > Cl- 2(E 2 - E1 )
Hence, electron affinity of X(g) is .
Te2- > I- (They are isoelectronic) A0
4. (d) Due to inert pair effect the more common 10. (d) In covalent hydrides, as we move left to right
oxidation state for Tl, Pb and Bi are +1, +2 and in a period acidic strength increases.
+3 respectively. 11. (d) In any period, noble gas has largest atomic
5. (b) The relative positions of these elements in radius because for noble gases vander waal’s
periodic table:- radius is considered.

Acidic strength of Oxides µ Electronegativity
12. (d)
Mg P Cl of central atom.
Ca CO2 is more acidic than SiO2 hence, option ‘i’
Order of atomic radius:- is incorrect.
Ca > Mg 13. (b) The elements having atomic number 113
Mg > P > Cl belongs to group 13 and 7th period.
6. (b) The element ‘X’ is ‘Cu’. Its electronic 14. (c) The electronic configuration of Cr is (Ar) 3d5
configuration is (Ar) 4s1 3d10. In third shell 4s1 and it is a representative element. In HN3,
18e- and in fourth shell 1e- is present. oxidation state of nitrogen is -1/3.
1.28

15. (c) The biggest jump in successive ionization 31. (c) Z has biggest jump from IE1 to IE2 hence, it
energy is from IE3 to IE4. Hence, this element has 1 valence e-.
has 3 valence e-. 32. (c) The order of radius is:
16. (d) In s-block, all oxides are basic except BeO. Be > B > C > N > O
BeO is an amphoteric oxide. They are obtained 33. (a) Elements A, B, C and D are Ne, O, Na and F
by the electrolysis of fused chlorides. respectively. Their correct order of EA is:
17. (c) Na+ and F- are isoelectronic but Na belongs to Ne < Na < O < F
3rd period while F belongs to 2nd period. 34. (d) The first member of the lanthanoid series is Ce
18. (c) The element (P) is Lu, which is a lanthanoid. (cerium). Pricogens are group 15 elements.
All lanthanoids belong to the 3rd period. 35. (b) Smaller the value of IE, easier is the formation
19. (b) Ionic radius of Li+ (0.76Å) is larger than that of cation. Larger the value of EA, easier is the
of Mg2+ (0.72Å). formation of anoin.
20. (c) The bond having greater polarity (or, greater IE+EA
Electronegativity on Mulliken’s scale =
DEN) has greater chance of dissociation in 1 2
water. Size µ
Z eff
21. (d) The equation for second ionization energy of 36. (d) For isoelectronic species, as atomic number
M is, increases radius decreases.
M+(g) " M2+(g) + e- 37. (c) The relative order of these elements in periodic
This equation will be obtained by (V)–(iii) table is:
M(g) "M2+(g) + 2e- F Ne
– (M(g) "M+(g)+ e-)
P
M+(g) " M+2 + e- K Ca
22. (d) EA1 process is generally exothermic while
The correct order of IE1 is:
EA2 process is always endothermic. Hence,
K < Ca < P < F < Ne
DH1, DH2 and DH3 are negative whereas DH4
is positive. 38. (d) The number of d-electrons in
Fe2+ ([Ar] 4s03d6) are 6.
23. (d) As positive oxidation state increases, radius
p-electrons in Ne (1s2 2s2 2p6) are 6.
decreases.
s-electrons in Mg (1s2 2s2 2p6 3s2) are 6.
24. (c) Completely filled sub-shell is more stable than
d-electrons in Fe ([Ar] 4s23d6) are 6.
half filled sub-shell.
p-electrons in Cl- (1s2 2s2 2p6 3s2 3p6) are 12.
25. (a) The biggest jump in successive ionization
energy is from IE3 to IE4. Hence, the number 39. (b) Ni+2= [Ar] 4s0 3d8 (2 unpaired electrons)
of valence electron is 3. Ni+4= [Ar] 4s0 3d6 (4 unpaired electrons)
26. (d) Due to lanthanoid contraction 5d-series 40. (c) Magnetic moment = 1.73BM
elements have greater effective nuclear hence, The number of unpaired e- = 1
they have higher ionization energy. V = [Ar] 4s2 3d3
27. (d) IE2 for an element is higher than IE1 because V+4 = [Ar] 4s° 3d1
after removal of 1st electron, 2nd electron is In VCl4, oxidation state of ‘V’ is ‘+4’
removed from the cation. The cation is smaller 41. (d) The correct order of radius is:
than its parent atom and it has greater effective F < O < F-< O-2
nuclear charge (Zeff) than its parent atom.
42. (c) The correct trend of first ionization energy is:
28. (a) Due to extra stability of half-filled p-subshell,
elements of group 15 have higher IE than B > Al < Ga > In < Tl
elements of group 16.
29. (d) Fluorine has small size, high electron density Higher Higher
and an increased electronic repulsion. Zeff Zeff
30. (c) The order of radius is: B > Tl > Ga > Al > In (Based on practical
rvander waal > rmetallic > rcovalent values)
1.29

43. (d) Cs+ is largest cation and F- is smallest anion 10. (a, c, d) The correct order of DHeg(With negative
hence, CsF has the lowest anion to cation size sign) is:
ratio. Cl > F > Br >I
44. (d) F is more electronegative than that of Cl-. The remaining orders are correct.
Anions are less electronegative than neutral
11. (a, b, c, d) S-(g) + e-" S2-(g); EA2 of S
atoms.
Ne (g) + e-" Ne-(g); EA1 of Ne
45. (a)
N (g) + e-" N-(g); EA1 of N
Elements Atomic number
Al2+(g) + e-" Al3+(g); IE3 of Al
P 16
These all steps are endothermic.
Q 17
12. (c) rVan der Waal > rMetallic > rCovalent
R 18
13. (b) In isoelectronic series, as atomic number
S 19 increases, radius decreases.
The bond between S (alkali metals) and Q 14. (d) Both N3- and Al3+ are isoelectronic. The
(halogens) will be most ionic (least covalent) correct order of radius is:
N3- > Al3+
15. (d) Mn = 1s2 2s2 2p6 3s2 3p6 3d5 4s2
1. (a, b, c) Value of ionization energy increases from left Other (n-1) ns
to right in a period. s = 1 × 0.35 + 13 × 0.85 + 10 × 1
Hence, alkali metals have lowest IE and   = 21.40
noble gases have highest
IE in respective period. 16. (b) IE1 of Ga > Al, due to imperfect shielding of
Due to half filled outermost 2p-subshell, N 3d-orbitals in Ga.
has lower EA1 than O 17. (c) IE1 of the group 2 elements is more than that
(a, d) If ‘X’ Mn then this e- configuration represent of the group 3 elements.
cationic form X+ (Mn+) 18. (d) Order of the first ionization energy is:
If ‘X’ is ‘Cr’ then this e- configuration C<O<N<F
represents ground state. 19. (d) Relative positions of these elements in
3. . (b, c, d) The correct order of IE1 is: periodic table is:
N > O > C > B
The remaining orders are correct. Ga Ge
(b, c, d) The correct order of basic strength is: In Sn
SbH3 < AsH3 < PH3< NH3 Among these four elements, Ge has highest
The remaining orders are correct. first ionization energy.
5. (a, b, d) As positive charge increases, radius 20. (b) Correct orders of ionization energy:
decreases. Hence, Cr+3 is larger than Cr+6. Be  > Sr
6. (a, b, d) Due to higher effective nuclear charge, Ga Be+ > Sr+
has greater first ionization enthalpy than Al. Be+ > Be
7. (a, b) The correct orders of electron affinity are:
21. (a) Both N and P have stable half-filled p-orbitals
N<C<O<F in the outermost shell.
P < Si < S< Cl
22. (a, b, c, d) | EA1 | = |DegH1 |
8. (b, c) Down the group, electronegativity decreases.
As we move from left to right in a period Second period elements have lower EA1 than
electronegativity increases. third period elements.
9. (a, d) Due to poor shielding of nuclear charge by d 23. (a, d) Noble gases have stable outermost shell e-
or f-orbital electrons, 5d-series elements have configuration hence, DegH1 of noble gases
greater elective nuclear charge. have large positive values.
1.30

24. Due to inert pair effect, the most stable 13. (a) Electronic configuration of Cr is (Ar) 4s1 3d5
oxidation stable of Tl is +1. after removal 1e-, next will be removed from
25. Order of ionization energy is: half filled d-subshell.
Li < Be >B < C < N > O < F < Ne 14. (c) K+, Ca+2, Sc+3 and Cl- all have 18 electrons.
26. B, S, P, At, H, Li 15. (a) Correct order of radius:
27. Cs, Ba, F, Zn, Be, Al, Sr B3+ < Li+ < F- < O-2
16. (d) Acidic strength of oxides µ electronegativity
28. Li and Li+ (Li < Li+)

Ba and Sr (Ba < Sr) } Order
energy
of ionization
of central atom.
17. (a) O-(g) + e–"O-2 (g). This process is endo-
29. a " r b " s c" p d" t
  

thermic because anion will tend to resist the


30. a "q, r b " p, s c" t d" r addition of another electron.
e " u f"t 18. (a) Due to very high hydration enthalpy of F-, F2 is
strongest oxidizing halogen.
19. (d) H3O+, NH3 and CH3- all have 10e-.
20. (c) The main cause of lanthanoid contraction
is poor shielding by 4f electrons on outer
electrons.
1. (a) Acidic strength of oxides µ electronegativity 21. (b) The correct order of first ionization enthalpy:
of central atom. B<C<O<N
2. (b) In isoelectronic species, as atomic number 22. (c) Due to lanthanoid contraction, 4d and 5d-series
increases radius decreases. elements have almost similar radius.
3. (b) Down the group ionization potential decreases. 23. (a) SnO2 is an amphoteric oxide.
4. (b) Alkaline earth metals anion are unstable 24. (d) The correct order of first ionization enthalpy:
because they have fully filled outermost sub-
S<P
shell.
B<F
5. (a) Acidic strength of oxides µ electronegativity
25. (c) K , Cl-, Ca+2 and Sc+3 all have 18 electrons.
+
of central atom.
26. (b) Cause of lanthanoid contraction is the
6. (c) The lower oxidation states for the group
imperfect shielding on outer electrons by 4f
14 elements are more stable for the heavier
electrons from the nuclear charge.
members for the group due to inert pair effect.
27. (b) Order of reactivity in alkali metals:
7. (b) Relative positions of these elements in periodic
Li < Na < K < Rb < Cs
table,
Order of reactivity in halogens:
F2 > Cl2 > Br2 > I2
S Ar
28. (a) The correct order of ionic radius is:
Ca Se
Na+ > Li+
Ba
Li+ > Mg+2
The correct order of increasing IE1: Mg+2 > Be+2
Ba < Ca < Se < S < Ar 29. (d) In isoelectronic species, as atomic number
8. (a) In lanthanoids, as atomic number increases increases ionic radius decreases. The correct
radius decreases. order of radius is:
9. (b) In lanthanoids, as atomic number increases O-2 > F- > Na+ > Mg2+> Al3+
ionic radius decreases. 30. (d) The electronic configuration of Gd (atomic
10. (b) The variation in properties of elements is number 64) is:
related to their atomic numbers. [Xe] 6s2 4f7 5d1
11. (b) It is due to lanthanoid contraction. 31. (b) The correct order of negative e- gain enthalpy
12. (a) N-3, F- and Na+ all have 10 electrons. (electron affinity) is:
Cl > F > Br > I
1.31

32. (a) Basic strength of oxides µ 1 38. (c) The relative position of elements in periodic
Electronegativity of
central atom table is:

33. (c) In isoelectronic species, as atomic number Se Ar
increases ionic radius decreases. The order of Ca Se
radius is:
Ca2+ < K+ < Cl-< S2-
Ba
34. (d) IE of M = IE1 of M+
C+ N+ O+ F+ The correct order of first ionization enthalpy
2p1 2p2 2p3 2p4 is:
The correct order of second ionization enthalpy Ba < Ca < Se < S < Ar
is: 39. (b) Na (g) + 5.1ev " Na+ (g) +e-
C<N<F<O Na+ (g) + e- "Na + 5.1ev; DHeg = - 5.1ev
35. (a) The correct order of ionization enthalpy is: (e- gain enthalpy)
B<C<O<N 40. (d) All elements in a group have similar number of
36. (c) F-atom has smaller size and incoming e- feels valence electrons.
more repulsion from already present e- in 41. (a) P-3 and S-2 are isoelectronic hence, their order
F-atom hence, electron gain enthalpy with of radius is S-2 < P-3
negative sign (electron affinity) of fluorine is 42. (c) N-3, O-2and F- are isoelectronic hence, their
less than that of chlorine. order of radius is
37. (d) The relative position of elements in periodic N-3 > O-2 > F-
table is: (1.71 Å) (1.40 Å) (1.36Å)
O 43. (c) Order of first ionization energy is:
S Sc > Na > K > Rb
As Se
The correct order of atomic radius is:
O < S < Se < As
Chapter

Key Concepts

INTRODUCTION
The formation of a chemical bond between two atoms
In Lewis structure, a bond between the two atoms is shown
implies that the system consisting of these two atoms at
by Lewis electron-dot symbols in which valence electrons
stable internuclear distance is energetically more stable
are shown by dots around the letter symbol of the atom.
than the two isolated atoms. A general study on the
The dots are placed as follows.
reactivity of different elements revealed that noble gases
have little tendency to combine with other elements. This Place a single dot on the four sides of the letter symbol
leads to the fact that the noble gases have stable outer followed by the second dot till all the valence-electrons
configuration (ns)2 (np)6 (octet configuration). All other have been accounted for.
atoms combine to achieve the stable octet configuration Illustrations:
either by mutual sharing of electrons (covalent bond) or Lithium Beryllium
by complete transfer of electron(s) from one atom to other (2s1) (2s)2 Be
(ionic bond).
Boron Carbon
B (2s)2(2p)2 C
(2s)2(2p)1
KÖSSEL LEWIS THEORY Nitrogen Oxygen
OF CHEMICAL COMBINATION N (2s)2(2p)4 O
(2s)2(2p)3
According to this theory, atoms can combine either Fluorine Neon
by transfer of outer-shell electrons, known as valence (2s)2(2p)5 F (2s)2(2p)6 Ne
electrons, from one atom to another or by sharing the
valence electron(s) in order to achieve octet configuration
(i.e., a total of eight electrons) in their respective valence
shells.
The sharing of electron(s) leads to the formation of A covalent bond involves mutual sharing of valence
covalent bond while transferring of electron(s) leads to electrons between two atoms. The sharing of two, four and
the formation of ionic bond between the two involved six electron leads to the formation of a single, double and
atoms. triple bond, respectively.
2.2

A covalent bond is formed if the atoms have lesser number


of valence electrons as compared to the nearby noble gas
which has octet configuration. Such elements are known
as electronegative elements. Thus, the criterion of the
formation of covalent bond is: If a pair of electrtons shared between two atoms comes
exclusively form one of the atoms, the bond formed is
Electronegative element + Electronegative element
said to be a coordinate covelent (or dative) bond. To keep
Covalent bond track of electrons, a coordinate covalent bond may be
Exception to the octet rule is the hydrogen atom which can represented by an arrow ( ). Once a coordinate bond is
accommodate only two electrons which corresponds to the formed, it behaves like a covalent bond.
electronic configuration of nearby helium (1s2) atom. H3N – BF3
H F H F
Illustrations:
H–N + B–F H–N B–F

Cl2 Cl + Cl Cl Cl or Cl - Cl H F H F
Formation of Double Bond(s) Writing a Lewis Structure

O2 O + O O O or O=O The structure of a molecule or ion may be written by


following the steps listed below:
Formation of Triple Bond(s) 1. Calculate the total number of valence electrons of the
atoms in the molecule. For an anion, add the number
N2 N + N N N or N∫N
of negative charges and for a cation, subtract the
number of positive charges.
2. Write the skeleton structure of the molecule or ion
connecting every bonded pair of atoms by a single
bond, i.e., a pair of electron dots.
The octet rule is generally obeyed by the elements of
second and third periods with the following exceptions:- 3. Assign a total of eight electrons in each atom (except
hydrogen) surrounding the central atom.
The Incomplete Octet
4. Distribute the remaining electrons (if any) as pairs to
BeCl2
the central atom.
Cl Cl Cl Be Cl If there are fewer than eight electrons on the central atom,
move one or two pairs of electron from a surrounding
4 electrons atom to form double or triple bond between the two atom.
Odd-Electron Molecules
Atoms that often form multiple bond are C, N, O and S.
All atoms of a compound containing odd number of
Lewis structure of COCl2
electrons will not satisfy octet rule as even number of
electrons are required for pairing of electrons. Step 1 Valence electrons are 4 + 6 + 2 × 7 = 24
NO Step 2 Carbon being the most electropositive atoms
occupies the central position to which other atoms
N + O N + O N O
are bonded.
7 electrons O
The Expanded Octet
Cl : C : Cl
Elements of third period and beyond can accommodate Step 3 Assign 8 electrons each to surrounding atoms
more than 8 electrons due to the availability of vacant d
O
orbitals.
Cl : C : Cl
PCl5 Cl
Cl Cl There were 24 valence electrons and all of them
5 Cl + P 5 Cl + P P have been distribuled. However, the central C
Cl Cl atom has only 6 electrons. In order that this atom
(P has 10 electrons) also has 8 electrons, move one pair of elctrons
2.3

from O to the bond connecting C atom, thus


forming a double bond.
O O O
||
Each atom in a molecule has its own ability to attract
Cl : C : Cl Cl : C : Cl   or  Cl – C – Cl
the bonded pair of electrons. This ability is known
as electronegativity. The bonded pair of electrons in
homonuclear diatomic molecules (such as H2, O2, F2, Cl2,
etc.) is shared equally by both atoms. This is not correct
in the case of heteronuclear diatomic molecules (such as
The formal charge an atom is the difference between the
HCl, HF, NO, etc.) The bonded pair of electrons is closer
valence electrons in an isolated atom and the number of
to the atom having larger electronegativity. Consequently,
electrons assigned to that atoms in a Lewis structure. The
this atom acquires a partial negative charge while the other
equation for computing formal charge is,
atom acquires equal partial positive charge. Because of the
Formal charge = Valence electrons on free atom – charge separation, the covalent bond between these two
1
Number of (Nonbonding + bonding) electrons in atoms is said to be a polar covalent bond.
a Lewis structure 2

The sum of the formal charges of atoms in a Lewis


structure is equal to the charge on the molecular species.
The polarization of bonded pair of electrons between two
Illustration atoms is expressed in terms of physical quantity known as
COCl2 molecule   Lewis structure Cl : C : Cl dipole moment (symbol : µ). It is defined as
µ = ( q) (r)
O
where q is the partial charge separation between two
Atom Valence Electrons in Lewis Formal atoms and r is the distance between the two atoms.
electrons structure Charge Representation of Dipole Moment
in a free Non- Bonding
atom Dipole moment is a vector quantity, i.e., it has magnitude
boding
as well as direction. In chemistry, dipole moment is
1 indicated by the crossed arrow as shown in the following.
Cl 7 6 2 7 − (6 + × 2) = 0
2 positive end negative end
1 that is, it is directed from positive end to the negative end.
O 6 4 4 6 − ( 4 + × 4) = 0
2
Unit of Dipole Moment
1
C 4 0 8 4 − ( 0 + × 8) = 0 In SI system, unit of dipole moment = (unit of q)
2 (unit of ) = Cm
In CGS system, unit of dipole moment = (esu)
(cm)
Most molecules have dipole moment of the order of 10–18
esu cm. This value of dipole moment is known as 1 debye
Computing formal charge of atoms in a molecule or ion (written as 1 D).
helps deciding a possible Lewis structure of the species.   1.6 × 10 −19 C  
1D = 10–18 esu cm = (10–18) (1esu ) 
The guiding principles are as follows:  4.8 × 10 −10 esu  
   
(10 m) = 3.33 × 10 cm
–2 –30
having the lowest magnitude of formal charge is the
preferred structure.

of formal charges, the one having negative formal


Each bond in a molecule has a dipole moment, known
charges one the more elecronegative atoms is the
as bond moment. The dipole moment of a molecule is
preferred structure.
obtained by the vector addition of these bond moments.
2.4

Illustration µ AB
Percent ionic character = ×100
The bond moment of O – H bond is 1.52 D. The bond µ ionic
angle of H2O is 105º. The dipole moment of H2O molecule where µionic = (e rAB) corresponds to 100% ionic character
will be of the bond.
µH2O = 2µOH cos (105º/2) = 2 (1.52 D) (0.609) = 1.85 D Illustration:
H
52.5º The bond moment of O – H bond is 1.52 D. If bond length
O
O – H is 95 pm, its percent ionic character will be
H µ
Percent ionic character = OH ×100
e rOH
(1.52 D)(3.33 × 10 −30 Cm / 1D)
= × 100
(1.6 × 10 −19 C)(95 × 10 −12 m )
The dipole moment of a nonpolar polyatomic molecule = 33.3 %
is zero inspite of the fact that the bond moments of the
molecule is not zero. This is due to the fact that the
individual bond moments in the molecule is symmetrically
placed so that their vector additions is zero.
Sometimes, one can write more than one equivalent Lewis
Illustration: structures differing in the distribution if electrons over a
F given skeleton of atoms in a molecule. None of the individual
structures adequately explains the characteristics of the
B O == C == O molecule. However, these can be explained if the actual
F structure of the molecule is considered as the superposition
of individual structures. This phenomenon is known as
F resonance and the individual structures are known as
resonating structures. It is represented by a double-headed
arrow ( ) inserted between the resonating structures.
Illustrations:

Both NH3 and NF3 have pyramdial shapes with lone pair || S | S S
of electrons on nitrogen atom. O O O O written as O O
The resonance hybrid of the two Lewis structures makes
both the S – O bond lengths equal in size.
N N

H H F F

H F
In NH3, orbital dipole acts in the same direction as the sum
of bond vectors of the three N – H bond bonds. In NF3,
orbital dipole acts in the opposite direction to the sum of
bond vectors of the three N – F bond bonds. These facts
make the dipole moment of NH3 (µ = 1.57 D) larger than written as
that of NH3 (µ = 0.24 D).

The percent ionic character of a polar bond A – B is


defined as: The transfer of valence electron(s) from one atom of an
element to the valence shell of the atom of some other
2.5

element leads to the formation of positive and negative Formation of 1 mol of Solid Ionic Compound from
ions, respectively. The electrostatic attraction between the Constituent Elements
positive and negative ions results in the formation of an Taking an example of sodium chloride, we have the
ionic bond between the involved ions. following steps in the formation of solid compound.
+ –
Na + :Cl:
: :

: :
Na + Cl: (i) Na(s) Na(g) H1 = 108 kJ mol–1
2 5 2 6
(3s)1 (3s) (3p) (3s)0 (3s) (3p) 1
(ii) Cl2(g) Cl(g) H2 = 120 kJ mol–1
:F . 2
+ . 2F : 5

+ . Mg .
: :

: :

: :
2+
Mg + : F :
(2s)2 (2p)
5
(3s)
2
(2s) (2p)
2
(3s) (2s) (2p)
0 6 (iii) Na(g) Na+(g) + e– H3 = 496 kJ mol–1
(iv) Cl(g) + e– Cl–(g) H4 = –349 kJ mol–1
Energies Involved in the Formation of One Molecule of Na + (g) + Cl − (g) → Na + Cl − (s) ∆H 5 = −788 kJ mol −1
Sodium Chloride (v)
1 −1
Na(s) + Cl2 (g) → Na + Cl(s) ∆H = −313kJ mol
The formation of Na+Cl–(g) from Na(g) and Cl(g) involves 2
the following steps. Since H is negative, the formation of solid NaCl is
energetically favorable. From the values of H’s listed
(i) Na(g) Na+(g) + e– Ei = 8.24 × 10–19 J above, it is obvious that the step (v) is the most favourable
(ii) Cl(g) + e– Cl–(g) Eea= –5.78 × 10–19 J step since its highly exothermic nature counter acts the
endothermic steps (i) to (iii).
(iii) Na+(g) + Cl–(g) Na+Cl–(g) PE
The enthalpy involved in the reversal of step (v), i.e.,
where PE is the potental energy in the formation of ionic NaCl(s) Na+(g) + Cl–(g) is known as lattice energy of
bond. This is evaluated by the expression the ionic solid.
QQ
PE = 1 2 By definition, lattice energy of an ionic solid is the energy
(4πε0 )r required to completely separate one mole of solid ionic
where Q1 = Q2 = 1.60 × 10–19C and r = rNa+ + rCl– compound into gaseous constituent ions. Larger the value
= 95 pm + 181 pm = 276 pm. Considering Q1 and Q2 as of lattice energy, more stable the ionic compound.
point charges, we have
The lattice energy is determined indirectly through the use
(1.60 × 10 −19 C)(−1.60 × 10 −19 C)
PE = of Born-Haber cycle. The latter involves the steps (i) to (v)
(4)(3.14)(8.854 × 10 −12 C2 N −1m −2 )(276 × 10 −22 ) listed above for the formation of solid ionic compound. In
= –8.34×10–19J this cycle, H is determined experimentally. Subtraction
of H1, H2, H3 and H4 from the value of H gives the
Hence, for the reaction Na(g) + Cl(g) Na+Cl–(g)
value of H5. The lattice energy is negative of the value
we have E = E1 + Eea + PE of H5.
 = (8.24 – 5.78 – 8.34) × 10–19 J
Fajan Rules: An ionic compound has partial covalent
 = – 5.88 × 10–19 J
character and vice versa. The partial covalency in an ionic
The negative value of E indicates that the compound my be explained qualitalively with the help of
formation of an isolated ionic bond Na+Cl–(g) is feasible Fajan rules described in the following.
as the combination is energetically more stable than Na(g)
High charge and small size of the cation: Such an ion
and Cl(g) taken together.
will exert a greater effect in polarizing anions causing
cationic electronic charge to penetrate partially into the
For E in Equation (3) to be negative, we must have anionic electronic cloud resulting into the partial covalent
(i) Low value of Ei. This is shown by electropositive bond character to the ionic bond.
element(s).
The elecronic
(ii) High value of Eea. This is shown by electronegative cloud of such an anion is most easily polarized by the
element(s). cation because the anionic charge cloud is less influenced
Hence by the nuclear charge of the anion.
Electropositive + Electronegative Ionic Electron configuration of the cation: For two cations of
element element Bond the same size and charge, the cations of electronic
(eg, elements of Gp.1) (eg, elements of Gp. 17) configuration (n–1)dxns0 (i.e., transition metal ions) have
2.6

more polarizing power than the cation of electronic A simple theory, known as VSEPR theory, was proposed
configuration (n – 1)s2(n–1)p6 ns0 (i.e., alkali and alkaline by Sidgwick and Powell in 1940 and was developed and
earth metal ions). This is due to less shielding of nucleus by refined by Nyholm and Gillespie in 1957. The guiding
the electronic cloud of transition metal ions as compared rules of this theory are as follows.
to that in the alkali and alkaline metal ions. The number of electrons pairs in the valence shell of
Hg2+ ion has larger polarizing effect than Ca2+ ion. Lithium the central atom of a molecule decides the shape of the
salts have more covalent chracter than the alkali salts. I– molecule. These pairs of electrons occupy the specific
ion is more easily polarized than Cl– ion by Ag+ ion. positions so as to minimize the mutual electronic repulsion.
A multiple bond is treated as if it is a single electron pair.
The repulsive interaction of electron pairs decrease in the
order
Lone pair (lp) - Lone pair (lp) > Lone pair (lp) - Bond pair
(bp) > Bond pair (bp) > Bond pair (bp)
The covalent-bonded molecules have definite shapes The shapes of molecules as predicted by VSEPR theory
which cannot be accounted for by Lewis structures. are shown in Table.
Shapes of some molecules on the basis of VSEPR model

Molecules Number of valence Basic shape

Electrons Electron Bonding Lone pairs


around the pairs pairs
central atom

(i) BeCl2 4 2 2 0 Linear

(ii) BCl3 6 3 3 0 Triangular


planar

CH4 8 4 4 0
(iii) NH3 8 4 3 1 Tetrahedron
H2O 8 4 2 2

PF4 10 5 5 0
10 5 4 1 Trigonal
(iv) SF4 bypramid
CIF3 10 5 3 2

SF4 12 6 6 0
(iv) 12 6 5 1 Octahedron
IF5
2.7

A few examples of molecules containing lone pair s In this


electrons along with their geometry are described in the overlapping, the electronic charge is concentrated between
following the nuclei of bonding atoms.
.. ..
S N p In this
O O H H overlapping, the electronic charge is concentrated above
H the plane of nuclei of the bonding atoms.
Shape : Bent Shape : Trigonal pyrmidal End to end overlap
Bond angle : 109.5° Bond angle : 107°

O
..
..

H H 2pA 2pB

Shape : Bent
Bond angle : 104.5°

sideways
overlap

The quantitative description of chemical bond is provided


by the quantum mechanical theories. Two theories,
namely, valence bond (VB) and molecular orbital (MO)
have been developed. 2pA 2pB
The essential guidelines of VB method are as follows. The electronic configuration of oxygen
(i) A molecule is considered to be a collection of atom is (1s)2 (2s)2 (2px) 2(2py)1 (2pz) 1. There are two 2p
atoms with electrons occupying their respective atomic orbital, each containing one electron. Thus it can
atomic orbitals. form two bonds — and bonds.
z z
(ii) The formation of molecule is analysed in terms
of interactions amongst electrons-electrons,
electrons-nuclei and nuclei-nuclei.
(iii) For a molecule to be stable, the electrostatic y y
attractions must predominate over the electrostatic
repulsion. The difference in these two is released in
the form of heat. Thus, a molecule is energetically Oa Ob
more stable than the individual atoms. -bond
Electron associated with atom HA can go to the atom HB
and vice versa through the overlap region. Also in the -bond
overlap region, there will be some probability of finding
both the electrons and thus according to Pauli’s exclusion
principle, these two electrons must have opposite spins.

The electronic configuration of


nitrogen atom is (1s)2(2s)2(2px)1(2py)1(2pz)1. There are
The intervening electronic charge between the two
three 2p orbitals, each containing one electron. Thus it can
nulcei has an affect of decreasing nuclear repulsion and
form three bonds—one and two -bonds.
maximises electron-nuclei attractions. This lead to the z z
stale H2 molecule. y y

x x

In a molecule, two of overlapping of orbitals having


directional chracteristics may be distinguished.
2.8

-bond orbitals are directed to the corners of trigonal bipyramid


three are in the same plane making an angle 120° with
each other, fourth and fifth are directed perpendicular to
the plane (one above and the other below).
sp3d2 or d3sp3 Hybridisation: The resultant six hybrid
orbitals are directed to the corners of regular octahedron
four are in the same plane making an angle 90° with each
-bond
other, fifth and sixth are directed perpendicular to the
plane (one above and the other below).
sp3d3 or d3sp3 Hybridisation: The resultant seven
orbitals are directed to the corners of a regular pentagonal
According to the valence bond theory, bipyrimide five are in the same plane and sixth and seventh
are directed perpendicular to the plane (one above and the
4Be (1s , 2s ) Should form no chemical bond
2 2

as it does not contain any other below).


unpaired electron.
5 B (1s2, 2s2, 2p1) Should form a single bond as
it contains only one unpaired
electron.
Molecular orbital theory provides the explanation for the
6C (1s , 2s , 2px , 2p y) Should form two bond as it
2 2 1 1
formation of bond in a molecule on the lines very similar
contains two unpaired electron.
to those of atomic orbitals. The essential guidelines of this
Experimentally it is found that Be is divalent, B is trivalent theory are as follows.
and C is tetravalent.
Like atomic orbitals in an atom, there exists molecular
To explain this, the concept of hybridisation is introduced. orbitals in a molecule. The only difference is that an
In this concept we have, atomic orbital is a monocentric (i.e., exists around a single
Two or more atomic orbitals of the same atom mix nucleus) while a molecular orbitals is a polycentric (i.e.,
each other to provide a new set of indentical number of exists around more than one nucleus and thus belongs to
degenerate orbitals. These orbitals, known as hybrid the molecule as a whole).
orbitals, are completely identical in size, shape and Each molecular orbitals in a molecule describes different
orientations. electronic behaviour and has a fixed energy.
sp3 Hybridisation: In sp3 (pronounced as ‘s-p three’)
Electrons in a molecule occupy molecular orbitals in
hybridisation, one s orbital and three p orbitals of the same
accordance with aufbau principle, Pauli’s exclusion
valence shell of an atom combine to give four degenerate
principle and Hund’s rule.
equivalent sp3 hybrid orbitals. These four orbitals are
directed towards the four corners of a regular tetrahedron The square of molecular orbital (which a mathematical
making an angle of 109°28' with respect to each other. expression) evaluated at a point around the nuclei of the
molecule gives the probability of finding electron at that
point.
The shape of a molecular orbital is the region around the
nuclei where there exists 90–95% probability of finding
Three main hybridisation involving d orbitals are as the electron.
follows :
The designation of
sp2d or dsp2 Hybridisation: The resultant four hybrid a molecular orbital starts by starting its or nature
orbital lie in a plane with bond angle of 90° with respect followed by the atomic orbitals into which it separates at
to each other. larger distance. The antibonding orbital is designated by
In dsp2, d orbital belongs to penultimate shell while in placing an asterisk on the symbol or .
sp2d, it belong to the valence shell. The effective combinations of atomic orbitals of atoms A
sp3d or dsp3 Hybridisation: The resultant five hybrid and B are as follows:
2.9

1. Combination Involving 2s(A) and 2s(B) The correlation diagram shown in figure 30 is found to be
Destructive interaction Nodal line
applicable only for O2 and F2 molecules.
+ +
The relative energies of molecular orbitals is
 *2s
+ + 2s < *2s < 2pz < 2px = 2py < *2px = *2py < *2pz

+
For the molecules Li2 to N2, the energies difference in 2s(A)
2s(A) 2s(B) and 2p(A) is not large and thus these two orbitals jointly
Constructive interaction
combine with the orbitals jointly combine with the orbitals
 2s 2s(B) and 2p(B). The resultant correlation is modified and is
2. Combination Involving 2sz(A) and 2pz(B) known in figure. The noteable modification is that
Destructive interaction Nodal plane
+ +
E( 2p) > E( 2p)
+ + s*2pz The relative energies of molecular orbitals is
2pz(A) 2pz(B) 2s < *2s < 2px = 2py < 2pz < *2px = *2py < *2pz
+
Constructive interaction s2pz

3. Combination Involving 2px(A) and 2px(B)


Destructive interaction
+

+ The following guidelines are helpful in describing


+

molecular characteristics.
*2px
1. The valence electrons are distributed among he
+

molecular orbitals in accordance with aufbau principle,


Pauli’s exclusion principle, and Hund’s rule.
+

Aufbau principle: Moleular orbitals are filled in the


2p x(A) 2p x(B)
increasing order of energy.
Constructive interaction 2px Pauli exclusion principle: No more than two
4. Combination Involving 2py(A) and 2py(B) electrons can occupy a single molecular orbital. The
The molecular orbitals formed are similar to 2px and two electrons have opposite spins.
*2p in the direction of y-axis.
x Hund’s rule: The degenerate orbitals are singly
Relative Energies of Molecular Orbitals correlation
occupied by electrons with parallel spins followed by
Diagram displays the correlation diagram expected for the
orbitals of atoms of second period. In this diagram, double occupancy.
E( 2p) < E ( 2p) since the end-to-end overlap 2p orbitals *2p
is expected to be larger than side-ways overlap.
σ*2p

*2p

π*2p 2p 2p
2p

2p π2p 2p

Energy
2p

Energy
σ2p
*2s

σ*2p 2s 2s

2s 2s 2s

Atom A Molecular Orbitals Atom B


σ2s

Atom A Molecular Orbitals Atom B Correlation diagram for Li2 to H2 and F2

Correlation diagram for O2 and F2


2.10

2. The number of bonds in a molecule is defined by a 4. Addition of an electron in the bonding orbital or
physical quantity, known as bond order. It is defined removal of an electron form the antibonding orbital
as one half of the net excess of bonding electrons, i.e., increases bond order and hence increases stability of a
molecule.
Number of (bonding antibonding) electrons 5. Removal of an electron form a bonding orbital or
Bond order=
2 addition of an electron in the antibonding orbital
3. The strength of a bond depends on the bond order of decreases bond order and hence decreases stability of
the molecule. The larger the bond order, the stronger a molecule.
the bond, the larger the dissociation energy of the 6. Paramagnetism in a substance is due to the presence
molecule. of unpaired electrons in the molecules.

Solved Examples

1. An element (X) forms compounds of the formula (c) d – orbital taking part in sp3d2 are d X2– Y2 and
XCl3, X2O5 and Ca3X2 but does not form XCl5. dZ2
Which of the following is the element(X)? (d) d – orbital taking part in sp3d3 are dXY, dZ2
(a) B (b) Al and dX2– Y2
(c) N (d) P Sol.(b) dsp2 (s + pX + pY + dX2–Y2)
Sol.(c) ‘N’ can form NCl3, N2O5 and Ca3N2 but can not sp3d (s + pX + pY) (pZ + dZ2)
form NCl5. Due to absence of d – orbital’s, ‘N’can sp3d2 (s + pX + pY + dX2–Y2) (pZ + dZ2)
not expand its valency to 5.
sp3d3 (s + pX + pY + dXY + dX2–Y2) (pZ + dZ2)
2. Which of the following anions can not be formed
by boron? 5.
Which of the following set is not correct?
(a) N2O, O3, NH4+ all have co – ordinate bonds.
(a) BF63- (b) BH4-
(b) H2O, NO2, ClO2 all are ‘V’ shape molecules.
(c) B(OH)4- (d) BO2-
(c) I3–, ICl2–, NO2+ all are linear molecules.
Sol.(a) Any second period element can form maximum 4 (d) SF4, SiF4, XeF4 all are tetrahedral in shape.
bonds (covalent and co-ordinate). After formation
of 3 covalent bonds, ‘B’ can form only one co- Sol.(d) N∫ N O
ordinate bond because it has only one vacant
orbital. O  H3N H+ (all have co– ordinate bond)

2S 2P O O
B(I excitation) O N CI
3. Which of the following would result in the ('V' shape)
H O O
formation of strongest p- bond if the molecular H O O
axis is x – axis? I3-, ICl2-, NO2+ (all are linear)
(a) 2px + 2px (b) 2py + 2py SF4 (See-saw shape) SiF4 XeF4 (square
(c) 2py + 3dxy (d) 2pz + 4pz (Tetrahedral) planar shape)
Sol.(b) 2px + 2px will form s – bond 6. The incorrect order of bond dissociation energy
2py+2py and 2px+3dxy will form p-bond but 2pp + will be :
2pp bond is stronger than 2pp + 3dp (a) H – H > Cl – Cl > Br – Br
4. Which of the following statement is wrong? (b) Si – Si > P – P > Cl – Cl
(a) d – orbital taking part in dsp2 is dX2– Y2 (c) C – C > N – N > O – O
(b) d – orbital taking part in sp3d is dXY (d) H – Cl > H – Br > H – I
2.11

Sol.(b) Bond length depends on size of atoms order of 9. In which of the following processes, the value of
bond length is:- magnetic moment does not change?
Si – Si < P – P < Cl – Cl (a) N2 N2-2 (a) B2 B2-2
Bond energy µ 1
Bond length (c) O2+ O2- (d) O2- O2-2
Sol.(c) By using molecular orbital theory, we can calculate
Hence, correct order of bond energy is: number of unpaired e-in all these species.
Si – Si > P – P > Cl – Cl
N2 N2-2 B2 B2-2
7.
The incorrect order of bond dissociation energy
(0) (2) (2) (0)
will be :
(a) CO < CO2 < CO32- (C – O bond) O2+ O2- O2- O2-2
(b) CN- < NCN2- < R-CO-NH2 (C – N bond) (1) (1) (1) (0)
(c) ClO- < ClO2- < ClO3- < ClO4- (Cl – O bond)
10. Which of the following statement(s) is not true
(d) SO2 < SO42- < SO32- (S – O bond) for the given species?
Sol.(a) CO CO2 CO32- N2, CO, CN- and NO+
(3) (2) (1.33)
(a) All species have linear shape
CN- NCN2- R-CO-NH2 (b) All species have some dipole moments
(3) (2) (1.5) (c) All species are isoelectronic
ClO- ClO2- ClO3- ClO4- (d) All species have identical bond order and
(1) (1.5) (1.67) (1.75) they are diamagnetic in nature.

SO2 SO42- SO32- Sol.(b) All diatomic species are linear.


(1.5) (1.5) (1.33) Dipole moment of N2 is zero but remaining
In bracket bond order is given. species have some dipole moments.
8.
In which of the following change, adjacent bond All species have 14 e-, bond order is 3 and
angle increases? diamagnetic in nature.
x
-
(a) BeF2 + 2F BeF42-
11. Dipole moment of is 1.5 D. Calculate
(b) SiF4 + 2F- SiF62-
x
-
(c) BF3+ F BF4-
Dipole moment of x
x
(d) NH3+ H+ NH4+
x
- 2-
ol.(d) BeF2+ 2F
S BeF4
sp sp3 (a) 1.5 D (b) 1 D
(180°) (109° 28') (c) 2.35 D (d) 3 D
x
SiF4 + 2F-
SiF62-
sp3 sp3d2 Sol.(c)
(109° 28') (90°) x x
x
BF3 + F-
BF4- Three bond moments are equal in magnitude and
sp2 sp3 they are 120° apart hence, their result is zero. Net
(120°) (109° 28') dipole moment of this molecule equals to the
x
NH3+ H+ NH4+
that of
sp3 sp3
(107°48') (109° 28')
2.12

12. Which of the following pair of compounds are Solid form of PCl5 is PCl4+ PCl6–
polar, planar and sp2 hybridisation? sp3  sp3d2
(a) H2CO3, SO2 (b) HClO2, H2CO3 Solid form of PBr5 is PBr4+ Br–
(c) BFClBr, ClF3 (d) SO3, O3 sp3
Sol.(a) H2CO3 and SO2 Solid form of XeF6 is XeF5+ XeF7–
(Both are polar, planar and have sp2 hybridisation) sp3d2 sp3d4
In HClO2, hybridisation of Cl is sp3 17. In (B4O5(OH)4)2- the number of boron atoms
In ClF3, hybridisation of Cl is sp3d having on octet of electron is :
SO3 is non – polar (a) 0 (b) 1
(c) 2 (d) 4
13. Back bonding in BF3 does not affect:
(a) Planarity, Lewis acidic strength and bond Sol.(c) Structure of (B4O5(OH)4)2- is:-
OH
angle
O B O
(b) Bond length, hybridization and bond strength
(c) Bond angle, planarity and geometry
(d) Lewis acidic strength, bond length and bond
HO B O B OH
order (B-F)
O B O
Sol.(c) Due to back bonding in BF3, it’s bond length ,
bond energy, bond strength and lewis acidic OH
strength changes but there is no change in bond In this structure 2 Boron atoms have octet of
angle, hybridization, geometry and planarity. electrons in their outermost shell.

14. Which of the following molecule has 2pp - 3pp 18. Which of the following statement is not correct?
back bonding? (a) The maximum number of atoms in one plane
in B3N3H6 are 12.
(a) OCl2 (b) BF3
(b) There is no S – S bond in S3O9.
(c) CCl2 (d) CCl3- (c) Maximum number of identical bond angles
Sol.(c) in ClO4- are 4.
Molecule Type of back bonding (d) Number of bridging oxygen in P4O10 are 6.
OCl2 2pp – 3dp H
BF3 2pp - 2pp H B H

CCl3- 2pp – 3dp Sol.(c) N N The complete structure is


:CCl2 2pp – 3pp Planar hence, all 12 atoms
B B. are in same plane.
In :CCl2, vacant 2p-orbital of ‘C’ and paired N H
H
3p-orbital of ‘Cl’ form back bond. H
15. In which of the following molecule 2C – 2e- bond Structure of S3O9 is:-
is absent? o o
(a) BeCl2 in Vapor state (b) Al2Cl6 s

(c) BeH2 in solid state (d) B2H6 o o There is no S – S bond


-
Sol.(c) In BeH2 all bonds are 3C-2e . o s o
s
16. In which of the following d – orbital’s are not o o o
used by central atom in hybridization?
Structure of ClO4- is:-
(a) PF5 (s) (b) PCl5 (s) O–
(c) PBr5 (s) (d) XeF6 (s)
In this tetrahedral structure
Sol.(c) Solid form of PF5 is PF4+ PF6– CI
identical bond angles are 6.
sp3 sp3d2 O O
O
2.13

Structure of P4O10 is:- /Z +/ /Z -/


Uo µ
r
Here, /Z+/ and /Z- / are charge of cation and anion.
In this structure 6, P – O – P r = r+ + r-
  linkage is present.
  r+ = radius of cation
r- = radius of anion
19.
Which of the following compound produces only In these four compounds, MgO has highest lattice
basic products on hydrolysis? energy.
(a) Mg3N2 (b) NCl3 23. The dipole moment of LiH is 1.964 × 10-29 Cm
(c) BBr3 (d) PCl5 and the inter atomic distance between Li and H
o
in this molecule is 1.596 A . What is the percent
Sol.(a) Mg3N2 + 6H2O 3Mg(OH)2 + 2NH3 ionic character in LiH?
(Both are basic) Sol. The dipole moment of 100% ionic molecule
(Li+H)
NCl3 + 3H2O NH3 + 3HOCl
(One is basic and other is acidic) = (1 electronic charge) (inter atomic distance)
= (1.602 x 10-19C) (1.596 x 10-10m)
BBr3 + 3H2O H3BO3 + 3HBr = 2.557 x 10-29 Cm
(Both are acidic)
Fractional ionic = Experimental dipole moment
character Theoretical value of dipole
PCl5 + 4H2O H3PO4 + 5HCl moment
(Both are acidic)
−29
1.94 × 10
20. Which of the following reactions is incorrect? = = 0.768
D 2.557 × 10 −29
(a) Pb(NO3)2 ææ Æ PbO2 + NO2 + O2
D The bond in LiH is 76.8% ionic.
(b) 2 NaNO3 ææ Æ 2NaNO2 + O2
D 24. KF Combines with HF to form KHF2. The
(c) Fe2(SO4)3 ææ Æ Fe2O3 + SO3
D Compound contains the species-
(d) 2NaHCO3 ææ Æ Na2CO3 + H2O + CO2
(a) K+, F- and H+ (b) K+, F- and HF
Sol.(a) The correct reaction is: (c) K+ and (HF2)- (d) (KHF)+ and F-
Pb(NO3)2 æDæ
Æ PbO + NO2 + O2
Sol.(c) KHF2 " K+ + HF2-
21. Which of the following is not correctly matched?
25. Two elements X and Y have following electronic
(a) NH3 > SbH3 > PH3 > AsH3 (Melting point)
configuration:
(b) NH3 > PH3 > AsH3 > SbH3 (Proton affinity)
X : 1s2 2s2 2p6 3s2 3p6 4s2
(c) CH4> SiH4> GeH4 > SnH4 (Thermal stability)
(d) H2O > H2Te > H2Se > H2S (Boiling point) Y : 1s2 2s2 2p6 3s2 3p5
Sol.(a) The correct order of melting point is: The expected compound formed by combination
of X and Y Will be expressed as-
NH3 > SbH3 > AsH3 > PH3
(a) XY2 (b) X5Y2
The order depends on strength of intermolecular
bonding. In solid state, intermolecular bonding in (c) X2Y5 (d) XY5
NH3 is much stronger than in liquid state. Sol.(a) Valency of element X is 2 (2 electrons in the
22. Which of the following substance has the highest outermost shell) while that of element Y is
melting point? 1(1electron in the outermost shell). So the formula
of the compound between X and Y is XY2
(a) NaCl (b) KCl
(c) MgO (d) BaO 2 1
Sol.(c) For ionic compounds, melting point
µ lattice energy (Uo) X Y
2.14

26. Which of the following interaction lies in the DH = DH2 + DH3 + 2DH1

range of 8-42 kJ/mol? Or  = 737.7 + 1451 + 2 (-328)
-
(a) F  …..HF (b) Xe…..H2O Or  = 1532.7 KJ/mol
+ -
(c) Cs …..OH (d) O2N– –OH…H2O
29. Which of the following order is incorrect?
Sol.(d) The interaction present in O2N– –OH…H2O
(a) Ionic character : MCl < MCl2 < MCl3
is intermolecular H-bond. It’s bond energy lies
between 8-42 kJ/mol. (b) Polarisibility : F-< Cl– < Br– < I–
The interaction in F-…..HF is very strong H-bond. (c) Polarising power : Na+< Ca+2 < Mg+2< Al+3
It’s strength almost equals to the covalent bond. (d) Covalent character : LiF < LiCl < LiBr < LiI
+ -
The interaction in Cs …..OH is an ionic bond. Sol.(a) Ionic character µ 1
The interaction in Xe…..H2O is dipole – induced Polarising power of cation
dipole interaction which is weaker than H-bond. As positive oxidation state increases or, size of
27. Which of the following compound has highest cation decreases, polarising power of cation
lattice energy? increases hence, the correct order of ionic
(a) AlF3 (b) Na2S character is: MCl > MCl2 > MCl3

(c) Al2O3 (d) CaF2 30.


Select wrong statement:
(a) A transition metal ion has more polarising
/Z +/ /Z -/ power than S-block ions of comparable size
Sol.(c) Lattice energy (Uo) µ
r and charge.
Here, /Z+/ and /Z-/ are magnitude of charge of (b) Order of solubility in water is AgF > AgCl >
cation and anion respectively.
AgBr > AgI
r ª r+ + r- (c) LiCl is soluble in organic solvents
Al2O3 has highest lattice energy among these (d) The hydration of ions involves absorption of
four because it has highest value of /Z+/ /Z-/ heat.
28. What is the DH of the following reaction? Sol.(d) Out of cations having comparable size and charge
the one having
Mg (g) + 2F (g) Mg2+ (g) + 2F- (g)
Noble gas configuration has less polarizing
If DegH of F = -328 kJ/mol power.
First ionisation energy of Mg = 737.7 kJ/mol LiCl is a covalent compound hence, it is soluble
Second Ionisation energy of Mg = 1451 kJ/mol in organic solvents.
(a) 1532.7 kJ/mol (b) 1860.7kJ/mol Order of solubility of heavy metal halides depends
order on hydration energy.
(c) 2516.7kJ/mol (d) 1451kJ/mol
(AgF > AgCl > AgBr > AgI)
Sol.(a) Given that,
F (g) + e- F- (g); DH1 = -328 KJ/mol The hydration of ions involves evolution of heat.
Mg (g) Mg+ (g) + e-; DH2 = 737.7 KJ/mol
M+ (g) + H2O M+ (aq); DH = -Ve
Mg+(g) Mg+2 (g) + e-; DH3 = 1451 KJ/mol
Mg (g) + 2F(g) Mg+2 (g) + 2F- (g); DH
2.15

Exercise
Covalent and ionic
8. The fluorine molecules is formed by:
1. The phosphate of a metal has the formula M3PO4. p-p orbital’s (sideways overlap)
The formula of its chloride would be: p-p orbital’s (end-to-end overlap)
MCl (b) MCl2 sp-sp orbital’s
(c) MCl3
(d) M2Cl3 s-s orbital’s
2. Solid NaCl is a bad conductor of electricity 9. Among given species identify the isostructural
because: pairs:
In solid NaCl there are no ions [NF3 and BF3]
Solid NaCl is covalent [BF4-and NH4+]
In solid NaCl there is no mobility of ions [BCl3 and BrCl3]
In solid NaCl there are no electrons [NH3 and NO3-]
3. Knowing that Na+ > Mg2+ and S2- > Cl- (Order 10. The molecule exhibiting maximum number of
of size), predict which compound will be more non-bonding electron pairs (lp) around the central
covalent? atom is:
MgS (b) Na2S XeOF4 XeO2F2
(c) MgCl2 (d) NaCl XeF3- XeO3
4. Which of the following compound possesses the 11. The shapes of XeF4, XeF-5 and SnCl2 are:
largest lattice energy? Octahedral, trigonalbipyramidal and bent
LiF (b) NaCl Square pyramidal, pentagonal planar and
(c) KH (d) CsI linear
Square planar, pentagonal planar and angular
5. Select the incorrect statement:
See-saw, T-shaped and linear
Lithium is least reactive but the strongest
reducing agent among all the metals. 12. Which is not correctly matched?
Lithium hydrogen carbonate is not obtained XeO3 – Trigonalbipyramidal
in the solid form while all other alkali metals ClF3 – bent T-shape
forms solid hydrogen carbonates. XeOF4 – Square pyramidal
Lithium nitrate when heated gives lithium XeF2 – Linear shape
oxide whereas other alkali metal nitrates
13. The geometry of ammonia molecule can be best
decompose to give the corresponding nitrite.
described as:
Solubility of alkali metal hydroxides decreases
Nitrogen at one vertex of a regular tetrahedron,
down the group. It is due to decrement in
the other three vertices being occupied by
hydration energy from Li+ to Cs+.
three hydrogens
6. Which pair of element can form multiple bond
Nitrogen at the centre of the tetrahedron,
with itself and oxygen?
three of the vertices being occupied by three
F, N N, Cl hydrogens
N, P N, C Nitrogen at the centre of an equilateral
7. Bonds present in N2O5 (nitrogen pentaoxide) are: triangle, three corners being occupied by
Only ionic three hydrogens
Only covalent Nitrogen at the junction of a T, three open
ends being occupied by three hydrogens
Covalent and co-ordinate
2.16

14. Which one of the following compounds has the 22. How many resonance forms can be written for the
smallest bond angle? nitrate ion, (NO3-)?
OH2 SH2 1 (b) 2
NH3 SO2 (c) 3 (d) 4

15. The bond angles of NH3, NH4+ and NH2- are in 23. When the substances Si, KCl, CH3OH and C2H6
the order: are arranged in order of increasing melting point,
what is the correct order?
NH2- > NH3 > NH4+
Si, KCl, CH3OH, C2H6
NH4+ > NH3 > NH2-
CH3OH, C2H6, Si, KCl
NH3 > NH2- > NH4+
KCl, Si, C2H6, CH3OH
NH3 > NH4+ > NH2-
C2H6, CH3OH, KCl, Si
16. Which of the following molecules will have polar
bonds but zero dipole moment? 24. The H-O-H bond angles in H3O+ are approximately
107°. The orbitals used by oxygen in these bonds
O2 CHCl3 are best described as:
CF4 None of these p-orbitals
17. Which has maximum dipole moment? sp-hybrid orbitals
CI
sp2-hybrid orbitals
CI
sp3-hybrid orbitals
CI CI CI
(a) (b) 25. Which of the following fact is directly explained
by the statement oxygen is a smaller atom than
sulphur?
CI H2O boils at a much higher temperature than
CI H2S
CI CI
H2O undergoes intermolecular hydrogen
bonding
(c) (d)
H2O is liquid and H2S is gas at room
temperature
CI CI S-H bond longer than O-H bond
18. The hybridisation of central iodine atom in IF5, 26. Which of the following statement is correct about
I3- and I3+ are respectively: I3+and I3- molecular ions?
sp3d2, sp3d, sp3 Number of lone pairs at central atoms are
sp3d, sp3d, sp3 same in both molecular ions
sp3d2, sp3d2, sp3 Hybridisation of central atoms in both ions
are same
sp3d, sp3d2, sp3
Both are polar species
19. Which of the following species used axial set of
Both are planar species
d-orbital’s in hybridisation of central atom?
PBr4+ (b) PCl4- 27. Iodine molecule are held in the solid lattice
by……………………..
(c) ICl4- (d) None of these
London forces
20. Low melting point is expected for a solid:
Dipole-dipole interactions
Ionic solid Metallic solid
Covalent bonds
Molecular solid Covalent solid
Both are planar species
21. Which molecule does not exist?
28. Species having maximum Cl-O bond order is:
OF2 (b) OF4
(a) ClO3- (b) ClO3
(c) SF2 (d) SF4
(c) ClO2 (d) ClO2-
2.17

29. Which of the following species contains minimum 37. The bonds present in KHF2 are:
number of atoms in XY Plane ? Only ionic
(a) XeF5- (b) SF6 (b) Covalent and coordinate
(c) IF7 (d) All (c) Only covalent
30. In which of the following molecular shape dZ2– (d) Covalent, ionic and H-bond
orbital must not be involved in bonding? 38. The correct order of dipole moment is:
(a) Pentagonal Planar CH4 < NF3 < NH3 < H2O
(b) Trigonal Planar (b) NF3 < CH4 < NH3 < H2O
(c) Linear (c) NH3 < NF3 < CH4 < H2O
(d) Square Planar (d) H2O < NH3 < NF3 < CH4
31. The correct statement regarding SO2 molecule is: 39. Among the following which is polar?
Two pp - dp bonds (a) CO2 (b) SO2
Molecule has 2 lone pair, 2 s bonds and 2p
(c) BeCl2 (d) CI CI
bonds
Two pp - pp bonds 40. In the reaction, 2PCl5 PCl4++ PCl6-, the
One pp - pp and one pp -dp bonds change in hybridisation is from:
32. The molecule MLX is Planar with 7 pairs of (a) sp3d to sp3 and sp3d2
electrons around M in the valence shell. The (b) sp3d to sp2 and sp3
value of X is: (c) sp3d to sp3d2 and sp3d2
6 (b) 5 (d) sp3d2 to sp3 and sp3d
(c) 4 (d) 3 41. Out of the compounds shown below, the vapour
33. In which of the following pairs, both the species pressure of II at a particular temperature is
have the same hybridisation? expected to be
CHO
(I) SF4, XeF4 (II) I3-, XeF2
(III) ClO3+, PO4-3 (IV) ClO3-, PO43–
p-hydroxybenzaldehyde &
(a) I, II (b) II, III
(c) II, IV (d) I, II, III
34. Which of the following possess two lone pair of OH
electrons on the central atom and square planar CHO
OH
shape:
o-hydroxybenzaldehyde &
(I) SF4 (II) XeO4
(III) XeF4
(IV) ICl4-
(a) Higher than that of I
(a) I , III (b) II , IV
(b) Lower than that or I
(c) III , IV (d) All (c) Same as that of I
35. Which of the following does not follow the octet (d) Can be higher or lower depending upon the
rule? size of vessel
NaCl (b) CaCl2 42. Which of the following have been arranged in
(c) ClF3 (d) CCl4 increasing bond order as well as bond dissociation
energy?
36. Which of the following does not have coordinate
bonds? (a) O2-2 < O2- < O2+< O2
(b) O2-2 < O2- < O2 < O2+
HNO2 (b) O3
(c) O2 < O2+ < O22- < O2-
(c) NaBF4 (d) NH4Cl
(d) O2+ > O22- > O2- < O2
2.18

43. The statement true for azide ion (N3-) is: 7. Give the correct order of initials T or F for
(a) It has a non–linear structure following statements. Use T if statement is true
(b) It is called pseudo halogens and F if it is false:
(c) The oxidation state of N in this anion is- 1 (I) The order of repulsion between different pair
(d) It is isoelectronic with NO2 of electrons is:
44. The pair of strongest hydrogen bond is: lp – lp > lp– bp > bp– bp

(a) SiH4 and SiCl4 (II) In general, as the number of lone pair of
electrons on central atom increases, value
(b) CH3COOH and CH3OCH3
of bond angle from normal bond angle also
(c) CH3COOH and CH3COCH3 increases
(d) H2O and H2O2 (III) The number of lone pair on O in H2O is 2
45. The strongest force among the following is: while on N in NH3 is 1
(a) London force (IV) The structures of xenon fluorides and xenon
(b) Ion-dipole interaction oxofluorides could not be explained on the
(c) Dipole-induced dipole interaction basis of VSEPR theory
(d) Dipole-dipole interaction (a) TTTF (b) TFTF
(c) TFTT (d) TFFF
8. Among the following species, the least angle
around the central atom is in:
1. In which of the following species maximum
atoms can lie in same plane? O3 I3-
XeF2O2 PCl5 NO2- PH3
AsH+4 XeF4 9. BF3 and NF3 both are covalent compounds but
NF3 is polar whereas BF3 is non–polar. This is
2. The correct order of Cl - O bond order is:
because:
(a) ClO3- < ClO4- < ClO2- < ClO-
Nitrogen atom is smaller than boron atom
(b) ClO- < ClO4-< ClO3- < ClO2-
N – F bond is more polar than B – F bond
(c) ClO- < ClO2-< ClO3- < ClO4-
NF3 is pyramidal whereas BF3 is planar
(d) ClO4- < ClO3-< ClO2- < ClO- triangular
3. Resonance structures can be written for: BF3 is electron deficient whereas NF3 is not
O3 NH3 10. Which statement is incorrect?
CH4 H2O MP of H2O, NH3, HF are maximum in
4. The number of sp2 – s sigma bonds in benzene their respective group due to intermolecular
are: H-bonding
3 6 BP of CH4 out of CH4, SiH4, GeH4, and SnH4
12 None of these is least due to weak intermolecular force of
attraction
5. Which is the following pair of species have
identical shapes? Acetic acid forms dimer by H-bonding
NH3 has lower BP than SbH3
(a) NO2+ and NO2- (b) PCl5 and BrF5
11. The molecular size of ICl and Br2 is approximately
(c) XeF4 and ICI4- (d) TeCl4 and XeO4
same, but BP of ICl is about 40°C higher than of
6. The hybridisation of the central atom will change Br2. It is because:
when: ICl bond is stronger than Br-Br bond
(a) NH3 combines with H+ IE of iodine < IE of bromine
(b) H3BO3 combines with OH- ICl is polar while Br2 is nonpolar
(c) NH3 forms NH-2 I has larger size than Br
(d) H2O combines with H+
2.19

12. Number of S-S bond is H2SnO6: 19. Which of the following is the correct order for
(a) n (b) (n-1) increasing bond angle?
(a) NH3 < PH3 < AsH3 < SbH3
(c) (n-2) (d) (n+1)
(b) H2O < OF2 < Cl2O
13. The nodal plane in the p -bond of ethane is located
(c) H3Te+ < H3Se+ < H3S+ < H3O+
in:
(d) BF3 < BCl3 < BBr3 < BI3
(a) The molecular plane
(b) A plane parallel to the molecular plane 20. SbF5 reacts with XeF4 and XeF6 to form ionic
compound [XeF3]+[SbF6]- and [XeF5]+ [SbF6]-
(c) A plane perpendicular to the molecular plane
then molecular shape of [XeF3]+ ion and [XeF5]+
which bisects the carbon-carbon s bond at
ion respectively:
right angle
Square pyramidal, T– shaped
(d) A plane perpendicular to the molecular plane
which contain the carbon-carbon bond Bent – T – shape, square pyramidal
See – Saw, square pyramidal
14. Which set of compound in the following pair of
ionic compound has the higher lattice energy? square pyramidal, See–Saw
(i) KCl or MgO 21. Melting point of calcium halides decreases in the
(ii) LiF or LiBr order:
(iii) Mg3N2 or NaCl CaF2 > CaCl2 > CaBr2 > CaI2
CaI2 > CaBr2 > CaCl2 > CaF2
KCl, LiBr, Mg3N2 CaBr2 > CaI2 > CaF2 > CaCl2
MgO, LiBr, Mg3N2 CaCl2 > CaBr2 > CaI2 > CaF2
MgO, LiF, NaCl 22. Which of the following order is incorrect?
MgO, LiF, Mg3N2 NaCl> KCl > RbCl > LiCl (Order of melting
15. The bond having the highest bond energy is: point)
C = C (b) C = S LiCl > CsCl > RbCl > NaCl > KCl (Order of
solubility in water)
(c) C = O (d) P = N
LiCl > NaCl > KCl > RbCl > CsCl (Order of
16. Which of the following overlaps is incorrect lattice energy)
(assuming Z-axis is internuclear axis)? NaF > MgF2 > AlF3 (Order of melting point)
(I) 2pY + 2pY "p- Bond formation 23. Which of the following process is correct?
(II) 2pX + 2pX "s- Bond formation D 1
(a) Ca(NO3) ææÆ CaO + 2NO2 - + O2 -
(III) 3dXY + 3dXY"p- Bond formation 2
 .D
(IV) 2s + 2pY"p - Bond formation
1
(V) 3dXY + 3dXY"d- Bond formation Ca + O2 -
2
(VI) 2pZ + 2pZ"s- Bond formation
I, II, III (b) III, VI (b) 2FeSO4 æDæ
Æ Fe2O3 + SO2 - + SO3 -
(c) II, V (d) II, III, IV (c) Ca(HCO3)2 æDæ
Æ CaCO3 + CO2 - + H2O
17. In which of the following combination-
(d) Both (b) and (c)
hybridisation of central atom does not change?
H2O + CO2 24. Choose the correct code of characteristics for the
given order of hybrid orbitals of same atom, sp <
H3BO3 + OH-
sp2 < sp3
BF3 + NH3
(I) Electronegativity
None of these
(II) Bond angle between same hybrid orbitals
18. Which species has the same shape as NH3?
(III) Size
(a) SO32- (b) CO32-
(c) NO3- (d) SO3 (IV) Energy level
2.20

(a) (II), (III) and (IV) (c) (s2py or s2px) or (p2py* or p2px*)
(b) (III), (IV) (d) (p2py* or p2px*) or (p2py or p2px)
(c) (II) and (IV) 32. The decreasing (O - O) bond length order in the
(d) (I), (II), (III) and (IV) following point?
25. Select pair of compound in which both have (a) O2 > H2O2 > O3 (b) H2O2 >O3 > O2
different hybridisation but similar shape: (c) O3 > H2O2 >O2 (d) O3 > O2 > H2O2
BF3, BrF3 (b) ICl2–, BeCl2 33. Which of the following substance has the highest
(c) BCl3, PCl3
(d) PCl3, NCl3 melting point?
26. Assuming the bond direction to the z-axis, which (a) BaO (b) MgO
of the overlapping of atomic orbitals of two atom (c) KCl (d) NaCl
(A) and (B) will result in bonding?
34. Which of the following statement is correct?
s-orbital of A and px–orbital of B
(a) FeCl2 is more covalent than FeCl3
s-orbital of A and pz–orbital of B
(b) FeCl3 is more covalent than FeCl2
py-orbital of A and pz–orbital of B
(c) Both FeCl2 and FeCl3 are equally covalent
s-orbital of both (A) and (B)
(d) FeCl2 and FeCl3 do not have any covalent
(a) I, and IV (b) I and II character
(c) III and IV (d) II and IV 35. Which of the following compound does not have
27. The number and type of bonds between two hydrogen bonding?
C-atoms in SrC2 are: (a) K2HPO4
1s, 1p (b) 1s, 2p (b) K2HPO3
(c) 1s, 1.5p (d) 1s (c) NaHCO3(s)
  O
28. Which of the following is the decreasing order of  

their dipole moments? (d) CH3 - CH - C - CH2 - CH3
|
(a) NH3 > BF3 > NF3   O-H
(b) NH3 > NF3 > BF3
36. Which of the hydrogen halide forms salts like
(c) BF3 > NH3 > NF3 KHX2 (where X is a halogen atom)?
(d) NH3 > BF3 > NF3
(a) HF (b) HCl
29. The maximum number of 90° angles between bp-
(c) HI (d) HBr
bp of electrons is observed in:
(a) sp3d hybridisation 37. Which of the following is an incorrect statement
for ice:
(b) dsp3 hybridisation
(a) Less density than liquid water
(c) dsp2 hybridisation
(b) Each O atom is surrounded by four
(d) sp3d2 hybridisation
equidistance H-atoms
30. The bond angles of SO3, SO2 and SO32- are in the (c) Distance between two adjacent O-atoms is
order: equal
(a) SO3 > SO2 > SO32- (d) Open cage-like crystal structure.
(b) SO32- > SO3 > SO2
38. The intermolecular interaction that is dependent
(c) SO2 > SO3 > SO32-

on the inverse cube of distance between molecules
(d) SO3 > SO32- > SO2 is:
31. In forming (i) N2 " N2+ and (ii) O2 " O2+; the (a) Ion-ion interaction
electrons respectively are removed from: (b) Ion-dipole interaction
(a) (p2py* or p2px*) or (p2py* or p2px*) (c) London force
(b) (p2py or p2px) or (p2py or p2px) (d) Hydrogen bond
2.21

39. Which of these is not true for metallic bond ? 45. Which of the following has highest melting point?
(a) Metallic bond is non-directional in nature SF6 (b) NaCl
(b) Metallic bonds are weaker than covalent (c) SiC (d) Xe
bond.
(c) Energy required to vapourise a mole of
Cu metal is high to the energy required to
vapourise a mole of a covalent substance like ONE OR MORE THAN ONE CORRECT
diamond.
1. Which species have same bond order?
(d) The valency electrons in a metallic bond are
mobile. CO3-2 (b) NO3-
40. Which substance has the strongest London (c) NO2- (d) NO
dispersion forces? 2. In Which of the following molecule bonding is
SiH4 (b) CH4 taking place in excited state?
(c) SnH4 (d) GeH4 CH4 (b) BF3
(c) ICl3 (d) PCl3
41. The type of molecular forces of attraction present
in the following compound is: 3. Which is/are in linear shape?
OH NO2+ (b) XeF2
NO2 (c) I3 -
(d) I3+
4. The species which are paramagnetic is/are:
NO (b) NO2
HO
(c) ClO2 (d) N2O4
(a) Intermolecular H-bonding
5. Which of the following statements are correct?
(b) Intramolecular H-bonding
O–hydroxybenzaldehyde is a liquid at
(c) van der Waals’ force room temperature due to intramolecular H–
(d) All of these bonding.
42. Which among the following attraction is Order of boiling point is H2O > H2Te > H2Se
strongest? > H2S
(a) HF ……… H2O Order of boiling point is HF > HI > HBr >
(b) Na+ ……… HCl HCl
(c) H2O ……… Cl2 Order of boiling point is SbH3 > NH3 > AsH3
(d) Cl – Cl …… Cl – Cl > PH3
43. In which of the following species back- p bonding 6. The molecule that is/are having N – N bond:
exists? N2O (b) N2O3
(a) NF3 (b) NH3 (c) N2O5 (d) N2O4
(c) BF3 (d) BF-4 7. There is change in the type of hybridisation when:
44. Which statement about hybridisation is correct? NH3 combines with H+
It involves the mixing of atomic orbitals of AlH3 combines with H-
the atom at the time of their participation in NH3 forms NH2-
bonding. SiF4 forms SiF62-
(b) sp3d3 hybridisation orbital point out towards Select correct statement(s) regarding s and
8.
the corners of regular hexagon. p-bonds:
(c) Hybrid orbitals form weaker bonds than pure s-bond lies on the line joining the nuclei of
atomic orbitals. bonded atoms
(d) For hybridisationto occur, the atom must p-electron cloud lies on either side to the line
have vacant orbitals in the valence shell. joining the nuclei of bonded atoms
2.22

(2pp-3dp) pi-bond is stronger than (2pp-3pp) 1


pi-bond. (b) Bond order: Bond order = (Nb - Na)
2
s-bond has primary effect to decide direction A positive bond order means a stable
of covalent bond, while p-bond has no molecule while a negative or zero bond order
primary effect in direction of bond means an unstable molecule.
(c) Nature of the bond: Bond order 1, 2, or 3
9. In which of the following there is intermolecular
corresponds to single, double or triple bonds
hydrogen bonding?
respectively.
Water Ethanol (d) Bond length: Bond length decreases as bond
Acetic acid H-F order increases.
(e) Magnetic nature: Molecular orbital’s
10. Which of the following statements are correct
in a molecule are double occupied, the
about sulphur hexafluoride?
substance is diamagnetic and if one or more
All S-F bonds are equivalent molecular orbital’s are singly occupied, it is
SF6 is a planar molecule paramagnetic.
Oxidation number of sulphur is the same as 14. Select correct statement(s):
number of electrons of sulphur involved in (a) Among O2+, O2 and O2- the stability decreases
bonding as O2+ > O2 > O2-
Sulphur has acquired the electronic structure (b) He2 molecule does not exist as the effect of
of the gas argon bonding and anti-bonding molecular orbital’s
11. Correct order of decreasing boiling points is: cancel each other
(a) HF > HI > HBr > HCl (c) C2, O22- and Li2 are diamagnetic
(b) H2O > H2Te > H2Se > H2S (d) In F2 molecule, the energy of s 2pz is more
(c) Br2 > Cl2 > F2 than p2px and p2py
(d) CH4 > GeH4 > SiH4 15. N2 has greater bond dissociation energy than N2+,
where as O2 has a lower bond dissociation energy
12. Which of the following species does/do not exist?
than O2+ because :
OF4 NH2- Bond order is reduced when O2 is ionised
NCl5 ICl32- to O2+ and bond order increased when N2 is
ionised to N2+
13. Ionic compounds in general do not possess:
Bond order is increased when O2 is ionised to
high melting points and non-directional
O2+ and bond order is decreased when N2 is
bonds
ionised to N2+
high melting points and low-boiling points Bond order is deceased when O2 is ionised to
directional bond and low-boiling points O2+ and bond order is decreased when N2 is
high solubilities in polar and non-polar ionised to N2+
solvents None of these.
PASSAGE BASED QUESTIONS Passage # 2 (Q. 16 and 17)
Passage # 1 (Q. 14 and 15 ) Molecular geometry is the general shape
The distribution of electrons among various of a molecule as determined by the relative
molecular orbital’s is called the electronic positions of the atomic nuclei. VSEPR model
configuration of the molecule which provides predicts the shape of the molecules and ions
us the following very important information’s in which valence shell electron pairs are
about the molecule. arranged about the atom as far away from one
another as possible. Thus minimising electron
(a) Stability of molecule: The molecule is
pair repulsion gives information about the
stable if number of bonding molecular orbital
geometry of a molecule. Information about
electrons (Nb) is greater than the number of
the geometry of a molecule can sometimes
anti bonding molecular orbital electrons (Na)
be obtained from an experimental quantity
and vice – versa.
called dipole moment.
2.23

16. The dipole moment of a tri-atomic molecule AX2 22. Find the number of species having fractional
was found to be equal to the bond moment of bond order?
A – X bond. N2+, N2-, O2, O2+, F2, B2, C2+, CN-, NO+
Which of the following information regarding 23. P4O10 has two different types of P – O bonds.
geometry of the molecule can be drawn from the Find the number of P – O bonds with shorter bond
above observation? length.
(a) Molecule is linear
24. In a P4O6 molecule, the total number of P-O-P
(b) Molecule is V shaped with –X – A – X = 90° bonds is :
(c) Molecule is V shaped with –X–A – X = 120° 25. Find the total number of polar molecules.
(d) Molecule geometry cannot be predicted with
PF3Cl2, SF4, PCl5, PCl3F2, SF6, XeF2, NO2+,
the given information.
BF2Cl, BF3
17. Which of the following inter – halogen compounds
26. How many of the following have hydrogen
is non–polar in nature?
bonding?
(a) ClF3 (b) BrF5
NH3, CH4, H2O, HI, HF, HCOOH, B(OH)3,
(c) IF7 (d) BrCl CH3COOH, HCO2– ion
Passage # 3 (Q. 18 and 19) 27. In which of the following all bond length are not
When hybridisation involving d-orbitals are equal?
considered then all five d-orbitals are not PCl5, SF4, ClF3, XeF2, [SF5]+, [ClF4]+, [XeF3]+,
degenerate, rather dx2–y2, dz2 and dxy, dyz, dzx O3, P4 (white)
form two different sets of orbitals and orbitals of
COLUMN MATCHING TYPE QUESTIONS
appropriate set is involved in the hybridisation.
28. Match the following :
18. In sp3d2 hybridisation, which sets of d-orbitals is
involved? Column I Column II
(a) dx2–y2, dZ2 (b) dZ2, dxy (A) BF3 (a) sp3 hybridisation
(B) (SiH3) 3N (b) pp – pp back bond
(c) dxy, dyz (d) dx2 – y2, dxy
(C) B2H6 (c) pp – dp back bond
19. Molecule having trigonalbipyramidal geometry
(D) SiO2 (d) 3c – 2e bond
and sp3d hybridisation, d-orbitals involved is:
29. Match the following:
(a) dxy (b) dyz
Column I Column II
(c) dx2– y2
(d) dz2 -
(A) O2 and NO (a) Same magnetic property and
INTEGER TYPE QUESTIONS bond order as that in N2+
20. Consider y-axis as inter-nuclear axis, how many (B) O2+ and NO (b) Same bond order but not same
of following will lead to p bond formation? magnetic property as that in O2
(a) py –py (b) px – px (C) CO and CN- (c) Same magnetic property and
(c) pz – pz (d) dxy – dxy bond order as that N22-
(e) dyz – dyz (f) px – dxy (D) C2 and CN+ (d) Same magnetic property and
(g) dxy – pz (h) dxz – dxz bond order as that in NO+

21. How many of the following have pp - dp bonding? 30. Match the following:
-2
(a) CO3 (b) NO3- Column – I Column - II
(c) SO3-2 (d) SO4-2 (A) H3P3O9 (a) S-O-S bond is present
(e) PO4-3 (f) NO2- (B) H2S2O7 (b) Dibasic acid
(g) S2O8-2 (h) R3PO (C) H2S4O6 (c) P-O-P bond is present
(i) S2O3-2 (j) ClO4- (D) H4P2O5 (d) Central atom (S or P) in
(k) ClO3- (l) SO2 maximum oxidation state
2.24

A plane perpendicular to the molecular plane


which contains, the carbon-carbon bond.
[IIT-2002]
7. Which of the following molecular species has
1. Amongst H2O, H2S, H2Se and H2Te, the one with unpaired electrons?
the highest boiling point is: N2 (b) F2
(a) H2O because of hydrogen bonding
(c) O2- (d) O22-
H2Te because of higher molecular weight
[IIT-2002]
H2S because of hydrogen bonding
H2Se because of lower molecular weight 8. Which of the following are isoelectronic and
[IIT-2000] isostructural ? NO3-, CO32-, ClO3-, SO3
2. The hybridisation of atomic orbitals of nitrogen NO3-, CO32- (b) SO3, NO3-
in NO2+, NO3- and NH4+ are: (c) ClO3-, CO32- (d) CO32-, SO3
sp2, sp3, and sp2 respectively [IIT-2002]
sp, sp2, and sp3 respectively
9. According to molecular orbital theory which
sp2, sp, and sp3 respectively of the following statement about the magnetic
sp2, sp3, and sp respectively character and bond order is correct regarding
[IIT-2000] O2+?
3. The correct order of hybridisation of the central Paramagnetic and bond order < O2
atom in the following species NH3, [PtCl4]2-, PCl5 Paramagnetic and bond order > O2
and BCl3 is: Diamagnetic and bond order < O2
dsp2, sp3d, sp2 and sp3 Diamagnetic and bond order > O2
3
(b) sp , dsp2, sp3d and sp2 [IIT-2003]
(c) dsp2, sp2, sp3, sp3d
10. Which species has the maximum number of lone
(d) dsp2, sp3, sp2, sp3d pair of electrons on the central atom?
[IIT-2001]
(a) ClO3- (b) XeF4
-
4. The common features among the species CN , CO
(c) SF4 (d) I3-
and NO+ are:
Bond order three and isoelectronic [IIT-2005]
Bond other three and weak field ligands 11. The percentage of p-character in the orbitals
Bond order two and p - acceptors forming p-p bonds in P4 is:
Isoelectronic and weak field ligands 25 (b) 33
[IIT-2001]
(c) 50 (d) 75
5. Specify hybridisation of N and B atoms in a 1:1
complex of BF3 and NH3: [IIT-2007]
N : tetrahedral, sp3 ; B : tetrahedral, sp3 12.
Among the following, the paramagnetic
N : Pyramidal, sp3 ; B : Pyramidal, sp3 compound is:
N : Pyramidal, sp3 ; B : Planar, sp3 Na2O2 (b) O3
N : Pyramidal, sp3 ; B : tetrahedral, sp3 (c) N2O
(d) KO2
[IIT-2002]
[IIT-2007]
6. The nodal plane in the p bond of ethene is located
13. The species having bond order different from that
in:
in CO is:
The molecular plane.
A plane parallel to the molecular plane. NO- (b) NO+
A plane perpendicular to the molecular plane (c) CN- (d) N2
which bisects, the carbon-carbon s bond at
[IIT-2007]
right angle.
2.25

14. The structure of XeO3 is: 18. The nitrogen oxide(s) that contain(s) N-N bond(s)
Linear (b) Planar is(are):
N2O (b) N2O3
(c) Pyramidal (d) T-shaped
(c) N2O4 (d) N2O5
[IIT-2007] [IIT-2009]
15. Statement-1: p-hdroxybenzoic acid has a lower 19. The bond energy (in kcal mol-1) of a C-C single
boiling point then o-hdroxybenzoic acid. bond is approximately:
Statement-2: o-hdroxybenzoic acid has 1 (b) 10
intramolecular hydrogen bonding. (c) 100
(d) 1000
Statement-1 is True, Statement-2 is True, [IIT-2010]
Statement-2 is a correct explanation for 20. Assuming that hund’s rule is violated, the bond
Statement-1. order and magnetic nature of the diatomic
Statement-1 is True, Statement-2 is True, molecule B2 is:
Statement-2 is not a correct explanation for 1 and diamagnetic
Statement-1. 0 and diamagnetic
Statement-1 is True, Statement-2 is False. 1 and paramagnetic
Statement-1 is False, Statement-2 is True. 0 and paramagnetic
[IIT-2010]
[IIT-2007]
21. The species having pyramidal shape is:
16. Statement-1: In water, orthoboric acid behaves as
SO3 (b) BrF3
a weak monobasic acid.
(c) SiO32- (d) OSF2
Statement-2: In water, orthoboric acid acts as a
[IIT-2010]
proton donor.
Statement-1 is True, Statement-2 is True, 22. In allene (C3H4), the type(s) of hybridisation of
Statement-2 is a correct explanation for the carbon atoms is (are):
Statement-1. sp and sp3 (b) sp and sp2

Statement-1 is True, Statement-2 is True, (c) Only sp2 (d) sp2 and sp3
Statement-2 is Not a correct explanation for [IIT-2012]
Statement-1. 23. The shape of XeO2F2 molecule is:
Statement-1 is True, Statement-2 is False. Trigonalbipyramidal
Statement-1 is False, Statement-2 is True. (b) Square planar
[IIT-2007] (c) Tetrahedral
(d) See-saw
17. Statement-1: Pb4+ compounds are stronger [IIT-2012]
oxidizing agents than Sn4+ compounds.
24. Hybridisation of the underlined atom changes
Statement-2 : The higher oxidation states for the when,
group 14 elements are more stable for the heavier
AlH3 changes to AlH4-
members of the group due to inert pair effect.
H2O changes to H3O+
Statement-1 is True, Statement-2 is True,
NH3 changes to NH4+
Statement-2 is a correct explanation for
statement-1. All of these
[AlEEE-2002]
Statement-1 is True, Statement-2 is True,
Statement-2 is Not a correct explanation for 25. The maximum number of 90° angles between
statement-1. bond pair- bond pair of electrons is observed in,
dsp3 hybridisation
Statement-1 is True, Statement-2 is False.
(b) sp3d hybridisation
Statement-1 is False, Statement-2 is True. (c) dsp2 hybridisation
[IIT-2008] (d) sp3d2 hybridisation [AlEEE-2003]
2.26

26. Based on lattice energy and other considerations, 32. The decreasing values of bond angles from
which one of the following alkali metal chlorides NH3(106°) to SbH3 (101°) down group-15 of the
is expected to have the highest melting point? periodic table is due to:
RbCl (b) KCl Increasing bp-bp repulsion
(c) NaCl (d) LiCl Increasing p-orbital character in sp3
[AlEEE-2004] Decreasing lp-bp repulsion
27. The number and types of bonds between two Decreasing electronegativity
carbon atoms in calcium carbide are: [AlEEE-2006]
One sigma, One pi 33. In which the following ionisation processes, the
(b) One sigma, Two pi bond order has increased and magnetic behaviour
(c) Two sigma, One pi has changed?
(d) Two sigma, Two pi
C2 C2+ (b) NO NO+
[AlEEE-2005]

28. In silicon dioxide: (c) O2 O2+ (d) N2 N2+
Each silicon atom is surrounded by four [AlEEE-2007]
oxygen atoms and each oxygen atom is 34. The charge/size of a cation determines its
bonded to two silicon atoms. polarising power. Which one of the following
Each silicon atom is surrounded by two sequences represents the increasing order
oxygen atoms and each oxygen atom is polarizing power of the cationic species, K+,
bonded to two silicon atoms. Ca2+, Mg2+, Be2+?
Silicon atom is bonded to two oxygen atoms. Mg2+ < Be2+ < K+ < Ca2+
There are double bonds between silicon and Be2+ < K+ < Ca2+ < Mg2+
oxygen atoms. K+ < Ca2+ < Mg2+ < Be2+
[AlEEE-2005]
Ca2+ < Mg2+ < Be2+< K+
29. The molecular shapes of SF4, CF4 and XeF4 are: [AlEEE-2007]
Different with 1, 0 and 2 lone pairs of
35. The bond dissociation energy of B–F in BF3 is
electrons on the central atoms, respectively
646kJ mol-1 whereas that of C–F bond in CF4
Different with 0, 1 and 2 lone pairs of is 515kJ mol-1. The correct for higher B-F bond
electrons on the central atoms, respectively dissociation energy as compared to that of C – F
Different with 1, 1 and 1 lone pairs of is:
electrons on the central atoms, respectively Smaller size of B-atom as compared to that of
Different with 2, 0 and 1 lone pairs of C-atom
electrons on the central atoms, respectively Stronger s bond between B and F in BF3 as
[AlEEE-2005] compared to that between C and F is CF4
30. The number of hydrogen atom(s) attached to Significant pp–pp interaction between B and
phosphorus atom hypophosphorus acid is: F in BF3 whereas there is no possibility of
Zero (b) Two such interaction between C and F in CF4
(c) One (d) Three Lower degree of pp–pp interaction between
[AlEEE-2005] B and F in BF3 than that between C and F in
CF4
31. The correct order of the thermal stability of
hydrogen halides (H-X) is: [AlEEE-2009]
HI > HBr > HCl > HF 36. The hybridisation of orbitals of N atom in NO3-,
HF > HCl > HBr > HI NO2+ and NH4+ are respectively:
HCl < HF < HBr < HI sp, sp2, sp3 (b) sp2, sp, sp3
Hl < HCl < HF < HBr (c) sp, sp3, sp2 (d) sp2, sp3, sp
[AlEEE-2005] [AlEEE-2011]
2.27

37. The molecule having smallest bond angle is: H2-, He22+ (b) H2+, He22-
NCl3 (b) AsCl3 (c) H2-, He22- (d) H22+, He2
(c) SbCl3 (d) PCl3 [IIT-JEE Main-2013]
[AlEEE-2011] 45. Stability of the species Li2, Li2- and Li2+ increases
38. Boron cannot form which one of the following in the order of:
anions? Li2- < Li2< Li2+
BF63- (b) BH4- (b) Li2 < Li2+< Li2-
(c) Li2-< Li2+< Li2
(c) B(OH)4- (d) BO2-
(d) Li2< Li2-< Li2+
[AlEEE-2011]
[IIT-JEE Main-2013]
39. Among the following the maximum covalent
46. For which of the following molecule significant
character is shown by the compound:
m   π 0?
FeCl2 (b) SnCl2 Cl CN
Cl
(c) AlCl3 (d) MgCl2
[AlEEE-2011] (i) (ii)

40. Ortho-nitrophenol is less soluble in water than Cl CN


p-and m-nitrophenols because:
OH SH
O-nitrophenol is more steam volatile than
those or m-and p-isomers.
(iii) (iv)
O-nitrophenol shows intramolecular
H-bonding OH SH
O-nitrophenol shows intermolecular (i) and (ii) (b) Only (iii)
H-bonding (c) (iii) and (iv) (d) Only (i)
Melting point of O-nitrophenol is lower than [IIT-JEE Main-2014]
those of m-and p-isomers.
47. The correct statement for the molecule, CsI3, is:
[AlEEE-2012]
It contain Cs+ and I3- ions
41. In which of the following pairs the two species
It contains Cs3+ and I- ions
are not isostructural?
It contains Cs+, I- and lattice I2 molecule
CO32-and NO-3
It is a covalent molecule
(b) PCl+4 and SiCl4
[IIT-JEE Main-2014]
(c) PF5 and BrF5
(d) AlF63- and SF6 48. The intermolecular interaction that is dependent
on the inverse cube of distance between the
[AlEEE-2012]
molecules is:
42. Which one of the following molecules is expected (a) ion-ion interaction
to exhibit diamagnetic behaviour?
(b) ion-dipole interaction
S2 (b) C2
(c) N2 (d) O2 (c) London force

[IIT-JEE Main-2013] (d) hydrogen bond
43. Which of the following exists as covalent crystals [JEE-Main 2015]
in the solid state? 49. The species in which the N-atom is in a state of sp
Phosphorus (b) lodine hybridisation is:
(c) Silicon (d) sulphur
NO2- (b) NO3-
[IIT-JEE Main-2013]
(c) NO2 (d) NO2+
44. In which of the following pairs of molecules/ions,
both the species are not likely to exist ? [JEE-Main 2016]
2.28

Answer Key

1. (a) 2. (c) 3. (a) 4. (a) 5. (d) 6. (d) 7. (c) 8. (b) 9. (b) 10. (c)
11. (c) 12. (a) 13. (b) 14. (b) 15. (b) 16. (c) 17. (b) 18. (a) 19. (c) 20. (c)
21. (b) 22. (c) 23. (d) 24. (d) 25. (d) 26. (d) 27. (a) 28. (b) 29. (b) 30. (c)
31. (d) 32. (b) 33. (c) 34. (c) 35. (c) 36. (a) 37. (d) 38. (a) 39. (b) 40. (a)
41. (a) 42. (b) 43. (b) 44. (d) 45. (b)

1. (d) 2. (c) 3. (a) 4. (b) 5. (c) 6. (b) 7. (b) 8. (d) 9. (c) 10. (a)
11. (c) 12. (b) 13. (a) 14. (d) 15. (c) 16. (d) 17. (c) 18. (a) 19. (c) 20. (b)
21. (a) 22. (d) 23. (d) 24. (b) 25. (b) 26. (d) 27. (b) 28. (b) 29. (d) 30. (a)
31. (c) 32. (b) 33. (b) 34. (b) 35. (b) 36. (a) 37. (b) 38. (d) 39. (c) 40. (c)
41. (d) 42. (b) 43. (c) 44. (a) 45. (c)

1. (a, b) 2. (a, b, c) 3. (a, b, c) 4. (a, b, c) 5. (b, c, d) 6. (a, b, d) 7. (b, d) 8. (a, b, c, d)


9. (a, b, c, d) 10. (a, c) 11. (a, b, c) 12. (a, c, d) 13. (b, c, d) 14. (a, b, c) 15. (b) 16. (c)
17. (c) 18. (a) 19. (d) 20. (5) 21. (9) 22. (4) 23. (4) 24. (6)
25. (3) 26. (6) 27. (5)
28. A b;   B c;  C a,d; D a,c
29. A c;  B a;  C b;d; D b,c
30. A c,d; B a,b,d; C b; D b,c

1. (a) 2. (b) 3. (b) 4. (a) 5. (a) 6. (a) 7. (c) 8. (a) 9. (b) 10. (d)
11. (d) 12. (d) 13. (a) 14. (c) 15. (d) 16. (c) 17. (c) 18. (a, b, c) 19. (c) 20. (a)
21. (d) 22. (b) 23. (d) 24. (a) 25. (d) 26. (c) 27. (b) 28. (a) 29. (a) 30. (b)
31. (b) 32. (d) 33. (b) 34. (c) 35. (c) 36. (b) 37. (c) 38. (a) 39. (c) 40. (b)
41. (c) 42. (b, c) 43. (c) 44. (d) 45. (c) 46. (c) 47. (a) 48. (d) 49. (d)

Hints and Solutions

| Z+ | . | Z− |
4. (a) Lattic energy (U0) ∝
1 r0
1. (a) M 3 PO 4 |Z | and |Z | are magnitude of charge of cation
+ –
Formula of chloride of M is MCl and anion.
2. (c) Due to presence of lettice, there is no free ions r0 r + + r –
in NaCl hence, there is no mobility of ions. In LiF, both cation and anion are very small.
3. (a) According to Fazan’s rule, 5. (d) Most reactive metal is Cs but Li is the strongest
smaller cation and larger anion leads more reducting agent among all the metals.
covalent character. Only bicarbonates of Na+, K+, Rb+ and Cs+
2.29

exist in solid state. 17. (b) Cl


2LiNO3 ∆ → Li2O + 2NO2­- + 1/2O2­- Cl Cl

2NaNO3  → 2NaNO2 + O2-­ 0 =0
(K+, Rb+ or Cs+)
Order of solubility in water :
Cl Cl
LiOH < NaOH < KOH < RbOH < CsOH
Cl Cl
6. (d) F can not form multiple bond. Cl and P can
not form multiple bond with itself because Cl Cl Cl
3pp – 3pp bond is not stable.
7. (c) O O
O
N N
Cl Cl
O O µ = x µ = 2x
8. (b) In F2 molecule,
18. (a) Species lp + bp (s) Hybridization
F σ → F (p – p end-to-end overlapping) IF5 6 sp3d2
9. (b) Species Shape I 3
– 5 sp3d
NH3, NF3 Pyramidal I3
+ 4 sp3
BF3, BCl3, NO3– Trigonal planar 19. (c) In ICl4–, hybridization of central atom is
BF4 , NH4 – + Tetrahedral sp3d2. In sp3d2, axial d-orbitals dx2–y2 and dz2
BrCl3 ‘T’-shape participates.
10. (c) Species lp on central atom 20. (c) General order of melting point:
Covalent network solid > Ionic solid > Metallic
XeOF4 1
solic > molecular solid.
XeO2F2 1 21. (b) OF4 does not exist becuase, due to absence of
XeF3 – 3 d-orbitals ‘O’ can form only 2 covalent bonds.
XeO3 1 22. (c) Q
OQ O O
11. (c) Species Shape
XeF4 Square planar N N N
Q
XeF5 – Pentagonal planar O O O O O O
SnCl2 Angular or ‘v’-shape 23. (d)
12. (a) XeO3 (Pyramidal shape) C2 H 6 < CH3OH < KCl < Si
( Molecular with van der Waal ’s ( Molecular with H − bond ) ( Ionic ) ( Covalent network )
force of attraction )

Xe 24. (d) Hybridisation of ‘O’ in H3O+ is sp3.


O O 25. (d) Bond length directly depends on size of atoms.
+
O 26. (d) I3 I3−
13. (b) ’V ’−shape Linear
( Planar ) ( Planar )

27. (a) In I2 solid, I2 molecules are attached by London


N
H H forces (because I2 is a non-polar molecule)
H 28. (b) In ClO3, all Cl–O bonds are double bond.
Geometry is tetrahedral, shape is pyramidal

14. (b) H 2 O H 2 S NH3 SO2 Cl


( ≈ 104.5°) ( ≈ 92°) ( ≈ 107°.48′) ( ≈ 109.5°) O O

15.
+
(b) NH 4 > NH3 > NH 2− O
( ≈ 109.5°) ( ≈ 107° 48′) ( <<109°⋅5°) 29. (b) Species Shape
16. (c) If CF4, all C–F bonds are polar but due to XeF5– Pentagonal planar (6 atoms are
regular geometry it has zero dipole moment. in XY-plane)
2.30

SF6. Octahedral (5 atoms are in XY- 43. (b) N3– has linear structure. It is called as
plane) pseudo halogen. Oxidation state of
IF7. Pentagonal bipyramidal (6 ‘N’ is –1/3. Number of electrons in
atoms are in XY plane) N3– and NO2 are 22 and 23 respectively.
30. (c) Shape Possible hybridisation 44. (d) Only H2O and H2O2 can form H-bond.
Pentagonal planar sp3d3 45. (b) Order of strength of bond:
Trigonal planar sp2 Ion-dipole > Dipole-dipole > Dipole-induced
Linear sp, sp3d dipole > London force
Square planar dsp2, sp3d2
31. (d) In SO2, 2pp–3pp and 2pp–3dp bonds are
present.
1. (d)
32. (b) ML5 must be pentagonal planar with 5 bond F
O CI
pair and 2 lone pair around M.
CI

••
Xe
33. (c) Species lp + bp ( ) Hybridisation P CI (4 atoms in same plane)
O
I3–, XeF2, SF4 5 sp3d F CI
(4 atoms in same plane) CI
XeF4 6 sp3d2
ClO3–, PO43– 4 sp3 H
ClO3+ 3 sp2
As F F
34. (c) Species Shape Xe (5 atoms in same plane
H H
SF4 see-saw H F F
(3 atoms in same plane)
XeO4 Tetrahedral
XeF4, ICl4 – Square planar 2. (c)
35. (c) In ClF3, Cl has 10 electrons in its valence shell O
O

after sharing. CI O
CI CI CI O
O
36. (a) H – O – N = O O O OO O
O O
37. (d) K+ ionic HF2– O
O

F F BO = 1.75 BO = 1.67 BO = 1.5 BO = 1
H 3. (a)
H–bond Covalent bond O
38. (a) Dipole moment of CH4 is zero (it has regular O
O O O O
geometry).
NF3, NH3 and H2O have non-zero dipole 4. (b)
H
moment becuase they have irregular geometry.
39. (b) SO2 is polar because it has irragular geometry. H C H
C C
40. (a) Species lp + bp(s) Hybridisation
PCl5 5 sp3d
C C 6, sp2 – s sigma bonds are present
H C H
PCl4+ 4 sp3
PCl6 – 6 sp3d2 H
41. (a) Due to intramolecular H-bond, O-hydroxy- 5. (c)
benzaldehyde has weaker intermolecular Species Shape
bonding hence, it has higher vapour pressure NO2 linear
than p-hydroxybenzaldehyde.
NO2 'V'– shape
42. (b) Species Bond order
O2 –2 1 PCl5 Trigonal bipyramidalr
BrF5 Square pyramidal
O2– 1.5
XeF4, ICl4 Square planar
O2+ 2.5 TeCl4 See-saw
O2 2 XeO4 Tetrahedral
2.31

6. (b) 14. (d) Order of lattice energy :


NH3 + H +  → NH 4+ MgO > KCl
3
sp3
sp LiF > LiBr

H3BO3 + OH [B(OH)4]

Mg3N2 > NaCl
or sp 3

B(OH)3 15. (c) Size of O < size of C


16. (d) 2px + 2px p-bond
2
sp

NH3 NH2– + H+ 3dxy + 3dxy d-bond


3
sp3 sp 2s + 2py no bond
17. (c) H 2 O + CO2 → H 2 CO3 or (HO)2 CO
+ +
HO+H H3O
2 3
sp3 sp sp2
sp
H3 BO3 + OH → [ B (OH)4 ]−
7. (b) sp
2
sp3

order of repulsion is : BF3 + NH3 H3 Æ [H3 N Æ BF3 ]


sp3 sp3
lp – lp > lp – bp > bp – bp
18. (a)
As number of lp on central atom increases,
bond angle decreases. Species Shape
The number of lp on O in H2O is 2 while on N Pyramidal
SO32 , NH3
in NH3 is 1. The structures of xenon fluorides
and xenon oxofluorides could be explained on Trigonal planer
SO3, NO3 , CO32
the basis of VSEPR theory.
8. (d) 19. (c) Correct order of bond angle:
NH3 > PH3 > AsH3 > SbH3
Species Bond angle Cl2O > H2O > OF2
H3O+ > H3S+ > H3Se+ > H3Te+
O3 116°
BF3 = BCl3 = BBr3 = BI3
I3– 180° 20. (b) Species Shape
[XeF3]+ Bent T-shape
< 120°
NO2 [XeF5]+ Squar pyramidal
21. (a) Order of melting point:
PH3 93.8°
CaF2 > CaCl2 > CaBr2 > CaI2
22. (d) Correct order of melting point is,
9. (c) NF3 has irregular geometry while BF3 has
regular geometry. NaF < CaF2 < AlF3 (depends on lattice energy)
23. (d)
10. (a) Order of melting point: ∆ 1
CaO Ca + O2 ­
H2O > H2Te > H2Se > H2S 2
(Metal oxides have very high thermal stability)
NH3 > BiH3 > SbH3 > AsH3 > PH3
HI > HF > HBr > HCl 24. (b) Order of electronegativity:
sp > sp2 > sp3
11. (c) ICl is polar and Br2 is non-polar
order of bond angle between same hybrid
12. (b) O O orbitals:
|| ||
HO—S—(S) —S—OH sp > sp2 > sp3
n–2
|| ||
O O 25. (b)
Both ICl2– and BeCl2 have linear shape but
Total number of S-S bonds = (n – 1) they have different hybridisation of central
13. (a) Electron density of p-bond is present above atom.
and below of internuclear axis, not in the ICl2– BeCl2
molecular plane. sp3d sp
2.32

26. (d) S + px no bond 42. (b) Ion-dipole interaction is stronger than H-bond.
py + pz no bond 43. (c) In BF3, pp – pp back bonding is present.
27. (b) SrC2 (Sr – C C–)
+2 – 44. (a) Hybridization involves the mixing of atomic
28. (b) Due to regular geometry, BF3 has zero dipole orbitals of the atom at the time of their
moment. participation in bonding.
NH3 has higher dipole moment than NF3 45. (c)
because in NH3 lp moment supports dipole
moment while in NF3 lp moment opposes
dipole moment.
29. (d) In sp3d2 hybridisation (octahedral geometry),
12, 90° angles are observed between bp - bp of 1. (a, b)
electrons. Species Structure Bond order
30. (a) Order of bond angle:
O
SO3 > SO2 > SO32–
CO3–2 1.33
(120°) ( 109.5°) (< 109.5°) Θ
C
O Θ
O
31. (c) N2 N2+
1e– is removed from s2px or s2py or s2pz O

O2 O2+ NO3 N 1.33
Θ
O
1e– is removed from *2px or *2py or *2pz O

32. (b) Order of bond length:



H2O2 > O3 > O2 NO2 N 1.5
Θ
O O
(Bond order = 1 1.5 2)
33. (b) MgO has high lattice energy NO N O 1.5
34. (b) Fe+3 has greater polarising power than Fe+2.
2. (a, b, c)
35. (b) K2HPO3 (2K+ HPO3–2)
O In CH4, 2s 2p

C (I excitation)
P In BF3,
H O
O 2s 2p
O B (I excitation)
O
In ICl3,
36. (a) HF forms KHF2 because HF can form H-bond.
I (I excitation)
37. (b) ‘O’ forms covalent bond with 2 H-atoms and
5s 5p 5d
H-bond with 2 H-atoms.
1
38. (d) E (This relation is valid for dipole-dipole
r3 In PCl3,
interaction) 3s 3p
H-bond is a type dipole - dipole interaction P (Ground state)
39. (c) Cu is a metallic substance. 3. (a, b, c)
For metallic substance heat of vapourisation is Species Molecular shape
lower than for covalent network substance like NO2+ Linear
diamond. .
XeF2 Linear
40. (c) Strength of London dispersion forces depends
I3– Linear
on molecular mass.
41. (d) In this compound, intermolecular H-bond, I3+ ‘V’-shape
intramolecular H-bond and van der Waal’s 4. (a, b, c)
force, all are present. NO, NO2 and ClO2 are odd e– molecules.
2.33

5. (b, c, d) 12. (a, c, d)


Cause of a particular physical state of a molecular ‘O’ can form only 2 covalent bonds hence, OF4 does
compound is strength of intermolecular bonding not exist.
not intramolecular bonding ‘N’ can form only 3 covalent bonds hence,
6. (a, b, d)
NCl5 does not exist.
N2O N≡N→ O
In octahedral geometry 3lp are not possible hence
ICl32– does not exist.
O
N2O3 O 13. (b, c, d)
N–N O Ionic compounds have high melting point, boiling
point, solubility in polar solvents and have non-
O
O O directional bonds.
N2O5 N
N O 14. (a, b, c)
O
     Order of stability :
           O2+ > O2 > O2–
O
O Bond order = 2.5 2 1.5
N2O4 N–N O He2 does not exist because bonding and antibonding
O electrons are equal.
7. (b, d) In C2,O22– and Li2, all molecular orbitals are paired
NH3 + H + → NH 4+ hence, they are diamagnetic.
sp3 sp3 In F2, the energy of is less than and
2pz 2px 2py.

AlH3 + H → Al H 4−
sp2 sp3 15. (b) N 2 → N 2+
Bond order =3 Bond order = 2.5
NH3 → N H 2− + H +
sp3 sp3 O2 → O2+
Bond order = 2 Bond order = 2.5
SiF4 + 2F − → SiF6−2
sp3 sp3 d 2 16. (c)
Let the bond moment of A–X bond = x
8. (a, b, c, d) The dipole moment of AX2 =
s x 2 + x 2 + 2 x 2 (cos120°) = x
p
A

s X X
p
p p p p p p
9. (a, b, c, d)
In all these compounds H-atom is attached with
highly electronegative atom.
10. (a, c) s
p
SF6 has octahedral geometry (Non-planar)
p
In SF6, ‘S’ has 12e– in outermost shell after sharing
p
hence, it does not acquire e– configuration of Ar gas.
p
11. (a, b, c)
p
The correct order of boiling point is:
d
CH4 < SiH4 < GeH4
2.34

2. (b) lp + bp (s) Hybridisation


+
NO2 2 sp
CO3−2 , NO3− , NO2− → pπ − pπ –
NO3 3 sp2
SO3−2 , SO 4−2 , PO −4 3 , S2 O8−2 , R 3 PO, S2 O3−2 , ClO −4 , ClO3− , SO2 → pπ − dπ +
NH4 4 sp3
3. (b) lp + bp (s) Hybridisation
NH3 4 sp3
PCl5 5 sp3d
BCl3 3 sp2
In [PtCl4] , hybridisation of Pt is dsp2.
2–

4. (a) They all have 14 e– and bond order 3.


5. (a) H3N BF3
Or
≈ O
-
H3 N - B F3
sp3 sp3

6. (a) Electron density of p-bond is present above or


below of the molecular plane.
7. (c) O2– is an odd e– species. It has 1 unpaired e–.
O 8. (a) Both NO3– and CO2– 3 have 32 e and they have

P trigonal planar shape.
O O
P
9. (b) Bond order Magnetic character
O
P P O2   2 paramagnetic
O O2   2.5
+ paramagnetic
O
10. (d) Number of lp on central atom
ClO3– 1
XeF4 2
SF4 1
I3– 3
11. (d) Hybridisation of P in P4 is sp3.
12. (d) Anion of KO2 is O2– (superoxide). O2– is an
odd e– species.
13. (a) Bond order
CO 3
" NO– 2
NO+ 3
" CN– 3
" " " " N2 3
" " " "
14. (c) In XeO3, Xe has 1 lp and 3 bp ( ).
" " " "
Hence, its shape is pyramidal.
15. (d) p-hydroxybenzoic acid has intermolecular
H-bond while O-hydroxybenzoic acid has
intramolecular H-bond. Due to intermolecular
H-bond, intermolecular bond becomes weaker
1. (a) Order of boiling point is: hence, p-isomer has greater boiling point than
H2O > H2Te > H2Se > H2S O-isomer.
2.35

16. (c) In water, orthoboric acid behaves as OH– 28. (a)


acceptor O O
H3BO3 + H2O $ [B(OH)4]– + H+ ~ O – Si – O – Si – O ~
Or
B(OH)3 O O
17. (c) For heavier members of group 14, due to inert
~ O – Si – O – Si – O ~
pair effect, lower oxidation state is more stable.
18. (a, b, c) O O
N = N →O (N2O)
O O 29. (a) Molecule lp on central atom Shape
N–N (N2O3) SF4 1 see-saw
O
CF4 0 Tetrahedral
O O XeF4 2 Square planar
N–N O (N2O4)
O 30. (b) Hypophosphorus acid (H3PO2)
O
O O O P
N N (N2O5) H OH
O O H
19. (c) Bond energy of C–C bond 100 kCal mol–1. 31. (b) As bond length increases, bond breaking
becomes easier.
20. (a) If Hund’s rule is not followed then Bond order
32. (d) Order of bond angles :
of B2 is 1 and it is diamagnetic
NH3 > PH3 > AsH3 > SbH3
21. (d) 33. (b) NO NO+
(Pyramidal shape) Bond order : 2.5 3
S
O F
Magnetic character : paramagnetic Diamagnetic
F
34. (c) Order of polarizing power :
H
22. (b) H
C=C=C K+ < Ca+2 < Mg+2 < Be+2,
H 2 H 1
sp sp sp2 Polarising power
size of ion
F
23. (d) O 35. (c) Due to p –p back bonding, B – F bond has
: Xe (see-saw shape)
partial -character hence, its bond energy
becomes more than expected.
O
F
36. (b) lp + bp( ) Hybridisation
24. (a) AlH3 AlH4– NO3 – 3 sp2
sp2 sp3 NO2+ 2 sp
H2O H3O+ NH4+ 4 sp3
sp3 sp3 37. (c) Order of bond angle:
NH3 NH4+
NCl3 > PCl3 > AsCl3 > SbCl3 (Order depends
sp3 sp3
on electronegativity of central atom)
25. (d) After sp3d2 hybridisation, geometry is
octahedral. In octahedral geometry. 38. (a) Due to absence of d-orbitals, ‘B’ can maximum
form 4 bonds (3 covalent and 1 co-ordinate
12, 90° angles are present.
bond)
26. (c) Order of melting point : 39. (c) Covalent character µ polarising power of
NaCl > KCl > RbCl > LiCl           cation
27. (b) CaC2 (Ca+2 C C ) µ charge density of cation
2.36

40. (b) Due to intramolecular H-bond, O-nitrophenol 46. (c) H H


O S
is less soluble in water.
41. (c) Species Molecular shape
CO3 , NO3 Plane triangle
2– – µ ≠0 µ ≠0

PCl4+, SiCl4 Tetrahedral


PF5 Trigonal bipyramidal O S
H H
BrF5 Square pyramidal
 AlF6 , SF6 Octahedral
3– 47. (a) CsI3 Cs+ + I3–
42. (b, c) In C2 and N2, all molecular orbitals are 48. (d) For dipole-dipole interaction (or, H-bond).
paired. 1
E
43. (c) Phosphorus (P4), Iodine (I2) and sulphur (S8) r3
are molecular solids while silicon (Si) is 49. (d) Species Hybridization
covalent network crystal. NO2– sp2
44. (d) H22+ has no electron and bond order of He2 is NO3 – sp2
zero. NO2 sp2
45. (c) Molecular orbital Bond order NO2+ sp
electronic arrangement
Li2 s1s2, s1s*2, s2s2 (4–2)/2 = 1
Li2+ s1s2 , s1s*2 , s2s1 (3–2)/2 = 0.5
Li2 – s1s2, s1s*2 , s2s2, s*2s1
(4–3)/2 = 0.5
Both Li2+ and Li2– have similar
bond order but
Li2– has more number of antibonding e– than
Li2+ hence, Li2+, is more stable than Li2–.
Chapter

Key Concepts
A metal atom or ion may combine with neutral molecules or
anions to form a new identifiable species called a complex
or coordination compound. For example, [Co(NH3)6]3+
is a complex species which can be identified as a whole.
The groups that surround the metal ion in a coordination
compounds are called ligands. The total number of the
ligands bound around a metal ion is called the coordination
number of the metal ion. Ligands have been classified
depending upon the number of donor atoms it has. A few
examples are given below :-
Mono or Unidentate Ligand: Examples are F–, Cl–, Br–, (a) The name of negative ligands end in ‘–o’, e.g.
H2O, NH3, CN– and etc. fluoro (F–), chloro (Cl–), hydrido (H–), thio (S2–),
nitro (NO2–)
Bidentate: Examples are ethylenediamine (en), glycinate (b) Neutral groups have no special endings, e.g.
ion (gly) etc. ammine (NH3), aqua (H2O), carbonyl (CO) and
Tridadentate: Examples are diethylenetriamine and 2, 2’, nitrosyl, (NO).
2”- tripyridine etc. (c) Positive groups en ‘-ium’, e.g. hydrazinium
Tetradentate: Examples are triethylenetetrammine and (H2NNH3+).
ethylenebis (salicyladimine) ion.
Pentadentate: Examples is ethylenediaminetriacetate ion.
Hexadentate: Example is ethylenediamine tetracetate ion.

The International Union of Pure and Applied Chemistry


(IUPAC) have suggested the following basic rules for
naming a coordination compound.
3.2

2. The cation makes available a number of orbitals


equal to its coordination number, for the formation of
covalent bonds with the ligands.
3. The cation orbitals hybridize to form a new set of
equivalent hybrid orbitals with definite directional
characteristics.
4. The nonbonding metal electrons occupy the inner d
orbitals and do not participate in the hybridization.
5. Each ligand contains a lone pair of electrons. A covalent
bond is formed by the overlap of vacant hybrid metallic
orbitals and a filled orbital of the ligand.
The above rules are illustrated with the following
A few example of naming the complex compounds typical examples.
are given in the following:
[Cr(NH3)6]3+
[Co(NH3)6]Cl3. Hexaamminecobalt (III) chloride
The outer electronic configuration of Cr is (3d)5
[CoCl(NH3)5]2+ Pentaamminechlorcoblat (III) ion (4s)1. Chromium in the above complex is in +3
[CoSO4(NH3)4]NO3. Tetraamminesulphatocobalt (III) oxidation state, hence, Cr(III) has the configuration
nitrate of (3d)3. There are six ligands, so six empty orbitals
[Co(NO2)3(NH3)3] Triamminetrinitrocobalt (III) of chromium are required. These include two 3d
orbitals, one 4s orbital and three 4p orbitals. These
Na2[ZnCl4] Sodium tetrachlorozincate (III)
orbitals hybridize to give d2sp3 hybrid orbitals
K3[Fe(CN)5(NO)] Potassium pentacyanonitrosyl- directed towards the corners of an octahedron.
ferrate (II) The find configuration of the complex will contain
Fe(C5H5)2. Bis(cyclopentadienyl) iron (II) three unpaired electrons in three 3d orbitals of
chromium. Hence, the complex will be paramagnetic.
The above description is diagrametically represented
as follows:
3d 4s 4p

On of the earliest theories to explain the formation of Cr atom


coordination compounds has given by Alfred Werner.
According to him, each element exhibits two types of Cr ion
3+

valencies. There is primary valency (which corresponds


Cr3+ in
to the oxidation state of the central metal) and secondary [Cr(NH3)6]
valency (which represents the coordination number of
the central metal). The primary valency is satisfied by 2 3
ds p hybridization six pairs of electrons
anions whereas the secondary valency is satisfied by either from six NH3 molecules
negative ion or neutral molecules. The primary valencies
are shown by dotted lines while secondary valencies by
3–
[CoF6]
solid lines. The secondary valencies are always directed 3d 4s 4p
Co atom
towards fixed positions in space giving a definite geometry
4p
to the complex. 3+
Co in
Modern theories to explain the formation of complex 4d
3+
compounds are valence-bond theory area, crystal field Co in
3–
[CoF6]
theory. The salient features of the valence bond theory are
as follows:- sp3d2 hybridization six pairs of electrons
from six F– ligands
1. The central metal looses a requisite number of
electrons to form the ion. The number of electrons lost The complex [CoF6]3– is octahedral and is strongly
is the valency of the resulting cation. paramagnetic.
3.3

[Ni(CN)4]2– and [NiCl4]2– orbital complexes sp3d2 hybridisation is involved.


3d 4s 4p The inner orbital complexes involve lesser number or
Ni atom unpaired electrons and thus these are known as low-spin
complexes. On the other hand, the outer orbital complexes
involve comparatively larger number of unpaired electrons
2+
Ni ion and thus are known as high-spin complexes.
Example [Cr(NH3)6]3+ involves d2sp3 hybridisation while
2+
Ni in [CoF6]3– involves sp3d2 hybridisation.
2–
[Ni(CN)4] 3d 4s 4p
3–
Cr in
×× ×× ×× ×× ×× ××
[Cr(NH3)6]3+
dsp2 hybridization, four pairs of 2 3
electrons from four CN ligands d sp hybridisation (inner-orbital complex)

3+ 3d 4s 4p 4p
Co in
[CoF6]3– ×× ×× ×× ×× ×× ××
2+
Ni in
2–
[NiC14] 3 2
sp d hybridisatoin (outer-orbital complex)

2
sp hybridization, four pairs of electrons For octahedral complexes,
from four CN ligands

[Ni(CN)4]2– is a square planar (dsp2 hybridisation) and SFL


diamagnetic (no unpaired electrons) while [NiCl4]2– is d4 - d7 (Pairing occurs) d2Sp3 (inner orbital/low spin complex)
tetrahedron (sp3 hybridisation) and paramagnetic (two
unpaired electrons). The CN– ligand is strong and it forces WFL Sp3d2 (outer orbital/high spin complex)
the two unpaired electrons to pair up.
[Ni(NH3)6]2+
3d 4s 4p
Ni atom

Ni2+ ion
[Co(OX)3 ]−3 , [Co(H 2 O)6 ]+3 , [ NiF6 ]−2 , [Mn ( NH3 )6 ]+2 , [Fe( NH3 )6 ]+2
4d d 2sp3 d 2sp3 d 2sp3 sp3d 2 sp3d 2

sp3d 2 hybridisation, octahedral Coordination compounds exhibit various types of


isomerism. A brief description of these are as follows:-
[Ni(NH3)6]2+ has an octahedral structure and is
paramagnetic (two unpaired electrons).
Ni(CO)4 The isomers involving the exchange of ligand in the
3d 4s 4p coordination sphere and the anion present outside the
coordination sphere are known as ionization isomers. For
Ni atom
example, [Co(NH3) 5 Br]SO4 and [Co(NH3)5 (SO4)Br.

Ni in
Ni (CO)4 The isomers involving the attachment of different donor
3
sp hybridisation atom of a ligand to the metal in the coordination sphere are
Ni(CO)4 has a tetrahedral structure and is diamagnetic known as linkage isomers. For example [Co(NH3)5 (NO2)]
(no unpaired electrons). CI2 and [Co(NH3)5(ONO)]CI2

The isomers involving the exchange of ligands between


the complex catoin and complex anion of a complex
In the inner orbital octahedral complexes d2sp3 compound are known as coordinate isomers. For example,
hybridisation in the central metal occurs while in the outer
3.4
A A A B

M M

The isomers involving the exchange of H2O molecules are


B B B A
known as hydrate isomers. Cis Trans
For example : [Co(H2O)6]Cl3 and [Co(H2O)5]Cl2.H2O

In an octahedral complex, if the two similar ligands


The isomers involving the different geometrical occupy positions at the two ends of the twelve edges of the
arrangement of ligands around the central metal atom are octahedron, the complex is the cis isomer.
known as geometrical isomers. The two identical ligands If the two similar ligands occupy positions at the end of a
occupying he adjacent position is known as cis isomer straight line passing through the centre of the octahedron
while those occupying the opposite by the lower case (which is occupied by the central atom M), the complex is
alphabets (such as a, b, c,…) and bidentate by the upper the trans isomer.
case alphabets (such as AA and AB), the geometrical A brief account of geometrical isomers of a few complex
isomerism in complex compounds are as follows. AA is compound is given below.
symmetrical and AB is unsymmetrical bidentate ligand. The complexes [Ma6], [Ma5b] and [M(AA)3] do not show
geometrical isomerism.
The complexes [Ma4b2], [Ma3b3] and [Ma4bc] exhibit two
isomers each.
The complexes [Ma4] and [Ma3b] do not exhibit [Ma4b2]
geometrical isomerism. b b

The complexes [Ma2b2] and [Ma2bc] exhibit cis-trans a b a a


isomerism.
a b a b
M M

M M
a a a a

a b b a a b
Cis Trans
Cis-isomer Trans-isomer
Ma2b2 In cis-form, the two ‘b’ ligands have cis positions to
a b a b each other. In trans-form, the two ‘b’ ligands have trans
positions to each other.
M M
[Ma4bc]
b a

a a a b
a c c a
Cis Trans

Ma2bc M M

The complex [Mabcd] exists in three isomeric forms


a a a c
The have different pairs of ligands at the trans positions.
a c a b a b
c a
M M M Cis-isomer Trans-isomer
[Ma3b3]
d b d c c d
In cis-form, the three ligands ‘a’ occupy positions at
The complex [M(AB)2] also exists in cis-and trans-forms. the corners of a triangular face and the three ‘b’ ligands
3.5

occupy positions at the corners of the opposite face. This a a a

isomer is also known as facial (abbreviation-face) isomer a b a c a d

The trans-form is known as meri-donal (abbreviation-mer) M M M


isomer.
a a b c b d b c

b a b a d b b
(and its enantiomer) (and its enantiomer)

M M The complex [Mabcdef] exhibit fifteen isomers (= 6C2


= 6!/(4! 2!)) all of them have their enantiomers, making a
b a b b total of 30 isomers.
The complexes [M(AA)a2b2], [M(AA)2a2] and [M
b a (AA)2ab], where AA is a symmetrical bidentate, exists in
face-isomer mer-isomer two forms, namely, cis and trans forms, of which cis form
has its enantiomer.
The complex [Ma2b2c2] exists in five geometrical isomers,
a a
(three are five ways of distributing pairs of ligands in trans
positons), one of which also has its enantionmer, thus a A b A
there exists a total of six isomers.
a a a
M M
b c b c a c
b A b A
M M M
b a
c b b c c b Cis-form (Trans-form)
(and its enantiomer)
a a b
Trans positions, aa; bc; bc ab; ab; cc A a
aa; bb; cc
A A A A
a a
a c b c
M M
M M
A a A A
b c a b
A a
b c
(enantiomer) Cis-form (Trans-form)
ab; ac; bc ac; bb; ac (and its enantiomer)

Co-ordination compounds having no plane of symmetry


A a
or centre of symmetry exists in two optically active
configuration which are related to each other through the A a A A
mirror image of each other and are not super-imposable
on each other.
M M
The complex [Ma2b2cd] exhibits six geometrical isomers
there are six ways of distributing pairs of ligands in trans A
A b A
positions, two of which have enantiomers, thus making a
total of eight isomers. A A
a a a
Cis-form (Trans-form)
b c b c a c (and its enantiomer)

M M M The complex [M(AB)3], where AB is unsymmetrical


bidentate ligand in which A and B are two different donor
d b d d d b atoms, exists in cis-and trans-forms. Each of the two also
a a
has its enantiomer.
b
3.6

A A

B B B A

M M
2
B A A B 2

A B 2
Cis-form Trans-form
(and its enantiomer) (and its enantiomer) 2
3
Example of Geometrical isomers in octahedral complexes
2
The cis isomers involve the two identical ligands on any
of the twelve edges of the octahrdron and trans isomers 2
involve the ligands on either end of a straight line passing
through the metal. 2
Cl 2 +
Cl
+
2 2
H3N Cl H3N NH3 2
6
Co Co
8
H3N Cl H3N NH3 15
NH3 Cl 30
Cis-isomer trans-isomer
2
3
3
A complex having no plane of symmetry or centre of
inversion exists in two optically active isomers. Such a 2
complex is said to be asymmetric. The two isomers are 4
mirror image of each other and the are not superimposable 6
on each other. The two have identical physical and chemical 12
properties but differ in their action on the polarized light. 4
Examples are
8
Py 2 +
Py 2 + 11
Cl Py Py Cl 4
Pt Pt 6

Cl NH3 H3N 12
Cl
NH3 NH3 24
Mirror

N + N +
N Cl Cl N The stability of a complex compound depends on the
:(N–N is a nature of the metal and that of the ligand. In general, the
Co Co
bidentate higher the oxidation state of the metal, the more stable the
N Cl Cl N ligand) complex. The cyano complexes are far more stable than
N N those formed by halide ions.
Mirror
The increasing order of the influence of ligands on the
electronic configuration of central metal atom or ion
(known as spectrochemical series) is:-
I– < Br– < S2– < SCN– < Cl– < F– < OH– < Ox < O2– < H2O
< NH3 < NO2– < CN– < CO
3.7

Thus, halogens are said to be weak ligands while NO2–,


CN– are said to be strong ligands.
Thus stability of complexes also decreases with increases
in the ionic size of the central metallic ion having the same
charge number. For example, the stability of complexes
formed from the same ligands increases from Mn2+ to Cu2+
and then decreases at Zn2+, that is the order of stability is
Mn 2 + < Fe2 + < Co 2 + < Ni 2 + < Cu 2 + > Zn 2 +
91 pm 83 pm 82 pm 78 pm 91 pm 74 pm

This sequence is known as Iriving-Willam order of stability


of complexes of M2+ ions.

Many ionic and covalent compounds of transition elements


are coloured. When light passes through a material,
it absorbs some of the wavelengths due to electronic
excitation. If absorption occurs in the visible region of
the spectrum, the transmitted light is coloured with the
z2 complimentary colour to the light absorbed. The material
looks coloured corresponding to the transmitted light.
In transition elements, the electronic excitation is due
to d-d electronic excitation. In an isolated metal ions,
d-orbitals are degenerate. This degeneracy is lost when
eg t2
the ligands approach the metal ion. The energy of some
of d-orbitals is lowered while of the remaining orbitals the
3
5
0 2
5
t energy is raised. Electrons occupy the orbitals of lower
energy and ca be excited to higher ones with the absorption
0 t of wavelength which lies in the visible region.
2
5
0 3
t In some complexes, cause of colour is charge transfer
5
spectra. It is of three types :-
t2g eg

Octahedron arrangement of ligands Tetrahedral arrangement of ligands


MnO 4 , MnO 24 , CrO 24 , Cr2 O72

Solved Examples
1. Select the incorrect statement: 2. Aqueous solution of Mohr’s salt gives a positive
(a) Hydrazine can act as bidentate ligand. test for:
(b) Oxalate is a chelating ligand. (a) Ferrous ions only
(b) Sulphate ions only
(c) All bidentate ligands are chelating ligands.
(c) Ammonium and sulphate ions only
(d) Number of chelate rings = (denticity – 1) (d) Ferrous, ammonium and sulphate ions
Sol.(a) Hydrazine (H2N – NH2) is a monodentate ligand.
All bidentate ligands are chelating ligands,
oxalate is a bidentate ligand.
3.8

(a) Diamminechlorido dipyridinenitrito–N–


cobaltate (III) nitrate
(b) Diammine chloridonitrito–O–bipyridine
cobalt (III) nitrate
(c) Diamminechloridonitrito–O–dipyridine
cobalt (III) nitrate
(d) Chloridodiamminenitrito–O– bis (pyridine)
cobalt (III) nitrate
Sol.(c) Correct IUPAC name is:
O O Diamminechloridonitrito–O–dipyridinecobalt
Q || || Q (III) nitrate.
2–
C2O4 6. Which of the following pairs of name and formula
          O of complexes is not correct?
      ||   Q
(a) Tetraamminecopper (II) sulphate

   [Cu(NH3)4]SO4
(b) Diamminesilver   (I) chloride [Ag(NH3)2]Cl
(c) Potassium hexacyanidoferrate (III)
N
..
K4[Fe(CN)6]
(d) Potassium amminepentachloridoplatinate (IV)
K[Pt (NH3)Cl5]
N
.. N
.. c) The correct formula of Potassium hexacyanido
ferrate (III) is K3[Fe(CN)6]
7. Different hydrated isomer of CrCl3.6H2O can not
be differentiated by:
(b) [V(CO)6] can act as oxidising agent. (a) Conductivity measurements
(c) [Fe(CO)5] can act as both oxidising agent and (b) Precipitation by AgNO3
reducing agent. (c) Dipole moment
(d) [Mn(CO)5] show dimerisation to gain (d) Magnetic moment
stability.

Sol. (c) [Mn(CO)6] (i) [Cr(H2O)6] Cl3


EAN of Mn = 25 – 0 + 12 = 37 (ii) [Cr(H2O)5Cl] Cl2. H2O
To gain stability it has to loose an e– hence, it can (iii) [Cr(H2O)4Cl2] Cl. 2H2O
act as reducing agent.
[V(CO)6]
EAN of V = 23 – 0 + 12 = 35
To gain stability it has to gain an e– hence, it can
act as oxidising agent.
[Fe(CO)5]
EAN of Fe = 26 – 0 + 10 = 36 (a) participation of d-orbital in hybridization.
It follows EAN rule. It does not behave as (b) polarity and magnetic nature.
oxidising agent and reducing agent. (c) presence of synergic bonding.
[Mn(CO)5] can dimerise [Mn2(CO)10] to gain (d) shifting of ns electron into (n–1) d subshell.
stability.
Sol.(a) Hybridization of Fe in [Fe(CO)5] is dsp3 while
5. The IUPAC name for the complex, [Co(NH3)2Cl hybridization of Ni in [Ni(CO)4] is sp3.
(ONO) (Py)2] NO3 is:
3.9

3d 4s 4p


3d 4s 4p

3d 4s 4p

3 1
d sp

3
dsp
10. Select the correct match:
[Ni(CO)4]
Ni = [Ar] 4s2 3d8
3d 4s 4p

3
sp

9. In the following complex ions order of


paramagnetism is :


(a) P>Q>R>S
(b) Q>P>R>S (c) [Fe(H2O)6]SO4 green
(c) P=Q=R=S (d) [Ti(H2O)6]3+ purple
(d) P>R>Q>S
)

3d 4s 4p 4d

sp3d2 12. The complex exhibits lowest energy absorption


band is:
(a) [NiCl4]2–
(b) [Ni(CO)4]
3d 4s 4p 4d (c) [Ni(CN)4]2–
(d) [Ni(H2O)6]2+
3 2
sp d
∆t < ∆0 < ∆sp
( Tetrahedral ) ( Octahedral ) (Square planar )

3d 4s 4p

3 1
d sp
3.10

13. Which of the following can show both geometrical


and optical isomerism?
(a) [Co(Ox)3]–3 Sol.(a) The oxidation state of cobalt in the complex
(b) [Fe(NH3)4 Cl2]+ [Co(NH3)6]3+ is + 3
(c) [Fe(NH3) (H2O) (Py) (Br) (Cl) (OH)] The electronic configuration of Co3+ ion is :
(d) [Co(NH3)3(NO2)3] 3d 4s 4p

3+
[Co (NH3)6]
Rearangement
d2sp3 hydridization

Octahedral, zero magnetic moment


(b) The oxidation state of chromium in the complex
is +3. The electronic configuration
3d 4s 4p

14. Which of the following is not correct?

d2sp3 hydridization

15 = 3.87 BM
17. From the stability constants (hypothetical values),
given below, predict which one is the strongest
ligand?

 







15. Which of the following names are not correct? 



Point out the mistakes.
(a) [Cu(H2O) (NH3)] Br2; Amminoaquodibro- Sol.(b) Greater the value of stability constant, stronger is
mocopper (I) the ligand.
(b) Na3[Al(C2O4)3]; Trisodium trioxalatoalumi- 18. Each of the compounds Pt(NH3)6Cl4,
nate (III) Cr(NH3)6Cl3, Co(NH3)4Cl3 and K2PtCl6 were
(c) Na2[Ni(EDTA)]; Sodium ethylenedi- dissolved in water to make its 0.001 M solution.
aminetetraacetato nickelate (II) The correct order of their increasing conductivity
(d) [Co(NH3)5ONO] SO4 ; Pentaamminenitroco- in solution is-
balt (III) sulphate (a) K2PtCl6 < Co(NH3)4Cl3 < Cr(NH3)6Cl3 <
Sol. (a) Ammine aqua copper (II) bromide Pt(NH3)6Cl4
(b) Sodium trioxalato aluminate (III) (b) Cr(NH3)6Cl3 < Co(NH3)4Cl3 < K2PtCl6 <
(c) It is correct Pt(NH3)6Cl4
(c) Co(NH3)4Cl3 < K2PtCl6 < Cr(NH3)6Cl3 <
(d) Pentammine nitrito cobalt (III) sulphate
Pt(NH3)6Cl4
16. Find out the hybridization, geometry and magnetic (d) Pt(NH3)6Cl4 < Co(NH3)4Cl3 < Cr(NH3)6Cl3 <
moment of the complexes : K2PtCl6
3.11

Sol.(c) In aqueous solution (0.001 M), the complexes coordination number is attained by other Mn atom
will dissociate to give the ions : of (Mn–Mn) bond. Thus it can have structure :

 
 CO CO
CO CO

 OC Mn Mn CO


 OC CO
CO CO

 20. Identify the complexes which are expected to be
19. The EAN of each Mn (Z = 25) in its carbonyl coloured and explain.
is 36. What is the structure of the carbonyl with (a) Ti(NO3)4
molecular formula : Mn2 (CO)10 ? (b) [Cu(NCCH3)4]+BF4–
Sol.
Electrons from each Mn = 25 (c) [Cr(NH3)6]3+ 3Cl–
(d) K3[VF6]
Sol.(c & d) (c) and (d) are coloured because Cr3+ in
Thus is the complex five CO (ligands) are [Cr(NH3)6]3+ and V3+ in [VF6]3– has unpaired
coordinated to each Mn atom and sixth electron in d subshell.

Exercise
5. Chelating ligand is:
(a) thiocyanate (b) cyanide
(c) Oxalate (d) Ammonia
1. Incorrect statement for addition compound is: 6. Which can’t form chelates?
(a) Simple salts do not lose their identify in (a) Didentate ligand
double salt. (b) Ambidentate ligand
(b) Complex compounds retain their identity in (c) Tetradentate lignad
aqueous solution. (d) Flexidenate ligand
(c) Simple salts lose their identity in complex 7. An example for a double salt is:
compound. (a) Cuprammonium sulphate
(d) Double salts retain their identity in aqueous (b) Mohr’s salt
solution. (c) Potassium ferricyanide
(d) Cobalthexammine chloride
2. Neutral and symmetrical bidentate ligand is:
8. Which of the following complexes are
(a) Oxalate (b) dien heteroleptic?
(c) gly (d) dipyridyl (a) [Cr(NH3)6]3+ (b) [Fe(NH3)4Cl2]+
3. Which of the following pair contains only (c) [Mn(CN)6]4– (d) [Co(NH3)6]+2
ambidenate ligand? 9. Which of the following can not act as ligand?
(a) H2N—CH2—COO–
(b) H2N–NH2
4. EDTA is a:

(a) Polydenate ligand N H3
(b) Chelating ligand 10. The oxidizing agent is:
(c) Flexidenate ligand (a) Fe(CO)5. (b) Mn(CO)5
(d) All of these (c) Mn2(CO)10. (d) Fe2(CO)9
3.12

11. Which follows EAN Rule? (a) [Co(NH3)6]Cl3


(a) [Pd(NH3)6]Cl4. (b) [Cr(NH3)5Cl] SO4 (b) [Co(NH3)3Cl3]
(c) Na4[Fe(CN)6] (d) K3[Fe(CN)6] (c) [Co(NH3)4Cl2]Cl
12. In sodium nitroprusside the oxidation number, (d) [Co(NH3)5Cl]Cl2
coordination number and effective atomic number 19. Which of the following ligand act as WFL for
of iron are respectively: octahedral complex of Co+3 ion?
(a) +3, 6, 35. (b) +3, 6, 36 (a) F  (b) H2O
(c) +2, 3, 36. (d) +2, 6, 36 2
(c) C2 O 4 (d) All of these
13. EAN of metal carbonyl M(CO)x is 36 if atomic
20. [Ni(CN)4] and [Ni(CN)4]–4 are not differ in:
–2
number of metal M is 26, what is the value of x?
(a) 4 (b) 8 (a) Oxidation state and co-ordination number
(c) 5 (d) 6 (b) Hybridisation state and geometry
14. The IUPAC name of [Pt(NH3)3(Br)(NO2)(Cl)]Cl (c) Magnetic behavior
is: (d) None of these
(a) Triamminechlorobromonitroplatinum (IV) 21. Inner orbital complex which is paramagnetic in
chloride nature?
(b) Triamminebromochloronitroplatinum (IV) (a) [Ni(CN)5]–3
chloride
(b) [Fe(CN)6]–4
(c) Triamminechlorobromoplatinum (IV)
(c) [Fe(H2O)5(NO)]SO4
chloride
(d) Triamminechloronitrobromoplatinum (IV) (d) [Fe(CN)6]–3
chloride 22. The complex ion [Fe(CN)6]4– contains:
15. Formula of potassium dicyanodioxalato nickelate (a) total of 36 electrons on Fe2+ cation
(II) is: (b) sp3d2 hybrid orbitals with octahedral structure
(a) K4[Ni(CN)(OX)2] (c) Twelve coordinate bonds
(b) K3[Ni(CN)2(OX)] (d) six sigma bonds
(c) K4[Ni(CN)2(OX)2] 23. [NiCl4]–2 and [Ni(CN)4]–2 show similarity in:
(d) K2[Ni(CN)2(OX)2] (a) Geometry
16. Select the correct IUPAC name of [Co(NH3)6] (b) Magnetic nature
[Co(ONO)6]: (c) Hybridisation of state of Ni
(a) Hexaamminecobalt (II) hexanitrito–O (d) Primary valency of Ni
cobalt (II) 24. VBT can not explain the geometry of:
(b) Hexaamminecobalt (III) hexanitrito–N (a) [Cr(NH3)6]+3. (b) [Co(NH3)6]+3
cobaltate (III)
(c) [Cu(NH3)4]+2. (d) [Zn(NH3)4]+2
(c) Hexaamminecobalt (II) hexanitrito–O
25. Incorrect order of splitting energy is:
cobaltate (II)
(a) [NiF6]–4 < [NiF6]–2
(d) Hexaamminecobalt (III) hexanitrito–O
cobaltate (III) (b) [Fe(H2O)6]+3 < [Ru(H2O)6]+3
17. When 1 mol CrCl3.6H2O is treated with excess of (c) [Co(NH3)6]+3 < [Co(en)3]+3
AgNO3, 2 mol of AgCl are obtained. The formula (d) [Ni(CN)4]–2 < [Ni(CN)6]–4
of the complex: 26. A complex has magnetic moment 1.73 BM.
(a) [CrCl3(H2O)3].3H2O Which of the following configuration of metal ion
(b) [CrCl2(H2O)4].2H2O is not possible?
(c) [CrCl(H2O)5]Cl2.H2O (a) d7, octahedral, lowspin
(d) [Cr(H2O)6]Cl3 (b) d5, octahedral, lowspin
18. Which of the following shows maximum (c) d8, square planar
conductance? (d) d9, tetrahedral
3.13

(a) 2 and 2. (b) 2 and 3


(c) 3 and 2. (d) 3 and 3
36. The type of isomerism present in pentaamminenitro
chromium (III) ion is:
(a) Optical (b) Linkage
(c) Ionization (d) Polymerisation
37. Na2S forms violet colour complex when reacts
with:
(a) Brown ring complex
29. Which of the following is correct order of (b) Sodium nitroprussible
stability? (c) K4[Fe(CN)6]
(a) [NiCl4]–2 < [PdCl4]–2 < [PtCl4]–2 (d) Hypo solution
(b) [Co(H2O)6]+3 < [Co(NH3)6]+3 < [Co(CN)6]–3 38. The incorrect statement is:
(c) [Co(H2O)6]+3 < [Rh(H2O)6]+3 < [Ir(H2O)6]+3 (a) Fe+2 salt gives blue colloidal solution with
(d) All of these K3[Fe(CN)6]
30. The colour of a compound may be due to : (b) FeCl3 gives red colour with K SCN
(a) polarisation (c) Cu+2 salt gives blue colloidal solution with
K4[Fe(CN)6]
(b) d-d transition
(d) Light blue solution of CuSO4 turn into dark
(c) charge transfer spectra
blue in presence of ammonia.
(d) All of these
39. Select correct statement regarding [Ni(DMG)2]
31. Which is coloured but not due to d-d transition? complex compound:
(a) Cr2O72– (b) KMnO4 (a) It acts as oxidizing agent because Ni2+ cation
(c) AgBr (d) All of these is having EAN 35.
(b) It is extra stabilized by hydrogen bonding
32. The colour of light absorbed by an aqueous
(c) It’s IUPAC name is Bis (dimethylglyoximato)
solution of CuSO4 is:
nickelate (II)
(a) orange-red (b) blue-green (d) It’s ligand contains two different donar sites
(c) yellow (d) violet 40. Sodium thiosulphate is used in photography to
33. Which of the following isomerism is not present (a) Reduce AgBr to metallic Ag
in complex [Co(NH3)4 (SCN)2] Br? (b) Convert metallic Ag to Ag salt
(a) Geometrical isomerism (c) Remove undecomposed AgBr as a soluble
(b) Linkage isomerism silver thiosulphate complex
(c) Ionisation isomerism (d) Remove unreduced silver
(d) Optical isomerism
34. Which is correct statement?
(a) [Co(en)3] [Cr(CN)6] will display coordination
isomerism 1. A six coordinate complex of formula CrCl3.6H2O
(b) [Mn(CO)5(SCN)] will display linkage has green colour. A 0.1 M solution of the complex
isomerism when treated with excess of AgNO3 gave 28.7 g
of white precipitate. The formula of the complex
(c) [Co(NH3)5(NO2)]SO4 will display ionization
would be:
isomerism
(a) [Cr(H2O)6)]Cl3
(d) All are correct
(b) [CrCl(H2O)5]Cl2.H2O
35. How many isomers exist for [Co(NH3)4Cl2]+ and
(c) [CrCl2(H2O)4]Cl.2H2O
[Co(en)2Cl2]+ complex ions, respectively?
(d) [Cr(H2O)3Cl3]
3.14

2. Which is not true about metal carbonyls? 9. Cis-trans isomerism is exhibited by:
(a) Here CO acts as a Lewis base as well as (a) [PtCl(NH3)3]3+ (b) [Pt(NH3)4]2+
Lewis acid
(c) [PtCl4]2– (d) [PtCl2(NH3)2]
(b) Here metal acts as Lewis base as well as
Lewis acid 10. Which one of the following platinum complexes
is used in cancer chemotherapy?
(a) cis-[PtCl2(NH3)2]
3. Which of the following pair the EAN of central (b) trans-[PtCl2(NH3)]
metal atom is not same? (c) [Pt(NH3)4]2+
(a) [Fe(CN)6]3– and [Fe(NH3)6]3+ (d) [Pt(Cl4)]2–
(b) [Cr(NH3)6]3+ and [Cr(CN)6]3– 11. The cyanide complex of silver formed in the
(c) [FeF6]3– and [Fe(CN)6]3– silver extraction in Mac-Arthur’s Forrest cyanide

(d) [Ni(CO)4] and [Ni(CN)4]2– process is:

4. The IUPAC name for K2[Cr(CN)2O2(O2)NH3] is: (a) [Ag(CN)2]– (b) K2[Ag(CN)3]
2–
(a) Potassium amminedicyanotetraoxo chromi- (c) [Ag(CN)4] (d) Na3[Ag(CN)4]
um (III)
(b) Potassium amminedicyanodioxygendioxo
chromate (IV)
(c) Potassium amminedicyanosuperoxoperoxo
chromate (III)
(d) Potassium amminedicyanodioxoperoxo
chromate (VI)
5. The magnetic moment of [NiX4]2– ion is found to (a) 0.01, 0.02. (b) 0.02, 0.01
be zero. Then the ion is: (c) 0.01, 0.01. (d) 0.02, 0.02
(X = monodentate anionic ligand) 13. In Na2[Fe(CN)5NO], sodium nitroprusside:
(a) sp3 hydridised (b) spd2 hydridised (a) oxidation state of Fe is +2
(c) dsp2 hydridised (d) d2sp hydridised (b) this has NO+ as ligand
6. The magnetic moment of a complex ion is 2.83 (c) both are correct
BM. The complex ion is : (d) none is correct
(a) [V(H2O)6]3+ (b) [Cr(H2O)6]3+

14. Complexes formed in the following methods are:
(c) [Cu(CN)4]2– (d) [MnCl4]2–
(I) Mond’s process for purification of nickel
7. What is the magnetic moment (spin only) and (II) Removal of unreacted AgBr from
hybridisation of the brown ring complex photographic plate
[Fe(H2O)5NO]SO4? (III) Removal of lead poisoning from the body
3 BM, sp3 d2 3 BM, d2 sp3    I II III
15 BM, sp3 d2 15 BM, d2sp3 (a) Ni(CO)4. [Ag(CN)2] –
[Pb(EDTA)]2–
8. Select the correct code about complex [Cr(NO2) (b) Ni(CO)4. [Ag(S2O3)2]3– [Pb(EDTA)]2–
(NH3)5] [ZnCl4]:
(I) IUPAC name of compound is pentaammine- (c) Ni(CO)6. [Ag(S2O3)2]3– [Pb(EDTA)]4–
nitrito-N-chromium (III) tetrachlorozincate (d) Ni(CO)6. [Ag(S2O3)2]– [Pb(EDTA)]2–
(II)
Ø
(II) It shows geometrical isomerism
(III) It shows linkage isomerism C
(IV) It shows coordination isomerism (Soluble Complex)

(a) III, IV (b) I, III and IV D


(Soluble Complex)
(c) II, III and IV (d) I, II, III and IV
3.15

21. Ligand with two or more points of attachment to


(a) IUPAC name of complex D is Tetraammine single metal atoms are called:
cuprate (II) sulphate (a) Modnodentate ligand (b) Chelating ligand
(b) Complex C is [Ag(NH3)4]+ (c) Ambidentate ligand (d) None of these
(c) By adding HNO3, complex C further converts 22. Number of water molecules acting as ligands in
into AgCl CuSO4.5H2O, ZnSO4.5H2O, FeSO4.7H2O
(d) In complex D, hybridization of Cu2+ is sp3 respectively are
(a) 5,5,7. (b) 4,5,4
(c) 4,4,6. (d) 4,4,7
(a) dx2 – y2, dz2, dxy
(b) dxy, dyz, dxz C2 O24 OQ2
(c) dx2 – y2, dxy, dxz
(d) dx2 – y2, dz2, dxz x y z
(a) 1. 1. 1
17. A [M(H2O)6]2+ complex typically absorbs (b) 1. 1. 2
at around 600 nm. It is allowed to react with (c) 1. 2. 2
ammonia to form a new complex [M(NH3)6]2+ (d) 2. 1. 1
that should have absorption at:
24. Which of the following is most likely structure
(a) 800 nm (b) 580 nm
1
(c) 620 nm (d) 320 nm of CrCl3.6H2O if of total chlorine of the
3
compound is precipitated by adding AgNO3 to its
aqueous solution?
(a) Oxygen gives colour to it (a) [Cr(H2O)6]Cl3
(b) Charge transfer when Mn (+7) gives its (b) [Cr(H2O)3Cl3].3H2O
electron to oxygen and oxidise to Mn (+8) (c) [CrCl2(H2O)4]Cl.2H2O
temporarily (d) [CrCl.(H2O)5]Cl2.H2O
(c) Charge transfer when oxygen gives its 25. Which of the following pair contains complex salt
electron to Mn (+7) changing in Mn (+6) and double salt respectively?
(d) None is correct explanation (a) FeSO4,K4[Fe(CN)6]
(b) [Cu(NH3)4]SO4, FeSO4.7H2O
(c) [Cu(NH3)4]SO4, K2SO4Al2(SO4)3.24H2O
 (d) MgSO4.7H2O, CuSO4 .5H2O


26. Give EAN value of Mg in [Mg(EDTA)]2– :
(a) It is a ligand substitution reaction (a) 16 (b) 20
(c) 22 (d) 18
(b) NH3 is a relatively strong field ligand while
H2O is a weak field ligand 27. The type of isomerism present in pentaammine
(c) During the reaction, there is a change in nitro chromium (III) perchlorate is
colour from light blue to dark blue (a) Optical (b) Linkage
(c) Hydrate (d) Polymerisation
(d) [Cu(NH3)4]2+, has a tetrahedral structure and
is paramagnetic 28. The following complexes are given
20. Select the correct IUPAC name for [Cr(C6H6)
(CO)3]:
(C) trans-[Co(NH3)2(en)2]3+
(D) [NiI4]2–
(E) [TiF6]2–
(F) [CoF6]3–
3.16

Choose the correct code. H 2 O < NH3 < NOΘ


2 < CN
Θ

(a) D, E are coloured; F is colourless H 2 O < NH3 < CN Θ < NOΘ


2
(b) B is optically active; A, C are optically 35. Which of the following complex is inner orbital
inactive as well as low spin complex?
(c) A, B are optically active; C optically inactive (a) [Cr(H2O)6]3+ (b) [Fe(CN)6]3–
(d) D is coloured; E, F are colourless
(c) [Cu(CN)4]3– (d) [Ni(NH3)6]2+
29. One mole of complex compound Cr(NH3)5Cl3
give 3 moles of ions on dissolution in water. One 36. Which of the following is incorrect about
mole of the same, complex reacts with two moles Wilkinson’s catalyst?
of AgNO3 to yield two moles of AgCl(s). The (a) It is a diamagnetic complex.
complex is (b) It is a non-ionic complex.
(a) [Cr(NH3)4Cl]Cl2.NH3 (c) It is tetrahedral complex.
(b) [Cr(NH3)4Cl2]Cl.NH3 (d) It is very effective for selective hydrogenation
of organic molecule at room temperature and
(c) [Cr(NH3)5Cl]Cl2
pressure.
(d) [Cr(NH3)3Cl3].2NH3
37. [Cr(H2O)6]Cl3 (atomic number of Cr = 24) has
30. In which of the following pairs both the complexes
a magnetic moment of 3.83 BM. The correct
show optical isomerism ?
distribution of 3d electron in the chromium of the
(a) cis-[Cr(C2O4)2Cl2]3–, cis-[Co(NH­3)4Cl­2] complex is:
(b) [Co(en)3]Cl3 , cis-[Co(en)2Cl2]Cl 3d1xy ,3d1yz ,3d1z2
(c) [Co(NO3)3(NH3)3], cis-[Pt(en)2Cl2]
(d) [PtCl(en)Cl], [NiCl2Br2]2–

31. The correct order of magnetic moment (spin


values in BM) is
(Atomic number Mn = 25, Fe = 26, Co = 27) 38. What will be the correct order of absorption
(I) [MnBr4]2– of wavelength of light in the visible region,
(II) [Fe(CN)6]4– for the complex, [Co(NH3)6]3+, [Co(CN)6]3–,
[Co(H2O)6]3+?
(III) [CoBr4]2–
(a) [Co(CN)6]3– > [Co(NH3)6]3+ > [Co(H2O)6]3+
(a) II > III > I (b) I > II > III
(b) [Co(NH3)6]3+ > [Co(H2O)6]3+ > [Co(CN)6]3–
(c) II > I > III (d) I > III > II
(c) [Co(H2O)6]3+ > [Co(NH3)6]3+ > [Co(CN)6]3–
32. Which of the following compound is not
coloured? (d) [Co(CN)6]3– > [Co(NH3)6]3+ < [Co(H2O)6]3+
(a) Na2[CuCl4] (b) Na2[CdCl4] 39. An ion M2+, form the complexes
(c) K4[Fe(CN)6] (d) K3[Fe(CN)6] [M(H2O)6]2+, [M(en)3]2+ and [M Br6]4– . Colour

of these complexes may be:
(a) Green, blue & Red (b) Blue, Red & Green
(c) Green, Red & Blue (d) Red, Blue & Green
(a) [Co(CN)6]3– (b) [CoF6]3–
(c) [Co(NO2)6]3– (d) [Co(NH3)6]3+ 40. Cu2+ ions will be reduced to Cu+ ions by the
addition of an aqueous solution of:
34. The increasing order of the crystal field splitting
power of some common ligands is
NH3 < NOΘ Θ
2 < CN < H 2 O
3.17

(b) In [Fe(C2O4)3]2+, geometrical isomerism


does not exist, while optical isomerism exists.

ONE OR MORE THAN ONE TYPE QUESTIONS

(a) Ionization isomerism


7. The d-orbitals involved in sp3d2 or d2sp3
(b) Geometrical isomerism hybridisation of the central metal ion are:
(c) Optical isomerism
(d) Linkage isomerism
2. Find out correct IUPAC name of complex
compound. 8. Which of the following pairs of name and formula
of complexes, is correct?
(a) Pentaaminecyanidocyanidochromium (II)
hexanitrito-N-irridate (III) (a) Tetramminecopper (II) sulphate ………….
(b) Triamminetricyanidochromium (III) [Cu(NH3)4]SO4
hexanitrito-N-irridate (III) (b) Diamminesliver (I) chloride ………………
(c) Hexanitrito-N-irridium (III) [Ag(NH3)2]Cl
pentaamminecyanidochromate (II)
(c)
Potassium hexacyanidoferrate (III)
(d) Pentaamminecyanidchromium (III) ………………….. K4[Fe(CN)6]
hexanitrito-N-irridate (III) (d) Potassium amminepentachloridoplatinate
3. Complex ions [NiCl6]4–, [Ni(CN)6]4– similar in (IV) ………………. K[Pt(NH3)Cl5]
their given properties: 9. Which of the following is/are correctly matched?
(a) oxidation state, geometry (a) [Ni(CO)4]Cl2-dsp2 and diamagnetic.
(b) co-ordination number, EAN (b) [Ni(en)3] (NO2)2- sp3d2 and two unpaired
(c) magnetic moment, geometry electrons.
(d) stability, colour (c) [V (NH3)6]Cl3-sp3d2 and two unpaired
electrons.
4. Which of the following compound has/have (d) [Mn (NO+)3(CO)]-sp3 and diamagnetic.
effective atomic number equal to the atomic
number of noble gas?
(a) K[Co(CO)4] (b) K2[Fe(CO)4]
S1: Generally square planar complexes show
geometrical isomerism but do not exhibit optical
(c) [Co(NH3)6]Cl2. (d) [CoCl3(H2O)3]
isomerism because they do not posses plane of
5. A d-block element forms octahedral complex symmetry.
but its magnetic moment remains same either in 4
strong field or in weak field ligand. Which of the
9
following is/are correct?
(a) Element always forms colourless compound.
(b) Number of electrons in t2g orbitals are higher
than in eg orbitals.
(c) It can have either d3 or d8 configuration.
(d) It can have either d7 or d8 configuration.
(a) S1 and S3 are correct
6. Which of the following statement (s) is/are false?
(a) In [PtCl2(NH3)4]2+ complex ion, the cis- (b) S2 and S3 are correct
form is optically active, while trans-form is (c) S1 is incorrect
optically inactive. (d) S2 and S3 are incorrect
3.18

11. Three arrangements are shown for the complex or neutral molecules which have a lone pair of
[Co(en)(NH3)2Cl2]+. Pick up the wrong statement. electrons, if the ligand is a neutral molecule such
Cl Cl Cl
as NH3, the negative end of the dipole in the
Cl NH3
NH 3 molecule is directed towards the metal cation.
Co en Co en
en Co The electrons on the central metal ion are under
H2N NH 3
NH 3
repulsive forces from those on the ligands. Thus
NH3 Cl Cl the electrons occupy the d-orbitals remain away
(I) (II) (III) from the direction of approach of ligands.
(a) I and II are geometrical isomers
(b) II and III are optical isomers
(c) I and III are optical isomers
(d) II and III are geometrical isomers
12. Consider the following complexies [V(CO)6]–, 16. The crystal field-spliting order for Cr3+ cation in
[Cr(CO)6] and [Mn(CO)6]+. Then incorrect octahedral field for ligands CH3COO–, NH3,H2O
statement (s) about metal carbonyls is/are. CN– is:
(a) ‘C–O’ bond is strongest in the cation and (a) CH3COO– < H2O < NH3 < CN–
weakest in the anion (b) CH3COO– < NH3 < H2O < CN–
(b) ‘C–O’ bond order is less in the cation than in (c) H2O < CH3COO– NH3 < CN–
anion. (d) NH3 < CH3COO– < H2O < CN–
(c) ‘C–O’ bond longer in the cation than in
Passage #3: (Q. 17 and 18)
anionic or neutral carbonyl.
Double salts are addition compounds which
(d) ‘M–C’ bond order is higher in the cation than
lose their identity in aqueous solution whereas
in anionic or neural carbonyl.
complexes which are also addition compounds
BASED ON PASSAGE TYPE QUESTIONS do not lose their identify in aqueous solution.
The coordination compounds show isomerism
Passage # 1 (Q. 13 and 14) and find applications in photography, qualitative
An isomer of the complex CoBrCl2(en)2(H2O), on analysis, metallurgy, water purification and in the
reaction with concentrated H2SO4 (dehydrating treatment of various diseases.
agent), suffers no loss in weight and on reaction 17. Which of the following statements is incorrect?
with AgNO3 solution it gives only white
(a) Alum is a double salt.
precipitate, which is soluble in NH3 solution.
(b) EDTA salt of calcium is used in the treatment
13. The correct formula of the complex is: of lead poisoning.
(a) [CoBr(H2O)(en)2]Cl2 (c) Effective atomic number of the metals in
(b) [CoCl(en)2(H2O)]BrCl complexes [Ni(CO)4] and [Fe(CN)6]4– is
(c) [CoBrCl(en)2]Cl. H2O same.
(d) [CoCl2(en)2]Br. H2O (d) Chloridotris-(triphenylphosphine) rhodium
(I) is effective heterogenous catalyst for
14. The incorrect statement about complex is:
hydrogenation of alkenes.
(a) It can show geometrical isomerism
(b) cis isomer is optically active 18. Which of the following statements is true for the
complex [Co(NH3)4Br2] NO2?
(c) Trans isomer is optically active
(a) It shows ionisation, linkage and geometrical
(d) It can exhibit solvate isomerism isomerism.
Passage #2: (Q. 15 and 16) (b) It does not show optical isomerism because
According to CFT, attraction between the central its cis and trans forms each have at least one
metal ion and ligands in a complex is purely plane of symmetry.
electrostatic. The transition metal which forms the (c) Its ionisation isomers cannot be differentiated
central atom cation in the complex is regarded as by silver nitrate solution.
positive ion. It is surrounding by negative ligands (d) (a) and (b) both.
3.19

INTEGER TYPE QUESTIONS 27.


19. Brown colour of the complex [Fe(H2O)5(NO)]
SO4 is due to charge transfer spectrum which
causes momentary change in oxidation state. Find
out oxidation state of Fe in this complex.
20. Sum of denticity of following ligands are

Glycinate ion, Oxalate ion, o-phenathroline,
2,2'-bipyridyl, diethylenetriamine,
ethylenediamine
28.
21. Find the sum of number of geometrical isomers
for following complexes.
(1) [CoCl2Br2]2–
(2) [Rh(en)3]3+
(3) [Cr(en)2Br2]+ (4) [Pt(en)Cl2]
(5) [Co(NH3)3(NO2)3]
22. In the complex Fe(CO)x, the value of x is:
23. Count the number of ions which can form both
low spin and high spin complexes when co-
ordination number 6
Co+3, Ni+2, Cr+3, Fe+2, Fe+3, Cu+2, Ti+3, Co+2

24. The number of unpaired electrons present in
[NiF6]2– is …………………

MATCH THE COLUMN TYPE QUESTIONS


25.

(A) [MnCl4]2– (P) sp3 hybridisation


3 15
(B) [Ni(CN)4]–2 (Q) Diamagnetic
24 8
(C) [Ni(CO)4] (R) Paramagnetic
(D) [Cu(NH3)4]2+ (S) dsp2 hybridisation
26.

(c) Geometrical and optical


(d) Geometrical only

(A) [Ni(CO)4] and K2 (P) Magnetic moment 4. If the bond length of CO bond in carbon monoxide
[Ni(CN)4]
is 1.128 Å, then what is the value of CO bond
(B) [Cu(NH3)4] SO4 (Q) Oxidaton no. of length in Fe(CO)5?
and K3[Cu(CN)4] central metal (a) 1.15 Å (b) 1.128 Å
(C) K2[NiCl4] and K4 (R) Geometry (c) 1.72 Å (d) 1.118 Å
[Ni(CN)4] [IIT-2006]
(D) K2[NiCl4] and 5. Among the following metal carbonyls, the C—O
K2[PtCl4] bond order is lowest in
(a) [Mn(CO)6]+ (b) [Fe(CO)5]
(c) [Cr(CO)6] (d) [V(CO)6]–
[IIT-2007]
3.20

6. Both [Ni(CO)4] and [Ni(CN)4]2– are diamagnetic. (c) HOOCCH2 CH2COOH


The hybridisations of nickel in these complexes N–CH2–CH2 –N
HOOCCH2 CH2COOH
respectively, are
(a) sp3, sp3. (b) sp3, dsp2 (d) COOH
2 3
H 2C
(c) dsp , sp . (d) dsp2, dsp2 HOOC–H2C H
[IIT-2008] N–CH–CH–N
H CH2–COOH
7. Among the following, the coloured compounds is CH2
(a) CuCl (b) K3[Cu(CN)4] HOOC
(c) CuF2 (d) [Cu(CH3CN)4]BF4 [IIT-2010]
[IIT-2008]
13. The ionization isomer of [Cr(H2O)4Cl(NO2)]Cl is
8. The IUPAC name of [Ni(NH3)4] [NiCl4] is (a) [Cr(H2O)4(O2N)]Cl2
(a) Tetrachloronickel (II) - tetraamminenickel
(b) [Cr(H2O)4 Cl2] (NO2)
(II)
(b) Tetraamminenickel (II) - tetrachloronickel (c) [Cr(H2O)4 Cl(ONO)]Cl
(II) (d) [Cr(H2O)4 Cl2(NO2)]. H2O
(c) Tetraamminenickel (II) - tetrachloronickelate [IIT-2010]
(II) 14. Geometrical shapes of the complexes formed by
(d) Tetrachloronickel (II) - tetraammnenickelate the reaction of
(0) Ni2+ with Cl–, CN– and H2O, respectively, are
[IIT-2008] (a) octahedral, tetrahedral and square planar
9. Statement I: [Fe(H2O)5NO]SO4 is paramagnetic. (b) tetrahedral, square planar and octahedral
Statement II: The Fe in [Fe(H2O)5NO]SO4 has (c) square planar, tetrahedral and octahedral
three unpaired electrons. (d) octahedral, square planar and octahedral
(a) Statement I is true; Statement II is true; [IIT-2011]
Statement II is the correct explanation of
15. Among the following complexes (K to P)
Statement I.
(b) Statement I is true; Statement II is true; K3[Fe(CN)6] (K), [Co(NH3)6]Cl3(L),
Statement II is not the correct explanation of Na3[Co(ox)3] (M), [Ni(H2O)6]Cl2(N),
Statement I. K2[Pt(CN)4] (O), [Zn(H2O)6](NO3)2(P)
(c) Statement I is true ; Statement II is false.
(d) Statement I is false; Statement II is true. The diamagnetic complexes are
[IIT-2008] (a) K, L, M, N (b) K, M, O, P
10. The complex showing a spin only magnetic (c) L, M, O, P (d) L, M, N, O
moment of 2.82 BM is [IIT-2011]
(a) Ni(CO)4 (b) [NiCl4]2– 16. NiCl2{P(C2H5)2(C6H5)}2 exhibits temperature
(c) Ni(PPh3)4 (d) [Ni(CN)4]2– dependent magnetic behavior (paramagnetic/
[IIT-2010] diamagnetic) the coordination geometries of
11. The spin only magnetic moment value (in Bohr Ni2+ in the paramagnetic and diamagnetic states
magneton units) of Cr(CO)6 is respectively, are
(a) 0 (b) 2.84 (a) tetrahedral and tetrahedral
(c) 4.90 (d) 5.92 (b) square planar and square planar
[IIT-2010] (c) tetrahedral and square planar
12. The correct structure of ethylenediaminetetraacetic (d) square planar and tetrahedral
acid (EDTA) is [IIT-2012]
(a) HOOCCH2 CH2COOH
17. As per IUPAC nomenclature, the name of the
N–CH=CH–N
HOOCCH2 CH2COOH complex [Co (H2O)4 (NH3)2] Cl3 is
(b) HOOC COOH (a) tetraaquadiaminecobalt (III) chloride
N–CH–CH–N (b) tetraaquadiamminecobalt (III) choride
HOOC COOH
3.21

(c) diaminetetraquacobalt (III) chloride 22. How many EDTA (ethylenediaminetetraacetate


(d) diamminetetraaquacobalt (III) chloride ion) molecules are required to make an octahedral
[IIT-2012] complex with a Ca2+ ion?
(a) One (b) Two
18. Consider the following complex ions, P, Q and R.
(c) Six (d) Three
P = [FeF6]3–, Q = [V(H2O)6]2+ and [AIEEE-2006]
R = [Fe(H2O)6]2+. 23. The “spin only” magnetic moment [in units of
The correct order of the complex ions, according Bohr magneton] of Ni2+ in aqueous solution
to their spin-only magnetic moment values (in would be. (At. No. Ni = 28)-
BM) is (a) 0 (b) 1.73
(a) R < Q < P (b) Q < R < P (c) 2.84 (d) 4.90
[AIEEE-2006]
(c) R < P < Q (d) Q < P < R
24. The only of the ‘spin only’ magnetic moment for
[JEE-Advanced-2013]
one of the following configuration is 2.84 BM.
19. Match each coordination compound in Column I The correct one is:
with an appropriate pair of characteristics from (a) d 4 (in strong ligand field)
Column II and select and correct answer using the (b) d 4 (in weak ligand field)
codes given below the Columns (c) d 3 (in weak as well as in strong field)

(en = H2NCH2CH2NH2 ; atomic numbers : (d) d5 (in strong ligand field)
Ti = 22; Cr = 24; Co = 27; Pt = 78) [AIEEE-2006]
25. Which one of the following has a square planar
geometry–
(Co = 27, Ni = 28, Fe = 26, Pt = 78)-
(a) [CoCl4]2– (b) [FeCl4]2–
2–
(c) [NiCl4] (d) [PtCl4]2–
(C)
[AIEEE-2007]
26. The coordination number and the oxidation state
of the element ‘E’ in the complex [E(en)2(C2O4)]
NO2 (where (en) is ethylene diamine) are,
Codes: respectively–
A B C D A B C D (a) 6 and +2 (b) 4 and +2
(a) d b c a (b) c a d b (c) 4 and +3 (d) 6 and +3
(c) b a c d (d) a c d b [AIEEE-2008]
[JEE-Advanced-2014]
20. The IUPAC name for the complex [Co(NO2)
(NH3)5] Cl2 is: (a) [Co(CN)6]3– (b) [Co(C2O4)3]3–
(a) pentaammine nitro-N-cobalt (II) chloride (c) [Co(H2O)6]3+ (d) [Co(NH3)6]3+
(b) pentaammine nitro-N-cobalt (III) chloride
(c) nitrito-N-pentaammineocobalt (III) chloride
(d) nitrito-N-pentaammineocobalt (II) chloride
[AIEEE-2006]
29. Which of the following pairs represents linkage
isomers?
(a) [Pd (P Ph3)2(NCS)2] and [Pd (P Ph3)2(SCN)2]
(b) [Co(NH3)5(NO3)] SO4 and [Co(NH3)5SO4]
NO3
3.22

(c) [Pt Cl2 (NH3)4] Br2 and [Pt Br2 (NH3)4] Cl2 (c) [Co(NH3)2Cl2]+

(d) [Cu(NH3)4] [(PtCl4)] and [Pt(NH3)4] [CuCl4] (d) [Cr(NH3)2Cl2]+
[AIEEE Online-2012]
[AIEEE-2009]
37. Type of isomerism which exists between
30. Among the ligands NH3, en, CN– and CO the
[Pd(C6H5)2(SCN)2] and [Pd(C6H5)2(NCS)2]
correct order the their increasing field strength, is-
(a) Linkage isomerism
(a) NH3 < en < CN– < CO
(b) Coordination isomerism
(b) CN– < NH3 < CO < en (c) Ionisation isomerism
(c) en < CN– < NH3 < CO (d) Solvate isomerism
(d) CO < NH3 < en < CN– [AIEEE Online-2013]
[AIEEE-2011]
31. Which one of the following complex ions has (a) [Fe(CN)6]3– (b) [Co(OX)3]3–
geometrical isomers? (c) [FeF6]3– (d) [Co(F6)]3–
(a) [Ni(NH3)5Br]+

(b) [Co(NH3)2 (en)2]3+
(c) [Cr(NH3)4(en)]3+
(a) [Co(H2O)6]2+ (b) [Co(CN)6]3–
(d) [Co(en)3]3+
[AIEEE-2011] (c) [Co(C2O4)3]3+ (d) [Co(NH3)6]3+
32. Which among the following will be named as
dibromidobis (ethylene diamine) chromium (III)
bromide?
(a) [Cr(en)2Br2] Br (b) [Cr(en)2Br2] Br
(c) [Cr(en) Br2] Br (d) [Cr(en)3]Br3
[AIEEE-2012]
33. The d-electron configuration of Cr2+, Mn2+, Fe2+ (c) [Fe(CN)6]3– is diamagnetic, [FeF6]3– is
and Co2+ are d4, d5, d6 and and d7 respectively. paramagnetic
Which one of the following will exhibit the lowest (d) [Fe(CN)6]3– is paramagnetic, [FeF6]3– is
paramagnetic behavior? diamagnetic
(At number Cr = 24, Mn = 25, Fe = 26, Co = 27)
(a) [Cr(H2O)6]2+ (b) [Co(H2O)6]2+ 41. An octahedral complex of Co3+ is diamagnetic.
(c) [Mn(H2O)6]2+ (d) [Fe(H2O)6]2+ The hybridisation involved in the formation of the
[AIEEE Online-2012] complex is:
34. The correct order of ligands in the spectrochemical (a) sp3 d2 (b) dsp2
series is:
(c) d2sp3 (d) dsp3d
(a) NCS– > CN– > Cl– > en
[JEE-Main Online-2014]
(b) CN– > en > NCS– > Cl–
(c) Cl– > en > CN– > NCS– 42.
Which of the following name formula
(d) en > CN– > Cl– > NCS– combinations is not correct?
[AIEEE Online-2012]
35. Square-planar geometry is shown by:
(a) [NiCl4]2– (b) CrO42–
(c) MnO4– (d) [PtCl2(NH3)2]
[AIEEE Online-2012]
36. Which of the following complex ions will exhibit
optical isomerism? (en = 1.2 diamine ethane)
(a) [Co(en)2Cl2]+

(b) [Zn(en)2]2+ [JEE-Main Online-2014]
3.23

43. Consider the coordination compound, (a) [FeF6]3– (b) [Mn(CN)6]4–


[Co (NH3)6]Cl3. In the formation of this complex, (c) [CoF6]3– (d) [Co(NH3)6]2+
the species which acts as the Lewis acid is-
[JEE-Main Online-2015]
(a) [Co(NH)6]3+ (b) Cl–
50. When concentrated HCl is added to an aqueous
(c) Co3+ (d) NH3 solution of CoCl2, its colour changes from reddish
[JEE-Main Online-2014] pink to deep blue. Which complex ion gives blue
44. Which one of the following complexes will most colour in this reaction
likely absorb visible light? (a) [CoCl6]3– (b) [Co(H2O)6]2+
(At number Sc = 21, Ti = 22, V = 23, Zn = 30) (c) [CoCl6]4– (d) [CoCl4]2–
(a) [Sc (H2O)6]3+ (b) [Ti (NH3)6]4+
[JEE-Main Online-2015]
(c) [V(NH3)6]3+ (d) [Zn(NH3)6]2+
[JEE-Main Online-2014]
45. An octahedral complex with molecular
composition M.5NH3. ClSO4 has two isomers,
A and B. The solution of A gives a white
precipitate with AgNO3 solution and the solution
of B gives white precipitate with BaCl2 solution.
The type of isomerism exhibited by the complex
52. The equation which is balanced and represents
is:
the correct product (s) is
(a) Linkage isomerism
(b) Ionisation isomerism
(c) Coordinate isomerism
(d) Geometrical isomerism Excess
 NaOH

[JEE-Main Online-2014]
46. Nickel (Z = 28) combines with a uninegative
monodentate ligand to form a diamagnetic
[JEE-Main-2015]
complex [NiL4]2–. The hybridisation involved and
the number of unpaired electrons present in the 53. The number of geometric isomers that can exist
complex are respectively. for square planar [Pt(Cl)(py)(NH3)(NH2OH))]+ is
(a) sp3, two (b) dsp2, zero (py = pyridine).
2
(c) dsp , one (d) sp3, zero (a) 2 (b) 3
[JEE-Main Online-2014] (c) 4 (d) 6
[JEE-Main-2015]
47. The octahedral complex of a metal ion M3+
with four monodentate ligands L1, L2, L3 and L4 54. The pair having the same magnetic moment is
absorb wavelengths in the region of red, green, [at number Cr = 24, Mn = 25, Fe = 26 and Co =
yellow and blue, respectively. The increasing 27]
order of ligand strength of the four ligands is (a) [Cr(H2O)6]2+ and [Fe(H2O)6]2+
(a) L4 < L3 < L2 < L1. (b) L1 < L3 < L2 < L4 (b) [Mn(H2O)6]2+ and [Cr(H2O)6]2+
(c) L3 < L2 < L4 < L1. (d) L1 < L2 < L4 < L3 (c) [CoCl4]2– and [Fe(H2O)6]2+
[JEE-Main-2014] (d) [Cr(H2O)6]2+ and [CoCl4]2–
48. Which molecule /ion among the following cannot [JEE-Main-2016]
act as a ligand in complex compounds 55. Which one of the following complexes shows
(a) CH4. (b) CN – optical isomerism?
– (a) cis [Co(en)2 Cl2]Cl
(c) Br (d) CO
(b) trans [Co(en)2 Cl2]Cl
[JEE-Main Online-2015] (c) [Co(NH3)4 Cl2]Cl
49. Which of the following complex ions has electrons (d) [Co(NH3)3 Cl3]
that are symmetrically filled in both t2g and eg orbits? [JEE-Main-2016]
3.24

Answer Key

1. (d) 2. (d) 3. (b) 4. (d) 5. (c) 6. (b) 7. (b) 8. (b) 9. (c) 10. (b)
11. (c) 12. (d) 13. (c) 14. (b) 15. (c) 16. (d) 17. (c) 18. (a) 19. (a) 20. (c)
21. (d) 22. (a) 23. (d) 24. (c) 25. (d) 26. (c) 27. (c) 28. (c) 29. (d) 30. (d)
31. (b) 32. (a) 33. (d) 34. (d) 35. (b) 36. (b) 37. (b) 38. (c) 39. (b) 40. (c)

1. (b) 2. (d) 3. (d) 4. (d) 5. (c) 6. (a) 7. (c) 8. (b) 9. (d) 10. (a)
11. (a) 12. (c) 13. (c) 14. (b) 15. (c) 16. (b) 17. (b) 18. (c) 19. (d) 20. (d)
21. (b) 22. (c) 23. (b) 24. (c) 25. (c) 26. (c) 27. (b) 28. (b) 29. (c) 30. (b)
31. (d) 32. (b) 33. (a) 34. (c) 35. (b) 36. (c) 37. (d) 38. (c) 39. (b) 40. (d)

1. (a,b,d) 2. (a,d) 3. (a,b,c) 4. (a,b,d) 5. (b,c) 6. (a,c,d) 7. (a.d) 8. (a,b,d)


9. (a,b,d) 10. (b,c) 11. (b,c,d) 12. (b,c,d) 13. (a) 14. (c) 15. (b) 16. (a)
17. (d) 18. (d) 19. (1) 20. (13) 21. (4) 22. (5) 23. (4) 24. (0)
25. A " P, R; B " Q, S; C " P, Q; D " R, S
26. A " Q, R, S ; B " P, Q, R; S; C " P, R; D " P, R, S
27. A " R, B " Q; C " P, D " S
28. P " 4; Q " 1; R " 1, 2, 4; S " 3

1. (d) 2. (b) 3. (a) 4. (a) 5. (d) 6. (b) 7. (c) 8. (c) 9. (a) 10. (b)
11. (a) 12. (c) 13. (b) 14. (b) 15. (c) 16. (c) 17. (d) 18. (b) 19. (b) 20. (b)
21. (d) 22. (a) 23. (c) 24. (a) 25. (d) 26. (d) 27. (a) 28. (c) 29. (a) 30. (a)
31. (b) 32. (a) 33. (b) 34. (b) 35. (d) 36. (a) 37. (a) 38. (b) 39. (b) 40. (a)
41. (c) 42. (b) 43. (c) 44. (c) 45. (b) 46. (b) 47. (b) 48. (a) 49. (a) 50. (d)
51. (c) 52. (b) 53. (b) 54. (a) 55. (a)
3.25

Hints and Solutions

[M(CO)5]
14. (b) 
Triamminebromochloronitroplatinum (IV)
1. (d)
Double salts completely ionise in aqueous chloride
solution. 15. (c)
Potassium dicyanodioxalatonickelate (II)
2. (d)
Oxalate is an anionic bidentate ligand. K4[Ni(CN)2(OX)2]
gly is an unsymmetrical and anionic ligand.
dien is a trtidentate ligand. 16. (d)
dipyridyl Hexaamminecoblat (III) Hexanitrito-O-
cobaltate (III)
17. (c)
N N
The formula of the complex is [Cr(H2O)5Cl]
is a neutral and symmetrical bidentate ligand. Cl2. H2O
3. (b) NO–2 and SCN– are ambidentate ligands. 18. (a)
Conductance depends on number of
4. (d) EDTA is a polydentate ligand hence, it is also ions:
a chelating ligand. EDTA–3 is a pentadentate
while EDTA–4 is a hexadentate ligand hence, it
is a flexidentate ligand.
5. (c) Oxalate is a bidentate ligand hence, it is a
chelating ligand.
6. (b) Ambidentate ligands are monodentate ligands.
7. (b) Mohr’s salt (FeSO4.(NH4)2SO4.6H2O) is a
double salt.
8. (b) [Fe(NH3)4Cl2]+ is a heteroleptic complex
because it has more than one type of ligands.
9. (c) In the structure, [H3N–CH2–CH2–NH3]2+, no +2
Ni = [Ar] 4s° 3d
8

atom has any lone pair. 3d 4s 4p


10. (b)
EAN of Mn in Mn(CO)5 is 35. To gain
stability, it will gain an e– hence, it will behave
as oxidizing agent.
11. (c) After pairing

2
dsp
Magnetic behaviour is diamagnetic
[Ni(CN)4]–4
Ni = [Ar] 4s23d8
3d 4s 4p
12. (d)
Sodium nitroprusside is
   +2     +1
Na2 [Fe(CN)5 NO] After pairing
EAN of Fe = 26 – 2 + 12 = 36
13. (c)
36 = 26 – 0 + 2x
x = 5 sp
3
3.26

21. (d)
[Ni(CN)5]–3, dsp3, (Inner orbital),
diamagnetic
[Fe(CN)6] , d2sp3, (Inner orbital),
–4

diamagnetic
[Fe(H2O)5, (NO)] SO4 sp3d2, (Outer orbital),
paramag netic
[Fe(CN)6] , d2sp3, (Inner orbital),
–3

paramagnetic
22. (a)
EAN of Fe in [Fe(CN)6]4– = 36
Hybridisation of Fe is d2sp3. It has 6 co-
ordinate bonds and 12 sigma bonds.
23. (d)
[NiCl4]–2 Tetrahedral, sp3, paramagnetic Æ
[Ni(CN)4]–2 Square planar, dsp2, diamagnetic


O......H – O

CH–
3 C = N N = C – CH3

Ni+2
CH–
3 C = N N = C – CH3

O – H ....O


"
4
Dt ª D0
9
4
D t ª ¥ 18000 = 8000 cm -1
9

"

28.7
= 0.2 mol
143.5

p*

&
& - -

&
3.27

5. (c)
[NiX4]–2 has zero unpaired e– hence, pairing of "
electrons ocurs. X is a strong field ligand.
+2 8
Ni = [Ar] 4s° 3d
3d 4s 4p
+2
" [Cu(NH 3 )4 ] SO 4

hc 1
2
dsp
∆0 = or ∆ 0 ∝
6. (a)
Magnetic moment = 2.83 BM λ λ
Number of unpaired e– = 2

"

7. (c)
In brown ring complex, [Fe(H2O)5NO] SO4, Fe
has three unpaired e– hence, magnetic moment
= 15 BM and, hybridization is sp3d2.
8. (b)
IUPAC name is.
Pentaaminenitrito-N-chromium (III) tetra-
chlorozincate (II)
This compound shows linkage and co- 1
ordination isomerism but not geometrical
3
isomerism.
9. (d)
Square planar complex Ma2b2 shows
geometrical isomerism.

"
"
"

"
"

+2 +1
Na 2 [ Fe(CN)5 NO]

"
"
"
"
" .
3.28

1. (a,b,d) Complex compound [Co(SCN)2 (NH3)4] Cl


Ma4b2 does not show optical isomerism
2. (a, d) Pentaamminecynidochromium (II) hexa
nitrito-N-irridate (III)
[Cr(NH3)5(CN)]3 [Ir(NO2)6]
Pentaaminecyanidochromium (III) hexa
nitrito-N-irridate (III)
[Cr(NH3)5(CN)]3 [Ir(NO2)6]2
3. (a,b,c)
+2
Ni = [Ar] 4s° 3d
8 [NiCl]64–
3d 4s 4p 4d

sp3d2

Number of unpaired e– = 2
Octahedral geometry
4–
+2 8 [Ni(CN)] 6
Ni = [Ar] 4s° 3d
3d 4p 4d
4s

sp3d2
Number of unpaired e– = 2
Octahedral geometry
Both complex have different ligands hence,
they have different colour and stability.
4 (a,b,d)

3.29

5. (b,c) Electronic Hybridization


configuration of central metal
d – d 0 3 d2sp3
d –d
8 10 sp3d2
In these cases, magnetic moment remains
same either in strong field or in weak field
ligand.
6. (a,c,d) In Ma4b2 complex both cis and trans-form are
optically inactive.
M(AA)3 complex shows optical isomerism
but not geometrical isomerism.
Mabcd tetrahedral complex shows optical
isomerism but Mabcd square planar complex
does not show optical isomerism.
7. (a,d) In sp3d2 or d2sp3 hybridization, d-orbitals
involved are dx2– y2 and dz2
Complex
8. (a,b,d) Potassium hexacyanidoferrate (III) is
K3[Fe(CN)6] Glycinate 2
9. (a,b,d) 2
3d 4s 4p
+2
[Ni(CO)4]Cl2 Ni = [Ar]
4s° 3d8
diamagnetic
2
dsp2
2
3d 4s 4p 4d
+2
[Ni(en)3] Ni = [Ar] paramagnetic
(NO2)2 4s° 3d8 3
3 2
sp d
2
3d 4s 4p
[V(NH3)6] Cl3 V+3 = [Ar] paramagnetic
4s° 3d2
2
d sp
3 Complex
3d 4s 4p [CoCl2Br2]2– Zero (tetrahedral complex)
+1 –3
[Mn(NO)3(CO)] Mn 2 = [Ar] diamagnetic
4s 3d8
3
[Rh(en)3]3+ Zero
sp
+
[Cr(en)2Br2] 2
10. (b, c)
S1 is incorrect but S2 and S3 are correct. [Pt(en)Cl2] Zero
11. (b, c, d) [Co(NH3)3(NO2)3]
I and II are geometrical isomers.
I and III are geometrical isomers.
II and III are identical.
12. (b, c, d)
Order of C – O bond order:
[V(CO)6]– < [Cr(CO)6] < [Mn(CO)6]+
Order of M – C bond order:
[V(CO)6]– > [Cr(CO)6] > [Mn(CO)6]+
13. (a)
If, there is no loss in weight with concentrated
H2SO4 then complex does not have water
of crystalisation. complex gives white
precipitate of AgCl.
3.30

[Ni(CO4)] [Ni(CN)4]
2–

+2 8
Ni = [Ar] 4s 3d 2 8
Ni = [Ar] 4s°3d
3d 4s 3d 3d 4s 4p

3d 4s 3d 3d 4s 3d
24. Metal ion having e– configuraton from d4 to d7
can form both low spin and high spin complexes. 3
sp dsp2

3d 4s 4p [Cu(CH3 CN)4 )] BF4

1
K 3 [Cu(CN)4 ]
d2Sp3

" " " "


" " " "
" " " "
1 1
" " " " [ Fe(H 2 O)5 NO] SO 4

1. (d) [NiCl4]2–
Ni+2 =
[Ar] 4s° 3d8
3d 4s 4p 3d 4s 4p


sp3
Tetrahedral geometry
2. (b) Hg[Co(SCN)4] 2
d Sp
3

Co+2 = [Ar] 4s° 3d7


3d 4s 4p 12. (c) Correct structure of EDTA is
HOOCH2C CH2COOH
N – CH2 – CH2 – N2

HOOCH2C CH2COOH
sp3
Number of unpaired = 3 e– 13. (b) The ionization isomer of [Cr(H2O)4Cl(NO2)]
Magnetic moment = 15 Cl is [Cr(H2O)4Cl2](NO2)
3. (a) [Co(NH3)4Br2] Cl and [Co(NH3)4BrCl] Br are 14. (b) [NiCl4]2– Tetrahedral
isonization isomers. Ma4b2 and Ma4bc complexes [Ni(CN)4]2– Square planar
show geometrical isomerism but not optical [Ni(H2O)6]2+ Octahedral
isomerism.
3.31

NiCl2{P(C2H5)2(C6H5)}2

5d 6s 6p
Paramagnetic Diamagnetic
+2 8
Ni = [Ar] 4s° 3d Ni+2 = [Ar] 4s° 3d8
3d 4s 4p 3d 4s 4p 2
dsp

sp 3 dsp 2
Tetrahedral Square planar

20. (b) IUPAC name of [Co(NO2)(NH3)5]Cl2 is:


Pentaammine nitro-N-cobalt (III) chloride

Ni+2 = [Ar] 4s° 3d8


3d 4s 4p

2(2+2) c 3d 4s 4p

d2sp3

(n–1)d ns np
3.32

[Fe(CN)6]
–3
[FeF6]–3
V I B G Y O R hc
+3
Fe = [Ar] 4s°3d
5
+3 5
 ;  
Fe = [Ar] 4s°3d
3d 4s 4p 3d 4s 4p 4d absorb wavelength ()
L1 Red
3 2
sp d
due to SFL, pairing occurs L2 Green
due to WFL, pairing does not occur.
paramagnetic complex
L3 Yellow
2 3
d sp
L4 Blue
paramagnetic complex

1
∆∝
λ

3d 4s 4p

d2sp3

eg

eg

"

"
"

a c
M.5NH3.ClSO4 a b a c

M M M

A B c d d b b
d
[M(NH3)5SO4]Cl [M(NH3)5Cl]So4
2+

AgNO3
[Cr(H2O)6 ]
BaCl2 +2
Cr =[Ar] 4s°3d4
AgCl BaSO4 3d 4s 4p 4d
(white) (white)

3 2
sp d

Number of unpaired e = 4
2+
[Fe(H2O)6 ]
+2
3d 4s 4p Fe =[Ar] 4s°3d 6
3d 4s 4p 4d

dsp 2
3 2
sp d
Number of unpaired e– = 4
Chapter

Key Concepts
Introduction: The process of extraction of metal from its (iii) Lighter impurity particles washed off with
ore in profitable manner is called metallurgy. water and heavier ore particles settle down at
(i) Mineral is a substance in which metal is the bottom
present in either native state or combined (iv) Usually employed for oxide and carbonate
state. ores.
(ii) ‘Ore’ is the mineral from which the metal can
(b) Magnetic separation:
be economically and conveniently extracted.
(i) Ore and gangue are separated, if only one of
(iii) ‘Gangue or matrix’ is the non metallic
them having magnetic property
impurities present in the ore.
(ii) Mixture of two minerals can also be separated
if one of them is non magnetic and the other
is magnetic.

(c) Froth floatation process:


(i) Employed for sulphide ores
(ii) It is based on the different wetting
Operation in which size reduction of large lumps to small
characteristics of the ore and gangue particles
pieces followed by finely ground material by the use of
with water and oil.
crushers and grinders.
(iii) Usually ore particles are making as aerofillic
and gangue particles as aerophobic by using
different reagents.
Operation in which the removal of impurities (gangue)
(iv) Ore particles raised to the surface along with
from ore by the following methods:
air bubbles and collected at the surface where
(a) Levigation or gravity separation: as gangue particles are wetted and settled
(i) This method is based on the difference in down at the bottom of the tank.
densities of the ore particles and gangue (v) Reagents acts as frothing agents (pine oil),
particles. collectors (ethyl xanthate and potassium
(ii) The powdered ore with gangue particles ethyl xanthate), activators (copper sulphate)
introduced in the running steam of water. and depressors (sodium cyanide, alkali).
4.2

(d) Leaching:
2ZnS + 3O2 2ZnO + 2SO2
(i) Chemical method of concentration 2PbS + 3O2 2PbO + 2SO2
(ii) Selective dissolution of ore in strong reagents (b) Metal sulphides converted into sulphates
where as gangue particles are undissolved
and gets separated.
PbS + 2O2 PbSO4
(iii) Employed for concentrating ores of ZnS + 2O2 ZnSO4
aluminium, silver gold etc. (c) Metal sulphides converted into chlorides
Working of the concentrated ore:
Ag2S + 2 NaCl 2AgCl + Na2S
(i) It depends upon the nature of the ore as well as (d) Conversion of amalgams
the nature of impurities.
AgCl + 2Hg Ag – Hg + HgCl
(ii) Processes involved:
(a) Conversion of the concentrated ore to its Conversion of the oxide to metallic form:
oxide form. The roasted or calcined ore is converted into metallic form
(b) Conversion of the oxide to the metallic form through reduction by using different reducing techniques
(c) Hydrometallurgy which will depends upon the nature of the ore, some of the
methods are mentioned below.
Conversion of the concentrated ore into its oxide form:
(i) Reduction by carbon (smelting):
(i) Calcination:
(a) Ore is heated in absence of air to remove The oxides of less electropositive metals like
water or CO2 from hydrated oxides or Pb, Zn, Fe, Sn, Cu etc, are reduced by strongly
carbonates respectively. heating with coal or coke.
(b) Process temperature is below the melting (a) Reduction of the oxide with carbon at high
points of treated ores. temperature is known are as smelting
(c)
During calcination moisture, volatile (b) Flux is added smelting, which reduces the
impurities are removed there by one becomes melting point of impurities to form as easily
porous. fusible substance called as ‘slag’ and can be
Example: separated easily because of its lower density.


Al2O3. 2H2O Al2O3 + 2H2O (c) Selection of flux depends upon nature of
impurity present. Its impurity is acidic, basic
2Fe2O3. 3H2O 2Fe2O3 + 3H2O flux is employed and vice versa.
CaCO3 CaO + CO2 (d) Smelting is usually carried out in blast
MgCO3 MgO + CO2 furnaces or reverberatory furnace.
(ii) Roasting: (ii) Reduction by aluminium (Alumino-thermite
(a) Ore is heated strongly with other substances, reduction):
usually with oxygen (a) Aluminium acts as reducing agent due to its
(b) Employed for sulphide ores high electropositive nature.
(c) Process temperature is below the melting (b) Oxides such as Cr2O3, Mn3O4 are reduced
points of treated ore by this method because carbon or CO are not
(d) Chemical conversion of ore is taking place. efficiently reduced.
(e) Some of the impurities removed as volatile (c) The process is also known as “Gold Schmidt
substances. thermite process”.


S + O2 SO2 (iii) Reduction by heating in air (Auto-reduction):
(a) Employed for metals of less active such as
4As + 3O2 2As2O3
Hg, Cu and Pb
P4 + SO2 2P2O5 (b) Due to unstable nature in the oxide form
Examples: at high temperature, no reducing agent is
(a) Conversion of metal sulphides into oxides required for their reduction .
4.3

Example : of zinc powder.


HgS + O2 ∆
→ Hg + SO2 Ag2S + 4 NaCN 2Na [Zn(CN)2] + Na2S
∆ Sodium dicyanoargentate (I)
2Cu2S + 3O2 
→ 2Cu2S + 2O2 (Roasting)
2Na[Ag(CN)2] + Zn Na2 [Zn(CN)4] + 2Ag .
2Cu2O + Cu2S ∆
→ 6C + SO2 (Auto-reduction)
(iv) Electrolytic reduction (Electro-metallurgy):
(a) Employed for highly electropositive metals
such as Na, K, Ca, Mg, Al, etc. Refining or purification:
(b) These metals are extracted by the electrolysis (i) The metals after reduction process consists of
of their oxides hydroxides or chlorides in number of impurities like Si, P, Slag, oxides,
fused state. other metals etc.
Example: (ii) Removal of all these impurities to get pure

On fusion: metal is called as refining
(iii) Methods as refining are:
NaCl  Na+ + Cl- (ions become mobile)
On electrolysis: (a) Liquation:
At cathode: (i) This is based on the principle of difference in
melting points of metal and impurity.
Na+ + e- Na
(ii) Employed for purification of low melting
At anode: point metals like Pb, Sn, etc.
Cl- Cl + e-
(b) Distillation process:
Cl + Cl Cl2
(i) This is based on difference in boiling points
(c) Aluminium is obtained by the electrolysis of metals and impurities.
of electrolyte which consists of mixture
(ii) Employed for low boiling point metals like
of alumina, cryolite and calcium fluoride.
Zn, Hg etc.
(cryolite and fluorospar are added to reduce
melting point of electrolyte and to increase (c) Oxidation process:
conductivity). (i) This is a selective oxidation method
(v) Other methods are: (ii) Used for refining those metals in which
(a) Reduction by carbon monoxide (employed the impurities have greater tendency to get
for iron (III) oxide) oxidized than the metals it self
(b) Reduction by water gas (employed for nickel (iii) The impurities converted into oxide and
oxide) skimmed off from the metal
(iv) Various oxidation processes used for different
(c) Amalgamation method (employed for noble
metals bear different names, eg, poling,
metals)
pudding, bessemerisation and cupellation
Hydrometallurgy (Reduction by precipitation): (for Ag)
(i) Process in which more electropositive metals (d) Electrorefining:
displace less electropositive metals from salt
(i) Employed for refining of highly electro
solution.
positive metals like Al, Cu, Ag, Zn, Sn, Cr
(ii) First the concentrated ore is dissolved and Ni
in strong reagent and removed insoluble
(ii) Impure metal is made as anode, thin pure
precipitates.
metal sheet is kept as cathode and the
(iii) Now the metal is precipitated by addition of electrolyte is comprising with soluble salt
more electropositive metal. solution of the metal.
Example: (iii) On passing the electric current, pure metal
Silver sulphide dissolved in sodium cyanide which forms a from the anode dissolved and is deposited on
soluble complex, then silver is precipitated by the addition the cathode.
4.4

(iv) The soluble impurities go into the solution


523K
(remains in the solution after the completion Ti (s) + 2I2 (g) TiI4 (g)
of refining) while the insoluble impurities
settle down below the anode as ‘anode mud’.  
1700K
(e) Van-Arkel process: Til4 (g) Ti (s) + 2I2 (g)
(i) Employed to get metal in very pure form of (f) Zone refining:
small quantities. (i) Employed for metals which require very high
(ii) In this method, the metal is converted into purity like semi conductors.
volatile unstable compound (eg iodide), and (ii) The method is based on the principle that an
impurities are not affected during compound impure metal on solidification will deposit
formation. crystals of pure metal and the impurities will
(iii) The compound thus obtained is decomposed remain behind in the molten part of the metal.
to get the metal. (iii) Used to purify the elements such as silicon,
(iv) Employed for purification of metals like germanium, etc.
titanium and zirconium

Solved Examples
1. Which of the following is not a concentration volatile impurities are removed and process is
technique? carried in absence of air or in limited supply of
Levigation air.
Froth floatation 4. The purpose of adding Na3AlF6 to Al2O3 during
Leaching electrolysis is:-
Calcination To decrease melting point of Al2O3
Sol.(d) Concentration is the method employed to remove To increase conductivity of electrolyte
gangue materials by mechanical separation. In To provide reducing conditions in the bath
calcination, volatile impurities are removed by Both (a) and (b)
heating ore below its melting point in absence or Sol.(d) Al2O3 is a poor conductor of electricity and
limited supply of air. having very high melting point. So, to increase
2. The ores that are concentrated by froth floatation the conductivity Na3AlF6 is added and to decrease
method are:- the melting point Na3AlF6 and CaF2, AlF3 are
Carbonates added so that melting point of electrolyte comes
Sulphides to around 930°C.
Oxides 5. During the process of electro refining of copper,
Phosphates some metals present as impurity settle down as
anode mud. These are:
Sol.(b) In floatation process, the ore particles should
Sn and Ag
have aerofillic in preference to gangue particles.
Sulphide ores having this character. Pb and Zn
Ag and Au
3. Calcination is the process in which:
Fe and Ni
Heating the ore in presence of air
Heating the ore in presence of sulphur Sol.(c) In anode mud, less electropositive elements are
present. Ag and Au are less electropositive than
Heating the ore in absence of air
Cu.
Heating the ore in presence of chlorine
6. The method of Zone refining of metals is based
Sol.(c) Calcination is the process in which moisture and on the principle of:
4.5

Greater solubility of the impurities in the Poling


molten state than in the solid. Electrolytic process
Greater solubility of pure metal than that of Sol.(a) In cupellation process, impure metal is heated
impurity. with borax and silica. Impurities are removed as
Higher melting point of the impurity than their oxide.
that of pure metal.
9. What is the appropriate process for the extraction
Greater noble character of the solid metal of lead from galena?
than that of the impurity.
(A) Froth floatation (B) Calcination (C) Roasting
Sol.(a) Zone refining is used when pure metal has greater (D) Self reduction (E) Electrolytic reduction
melting point than impurities and impurities have (F) smelting (G) Leaching
greater solubility in molten state than in the solid.
G " B" D " E
7. Which of the following is not correctly matched? A"C"D
ZnCO3 " Zn (Calcination followed by A"C"E
smelting)
A"C"F
Al(OH)3 " Al (Calcination followed by
smelting) Sol.(b) Galena is a sulphide ore (PbS). It is concentrated
by froth floatation. After partial roasting, self
PbS " Pb (Partial roasting followed by self
reduction process is used.
reduction)
Cu2S " Cu (Complete roasting following by 10. In purification of bauxite ore, it is mixed with coke
carbon reduction) and heated at 1800°C in presence of nitrogen.
This is:
Sol.(b) ZnCO3 " Zn
Hall’s process
ZnCO3 ∆
→ ZnO + CO2- (Calcination) Serpeck’s process
ZnO + C ∆
→ Zn + CO- (Smelting) Baeyer’s process
Electrolytic reduction
Al(OH)3 " Al Sol.(b) Serpeck’s process:-

Al(OH)3 
→ Al2O3 + H2O- (Calcination) Al2O3. 2H2O + 3C + N2 ∆
→ 2AlN+3CO+2H2O
Al2O3 ∆
→ Al (electrolytic reduction) AlN + 3H2O " Al(OH)3. + NH3-
2Al(OH)3 ∆
→ Al2O3 + 3H2O-
PbS " Pb 11. Select the reaction which does not occur in

PbS + O2 
→ PbO + PbSO4 + SO2- (Partial Bessemer’s convertor:
roasting) FeS + O2 " FeO + SO2
PbO + PbS ∆
→ Pb + SO2- 2Cu2S + 3O2 " 2Cu2O + 2SO2
(Self FeO + SiO2 " FeSiO3
reduction) 2CuFeS2 + O2 " Cu2S + 2FeS + SO2
Or PbS + PbSO4 ∆→ Pb + SO2-

Sol.(d) The reaction,
Cu2S " Cu 2CuFeS2 + O2 ∆
→ Cu2S + 2FeS + SO2
Cu2S + O2 ∆
→ Cu2O + SO2- (Complete represents partial roasting of ore. It is carried out
roasting) in reverberatory furnace.
Cu2O + C ∆
→ Cu + CO- (Carbon reduction) 12. Which of the following process is not involved in
8.
When an impurity in metal has greater affinity for the extraction of Ag from argentite?
oxygen and is more easily oxidized than metal Hydrometallurgy
itself then the metal is refined by: Formation of cyanide
Cupellation Levigation
Zone refining Reduction by Zn metal
4.6

Sol.(c) Levigation is a concentration process, used Removal of Pb impurity from impure


for oxide ores. Concentration process used for Ag " Cupellation
argentite ore is cyanide leaching. Obtaining wrought iron from cast
Ag2S + CN- + H2O + O2 " [Ag(CN)2]- iron " Bessemerisation
Reduction process used is Hydrometallurgy Refining of Nickel " Mond’s process
(reduction by Zn). Sol.(c) Wrought iron is obtained from cast iron in
-
[Ag(CN)2] + Zn " Ag + [Zn(CN)4] -2 reverberatory furnace.

13. Select the incorrect statement: 15.


Which of the following statement is not correct?
Based on reactivity series, occurrence of Tin stone is separated from non-magnetic
certain elements takes place in native state. impurity of wolframite by electrolytic
separation.
Due to the basic nature of oxides of alkaline
earth metals, they combine with atmospheric CO acts as reducing agent is most parts of
acidic oxides giving salts. blast furnace during extraction of iron.
Chalcopyrite ore contains both Cu and Fe. A silicate slag is obtained during the
extraction of Cu from CuFeS2 and iron from
Both anglesite and cassiterite contain Pb.
haematite.
Sol.(d) Due to less reactivity, Au occurs in native state. In Puddling process, impurities present in
Due to basic nature of alkaline earth metals cast iron are oxidized by haematite.
oxides, they occur in the form of their salts.
Anglesite is PbSO4 and cassiterite is SnO2. Sol.(a) Tin stone (SnO2) is non-magnetic while
wolframite (FeWO4) is magnetic Tin stone is
14. Which of the following is not correctly matched? separated from wolframite by electromagnetic
Removal of oxide impurity from impure separation.
Cu " Poling process

Exercise
5. Which one of the following is not an ore of
aluminum?
1. Three most occurring elements into the earth (a) Bauxite (b) Corundum
crust are: (c) Epsomite (d) Cryolite
(a) O, Si, Al (b) Si, O, Fe 6. Which one contains both Ca and Mg?
(c) Fe, Ca, Al (d) Si, O, N (a) Limestone (b) Dolomite
2. An example of an oxide ore is (c) Chalk (d) Feldspar
(a) Bauxite (b) Malachite 7. Which of the following minerals does not contain
(c) Zinc blende (d) Feldspar iron?
3.
Which of the following set of elements mostly (a) Magnetite (b) Magnesite
occur as sulphide ores? (c) Haematite (d) Limonite
(a) Zn, Cu, Na (b) Zn, Cu, Pb 8. Metals which exist as native ore?
(c) Fe, Al, Ti (d) Cu, Ag, Au (a) Ni, Pt (b) Zn, Cd
4. Which of the following is not an ore of (c) Pt, Au (d) Sn, Pb
Magnesium? 9. Which of the following contain Fe as well as Cr?
(a) Carnallite (b) Magnesite (a) Wolframite (b) Chromite
(c) Dolomite (d) Gypsum (c) Pyrolusite (d) Chalcopyrite
4.7

10. Which ore is concentrated by ‘wetting by oil’? (d) Oxide ores are concentrated by froth
(a) Oxide ore (b) Sulphate ore floatation process.

(c) Carbonate ore (d) Sulphide ore 19. Purification of silicon element used in
semiconductors is done by:
11. Haematite ore is concentrated by:
(a) Zone refining
(a) Gravity separation method
(b) Heating
(b) Froth floatation process
(c) Froth floatation
(c) Amalgamation
(d) Heating in vacuum
(d) Hand picking
20. Which one of following beneficiation processes is
12. In the extraction of copper from its sulphide ore, used for the mineral Al2O3.2H2O?
the metal is formed by reduction of Cu2O with:
(a) Froth floatation (b) Leaching
(a) FeS (b) CO
(c) Liquation (d) Magnetic separation
(c) Cu2S (d) SO2
21. In blast furnace, maximum temperature is in:
13. The element which is recovered from electrolytic
process is: (a) Zone of fusion

(a) Iron (b) Lead (b) Zone of combustion

(c) Aluminium (d) Zinc (c) Zone of slag formation

14. Refining of silver can be done by: (d) Zone of reduction

(a) Cupellation 22. When a metal is to be extracted from its ore and
if the gangue associated with the ore is silica,
(b) Electrorefining then:
(c) Both (a) and (b) (a) An acidic flux is needed
(d) None of these (b) A basic flux is needed
15. Impure aluminium is purified by: (c) Both acidic and basic fluxes are needed
(a) Baeyer’s process (d) Neither of them is needed
(b) Hall’s process 23 Silver containing lead as an impurity is removed
(c) Hoop’s Process by:
(d) Serpeck’s process (a) Poling (b) Cupellation
16. In the alumino-thermite process, Al metal acts as: (c) Lavigation (d) Distillation
(a) Oxidizing agent 24. Cassiterite is concentrated by:
(b) Reducing agent (a) Levigation
(c) Catalyst
(b) Electromagnetic separation
(d) Flux
(c) Froth-floatation
17. Slag is formed by reaction between:
(d) Liquification
(a) Impurities and coke
25. For which ore of the metal, froth floatation
(b) Impurities and ore
method is used for concentration?
(c) Impurities and flux
(a) Horn silver (b) Bauxite
(d) Flux and coke
(c) Cinnabar (d) Haematite
m18. Which of the following statement is correct?
(a) All ores are minerals 26. In which of the following minerals, aluminium is
not present?
(b) All minerals are ores
(c) Calcination is generally carried out in blast (a) Cryolite (b) Mica
furnace (c) Feldspar (d) Fluorspar
4.8

27. Gravity separation process may be used for the 5. Which one of the following reactions is an
concentration of: example for calcination process?
(a) Chalcopyrite (b) Bauxite (a) 2Ag + 2HCl + (O) " 2AgCl + H2O
(c) Haematite (d) Calamine (b) 2Zn + O2 "2ZnO
28. Electrolytic reduction method is used in the (c) 2ZnS + 3O2 " 2ZnO + 2SO2
extraction of:
(d) MgCO3 " MgO + CO2
(a) Highly electropositive elements
6. Cupellation process is used in the metallurgy of:
(b) Highly electronegative elements
(a) Cu (b)Ag
(c) Transition metals
(c) Zn (d)Al
(d) Noble metals
7. In electrorefining of metal the impure metal
29. Bauxite is leached with: is made the anode and a strip of pure metal the
(a) KCN (b) NaCN cathode during the electrolysis of an aqueous
solution of a complex metal salt. This method
(c) NaOH (d) Na2CO3
cannot be used for refining of:
30. In the equation: (a) Silver (b) Copper
4M + 8CN- + 2H2O + O2 " 4[M(CN)2]- + 4OH- (c) Aluminium (d) Gold
Identify the metal M: 8. Among the following groups of oxides, the group
(a) Cu (b) Fe that cannot be reduced by carbon to give the
respective metals is:
(c) Au (d) Zn
(a) Cu2O, SnO2 (b) Fe2O3, ZnO
(c) CaO, K2O (d) PbO, Fe3O4
9. The reason for floating of ore particles in
1. Which of the following process is not a physical concentration by froth floatation process is that:
process of separation? (a) They are light
(a) Levigation (b) They are insoluble
(b) Magnetic separation (c) They are charged
(c) Leaching (d) They are hydrophobic
(d) Froth floatation 10. Which of the following statements is correct
2. In the extraction of copper from copper pyrites, regarding the slag obtained during the extraction
iron is removed as: of a metal like copper or iron?
(a) FeSO4 (b) FeSiO3 (a) The slag is lighter and has lower melting
(c) Fe3O4 (d) Fe2O3 point than the metal
(b) The slag is heavier and has lower melting
3. In zone refining method, the molten zone:
point than the metal
(a) Consists of impurities only
(c) The slag is lighter and has higher melting
(b) Consists more impurity than the original
point than the metal
metal
(d) The slag is heavier and has higher melting
(c) Contains the purified metal only
point than the metal
(d) Moves to either side
11. Consider the following reactions at 1000°C.
4. Bauxite, Siderite and argentite are respectively:
1
(a) Sulphide, oxide and carbonate ore (i) Zn(s) + O2 (g) "ZnO(g); DG°= -360 kJ
2
(b) Oxide, carbonate and sulphide ore mol-1
(c) Oxide, oxide and sulphide ore 1
(ii) C(s) + O2 (g) " CO(g); DG°= -460 kJ
(d) Oxide, sulphide and oxide ore 2
mol-1
4.9

(a) ZnO is more stable than CO (a) In froth floatation process, pine oil decreases
(b) ZnO can be reduced to Zn by C the surface tension of the solution.
(c) ZnO and CO are formed at equal rate (b) In Poling refining, non volatile oxides are
removed as sucm.
(d) ZnO can not be reduced to Zn by C
(c) Dolomite ore can be considered as ore of
12. DG° vs T plot in the Ellingham’s diagram slopes both Ca and Mg.
downward for the reaction: (d) Aqueous Al2(SO4)3 is used for electrorefining
1 of Al.
(a) Mg + O2" MgO
2 19. Which is incorrectly matched for refining of
1 elements?
(b) 2Ag + O2" Ag2O (a) Si, Ge from impurities zone refining
2
1 method
(c) C + O2" CO (b) Sn refined from impurities with high boiling
2 point poling method
1 (c) Zn, Cd from impurities with high boiling
(d) CO + O2 " CO2
2 point distillation
13. The incorrect match in the following is:
(d) Al from impurities Cu, Fe hoop method
(a) Purification of Al metal : Baeyer’s method
20. A mixture of alumina and coke is heated in a
(b) Polling : Reduction of Cu2O current of nitrogen to about 1800°C. And the
(c) FeCr2O4 (Chromite ore) : NaOH/ Na2CO3 product obtained is treated with water. A gas is
evolved. The gas is:
(d) Ag : Mac Arthur cyanide process
(a) N2 (b) N2O
14. An ore after levigation is found to have acidic
impurities. Which of the following can be used as (c) NH3 (d) NO
flux during smelting operation? 21. The slag obtained during the extraction of copper
(a) H2SO4 (b) CaCO3 pyrites is composed mainly of:
(a) SiO2 (d) Both CaCO3 and SiO2 (a) MgSiO3 (b) CuSiO3
15. Which of the following metals never occurs in (c) FeSiO3 (d) CuFeS2
free sate in nature? 22. Consider the following at 1000°C
(a) Gold (b) Copper (A) 2Zn (s) + O2 (g) " 2ZnO(g); DG° = –360kJ
(c) Silver (d) Sodium mol-1
16. Select the incorrect statement in the following: (B) 2C (gr) + O2 (g) " 2CO(g); DG° = –500kJ
(a) Silica present in bauxite is removed by mol-1
addition of NaOH(aq.)
Choose the correct statement at 1000°C:
(b) Silica present in haemetite is removed by the
(a) Zinc can be oxidized by carbon monoxide
addition of CaCO3 during smelting
(b) Zinc oxide can be reduced by graphite
(c) Fe2O3 present in bauxite is removed by
(c) Both statements (a) and (b) are true
Bayer’s process
(d) Both statements (a) and (b) are False
(d) Magnetic separation is used for cassitarite
23. When ZnS and PbS minerals are present together,
17. In which of the following process, roasting and
them NaCN is added to separate them in the froth
self reduction is required?
floatation process as a depressant because:
(a) Cu2S " Cu2O (a) Pb(CN)2 is precipitated while no effect on
(b) CaCO3" CaO ZnS.
(b) ZnS forms soluble complex Na2[Zn(CN)4].
(c) ZnS " Zn
(c) PbS forms soluble complex Na2[Pb(CN)4].
(d) PbS " Pb
(d) It decreases the floatation property of PbS by
18. Which of the following statement is not correct? making it hydrophilic
4.10

24 The materials mixed, before haemetite ore is (c) Zinc blende and iron pyrites are sulphides.
subjected to smelting in the extraction of iron, (d) Malachite and azurite are ores of copper.
are:
(a) Coke and silica
(b) Coke and limestone
(c) Limestone and silica
ONE OR MORE THAN ONE CORRECT TYPE
(d) Cock, silica and limestone
1. Which of the following is (are) manufactured by
25. Which of the following reaction forms the basis the electrolysis of their fused salts?
of Goldschmidt alumino-thermite process?
(a) Copper (b) Sodium
(a) 2Al + N2 " 2AlN
(c) Aluminium (d) Platinum
(b) 2Al + 3Cl2 " 2AlCl3
2. Complexes formed in the cynide process are:
(c) 2Al + 6HCl " 2AlCl3 + 3H2 (a) [Au(CN)2]- (b) [Ag(CN)2]-
(d) 2Al + Fe2O3 " Al2O3 + 2Fe (c) [Cu(CN)4]2- (d) [Zn(CN)4]2-
26.
In the extraction of copper, metal is formed in the 3. Which of the following process (es) occur(s) during
Bassemer converter due to reaction: the extraction of copper from chalcopyrites?
(a) Cu2S + 2Cu2O "6Cu + SO2 (a) Froth floatation (b) Roasting
(b) Cu2S " 2Cu + S (c) Bessemerisation (d) Calcination

(c) Fe + Cu2O " 2Cu + FeO 4. Calcium silicate (slag) formed in the slag
formation zone in extraction of iron from
(d) 2Cu2O " 4Cu + O2 Haematite ore:
27. The extraction of zinc from zinc blende is (a) Does not dissolve in molten iron.
achieved by: (b) Being lighter floats on the molten iron.
(a) Electrolytic reduction (c) Is used in cement industry and as building
(b) Roasting followed by reduction with carbon material.
(c) Roasting followed by reduction with another (d) Prevents the re-oxidation of molten iron.
metal 5. Liquation process may be applied for the
(d) Roasting followed by self-reduction purification of:
28. Which method is not correct given for refining of (a) Copper (b) Tin
crude metals? (c) Iron (d) Zinc
(a) Distillation : Zinc and mercury
6. Roasting of copper pyrites is done:
(b) Liquation : Tin
(a) To remove moisture.
(c) Van Arkel : Zirconium
(b) To oxidize free sulphur and antimony.
(d) Mond process : Lead
(c) To convert pyrites completely into Cu2O and
29. Poling process is used for: FeO.
(a) The removal of Cu2O from Cu (d) To remove volatile organic impurities.
(b) The removal of Al2O3 from Al
7. Select the correct statement:
(c) The removal of Fe2O3 from Fe
(a) Dolomite contains both magnesium and
(d) All of these calcium.
30. Among the following statements, the incorrect (b) Extraction of lead from galena involves
one is roasting in limited supply of air at moderate
(a) Calamine and siderite are carbonates. temperature followed by self reduction at
(b) Argentite and cuprite are oxides. higher temperature (to melt the charge).
4.11

(c) Extraction of zinc from Zinc blende involves (d) Any metal will not reduce the oxide of other
roasting followed by reduction with carbon. metals which lie above it in the Ellingham
(d) The chemical composition of ‘slag’ formed diagram.
during the extraction of iron and copper is PARAGRAPH BASED QUESTIONS
FeSiO3.
Paragraph # 1 (Q. 14 to 15)
8. Which of the following is a correct statement?
Metallic gold frequently is found in
(a) Calamine is the ore of zinc. aluminosilicate rocks and it is finely dispersed
(b) Proustite is the ore of silver. among other minerals. It may be extracted by
(c) Cassiterite is the ore of tin. treating the crushed rock with aerated Sodium
(d) Diaspore is the ore of aluminium. cyanide solution. During this process metallic
gold is slowly converted to [Au(CN)2]-, which
9. Froth floatation:
is soluble in water. After equilibrium has been
(a) Is a physical method of separating mineral reached, the aqueous phase is pumped off and
from the gangue the metallic gold is recovered from it by reacting
(b) Is a method of concentration of ore depending the gold complex with zinc which is converted to
on the difference in wetability of gangue and [Zn(CN)4]2-gold in nature is frequently alloyed
the ore particles. with silver which is also oxidized by aerated
(c) Is used for the concentration of sulphide ores sodium cyanide solution.
(d) Is a method in which impurities sink to 14. The correct ionic reaction for the process is:
the bottom and ore particles pass on to the
(a) 4Au + 8CN- + 2H2O + O2(air) " 4[Au(CN)2]-
surface with froth.
(soluble) + 4OH-
10. Which of the following are correct processes? (b) Au + 2CN- " [Au(CN)2]-
(a) Fe + Al2O3 " 2Al + Fe2O3 (c) Zn + 2CN- " [Zn(CN)2]-
(b) ZnO + C " Zn + CO (d) Zn + 4CN- " [Zn(CN)4]2-
(c) Cr2O3 + 2Al " 2Cr + Al2O3 15. The process described above in the paragraph
(d) 2[Ag(CN)2]- + Zn " 2Ag + [Zn(CN)4]2- represents:
11. Which of the following reactions occur during (a) Ore concentration
calcination? (b) Pyrometallurgical extraction
(c) Hydrometallurgical extraction
(a) CaCO3 " CaO + CO2
(d) Purification of metal
(b) 2Al(OH)3 " Al2O3 + 3H2O
Paragraph # 2 (Q. 16 to 17)
(c) 4FeS2 + 11O2 " 2Fe2O3 + 8SO2
Extraction of copper is done using copper pyrites.
(d) Cu2S + 2CuO " 4Cu + SO2
After roasting, the ore is mixed with silica and
12. Silver containing lead as an impurity is not coke and then smelted in a blast furnace. The
purified by: matte obtained from the blast furnace is charged
(a) Poling (b) Cupellation into a silica-lined converter. Some silica is also
assed, and a hot air blast is blown into the mixture
(c) Levigation (d) Distillation
to obtain blister copper, which is purified by
13. Select the correct statements for Ellingham electrorefining.
diagram:
16. The chemical formula of copper pyrites is:
(a) Any metal will reduce the oxide of other
metals which lie above it in the Ellingham (a) CuFeS2 (b) Cu2O
diagram. (c) Cu2S (d) CuCO3. Cu(OH)2
(b) According to Ellingham diagram, Al will not 17. The chemical composition of the slag formed
reduce MgO at temperature below 1350°C during smelting is:
(c) According to Ellingham diagram, Al will (a) CuSiO3 (b) FeSiO3
reduce MgO at temperature below 1350°C (c) CaSiO3 (d) Cu2O. SiO2
4.12

Paragraph # 3 (Q. 18 to 19) (a) Bauxite (b) Corundum


Extraction of aluminium can be understood by: (c) Dolomite (d) Malachite
Pure alumina Concentration of ore by Bauxite
(e) Magnetite (f) Pyrolusite
(Al2O3) Chemical method (Al2O3.2H2O) (g) Argentite (h) Horn silver
(i) Quartz (j) Cryolite
Reduction
of alumina
(k) Siderite (l) Zincite
By electrolysis Electrolysis (m) Calamine (n) Sylvine
Impure Al Pure Al
(o) Carnellite
Using suspended graphite rods an anode and 23. Poling process is applied when impurity is a
C-lining inside the Fe container. compound of a metal and a non-metal. Atomic
18. The purpose of adding cryolite is: number of non-metal is…………………
(a) To remove the impurities as slag MATCH THE COLUMN TYPE QUESTIONS
(b) To lower the melting point of Al2O3 24.
(c) To decrease the electrical conductivity of
pure aluminium
(d) To increase the Al percentage in the yield
19. The molten electrolytes contain Na+, Al3+ and
Ca2+ but only Al gets deposited at Cathode
because,
(a) Standard reduction potential of Al is more
than that of Na and Ca
(b) Standard oxidation potential of Al is more
than that Na and Ca
(c) Graphite reacts only with Al3+ and not with 25.
Na+ and Ca2+ Column I Column II
(d) Discharge potential of Al3+ is higher than Na+ A Copper pyrites (a) Fluoride ore
and Ca2+
B Cryolite (b) Sulphate ore
INTERGER TYPE QUESTIONS
C Rock salt (c) Oxide ore
20. Number of metals among following which are
D Alumina (d) Sulphide ore
obtained by electrometallurgy in molten state are:
E Dolomite (e) Chloride ore
Li, Ba, Na, Al, Fe, Cu, Pb, Sn, Ag, Au, Zn, Ca,
Mg F Gypsum (f) Carbonate ore
21. How many of the following process of refining is/
are chemical methods:
(a) Liquation process
(b) Fractional distillation process
(c) Zone refining method 1. Electrolytic reduction of alumina to aluminium
(d) Chromatographic method by Hall – Heroult process is carried out:
(e) Cupellation (a) In the presence of NaCl
(f) Poling process (b) In the presence of fluorite
(g) Hoop’s process (c) In the presence of cryolite which forms a
(h) Kroll’s process melt with lower melting temperature
(d) In the presence of cryolite which forms a
22. How many of the following minerals are oxides
melt with higher melting temperature
of metals/ metalloids.
[IIT-2000]
4.13

2. The chemical composition of ‘slag’ formed during 8. Extraction of zinc from Zinc blende is achieved
the smelting process in the extraction of copper is by:
(a) Cu2O + FeS (b) FeSiO3 (a) Electrolytic reduction
(c) CuFeS2 (d) Cu2S + FeO (b) Roasting followed by reduction with carbon
[IIT-2001] (c) Roasting followed by reduction with another
metal
3. Which of the following process is used in
extractive metallurgy of magnesium? (d) Roasting followed by self-reduction
(a) Fused salt electrolysis [IIT-2007]
(b) Self - reduction 9. Native silver metal forms a water soluble complex
(c) Aqueous solution electrolysis with a dilute aqueous solution of NaCN in the
(d) Thermite reduction presence of
[IIT-2002] (a) Nitrogen (b) Oxygen
4. In the process of extraction of gold, (c) Carbon dioxide (d) Argon
 O [IIT-2008]
2
- -
Roasted gold ore + CN + H2O [X] + HO 10. Match the conversions in Column I with the
         [X] + Zn [Y] + Au type(s) of reaction(s) given in Column II.
Identify the complexes [X] and [Y] Column I Column II
- 2-
(a) X = [Au(CN)2] , Y = [Zn(CN)4] A PbS " PbO (a) Roasting
3- 2-
(b) X = [Au(CN)4] , Y = [Zn(CN)4] B CaCO3 " CaO (b) Calcination
- 4-
(c) X = [Au(CN)2] , Y = [Zn(CN)6] C ZnS " Zn (c) Carbon reduction
- 2-
(d) X = [Au(CN)4] , Y = [Zn(CN)4] D Cu2S " Cu (d) Self-reduction
[IIT-2003]
5.
The methods chiefly used for the extraction of Passage (Q. 11 to 13)
lead and tin from their ores are respectively: Copper is the most noble of the first row transition
(a) Self-reduction and carbon reduction metals and occurs in small deposits in several
countries. Ores of copper include chalcanthite
(b) Self-reduction and electrolytic reduction
(CuSO4.5H2O), atacamite (Cu2Cl(OH)3),
(c) Carbon reduction and self-reduction cuprite (Cu2O), copper glance (Cu2S) and
(d) Cyanide process and carbon reduction malachite (Cu2(OH)2CO3). However, 80% of
[IIT-2004] the world copper production comes from the ore
6. Which ore contains both iron and copper: chalcopyrite (CuFeS2). The extraction of copper
from chalcopyrite involves partial roasting,
(a) Cuprite (b) Chalcocite
removal of iron and self–reduction. [IIT-2010]
(c) Chalcopyrite (d) Malachite
11. Partial roasting of chalcopyrite produces:
[IIT-2005] (a) Cu2S and FeO
7. Match entry in Column (I) is in some way related (b) Cu2O and FeO
to the entries in columns (II) (c) CuS and Fe2O3
Column I Column II (d) Cu2O and Fe2O3
A Self - reduction (a) Lead 12. Iron is removed from chalcopyrite as:
B Carbon reduction (b) Silver (a) FeO (b) FeS
C Complex formation and (c) Copper (c) Fe2O3 (d) FeSiO3
displacement by metal 13. In self-reduction, the reducing species is:
D Decomposition of iodide (d) Boron (a) S (b) O2-
[IIT-2006] (c) S2- (d) SO2
4.14

14. Oxidation states of the metal in the minerals 20 Calcination is the process in which –
haematite and magnetite, respectively, are: (a) Ore is heated in absence of air
(a) II, III in haematite and III in magnetite (b) Used for sulphides ores
(b) II, III in haematite and II in magnetite (c) Ore is heated in presence of air
(c) II in haematite and II, III magnetite (d) None of these
(d) III in haematite and II, III in magnetite [JEE Main Online – 2013]
[IIT-2011] 21. In Goldschmidt alumino thermite process which
15. In the cyanide extraction process of silver from of the following reducing agents is used –
argentite ore, the oxidizing and reducing agents (a) Calcium (b) Coke
used are
(c) Al - powder (d) Sodium
(a) O2 and CO respectively
(b) O2 and Zn dust respectively [JEE Main Online – 2013]
(c) HNO3 and Zn dust respectively 22. The metal that cannot be obtained by electrolysis
(d) HNO3 and CO respectively of an aqueous solution of its salt is:

[IIT-2012] (a) Ag (b) Ca

16.
Sulphide ores are common for the metals (c) Cu (d) Cr
(a) Ag, Cu and Pb [JEE Main 2014]
(b) Ag, Cu and Sn 23. The form of iron obtained from blast furnace is:
(c) Ag, Mg and Pb (a) Steel (b) Cast Iron
(d) Al, Cu and Pb (c) Pig iron (d) Wrought Iron
[JEE Advanced 2016] [JEE Main Online – 2014]
17. During the process of electrolytic refining of 24. Calamine is an ore of:
copper, some metals present as impurity settle as
(a) Copper (b) Aluminium
‘anode mud’ these are –
(c) Iron (d) Zinc
(a) Pb and Zn (b) Sn and Ag
[JEE Main Online – 2015]
(c) Fe and Ni (d) Ag and Au
25. In the context of the Hall – Heroult process for
[AIEEE- 2005]
the extraction of Al, which of the following
18.
Heating mixture of Cu2O and Cu2S will give statements is false?
 [AIEEE- 2005] (a) CO and CO2 are produced in this process
(a) Cu + SO3 (b) Cu + SO2 (b) Al2O3 is mixed with CaF2 which lowers
(c) Cu2SO3 (d) CuO + CuS the melting point of the mixture and brings
conductivity
19. Which method of purification is represented by
(c) Al3+ is reduced at the cathode to form Al
the following equation:
(d) Na3AlF6 serves as the electrolyte

523K 1700K [JEE Main-2015]
Ti (s) + 2I2 (g) " TiI4(g) " Ti (s) + 2I2 (g)
26. Which one of the following ores is best
(a) Cupellation (b) Poling concentrated by froth floatation method?
(c) Van Arkel (d) Zone refining (a) Siderite (b) Galena
[AIEEE- 2012] (c) Malachite (d) Magnetite
[JEE Main-2016]
4.15

Answer Key

1. (a) 2. (a) 3. (b) 4. (d) 5. (c) 6. (b) 7. (b) 8. (c) 9. (b) 10. (d)
11. (a) 12. (c) 13. (c) 14. (c) 15. (c) 16. (b) 17. (c) 18. (a) 19. (a) 20. (b)
21. (b) 22. (b) 23. (b) 24. (a) 25. (c) 26. (d) 27. (a) 28. (a) 29. (c) 30. (c)

1. (c) 2. (b) 3. (b) 4. (b) 5. (d) 6. (b) 7. (c) 8. (c) 9. (d) 10. (a)
11. (b) 12. (c) 13. (a) 14. (b) 15. (d) 16. (a) 17. (d) 18. (d) 19. (b) 20. (c)
21. (c) 22. (b) 23. (b) 24. (b) 25. (d) 26. (a) 27. (b) 28. (d) 29. (a) 30. (b)

1. (b,c) 2. (a,b,d) 3. (a,b,c) 4. (a,b, c,d) 5. (b,d) 6. (a,b,c,d) 7. (a,b,c) 8. (a,b,c,d)


9. (a,b,c,d) 10. (b,c,d) 11. (a,b) 12. (a,c,d) 13. (a,b) 14. (a) 15. (c)

" " " " " "


" " " " " "

1. (c) 2. (b) 3. (a) 4. (a) 5. (a) 6. (c)   7. A " (a,c) ; B " (a,c) ; C " (b) ; D " (d)
8. (b) 9.(b)   10. A " (a) ;   B " (b) ;  C " (a,c) ;  D " (a,c,d) 11. (a)  12. (d)  13. (c)
14. (d) 15. (b) 16. (a) 17. (d) 18. (b) 19. (c) 20. (a) 21. (c) 22. (b) 23. (c)
24. (d) 25. (d) 26. (b)

Hints and Solutions

8. (c) Less reactive metals like Au, Pt exist as native


1. (a) Three most occurring elements into the earth ore.
crust are O, Si, Al 9. (b) Chromite ore is FeCr2O4
2. (a) Bauxite (Al2O3.2H2O) is an oxide ore. 10. (d) Sulphide ore is concentrated wetting by oil.
3. (b) Zinc blende, ZnS 11. (a) Oxide ore (Haematite, Fe2O3) is concentrated
chalcopyrites, CuFeS2 by gravity separation method.
Galena, PbS 12. (c) Cu2S + 2Cu2O $ 6Cu + SO2 -
4. (d) Gypsum is CaSO4.2H2O 13. (c) Highly electropositive elements are recovered
5. (c) Epsomite is MgSO4.7H2O from electrolytic process.
6. (b) Dolomite is CaCO3.MgCO3 14. (c) Refining of silver can be done by cupellation
7. (b) Magnesite is MgCO3 and electrorefining.
4.16

15. (c) Impure aluminium is purified by Hoop’s 4. (b) Bauxite (Al2O3 . 2H2O)
process. Siderite (FeCO3)
16. (b) In the alumino-thermite process, Al acts as Argentite (Ag2S)
reducing agent.
5. (d) MgCO3 $ MgO + CO2
17. (c) Slag is formed by reaction between impurities
Calcination is heating of ore in absence or
and flux.
limited supply of air.
18. (a) All ores are minerals but all minerals are not 6. (b) Cupellation is refining process for Ag.
ores.
7. (c) This method can not be used for highly
19. (a) Purification of Si is done by zone refining. electropositive matals.
20. (b) Concetration method used for bauxite ore is 8. (c) Highly electropositive metals are extrached by
leaching (chemical method). electrolysis of their fused salt.
21. (b) In blast furnane, maximum temperature is in 9. (d) Ore particles are hydrophobic and they are
combustion zone where combustion of coke wetted by oil.
(C) takes place. 10. (a) The slag is lighter and has lower melting point
22. (b) Silica (SiO2) is an acidic impurity hence, a than the metal hence it floats over the molten
basic flux is needed. metal.
23. (b) In cupellation method, Pb and Zn impurity 1
11. (b) (i) Zn(s) + O (g) $ ZnO(g) ;
present in silver are removed as their oxides. 2 2
24. (a) Cassiterite (SnO2) is an oxide ore. It DG° = – 360 kJ/mol.
is concentrated by gravity sepatation 1
(ii) C(s) + O (g) $ CO(g) ;
(Levigation). 2 2
25. (c) Froth floatation method is used for sulphide DG° = – 460 kJ/mol.
ore (cinnabar, HgS). (ii–i) ZnO(g) + C(s) $ CO(g) + Zn(s) ;
26. (d) Fluorspar is CaF2. DG° = – 460 + 360 = –100 kJ/mol.
27. (a) Gravity separation is used for oxide ore For a feasible reaction, DG° must be negative.
(Haematite, Fe2O3). It means, ZnO can be reduced by C into Zn.
28. (a) Electrolytic reduction method is used in the 12. (c) DG° Vs T plot in the Ellingham’s diagram
extraction of high electropositive elements like slopes downward for the reaction,
Na, Ca, Mg, K, Al etc. 1
C + O2 $ CO
2
29. (c) Bauxite is leached with NaOH 13. (a) Purification of Al metal is done by Hoop’s
Al2O3.2H2O + NaOH $ NaAlO2 + H2O process. (Baeyer’s process is used for
30. (c) Cyanide process is used for Ag and Au. purification of bauxite ore).
14. (b) To remove acidic impurities, basic flux is used.
15. (d) Because Na is highly reactive.
16. (a) Silica impurity present in bauxite is removed
by sepeck’s process (by addition of coke and
1. (c)
Leaching is a chemical process in which a
N2)
suitable reagent is used which forms a soluble
complex only with desired metal leaving 17. (d) PbS $ PbO + PbSO4 (Roasting)
behind impurities. PbS + PbO → Pb + SO2 
 Self Re duction
PbS + PbSO 4 → Pb + SO2 
2. (b)
FeO + SiO2 $ FeSiO3
Slag 18. (d) Electrorefining process is not used for Al.
3. (b) In zone refining method, the molten zone 19. (b) Poling process is used if oxide of metal is
consists more impurity than the original metal present as impurity in metal.
becuase impurities are moving in the direction 20. (c) Al2O3.2H2O+3C+N2 ∆ → 2AlN+3CO+2H2O
of movement of furnace. AlN + 3H2O $ Al(OH)3. + NH3-
4.17

21. (c) FeO + SiO2 $ FeSiO3 7. (a,b,c) Slag in the extraction of Fe is CaSiO3 while
(slag) slag in the extraction of Cu is FeSiO3.

22. (b) A reaction is feasible only when its G° is 8. (a,b,c,d)
negative. Calamine (ZnCO3)
(i): 2Zn(s) + O2(g) $ 2ZnO(g) ; Proustite (Ag3AsS3)
DG° = –360 kJ/mol Cassiterite (SnO2)
(ii): 2C(gr) + O2(g) $ 2CO(g) ; Diaspore (Al2O3.H2O)
DG° = –500 kJ/mol 9. (a,b,c,d) Froth floatation, is a physical method, mainly
(ii–i):2ZnO(s) + 2C(gr.) $ 2CO(g) + 2Zn(s) ; used for sulphide ores, depends on the
DG° = –140 kJ/mol difference in wetability of gangue and the ore
Hence, ZnO can be reduced by graphite. particles.
23. (b) Impurity ZnS forms a soluble complex 10. (b,c,d) Al is more electropositive than Fe hence, Fe
Na2[Zn(CN)4] with depressant NaCN. can not be used as reducing agent for Al2O3.
24. (b) In extraction of iron, coke (C) and limestone 11. (a, b) Calcination is the heating of ore in absence or
(CaCO3) are added with haematite ore (Fe2O3). limited supply of air.
25. (d) Fe2O3 + 2Al $ Al2O3 + 2Fe 12. (a,c,d) Lead impurity from silver can be removed by
It is an example of goldschmidt alumino- cupellation.
thermite process. 13. (a, b) Metal present lower in the Ellingham diagram
26. (a) In Bassemer converter copper is extracted by, can reduce metal oxide present higher in this
Cu2S + 2Cu2O $ 6Cu + SO2 diagram.
14. (a) In cyanide process, Au forms soluble complex
27. (b) ZnS (Zinc blende) $ Zn
with CN– in the presence of O2.
ZnS $ ZnO (Roasting)
15. (c) This process represents hydrometallurgical
ZnO $ Zn (C-Reduction process) extraction.
28. (d) Mond’s process is used for nickel (Ni) 16. (a) Copper pyrites is CuFeS2
29. (a) Poling process is used for the removal of Cu2O
17. (b) FeO + SiO2 " FeSiO3
from Cu. (impurity) (flux) (slag)
30. (b) Argentite (Ag2S)
18. (b) Cryolite (Na3AlF6) is added to lower the
Cuprite (Cu2O)
melting point of Al2O3.
19. (a) Al has higher SRP (standard reduction
potential) than Na and Ca.
20. Li, Ba, Na, Al, Ca, Mg
1. (b,c) Highly electropositive metals like Na, Al etc 21. Cupellation, Poling process, Hoop’s process,
are extracted by electrolysis of their fused salt. Kroll’s process.
2. (a,b,d) Cynide process is used for metallurgy of Ag 22.
and Au. Bauxite (Al2O3. 2H2O)
3. (a,b,c) Concentration by froth floatation followed by Magnetite (Fe3O4)
Roasting. Reduction by Bessemerisation
Quartz (SiO2)
4. (a,b,c,d) Slag (CaSiO3) is lighter than molten Fe Corundum (Al2O3)
5. (b,d) Liquation is the process used for metal having Pyrolusite (SnO2)
impurities greater melting point than metal.
Zincite (ZnO)
6. (a,b,c,d) Aim of roasting :
23. Poling is applied when metallic oxide is
(a) to convert ore into oxide of metal. present as impurity.
(b) to remove moisture. The atomic number of ‘O’ is 8.
(c) to remove volatile organic impurities. 24. (A " e ; B " b ; C " c ; D " a ; E " d ; F " f)
(d) to oxidize free sulphur and antimony. 25. (A " d ; B " a ; C " e ; D " c ; E " f ; F " b)
4.18

3
14. (d) Haematite (Fe 2 O3 )
2 3
Magnetite (Fe3 O 4 or FeO . Fe 2 O3 )
15. (b)
1. (c)
Cryolite is added into alumina to lower down
melting point. Ag2S+CN–+H2O+O2 $ [Ag(CN)2]– + S+OH–
  oxidizing agent
2. (b) FeO + SiO2 " FeSiO3
(impurity) (flux) (slag) [Ag(CN)2 ]– + Zn $ Ag + [Zn(CN)4]2–
Reducing agent
3. (a)
Highly electropositive metals, like Mg, are
16. (a) Argentite (Ag2S)
extracted by electrolysis of their fused salt.
Chalcopyrite (CuFeS2)
4. (a)
[X] is [Au(CN)2]–
Galena (PbS)
[Y] is [Zn(CN)4]2–
17. (d) Composition of anode mud in electrolytic
5. (a)
Pb " self reduction process
refining of copper is Ag, Au etc.
Sn " carbon reduction process
18. (b) Cu2S + 2Cu2O " 6Cu + SO2
6. (c)
Chalcopyrite is CuFeS2
19. (c) Van-Arkel process is used for Zr and Ti.
7. (A " a, c ; B " a, c ; C " b ; D " d)
20. (a) Calcination is the process in which ore is
8. (b)
ZnS $ ZnO (Roasting)
heated in absence or limited supply of air.
ZnO+C $ Zn+CO (Carbon reduction 21. (c) In Goldschmidt alumino thermite process, Al
process) is used as reducing agent.
9. (b) Ag2S + NaCN + H2O + O2 $ Na 22. (b) Higher electropositive metals, like Ca, are
[Ag(CN)2] + S + NaOH extracted by electrolysis of their fused salt.
10. (A " a ; B " b ; C" a, c ; D " a, c, d) 23. (c) The iron abtained from blasat furance is pig
11. (a) CuFeS2 + O2 $ Cu2S + FeO + SO2 iron.
12. (d) FeO + SiO2 $ FeSiO3 24. (d) Calamine is ZnCO3
25. (d) Cryolite (Na3AlF6) and Fluorospar (CaF2)
13. (c) Cu2 + 2Cu2O $ 6Cu + are added to lower down melting point and to
(S–2 ion acts as a reducing agent) increase conductivity of electrolyte (Al2O3).
26. (b) Galena (PbS) is concentrated by froth floatation
process.
Chapter

Key Concepts
 Covalent Hydrides: When element of p-block from
compound with hydrogen, they are called covalent
hydrides. Examples are NH3, H2O etc. These are
generally gaseous compound. Their stability with
Hydrogen is the first element of the periodic table. Its increase in atomic number of elements within group
electronic configuration is 1s1 and it behaves like an alkali decreases.
metal as well as a halogen. There are three isotopes of  Interstitial Hydrides: These are formed by some of the
hydrogen namely, hydrogen (1H1), deuterium (1H2 or D) transition metals with electronegativity ranging from
and tritium (1H3 or T). 1.2 to 1.4. Mostly these are non-stoichiometric solids
Based on the spinning of two nuclei in dihydrogen, two types and may be considered as interstitial compounds.
of dihydrogen may be distinguished. Ortho-dihydrogen Varying temperature and pressure may vary the
involves parallel spinning while para-dihydrogen involves proportion of hydrogen in the compound. Examples
antiparallel spinning of the two nuclei. are TiH1.73, ZrH1.92 , VH0.6 etc.
Dihydrogen is relatively inactive (because of high enthalpy  Polymeric Hydrides: Some elements with
435.kJ Mol-1) at ordinary temperature but quite reactive at electronegativity in the range 1.4 to 2.0 form polymeric
high temperature or in the presence of catalysts. hydrides. These are solids containing molecules
linked by hydrogen-bridged bonds. Examples include
(BeH2)n, (MgH2)n and (AlH3)n
Hydrogen finds many uses. For example, for the
preparation of NH3, CH3OH, synthetic petrol,
These are classified as follows: acetylene, vegetable ghee, H2 – F2 flame, an oxy-
 Ionic Hydrides: When elements of Groups 1 and 2 hydrogen flame etc.
(except Be and Mg) and lanthanides, (electronegativity
in the range 0.9 to 1.2) from compounds with hydrogen,
they are called ionic hydrides. These hydrides are
crystalline solids with higher melting points. The Water (H2O) is the most abundant liquid i.e., 75% of
stability of hydrides decreases with increase in atomic the earth’s surface is full oceans, lakes and rivers. It has
number of the element in a group. Examples are NaH, bent angular structure with H-O-H bond angle of 104.5°.
CaH2 etc. Because of hydrogen bondings, water unusually has high
5.2

melting and boiling points. Because of these bonds, ice is


less dense than liquid water. When ice (density = 0.917
g cm-3) is heated, its density increases to 1 g cm-3 at 4°C
followed by decrease in density as the temperature is The two O-H groups in hydrogen peroxide (H2O2) do not
further increased. lie in the same plane. The angle between two planes is
Water is termed as soft water if it is free from calcium or 111.5° and it reduces to 90.2° in the crystalline phase.
magnesium salts. If these salts are present, it is termed as The O-O-H bond on the other hand changes from 94.8°
hard water. The latter is not useful for washing purposes as to 101.9°.
soap forms insoluble scum of calcium or magnesium state
Hydrogen peroxide is a strong oxidizing agent. It oxidizes
in hard water.
ferrous to ferric, iodide to iodine, lead sulphide to lead
Temporary hardness in water is due to dissolved sulphate, potassium ferrocyanide to potassium ferricyanide
bicarbonates of calcium or magnesium. It can removed (in acidic medium) and manganese (II) to manganese (IV).
by boiling water when carbonates are precipitated out.
Hydrogen peroxide also acts as a reducing agent. It
Permanent hardness is due to soluble chloride or sulphate
reduces permanganate to manganese (II), iron (III) to
of calcium or magnesium. This hardness can be removed
iron (II), ferricyanide to ferrocyanide (in alkaline
by adding sodium hydroxide, carbonate or sodium
medium), periodate to iodate, ozone to oxygen and silver
phosphate. Ion exchange resins are also used to soften the
to metallic.
water.
5.3

SUMMARY OF PREPARATION AND PROPERTIES OF DIHYDROGEN


H2
DIHYDROGEN
PREPARATION PROPERTIES
From water (H2O) Physical Properties
(a) By reaction with metals  Colourless
 Active metals (Na, K, Ca) react at room temperature  Tasteless
 Metals like Mg, Zn, Al react on heating  Odourless
 Metals like Fe, Ni, Sn react with steam  Highly combustible
H2O + Na NaOH + H2  High value of DHH-H (436 KJ mol-1)
H2O + 2Mg 2MgO + H2

1000k

4H2O + 3Fe Fe3O4 + 4H2


Chemical Properties
(b) Electrolysis of acidic or alkaline water Metals react to form respective hydrides
Electricity 525K
2H2O O2 + 2H2 H2 + 2Na 2NaH
525K
From Acids by reaction with active metals H2 + Ca CaH2
2HCl + Mg MgCl2 + 2H2 Non-Metals form respective hydrides
H2SO4 + Zn ZnSO4 + H2 970K
2H2(g) + O2(g) 2H2O
or sunlight

From Conc. Alkalies by reaction with elements like Fe/Mo, 770K


Zn, Sn, Al, Be, B, Si etc. 3H2(g) + N2(g) 2NH3
2Al + 2NaOH + 2H2O NaAlO2 + 3H2 H2(g) + S(I) H2S
Sn + 2NaOH + H2O Na2SnO3 + 2H2 1275K
2H2(g) + 2C(s) C2H2
Electric arc
Laboratory Preparation H2(g) + X2(g) 2HX(X = F,Cl,Br I)
3000K
H2SO4 + Zn ZnSO4 + H2
dil. Metals Oxides are reduced to metals
H2 + PbO Pb + H2O
700K, 200atm.
2H2 + CO CH3OH
ZnO/Cr2O3

Uses Unsaturated hydrocarbons get hydrogenated


Pt
 In Preparation of ammonia H2 + CH2 = CH2 CH3-CH3
Pt
 Hydrogenation of oils
2H2 + CH ∫ CH CH3-CH3
 In oxy-hydrogen and atomic hydrogen torch Ni
 Liquid H2 is used as prospective fuel. H2 + Vegetable oils Fats
5.4

H2O
WATER

Physical Properties Chemical Properties

 The composition of hydrogen and oxygen elements Dissociation of water


by mass is 1:8 and by volume it is 2:1. High temp. 1
H2O H2 + O
 It is a covalent compound with bent structure. The 2 2
O-H bonds are sp3-ss - bonds and angle around O Dissociation is 0.2% even at 1500 K
is 104.5°. Amphoteric nature of water
 Its dipole moment is 1.83D and dielectric constant H2O + NH3 NH4+ + OH–
is 82.5. Acid
 Excellent solvents for large number of ionic as well H2O + H2O H3O+ + Cl–
as covalent substances. Base
 Interpartical forces in water and ice are H-bonds.
Self ionization of water
 Density of ice is less than water because of open
cage like structure of ice. H2O + H2O H3O+ + OH–

 Density of water is maximum (1.0 g cm-3) at 277 K. Active metals decompose water to liberate H2
H2O + 2Na 2NaOH + H2
 Dissociation constant (Kw) of water at 25° C is 1 ×
Heat
10-14 and pKw = 14. H2O + Zn ZnO + H2
In these reactions H2O acts as a weak oxidant.
It can reduce highly electronegative elements
such as halogens, carbon, etc.
2H2O + 2Cl2 4HCl + O2
C + H2O(g) CO + H2
Red Hot
Coke

Hydrolytic Reactions
3H2O + PBr3 H3PO3 + 3HBr
2H2O + SiCl4 SiO2 + 4HCl
6H2O + Ca3P3 Ca(OH)2 + 2PH3
2H2O + CaH2 Ca(OH)2 + H2
2H2O + CaC2 Ca(OH)2 + C2H2
12H2O + Al4C3 4Al(OH)3 + 3CH4
H2O + CaNCN CaCO3 + 2NH3
6H2O + Mg3N2 3Mg(HO)2 + 2NH3

Hydrate Reactions
CuSO4(s) + 5H2O CuSO4.5H2O(s)
AlCl3 + 6H2O AlCl3.6H2O
BeCl2 + 2H2O BeCl2.2H2O
5.5

H2O2
HYDROGEN PEROXIDE
PREPARATION PROPERTIES

 Merck’s method Physical Properties


Na2O2 + H2SO4 Na2SO4 + H2O2  Colourless, syrupy liquid with odour
 From Barium peroxide like nitric acid
 Soluble in alcohol, ether
BaO2.8H2O + H2SO4 BaSO4 + 8H2O + H2O2
 Density 1.44 g/cm3
BaO2.8H2O + H3PO4 Ba3(PO4)2 + 8H2O + 3H2O2  BP = 423K at 670 mm; 358K at 68 mm
 Electrolysis 50% H2SO4
Chemical Properties
H2SO4 H+ + HSO4
   Cathode:-           Anode:-  Decomposition
+
2H + 2e -
H2 2HSO4 -
H2S2O8 + 2e 2H2O2 2H2O + O2; DH = - 196 kJ
Decomposition is felicitated by Pt carbon
H2S2O8 + H2O 2H2O2 + H2SO4  Acidic Nature (Ka = 1.5 × 10-12)
 Electrolysis of (NH4)2SO4 (H2SO4 mixture) H2O2 + 2NaOH Na2O2 + 2H2O
(NH4)2SO4 + H4SO4 2NH4HSO4 H2O2 + NaCO3 Na2O2 + H2O + CO2
 Oxidising behaviour
NH4SO4- + H+
4H2O2 + PbS PbSO4 + 4H2O
Cathode:-    Anode:- -
 2H+ + 2e- H2 2NH4SO4- (NH4)2S2O8 + 2e- H2O2 + NO2 NO3- + H2O
(NH4)2S2O8 + H2O 2NH4HSO4 + H2O2 H2O2 + 2I- + 2H+ I2 + H2O
2+ +
Amm. Persulphate H2O2 + 2Fe + 2H 2Fe3+ + 2H2O
 Air oxidation of 2-enthraquinol H2O2 + AsO33- AsO43- + H2O
OH OH
C2H5
O2 C2H5 H2O2+2[Fe(CN)6]4- 2[Fe(CN)6]3- + 2H2O
+H2O2
Air
OH
4H2O2 + Cr2O72- + 2H+ CrO5 + H2O
O
 Reducing Nature
H2O2 + Ag2O 2Ag + H2O + O2
H2O2 + PbO2 PbO + H2O + O2
H2O2 + O3 H2O + 2O2
Concentration
5H2O2+2MnO4–+6H+ 2Mn2++8H2O+5O2
Dilute solution of H2O2 is concentrated by various steps
like slow evaporation on water bath, evaporation in H2O2 + Cl2 2HCl + O2
vacuum decicator, distillation under reduced pressure 2OH-
and finally freezing. H2O2 + 2[Fe(CN)6]3- 2[Fe(CN)6]4-
Storage: It is stored in a dark bottle and its decomposition is retarded + 2H2O + O2
by stabilizers like glycerol, acetanilide, uric acid, barbituric acid, etc. H2O2 + NaOBr NaBr + H2O + O2
H2O2 + CaOCl2 CaCl2 + H2O + O2

Bleaching action H2O2: H2O2 is used to bleach delicate articles like ivory silk, feather, wool, etc. The bleaching action
is due to its ability to oxidise the colouring matter.
   H2O2 H2O + O
Coloring matter + (O) Oxidised matter (Colourless)
5.6

S-Block Elements
Element of group 1 (or I A) and 2(or II A) are known as s-block elements.
General electronic configuration are
Group 1 [Inert gas]ns1 Alkali metals
Group 2 [Inert gas]ns2 Alkaline Earth metals

Property Alkali metal Alkaline earth metal
(a) All are silvery white. (a) All are grayish white.
(b) Light soft, malleable and ductile metals (b) Relatively harder.
Physical with metallic luster.
state
(c) Both are diamagnetic and colourless.

(a) Both produce characteristic colours in (a) Be and Mg do not show any colour as their elec-
Bunsen flame due to easy excitation of trons are more strongly bound.
electron to higher energy levels.

(b) Characteristic flame colours are (b) Ca – Brick red, Sr - Crimson


Li – Crimson, Na – Golden Yellow, Ba – apple green, Ra - Crimson
K – Pale violet, Rb and Cs – Violet
Flame (c) Energy released (c) Be and Mg atoms due to their small size, bind their
colour Li+ < Na+ < K+ < Rb+ < Cs+ electrons more strongly because of higher effective
nuclear charge. Hence these posses high excitation

energy and are not excited by the flame energy and
do not show any colour.
(d) The flame energy cause an excitation of
the outermost electron which on reverting
back to its initial position gives out the
absorbed energy as visible light.
(a) Due to unpaired lone electrons in ns sub (a) Due to smaller size, electrons are tightly held as
shell as well as due to their larger size, compared to alkali metal.
the outermost electron is far from the
lonisation nucleus, the removal of electron is easier
energy and these have low values of ionization
potential.
(b) IP of these metals decreases from Li to (b) The IP value decreases with increase of atomic
Cs radii from Be to Ba.
(a) Hydration represents for the dissolution
of a substance in water to absorb water
molecule by weak valency forces.
Hydration of ions in the process when
ions on dissolution in water get hydrated.
(b) Smaller the cation greater is the degree of (b) Hydration energy-
Hydration of ions hydration. Hydration energy-
  Li+ > Na+ > K+ > Rb+ > Cs+ Be+2 > Mg+2 > Ca+2 > Sr+2 > Ba+2

(c) Li+ being smallest in size has maximum


Degree of hydration and that is why
lithium Salts are mostly hydrated and
moves very slowly under the influence of
electric field.
5.7

Property Alkali metal Alkaline earth metal


These metals easily form univalent +ve ion The IP1 of these metals are much lower than IP2 and
Oxidation
by losing solitary ns electron due to low IP thus it appears that these metals should form univalent
1

value. ion rather than bivalent ions but in actual practice, all
these give bivalent ions.
(a) These metals are highly electropositive (a) Their electro negativities are also small but are
thereby posses low values of electro- higher than of alkali metals.
negativities
Electro negativity
(b) Electro-negativity of alkali metals (b) Electro-negativity decrease form Be to Ba.
decreases down the group
Li > Na > K > Rb > Cs
(a) Since alkali metals easily loose ns1 (a) They lose two electrons to give M+2 ion.
electron they have high value of oxidation
potential
Standard
   i.e., M M+(aq) + e-
oxidation
(b) Standard oxidation potential are listed (b) Standard oxidation potential are
potentials below Be Mg   Ca Sr
are reducing Li     Na       K   Rb    Cs 1.69     2.35  2.87 2.90
properties
3.05  2.71  2.93 2.99 2.99
(c) Li have greatest reducing nature due to
maximum hydration energy of Li+ ion.
(a) On exposure to moist air, all alkali metals (a) Except beryllium these metals are easily tarnish in
except lithium tarnish quickly. air as a layer of oxide is formed on their surface.
Action
with (b) They generally form oxides and (b) They give oxides of ionic nature M+2O-2 which are
air peroxides. crystalline in nature.
        O2    O2
M+O2 M2O M2O2 MO2
(a) Alkali metals decompose water with the (a) Ca, Sr, Ba and Ra decompose cold water readily
evolution of hydrogen with evolution of hydrogen.
  2M + 2H2O " 2MOH + H2   2M + 2H2O " 2M(OH)2 + H2
Action (b) Li decompose water slowly, sodium (b) Magnesium decomposes boiling water beryllium
with reacts water quickly. K, Rb and Cs react is not attacked even at high temperatures.
water with water vigorously.
(c) Alkali metals react with alcohols forming
alkoxides with the evolution of hydrogen:
   2Li + 2C2H5OH $ 2C2H5OLi + H2
   Ethy alcohol Lithium ethoxide
(a) These metals combine with H2 to give (a) Except Be, all alkaline earth on heating directly
white crystalline ionic hydrides of the with H2.
general formula MH.
(b) The metal hydrides react with water to (b) BeH2 is prepared by the action of LiAIH4 on
Hydride
give MOH and H2: BeCl2:
   MH + H2O $ MOH + H2   BeCl2 + LiAlH4 $ 2BeH2 + LiCl + AlCl3
(c) The ionic hydrides of Ca, Sr, Ba liberate H2 at
anode and metal at cathode.
5.8

Property Alkali metal Alkaline earth metal


(a) The carbonates (M2CO3) and (a) All these metal carbonates MCO3 are insoluble in
bicarbonates (MHCO3) are highly stable neutral medium but soluble in acids and decompose
to heat, where M stands for alkali metals on heating.

(b) The stability of these salts increases with (b) The stability of carbonates increase in
Carbonates the increasing electropositive character electropositive character metal.
from Li To Cs. Therefore Li2CO3
And bicarbonates
decompose on heating.
(c) Bicarbonates are decomposed
at (c) Bicarbonates of alkaline earth metals do not exit
relatively high temperature. in solid state but are known in solution only on
      heating their solution bicarbonate decomposed to
300°C
  2MHCO3 $ M2CO3 + H2O + CO2 liberate:
  M(HCO3)2 $ MCO3 + CO2 + H2O
(a) Alkali metals combine directly with (a) The alkaline earth metals directly combine with
halogens to from ionic halide MX. halogens on heating to give metal halides MX2.
(b) The ease with which the alkali metals (b) The ionic character of halides increases from Be to
form halides increases form Li to Cs due Ra.
to increase in electropositive character.
(c) LiX has more covalent character. (c) Beryllium halides have covalent Character due to
size and high effective nuclear charge and thus do
not conduct electricity in molten state.
(d) Halides having ionic nature high melting (d) The solubility of halides in water decreases down
point and are good conductor of current the group. Except fluorides, all are fairly soluble in
Halides in fused state. These are readily soluble in water.
water.
(e) Halides of potassium, rubidium and (e) The decrease is solubility of halide down the group
ceasium have property of combining is due to decreas in hydration energy because of
with extra halogen atoms forming increasing size of metal cation. Solubility OH
polyhalides: increase down the group
   KI + I2 – KI3
(f) The halides are hygroscopic and readily form
hydrates
  CaCl2.6H2O, BaCl2. 2H2O
   Otherwise down the group lattice and hydration
energy incomplete
(a) All these form sulphates of type M2SO4. (a) MSO4
(b) Except Li2SO4 rest are soluble in water. (b) The solubility of sulphates decreases on moving
Sulphates
Down the group stability and solubility down the group. BeSO4 is soluble in water while
increases. BaSO4 is completely insoluble
(a) Nitrates of both are soluble in water and
decompose on heating.
(b) LiNO3 decompose to give NO2 and O2 (b) On heating they decompose into their corresponding
Nitrates rest all give nitrites and oxygen. oxides with evolution of a mixture of nitrogen
2MNO3 2MNO2 + O2 (except Li) dioxide and oxygen
4LiNO3 2Li2O + 4NO2 + O2 1
M(NO3)2 MO + 2NO2 + O2
2
Forms deep blue solution with liquid Ammonia Except Be and Mg, all others form a deep blue-black
Solution of Liquid
which is conducting and paramagnetic in solution with liquid ammonia.
NH3
nature.
5.9

Solved Examples
1. Incorrect statement for H2O2 is: Sol. (b) Mg and Cu do not produce hydrogen on treatment
(a) Decomposition of H2O2 is a disproportion- with caustic soda (NaOH)
ation reaction. Zn + 2NaOH " Na2ZnO2 + H2 -
(b) Aqueous solution of H2O2 is weakly acidic.
Sn + 2NaOH + H2O " Na2SnO3 + 2H2 -
(c) Bleaching action of H2O2 is due to its
reducing nature. 2Al + 2NaOH + 2H2O " 2NaAlO2 + 3H2 -
(d) H2O2 is used in refreshing old lead paintings
5. In which reaction, hydrogen peroxide neither acts
PbS (black) converts into PbSO4 (white) in
as oxidizing agent nor reducing agent?
presence of H2O2.
(a) Na2CO3 + H2O2 "
Sol. (c) 2H2O2 " 2H2O + O2 (disproportionation) (b) PbS + H2O2 "
H2O2 is very good oxidizing and poor reducing (c) Cr2O72- + H+ + H2O2 "
agent. Its bleaching action is due to its oxidising
(d) SO32- + H2O2 "
nature.
PbS + H2O2 " PbSO4 + H2O Sol. (a) Na2CO3 + H2O2 " Na2O2 + CO2 + H2O
(black) (white) Here, H2O2 acts as acid.
2. In which property listed below hydrogen does not 6.
Alkali metals dissolve in liquid ammonia to give a
resemble alkali metals? blue colored solution which is due to the presence
(a) Tendency to form cation of –
(b) Nature of oxide (a) M – atoms (b) M + ions
(c) Combination with halogens
(c) Solvated anions (d) Solvated electrons
(d) Reducing character
Sol. (d) The blue colored solutions of an alkali metal in
Sol. (b) H2O is neutral while alkali metal oxides are basic
ammonia is explained on the basis of formation
in nature.
of ammoniated (solvated) metal cations and
3. Calgon causes the softening of hard water by: ammoniated (solvated) electrons in the metal
(a) Sequestraction of Ca2+ and Mg2+ ion ammonia solution in the following way:
(b) Sequestraction of Cl- and SO42- ion M M+ + e-
(c) Precipitation the Ca2+ and Mg2+ ions as M+ + xNH3
[M(NH3)x]+
phosphates
e- + yNH3 [e(NH3)y]-
(d) Precipitation the Ca2+ and Mg2+ ions as
sulphates M + (x + y)NH3 [M(NH3)x]+
(Solvated metal cation)
Sol. (a) Calgon is sodium hexametaphosphate Na6(PO3)6
+ [e(NH3)y]-
or Na2[Na4(PO3)6]
         (solvated electron)
2Ca+2+ Na2[Na4(PO3)6] " Na2[Ca2(PO3)6] + 4Na+ The blue colour of the solution is due to
Soluble complex excitation of free electrons to higher energy
It is sequestraction of Ca+2 and Mg+2 levels. The absorption of photons takes place in
the red region of the spectrum and hence, the
4. Which elements out of the following do not
solution appears blue in the transmitted light. As
produce hydrogen on treatment with caustic
the concentration of the alkali metal increases,
soda?
the metal ion cluster formation takes place and
A (Zn); B (Sn); C (Mg); D (Cu); E (Al) at very high concentration the solution becomes
(a) A, E (b) C, D (c) D, E (d) B, D colored like that of metallic copper.
5.10

7. Which of the following is an incorrect statement? CaO + H2O Ca(OH)2


(a) Sodium oxide is more basic then magnesium [Y]
oxide. Ca(OH)2 + CO2 Ca(HCO3)2
(b) Beryllium oxide is amphoteric. [Y] (excess) [Z]
(c) The thermal stability of beryllium carbonate
is more than of calcium carbonate. Ca(HCO3)2 ∆
→ CaCO3 + H2O + CO2
[Y] [X]
(d) Beryllium is amphoteric.
11. Number of crystal water in Gypsum, Plaster of
Sol. (c) The thermal stability of calcium carbonate is
Paris and Epsom salt respectively are:
more as compared to that of beryllium carbonate.
The ionic potential (F) value of Be2+ is more than (a) 2, 0.5, 7 (b) 7, 2,1
that Ca2+. So Be2+ attracts the oxygen of CO32- (c) 7, 0.5, 2 (d) 3, 4, 2
more and on heating beryllium carbonate looses
Sol. (a) The formulae of Gypsum, Plaster of Paris and
CO2 more easily.
Epsom salt are-
8. In the Solvay process of manufacture of sodium
CaSO4 .2H2O, CaSO4.0.5H2O and MgSO4. 7H2O
carbonate, the raw materials used are:
(a) aqueous NaOH, NH3 and CO2 12. Nitrolim (a nitrogenous fertilizer) is a mixture of:
(b) molten NaOH, NH3 and CO (a) Calcium carbide and calcium cyanamide
(c) brine NaCl, NH3 and CO (b) Calcium oxide and calcium carbide
(d) brine NaCl, NH3 and CO2 (c) Calcium cyanamide and carbon
Sol. (d)
The chemical reactions involved in Solvay (d) Calcium oxide and carbon
process are as below:
1000°c
NH3 + CO2 + H2O NH4 HCO3 Sol. (c) CaC2 + N2 CaCN2 + C
Cal. Cynamide
30°C
NH4HCO3 + NaCl NaHCO3. + NH4Cl
Nitrolim
250°C
2NaHCO3 Na2CO3 + H2O + CO2 13. On exposure to air, sodium hydroxide becomes
(used again) liquid and after sometimes it changes to whites
powder. Why?
2NH4Cl + Ca(OH)2 CaCl2 + 2H2O + 2NH3
     Slaked lime (used again) Sol. Sodium hydroxide continuously absorbs carbon
dioxide of atmosphere and is converted into
9. The ion of which of the following metals has least
sodium carbonate. A stage reaches when the
ionic conductivity in the aqueous solution?
solution becomes saturated and the crystals are
(a) Lithium (b) Sodium formed. These crystals, with the crystallization
(c) Potassium (d) Rubidium (efflorescence) and crumble to white powder.
Sol. (a) Li+ forms [Li(H2O)4]+ in water because of its 14. An aqueous solution of iodine becomes colourless
smallest size and highest charge to size ratio. The on adding excess of sodium hydroxide solution.
size of this hydrated ion is biggest and thus ionic Why?
conductivity is least.
Sol.
Iodine reacts with NaOH forming colourless
10. The compound X on heating gives a colorless compounds. Thus, the color of iodine disappears
gas. The residue is dissolved in water to obtain on addition of NaOH.
Y. Excess of CO2 is bubbled through aqueous
solutions of Y and Z is formed. Z on gentle 2NaOH + I2 NaI + NaIO + H2O
heating gives back X. The compound X is:
(a) CaCO3 (b) Na2CO3 Colourless products
(c) CaSO4. 2H2O (d) K2CO3 15. The addition of NaOH solution to a solution
Sol. (a) CaCO3 ∆
→ CaO + CO2 of ZnCl2 produces a white precipitate which
[X] dissolves on further addition of NaOH. Why?
5.11

Sol.
ZnCl2 reacts with NaOH to gives white precipitate 18. Element (A) burns in nitrogen to give an ionic
of Zn(OH)2 which later on gets dissolved in compound, (B) reacts with water to give (C) and
excess of NaOH because of formation of soluble (D). A solution of (C) becomes milky on bubbling
sodium zincate. carbon dioxide. Identify (A), (B), (C) and (D).
Sol. (a) Since element (A) burns in nitrogen to give
ZnCl2 + 2NaOH 2NaCl + Zn(OH)2 an ionic compound, therefore (B) must be a
(Insoluble) metal nitride
Zn(OH)2 + 2NaOH Na2ZnO2 + 2H2O (b) Since (B), a metal nitride reacts with water,
(Soluble) (B) is ionic nitride and the product formed,
16. NaCl in earlier days used to manufacture NaOH i.e., (C) and (D) are metal hydroxides and
and Cl2 involving following steps. Identify (A) to ammonia, (NH­3)
(I) in the following: (c) Since (C) become milky on bubbling CO2,
(C) must by calcium hydroxide, Ca(OH)2.
NaCl + conc. H2SO4 ∆
→ (A) + (B) (gas)
All reactions involved in the question can be
MnO2 explained as follows:
(B) gas (C) gas 3Ca(s) + N2(g) ∆

→ Ca3N2(s)
(A) + NaCl 
→ (D) + (C) gas     Calcium nitride
(A) (B)
(D) + carbon + CaCO3 (E) + (F)

Ca3N2 + 6H2O 3Ca(OH)2 + 2NH3(g)
  CaCO3 
→ (G) + (H) gas (B) Calcium Ammonia
(G) + H2O (I) Hydroxide(C) (D)

(I) + (E) NaOH + CaCO3 Ca(OH)2 + CO2 CaCO3 + H2O


Calcium
Sol.
NaCl + conc.H2SO4 NaHSO4 + HCl Carbonate
(A) (B) (milkiness)
MnO2 (D)
HCl Cl2 Thus, (A) = Ca, (B) = Ca3N2, (C) = Ca(OH)2
(B) (D) = NH3
Or MnO2 + 4HCl ∆→ MnCl2 + Cl2 + 2H2O 19. An aqueous compound of an inorganic compound
(B) (C) (X) shows the following reactions:

NaHSO4 + NaCl  → Na2SO4 + HCl (a) It decolourises an acidified KMnO4 solution
(A) (D) accompanied by the evolution of oxygen.
Na2SO4 + carbon + CaCO3 ∆ (b) It liberates I2 from an acidified KI solution.
→ Na2CO3 + CaSO4
(D)       (E)   (F) (c) It gives a brown precipitate with alkaline
KMnO4 solution with evolution of oxygen.
CaCO3 ∆ → CaO + CO2
(d) It removes black stains from old oil paintings.
(G) (H)
Identify X and give chemical equation for the
CaO + H2O Ca(OH)2 reactions at steps (a) to (d).
(G) (I)
Sol.
(X) is H2O2.
Ca(OH)2 + Na2CO3 2NaOH + CaCO3
(I) (E) a. 2KMnO4 + 3H2SO4 + 5H2O2
Purple   K2SO4 + 2MnSO4 + 8H2O + 5O2
17. Magnesium metal burns in air to give a white
Coloured Colourless
ash. When ash is treated with water, the odour of
ammonia can be detected. What is the reason? b. 2KI + H2SO4 + H2O2 K2SO4 + I2 +
Sol.
Mg burns in air to form MgO and Mg3N2. It is the 2H2O
Mg3N2 which on hydrolysis gives NH3 c. 2KMnO4 + 3H2O2
2Mg + O2 2MgO 2KOH + 2MnO2 + 2H2O + 3O2
      Brown
3Mg + N2 Mg3N2
d. PbS + 4H2O2 PbSO4 + 4H2O
Mg3N2 + 6H2O 3Mg(OH)2 + 2NH3 Black White
5.12

20. Calcium burns in nitrogen to produce a white Ca on heating with N2 produces calcium nitride,
powder which dissolves in sufficient water Ca3N2, a white powder. Ca3N2 on reacting with
to produce gas A and alkaline solution. The water produces ammonia gas NH3, i.e. A and
solution on exposure to air produces a thin solid alkaline solution, atmospheric CO2 to give
layer of on the surface. Identify the compounds insoluble CaCO3.
A and B. ∆
3Ca + N2  → Ca3N2
Sol. Ca + N2 ∆
→ white powder Ca3N2 + 6H2O 3Ca(OH)2 + 2NH3
H2O Calcium
Gas (A) + alkaline solution hydroxide
(alkaline solution)
Air
thin solid layer Ca(OH)2 + CO2 CaCO3 + H2O
(B) (air) (B)

Exercise
(a) passing CO2 gas through it
(b) passing SO2 gas through it
1. Which pair of species can undergo chemical (c) adding calculated amount of Ca(OH)2
reaction with each other? (d) adding calculated amount of sodium
(a) CO and NO hypophosphate.
(b) LiH and H2O
6. When temporary hard water containing
(c) CO2 and HCl
Mg(HCO3)2 is boiled the percipitate formed is of:
(d) CaH2 and SiH4
(a) MgCO3
2. Which type of element forms ionic hydrides?
(b) MgO
(a) Transition elements
(b) Metalloids (c) Mg(OH)2
(c) Elements with high electronegativity (d) None of these
(d) Elements with high electropositivity 7. In which of the following reactions hydrogen act
3. The three isotopes of hydrogen differ from one as oxidizing agent?
another in: (a) Ca + H2 "
(a) Atomic number (b) H2 + O2 "
(b) Number of protons (c) H2 + F2 "
(c) Nuclear charge (d) CuO + H2 "
(d) Nuclear mass
8. Which forces of attraction are responsible for
4. Electrolysis of which of the following liberates liquefication of H2?
hydrogen gas at anode? (a) Dispersion forces
(a) Aq. H2SO4 (b) Hydrogen bonding
(b) Aq. CuSO4 (c) Dipole force
(c) Molten calcium hydride (d) All of these
(d) Aq. barium hydroxide
9. Adsorbed hydrogen by Palladium is known as:
5. Which of the following operation would cause (a) Atomic (b) Nascent
removal of temporary hardness of water?
(c) Occuluded (d) Heavy
5.13

10. Which of the following is not a peroxide? 19. Which of the following properties of IA group
(a) Na2O2 (b) BaO2 metals increases as the atomic number rises?
(I) Metallic character
(c) PbO2 (d) H2O2
(II) Ionic radius
11. The ortho and para-hydrogens possess:
(III) Melting point
(a) Same physical properties but different
(IV) Density
chemical properties
(V) Ionization potential
(b) Different physical properties but same
chemical properties (a) I, II, III (b) I, II
(c) Same chemical and physical properties (c) III, IV, V (d) All
(d) Different physical and chemical properties 20. Which of the following statements is not true
12. Which is correct about the reaction between H2O2 about the dilute solutions of alkali metals in liquid
and O3? ammonia?
(a) It is a case of mutual reduction (a) They are deep blue coloured solutions
(b) O3 will oxidise H2O2 into O2 (b) They are highly conducting in nature
(c) It is not a redox reaction (c) They are diamagnetic in nature
(d) H2O2 being a stronger oxidizing agent will (d) Ammoniated cation and ammoniated anion
decompose ozone into oxygen are formed in the solution.
13. Alkali metal superoxides contain the (O2-) ion. 21. Which of the following equations is not involved
They are: in the Solvay process?
(a) Paramagnetic (a) CaCO3 " CaO + CO2
(b) Coloured compounds (b) NaCl + NH3 + H2O + CO2 " NH4Cl +
(c) Oxidizing agents NaHCO3
(d) All of these (c) CaO + 2NH4Cl " 2NH3 + H2O + CaCl2
14. On heating sodium metal in the current of dry (d) Na2CO3 + CO2 + H2O " 2NaHCO3
ammonia leads to the formation of which gas?
22. Which of the following property of alkaline earth
(a) NaNH2 (b) NaN3 metals increases with increasing atomic number?
(c) NH3 (d) H2 (a) Ionization potential
15. Which of the following s-block elements react (b) Solubility of hydroxides
with NaOH to give water soluble complex? (c) Solubility of sulphates
(a) Al (b) Ca (d) Density
(c) Be (d) Li 23. Among the carbonates of alkali metals which one
16. Which of the following element is common in has highest thermal stability?
microcosmic salt and Glauber’s salt? (a) Cs2CO3 (b) Rb2CO3
(a) N (b) Na
(c) K2CO3 (d) Na2CO3
(c) K (d) Both (a) and (b)
24. A solution of sodium in liquid ammonia is blue in
17. Which of the following elements does not form colour due to:
hydride by direct heating with dihydrogen?
(a) The presence of ions Na+
(a) Be (b) Mg (b) The presence of ammoniated electron
(c) Sr (d) Ba (c) The formation of NaNH2
18. A metal chloride, when placed on a platinum wire (d) The formation of sodium hydride
in Bunsen flame, does not produce any distinctive 25. The order of basic strength of the hydroxides of
colour. The cation of chloride is: alkali metals is:
(a) Li+ (b) Mg2+ (a) Li > Na > Rb > Cs
(c) Na+ (d) Ca2+ (b) Na > Li > Rb > Cs
5.14

(c) Cs > Rb > Na > Li 34. Which of the following compound is consumed
(d) Rb > Cs > Na > Li during the preparation of Na2CO3 by Solvay’s
Process?
26. Which of the following compounds liberate(s)
oxygen on heating? (a) NH3 + CaCO3 + NaCl
(b) NH4Cl + CaO + NaCl
(a) Li2CO3 (b) LiOH
(c) CaCO3 + NaCl
(c) LiNO3 (d) NaOH
(d) NaCl + NH4HCO3
27. When MgCl2.6H2O is strongly heated, then it
35. Select the correct statement:
forms:
(a) Be and Al show diagonal relationship
(a) MgO (b) Mg(OH)2
(b) Be forms tetrahedral complexes [Be(C2O4)2]2-
(c) Mg(OH)Cl (d) MgCl2 (c) Al forms AlF6-3, an octahedral complex
28. Magnesium liberates H2 on reaction with: (d) All are correct statements
(a) dil. HCl
(b) dil. H2SO4
(c) very dil. HNO3
(d) all of these 1. When a mixture of ammonium sulphate and 50%
29. Calcium hydride on hydrolysis forms: H2SO4 is electrolysed the products formed at
(a) CaO + H2 anode and cathode are:
(b) Ca(OH)2 only (a) H2 and H2O2
(c) Ca(OH)2 + H2 (b) (NH4)2S2O8 and H2
(d) Only CaO (c) H2 and NH4HSO4
30. Which one on reaction with NaOH solution gives (d) H2O2 and H2
inflammable gas? 2. When H2O2 is added to ice cold solution of
(a) S (b) Zn acidified potassium dichromate containing ether.
The contents are shaken and allowed to stand then
(c) NH4Cl (d) I2
(a) a blue colour is obtained in ether due to
31. Which of the following is the most important formation of Cr2(SO4)3
factor in making lithium metal, the strongest
(b) a blue colour is obtained in ether due to
reducing agent?
formation of CrO5
(a) Ionization energy
(c) CrO3 is formed which dissolves in ether to
(b) Hydration energy
give blue colour
(c) Heat of sublimation
(d) Chromyl chloride is formed.
(d) None of these
3. Which of the following species is reduced by
32. Compound having highest melting point: H2O2?
(a) LiCl (b) CsCl (a) [Fe(CN6)]4-
(c) NaCl (d) KCl (b) [Fe(CN6)]3- in alkaline medium
33. The solubility of metal halides depends on their (c) NO2-
nature, lattice enthalpy and hydration enthalpy of (d) I-/HCl
the individual ions. Amongst fluorides of alkali
metals, the lowest solubility of LiF in water is due 4. Which of the following on oxidation gives H2O2?
to: (a) 2-Ethylanthraquinol
(a) Ionic nature of lithium fluoride (b) 2-Ethylanthraquinone
(b) High lattice enthalpy of lithium and fluoride (c) Anthracene
ion (d) 2-Ethylanthracene
(c) High hydration enthalpy of lithium ion 5. One of the following is an incorrect statement.
(d) Low ionisation enthalpy of lithium atom Point out the incorrect one:
5.15

(a) Hardness of water depends upon its soap 11. Identify incorrect statement regarding H2O2:
consuming power (a) It can be prepared by acidifying BaO2 and
(b) Temporary hardness is due to bicarbonates of hydrolyzing H2S2O8 and H2SO5.
calcium and magnesium
(b) It is thermodynamically stable.
(c) Permanent hardness is due to soluble
sulphates and chlorides of Ca and Mg (c) It has non planar structure.
(d) Permanent hardness can be removed by (d) It is oxidizing as well as reducing agent.
boiling water. 12. In which of the following method of the removal
6. Incorrect statement about ortho and para of hardness, Ca+2 and Mg2+ are not separated
hydrogen: from sample of hard water?
(a) Para hydrogen is present in pure state at low (a) By boiling of temporary hard water
temperature (zero kelvin) (b) Addition of sodium carbonate
(b) The ratio of ortho : para hydrogen at room (c) Using sodium hexametaphosphate
temperature is 3:1 (d) Synethetic resins and zeolite method.
(c) Entropy of ortho hydrogen is more than para
13. Which of the following statement is not correct
hydrogen at high temperature.
regarding the diagonal relationship between Al
(d) 100% pure ortho hydrogen may be obtained and Be?
at high temperature (a) BeO and Al2O3 are amphoteric in nature.
7. Which of the following is an incorrect statement (b) Al4C3 and Be2C give same gas on hydrolysis.
for heavy water? (c) Both can from complexes with same
(a) It is used as moderator in nuclear reactor maximum co-ordination.
(b) It gives deuteromathane when react with (d) Both form electron deficient and covalent
Al4C3 hydride.
(c) Ionic compounds are more soluble in D2O 14. A+H2O " NaOH
than in H2O
B+H2O " NaOH+O2; A and B are respectively:
(d) Bond energy of D2O is higher than that of
(a) Na2O2 and Na2O
H2O
(b) Na2O and Na2O2
8. In which of the following reaction hydrogen
(c) NaO2 and Na2O2
peroxide is a reducing agent?
(d) Na2O and NaO2
(a) 2FeCl2 + 2HCl + H2O2 " 2FeCl3 + 2H2O
15. Which of the following pair of metal form nitride
(b) Cl2 + H2O2 " 2HCl + O2
on reaction with Nitrogen?
(c) HI + H2O2 " 2H2O + I2
(a) Li, Mg (b) Mg, Na
(d) H2SO3 + H2O2 " H2SO4 + H2O
(c) Al, K (d) Al, Na
9. Which one of the following removes temporary
hardness of water? 16. Which gas responsible for leaving holes in cakes
or pastries and making them light and fluffy?
(a) Slaked lime
(b) Plaster of Paris (a) O2 (b) CO2
(c) CaCO3 (c) H2 (d) CH4
(d) Hydrolith 17. When sodium is placed in moist air, finally change
into:
10. Which physical constant for H2O has higher
magnitude than D2O? (a) NaOH (b) Na2O2
(a) Boiling point (c) Na2O (d) Na2CO3
(b) Temperature of maximum density 18. Which of the following statement is not correct?
(c) Dielectric constant (a) AlCl3 is soluble in excess NaOH and form
(d) Bond dissociation energy soluble complex.
5.16

(b) LiHCO3 is not found in solid state. (c) BeO, Be(OH)2, Be3N2, BeCl2
(c) K2O2 is diamagnetic but KO2 is paramagnetic. (d) BeO, Be3N2, Be, Be(OH)2
(d) Hydrated MgCl2 gives anhydrous MgCl2 on 24. A solid compound X on heating gives CO2 gas
heating in dry air. and a residue. The residue mixed with water
19. Which of the following statement is not correct? forms Y on passing an excess of CO2 through Y
(a) BeF2 forms complex ion with NaF in which in water, a clear solution, Z is obtained. On boiling
Be goes with cation. Z compound X is reformed. The compound X is:

(b) BeCO3 is kept in the atmosphere of CO2 (a) CaCO3 (b) Na2CO3
since it is least thermally stable. (c) K2CO3 (d) Ca(HCO3)2
-2
(c) Be dissolves in alkali forming [Be(OH)4] . 25. Select the incorrect choice:
(d) BeH2 can exist as planar dimer in vapour (a) Solubility of alkaline earth metal’s carbonates,
state. sulphates and chromates decreases from Be
20. CO2 gas along with solid Y is obtained when to Ba.
sodium salt X is heated, X is again obtained when (b) Solubility of alkaline earth metal’s hydroxides
CO2 gas is passed into aqueous solution of Y. X is less than alkali metal hydroxides.
and Y are: (c) Solubility of alkaline earth Metal’s oxides
(a) Na2CO3, Na2O decreases from Be to Ba.
(b) Na2CO3, NaOH (d) SO2 on passing in lime water turns lime water
milky.
(c) NaHCO3, Na2CO3
26. Which of the following statement is not correct?
(d) Na2CO3,
NaHCO3
(a) All alkali-metal salts impart a characteristic
21. Which of the following statement is not correct? colour to the Bunsen flame.
(a) Lithium halide are most covalent among (b) The correct order of increasing thermal
alkali metal halides. stability of the carbonates of alkali metals is
(b) Li2O is more thermal stable than Li2CO3. Li2CO3 < Na2CO3 < K2CO3 < Rb2CO3 <
(c) Except Be halides, all other halides of II A Cs2CO3.
metals are ionic in nature. (c) Among the alkali metal’s cesium is the most
(d) Charge and size ratio for Be+2 and Al+3 is reactive
nearly same. (d) The reducing character of the alkali metal
22. NH3 + H2O + CO2 " A; hydrides follow the order:
A + H2O + CO2 " B LiH > NaH > KH > RbH > CsH.
B + NaCl " C + NH4Cl; 27. Identify the product A,B,C,D of reaction sequence
C " D + H2O + CO2. respectively:
Which of the following is incorrect statement? X NaCl " A + B + Cl2
(a) A is (NH4)2CO3     . Al
(b) D is Na2CO3     NaAlO2 + B(g)

(c) C is NaHCO3 Y A+Cl2 " C + D + H2O
(d) B is (NH4)2C2O4 (a) NaOH, NaCl, NaClO, H2O

(b) Na2CO3, H2, NaCl, NaClO3
23. When powered Be is heated with air, it form
A and B. Compound A gives C after reductive (c) NaOH, H2, NaCl, NaClO3
chlorination. C produces white fumes in presence (d) Na, H2, NaClO3, NaCl
of moisture and forms D. Then A, B, C and D, 28.
Which of the following metal, on burning in moist
respectively, are: air, does not give smell of ammonia?
(a) BeO, Be3N2, BeCl2, Be(OH)2 (a) Mg (b) Ca
(b) Be3N2, BeO, BeCl4-2, Be(OH)2
(c) K (d) Li
5.17

29. Mg2C3 reacts with water forming propyne gas.


C34- ions has:
(a) Two sigma and two pi bonds
ONE OR MORE THAN ONE CORRECT TYPE
(b) Three sigma and one pi bond
(c) Two sigma and one pi bond 1. The reagent(s) used for softening the temporary
hardness of water is (are):
(d) Two sigma and three pi bonds
(a) Ca3(PO4)2 (b) Ca(OH)2
30. The fluoride which is most soluble in water is:
(c) Na2CO3 (d) NaOCl
(a) CaF2 (b) BaF2
2. The oxidation states of the most electronegative
(c) SrF2 (d) BeF2 element in the products of the reaction between
31. Amongst the following hydroxides, the one BaO2 with dilute H2SO4 are:
which has the highest value of Ksp at ordinary (a) -1 (b) +1
temperature? (c) -2 (d) 0
(a) Mg(OH)2 (b) Ca(OH)2 3. Which of the following reaction(s) is/are correct?
(c) Sr(OH)2 (d) Ba(OH)2 (a) Cl2 + NaOH " NaCl + NaClO3 + H2O
32. At high temperature, nitrogen combines with (b) P4 + NaOH + H2O " NaH2PO2 + PH3
CaC2 to give:
D
(a) Calcium cyanide (c) S + NaOH Na2S2O3 + Na2S + H2O
(b) Calcium cyanamide
D
(c) Calcium carbonate (d) Si + NaOH Na2SiO3 + H2
(d) Calcium nitride 4. Which of the following is/are correct?
33. Which metal bicarbonates does not exist in solid (a) Sodium thiosulphate is called hypo.
state? (b) Sodium peroxide is called oxone.
(i) LiHCO3 (ii) Ca(HCO3)2 (c) Potassium carbonate is called pearl ash.
(d) Sodium nitrate is called Indian nitre.
(iii) Zn(HCO3)2 (iv) AgHCO3
5. Which of the following is/are found in the solid
(a) i, ii, iii, iv (b) i, ii, iii
state?
(c) i, ii, iv (d) ii, iii, iv
(a) LiHCO3 (b) KHCO3
34.
The reaction of sodium highly exothermic with
(c) NaHCO3 (d) NH4HCO3
water. The rate of reaction is lowered by:
(a) Lowering the temperature 6. Which of the following compound(s) will impart
a golden yellow colour to the Bunsen flame?
(b) Mixing with alcohol
(c) Mixing with acetic acid (a) KCl (b) K2CO3
(d) Making an amalgam (c) NaCl (d) Na2CO3
35. The alkali metals dissolve in liquid NH3, it is 7. Nitrogen dioxide cannot be obtained by heating:
found that: (a) KNO3 (b) NaNO3
(a) The dilute solution are blue but the (c) AgNO3 (d) Cu(NO3)2
colour changes to bronze with increasing 8. Which of the following metals dissolve in liquid
concentration. ammonia?
(b) The blue solutions is due to the presence of (a) Sr (b) Ca
solvated electrons. (c) Ba (d) Be
(c) The blue solutions are paramagnetic but 9. In which of the following, hydration enthalpy is
the bronze coloured solutions are greater than the lattice enthalpy?
diamagnetic.
(a) BaSO4 (b) BaCO3
(d) All the facts given above are found. (c) Na2SO4 (d) Na2CO3
5.18

PASSAGE BASED QUESTIONS formation of oxides, hydroxides and carbonates on their


Passage # 1 (Q. 10 and 11) surface. When heated in air or oxygen they burn vigorously
forming different types of oxides depending upon the
Hydrogen peroxide is a powerful oxidizing agent,
nature of the metal.
both in the acidic and alkaline medium.
In acidic medium: H2O2 + 2H+ + 2e- " 2H2O The formation and stability of these metals can be
explained on the basis of size of alkali metal ion and the
In alkaline medium: H2O2 + 2e- " 2-OH anion. Peroxides are colourless, while superoxides are
Hydrogen peroxide acts as a reducing agent coloured. The normal oxides are basic while peroxides
towards powerful oxidizing agents. and superoxides. Act as oxidizing agents.
In acidic medium: H2O2 " 2H+ + O2 + 2e- 15. On heating in excess of oxygen, lithium gives:
In alkaline medium, however, its reducing nature (a) Li2O (b) LiO
is more effective. (c) Li2O2 (d) LiO2
- -
H2O2 + 2OH " 2H2O + O2 + 2e 16. On heating excess of oxygen, potassium gives:
10. On addition of H2O2 to acidified KMnO4, KMnO4 (a) K2O (b) KO
gets decolourised due to:
(c) K2O2 (d) KO2
(a) Oxidation of KMnO4
(b) Reduction of KMnO4 Passage # 4 (Q. 17 and 18)
(c) Both oxidation and reduction According to Fajan’s rules, the percentage of covalent
(d) None of the above of KMnO4 character in an ionic compound increases if the cation is
highly charged or small in size and the anion is large or
11. H2O2 behaves as a bleaching agent due to:
cation has pseudo inert gas configuration. As a result of
(a) Oxidizing nature
the increased covalent character, solubility in less polar
(b) Reducing nature solvent increases and the melting point decreases.
(c) Acidic nature
17. Which of the following has the lowest melting
(d) Unstable nature point?
Passage # 2 (Q. 12 to 14) (a) KCl (b) LiCl
1000°c Steam (c) CsCl (d) RbCl
Red hot coke + Steam (X) (Z) + H2. 18. The correct order of increasing ionic character is:
500°c catalyst (y)
(a) BeCl2 < MgCl2 < CaCl2 < BaCl2
12. ‘X’ is: (b) BeCl2 < MgCl2 < BaCl2 < CaCl2
(a) Water gas (b) Producer gas (c) BeCl2 < BaCl2 < MgCl2 < CaCl2
(c) Coal gas (d) Oil gas (d) BaCl2 < CaCl2 < MgCl2 < BeCl2
13. Catalyst ‘Y’ is:
INTEGER VALUE TYPE QUESTIONS
(a) V2O5 (b) Cr2O3
19. What is the sum of protons, electrons and neutrons
(c) Fe2O3 (d) Fe2O3 + Cr2O3
in the lightest isotope of hydrogen?
14. ‘Z’ is removed by passing the gaseous mixture
through 20.
How many moles of phosphine are produced
when one mole of the calcium phosphide reacts
(a) acidic solution
with water?
(b) alkaline solution
(c) water under high pressure of 25 atm 21. Potassium iodide reacts with acidified K2Cr2O7.
How many moles of KI are required for one mole
(d) an organic solvent
of K2Cr2O7?
Passage # 3 (Q. 15 and 16) 22. How many water molecules are associated with
On exposure to air, alkali metals get tarnished due to washing soda?
5.19

MATCH THE COLUMN TYPE QUESTIONS statement-1.


23. (c) Statement-1 is True, statement-2 is False
Column I Column II (d) Statement-1 is False, statement-2 is True
A Calgon 1. More reactive form of hydrogen [IIT-2007]
as compared to H2 2. The reagent(s) used for softening the temporary
B D2O 2. Open book-type structure hardness of water is (are):
C Nascent hydrogen 3. Sodium polymetaphosphate (a) Ca3(PO4)2 (b) Ca(OH)2
D H2O2 4. Heavy water (c) Na2CO3 (d) NaOCl [IIT-2010]
3. Hydrogen peroxide in its reaction with KIO4 and
24.
NH2OH respectively, is acting as a:
Column I Column II
(a) Reducing agent, oxidizing agent
2+
A Sodium ion in zeolite gets 1. Ca
(b) Reducing agent, reducing agent
exchanged with
(c) Oxidizing agent, oxidizing agent
B Hardness 2. Mg2+
(d) Oxidizing agent, reducing agent
C Temporary hardness 3. Ca(HCO3)2
[JEE Advanced - 2014]
D Permanent hardness 4. MgSO4
4. A piece of magnesium ribbon was heated to
25. redness in an atmosphere of nitrogen and on
Column I Column II cooling water was added, the gas evolved was-
A Gives CO2 on heating 1. Na (a) Ammonia (b) Hydrogen

B Pink-violet flame colouration 2. Cs (c) Nitrogen (d) Oxygen


[AIEEE - 2005]
C Forms superoxide on heating 3. K2CO3
With O2 5. The ionic mobility of alkali metal ions in aqueous
D Used in photoelectric cells 4. NaHCO3
solution is maximum for-
(a) Rb+ (b) Li+
+
E Form monoxide on heating 5. K (c) Na (d) K+
with oxygen
[AIEEE - 2006]
F Forms peroxide on heating 6. Li
with oxygen 6. In context with the industrial preparation of
hydrogen from water gas (CO+H2), which of the
following is the correct statement?
(a) CO is removed by absorption in aqueous
Cu2Cl2 solution
(b) H2 is removed through occlusion with Pd
(c) CO is oxidized to CO2 with steam in the
Statement-1: Alkali metals dissolve in liquid
1. presence of a catalyst followed by absorption
ammonia to give blue solutions. of CO2 in alkali
(d) CO and H2 are fractionally separated using
Statement-2: Alkali metals in liquid ammonia

difference in their densities
give solvated species of the type [M(NH3)n]+
[AIEEE - 2008]
(M = alkali metals).
7. The products obtained on the heating LiNO3 will
(a) Statement-1: is True, statement-2 is True; be-
Statement-2 is a correct explanation for (a) Li2O + NO2 + O2
statement-1. (b) Li3N + O2
(b) Statement-1: is True, statement-2 is True; (c) Li2O + NO + O2
(d) LiNO2 + O2
Statement-2 is not a correct explanation for [AIEEE - 2011]
5.20

8. What is the best description of the change that (d) Reaction of methane with steam
occurs when Na2O(s) is dissolved in water? [AIEEE - 2012]
(a) Oxide ion accepts a pair of electrons 11. The solubility order for alkali metal fluoride in
(b) Oxide ion donates a pair of electrons water is-
(c) Oxidation number of oxygen increases (a) LiF < RbF < KF < NaF
(d) Oxidation number of sodium decreases (b) RbF < KF < NaF < LiF
[AIEEE - 2011] (c) LiF > NaF > KF > RbF
9. Which of the following on thermal decomposition (d) LiF < NaF < KF < RbF
yields a basic as well as an acidic oxide?
[JEE Main Online - 2013]
(a) KClO3 (b) CaCO3
12. In which of the following reactions H2O2 acts as a
(c) NH4NO3 (d) NaNO3 reducing agent?
[AIEEE - 2012] (i) H2O2 + 2H+ + 2e-" 2H2O
10. Pure hydrogen (99.9%) can be made by which of (ii) H2O2 – 2e- " O2 + 2H+
the following processes? (iii) H2O2 + 2e-" 2OH-
(a) Mixing natural hydrocarbons of high (iv) H2O2 + 2OH- - 2e- " O2 + 2H2O
molecular weight (a) (iii), (iv) (b) (i), (iii)
(b) Electrolysis of water (c) (ii), (iv) (d) (i), (ii)
(c) Reaction of slat like hydrides with water
[JEE Main - 2014]

Answer Key

1. (b) 2. (d) 3. (d) 4. (c) 5. (c) 6. (a) 7. (a) 8. (a) 9. (c) 10. (c)
11. (b) 12. (b) 13. (d) 14. (d) 15. (c) 16. (b) 17. (a) 18. (b) 19. (b) 20. (c)
21. (d) 22. (b) 23. (a) 24. (b) 25. (c) 26. (c) 27. (a) 28. (d) 29. (c) 30. (b)
31. (b) 32. (c) 33. (b) 34. (c) 35. (d)

1. (b) 2. (b) 3. (b) 4. (b) 5. (d) 6. (d) 7. (c) 8. (b) 9. (a) 10. (c)
11. (b) 12. (c) 13. (c) 14. (b) 15. (a) 16. (b) 17. (d) 18. (d) 19. (a) 20. (c)
21. (c) 22. (d) 23. (a) 24. (a) 25. (c) 26. (d) 27. (c) 28. (c) 29. (a) 30. (d)
31. (d) 32. (b) 33. (a) 34. (d) 35. (d)

      1. (b, c, d) 2. (a, c) 3. (a, b, c, d) 4. (a, b, c) 5. (b, c, d) 6. (c, d) 7. (a, b) 8. (a, b, c)


9. (c, d) 10. (b) 11. (a) 12. (a) 13. (d) 14. (c) 15. (a) 16. (d)
17. (b) 18. (a) 19. (2) 20. (2) 21. (6)    22. (10)
     23. (A 3; B 4; C 1; D 2)
    24. (A 1, 2; B 1, 2; C 3; D 4)
    25. (A 4; B 3; C 2, 5; D 2; E 1,2,5, 6; F 1)
5.21

1. (a) 2. (b, c, d) 3. (a) 4. (a) 5. (a) 6. (c) 7. (a) 8. (b) 9. (b) 10. (c)
11. (d) 12. (c)

Hints and Solutions


Mg+2 do not give flame test.
19. (b) In alkali metals (IA), metallic character and ionic
(b) LiH + H2O LiOH + H2 radius increases as the atomic number rises.

(d) Highly electropositive elements (s-block metals) 20. (c) Solutions of alkali metals in liquid ammonia are
can form ionic hydrides. paramagnetic in nature.

(d) Isotopes have different nuclear mass. 21. (d) Na2CO3 + CO2 + H2O " NaHCO3

(c) CaH2 (Molten) Ca+2 + 2H- (this reaction is not possible)

at anode:- 2H- H2 + 2e- The actual reaction is,



(c) Temporary hardness of water can be removed by 2NaHCO3  → Na2CO3 + CO2 + H2O
adding calculated amount of Ca(OH)2. 22. (b) Order of solubility of hydroxides:-

a) Mg (HCO ) 
3 2
→ MgCO + CO + H O
3 2 2 Be(OH)2 < Mg(OH)2 < Ca(OH)2 < Sr(OH)2 <
(a) Ca + H2 CaH2 Ba(OH)2

R.A O.A 23. (a) Order of thermal stability:-

8. (a) Intermolecular interaction present in H2 is Na2CO3 < K2CO3 < Rb2CO3 < Cs2CO3
dispersion forces. 24. (b) Cause of blue colour is presence of ammoniated
9. (c) Adsorption of H2 by various metals is also known electron.
as occulusion. 25. (c) Down the group, basic strength of hydroxides
10. (c) In PbO2, oxidation state of ‘O’ is ‘-2’. increases.
26. (c) 2LiNO3 

→ Li2O + 2NO2 + 1 O2
11. (b) Ortho and para – hydrogen possess different
2
physical properties but same chemical properties. ∆
27. (a) MgCl2. 6H2O  → MgO + HCl
12. (b) O3 is a stronger oxidizing agent than H2O2. O3
28. (d) Mg displaces H2 from acids
will oxidize H2O2 into O2
29. (c) CaH2 + 2H2O Ca(OH)2 + 2H2
13. (d) Due to presence of unpaired e- in superoxide ion,
they are paramagnetic, coloured and oxidizing 30. (b) Zn + 2NaOH Na2ZnO2 + H2 (Inflammable
agents. gas)

1 H 31. (b) Li is the strongest reducing agent because Li+ has



14. (d) Na + NH3  
→ NaNH2 + 2 exceptionally high hydration energy.
2
15. (c) Be + 2NaOH Na2BeO2 + H2 32. (c) Order of melting point is,
water soluble NaCl > KCl > CsCl > LiCl
16. (b) Microcosmic salt is Na(NH4)HPO4. 4H2O 33. (b) Both Li+ and F- ions are very small in size. Hence,
lattice energy of LiF is very high and it has lowest
Glauber’s salt is Na2SO4. 10H2O
solubility in water amongst alkali metal fluorides.
17. (a) Be is less reactive s-block metal. It does not form
34. (c) Raw materials used in Solvay process are CaCO3,
hydride by direct heating with H2.
NH3 and NaCl but only CaCO3 and NaCl are
18. (b) Due to high ionisation energy, salts of Be+2 and consumed during the preparation of Na2CO3.
5.22

35. (d) Be and Al show diagonal relationship. Maximum Na + O2 Na2O


co-ordination number of Be is 4 while that of Al Na2O + H2O NaOH
is 6. NaOH + CO2 Na2CO3 + H2O
Hydrated MgCl2 (MgCl2.6H2O) gives MgO on
heating in dry air.
BeF2 + 2NaF Na2[BeF4]
1. (b) at anode, (NH4)2S2O8 is formed.
In this complex, Be present in anionic part.
at cathode, H2 is formed.
Salt (X) is NaHCO3.
a blue colour is obtained due to formation of
2NaHCO3 ∆ → Na2CO3 + CO2 + H2O
chromic peroxide, CrO5 or CrO(O2)2.
(Y)
[Fe(CN)6]3– is reduced into [Fe(CN)6]4–
Halides of Mg are also covalent.
2-Ethylanthraquinol on oxidation give H2O2
B is NH4HCO3.
Permanent hardness can not be removed by
boiling water. Only temporary hardness can be Be + air ∆→ BeO + Be3N2
removed by boiling water.
(A) (B)
BeO + chlorination BeCl2
100% pure orthohydrogen can not be obtained
(C)
even at high temperature.
BeCl2 + moisture Be(OH)2
Ionic compounds are more soluble in polar (D)

solvents having higher value of dielectric CaCO3 ∆ → CaO + CO2
constant. H2O has higher dielectric constant than
(X)
D2O.

CaO + H2O Ca(OH)2


Cl2 acts as oxidizing agent and H2O2 acts as
(Y)
reducing agent.

Ca(OH)2 + excess CO2 Ca(HCO3)2


Ca(HCO3)2 + Ca(OH)2 2CaCO3 + 2H2O

(Z)
(Hardness) (Slaked lime)
Order of solubility :
H2O has higher dielectric constant than D2O.
BeO < MgO < CaO < SrO < BaO
It is thermodynamically unstable.
Correct order of reducing character :
2H2O2 2H2O + O2
LiH < NaH < KH < RbH < CsH
Sodium hexametaphosphate (Calgon) forms
soluble complex with Ca+2 and Mg+2. Electrolysis
NaCl(aq) NaOH + H2 + Cl2

Na2[Na4(PO3)6] + 2Ca+2    Na2[Ca2(PO3)6] + 4Na+
(A) (B)
13. (c) Be shows maximum co-ordination number 4

while due to pressence of vacant d-orbitals Al Al


shows maximum co-ordination number 6.
NaAlO2 + H2
Na2O + H2O NaOH
(B)
(A)

NaOH + Cl2 NaCl + NaClO3 + H2O


Na2O2 + H2O NaOH + O2
(B) Potassium (K) does not form nitride on burrning
in air.
15. (a) Na, K, Rb and Cs do not form nitride on reaction
with nitrogen. Structure of C 4–
3 is,


C C–C –3
Li and Mg form nitride with nitrogen.
It has 2 sigma and 2 pi bonds.

16. (b) In cakes or pastries, NaHCO3 (baking soda) is
added. During baking of cake, CO2 gas is released Order of solubility in water :
which makes cake light and fluffy. BeF2 > BaF2 > SrF2 > CaF2
5.23

Ksp µ solubility With H2O2, KMnO4 behaves as oxidising agent.


Order of solubility : H2O2 behaves as a bleaching agent due to its
Mg(OH)2 < Ca(OH)2 < Sr(OH)2 < Ba(OH)2 oxidizing nature. ∆ Steam

→ CO + H Catalyst
 → CO2 + H 2

CaC2 + N2  → CaCN2 Red hot coke + steam ∆
→ 
2 (Z)
Water gas
(X)
        (Calcium cyanamide)
∆ Steam
→ CO + H 2 Catalyst
 → CO2 + H 2
Only bicarbonates of Na+, K+, Rb+ and Cs+ exist is   (Z)
Water gas
solid state.         
Catalyst ‘Y’ is( XFe
)
O + Cr2O3
2 3
Hg is less reactive metal. Hence, by making an
CO2 is removed by passing the gaseous mixture
amalgam the rate of reaction of Na with water is
through water under high pressure of 25 atm.
lowered.
CO2 + H2O H2CO3
35 (d) The alkali metals dissolve in liquid NH3.

Li + O2(excess) 
This solution is blue coloured due to solvated → Li2O
electrons. As concentration of metal increases, K + O2(excess) ∆
→ KO2
colour changes to bronze.
Order of melting point:
KCl > RbCl > CsCl > LiCl
Covalent character µ polarising power of cation.
Order of ionic character is reverse of order of
1.(b, c, d)
covalent character.
Ca(OH)2 (slaked lime), Na2CO3 (washing soda)
Lightest isotope of hydrogen is protium. It has 1
and NaOCl are used for softening the temporary
proton, 1 electron and zero neutron.
hardness.
NaOCl + H2O " NaOH + HOCl Ca3P2 + 6H2O 3Ca(OH)2 + 2PH3
OH– + HCO3– "CO32– + H2O
1 mole 2 moles
BaO2 + H2SO4 (dilute) "BaSO4 + H2O2 K2Cr2O7 + 7H2SO4 + 6KI " Cr2(SO4)3 + 3I2
The most electronegative element in products is + 7H2O + 4K2SO4

oxygen. In H2O2, oxidation state of ‘O’ is –1 and
1 mole 6 moles
in BaSO4, oxidation state of ‘O’ is –2.
Washing soda is Na2CO3.10H2O
3. (a, b, c, d)
(A "3; B "4 ; C "1; D " 2)
All reactions are correct.
(A "1, 2; B "1, 2 ; C "3; D "4)
4. (a, b, c)
Indian nitre is potassium nitrate (KNO3) (A "4; B "3 ; C "2, 5 ; D "2 ; E "1, 2, 5,
6; F "1]
5. (b, c, d)
LiHCO3 exists only in solution.
6. (c, d) Na+ will impart a golden yellow colour to the
Bunsen flame.
7. (a, b) Nitrates of Na+, K+, Rb+ Cs+ do not release NO2
gas by heating. 1. (b) Both statements are correct but blue colour is due
to presence of solvated e–.
8. (a, b, c)
In s-block, Be and Mg do not dissolve in liquid 2. (b, c, d)
ammonia. Ca(OH)2 + Ca(HCO3)2 2CaCO3 + 2H2O
9. (c, d) For salts, which are soluble in water, hydration Na2CO3 + Ca(HCO3)2 CaCO3 + 2NaHCO3
enthalpy is greater than the lattie enthalpy. NaOCl + H2O NaOH + HOCl

Na2SO4 and Na2CO3 are soluble in water. OH– + HCO3– CO3–2 + H2O
5.24
1
+7 +5 2LiNO3 ∆
→ Li2O + 2NO2 + O2
KIO 4 + H 2 O2 → KIO3 + H 2 O + O2 2
Re ducing Na2O + H2O 2NaOH
agent
O + H
–2 +
OH –
−1 +3
NH 2 OH + H 2 O2 → N 2 O3 + H 2 O

Oxide ion (O–2) donates a pair of electrons
Oxidizing
agent CaCO3 ∆
→ (CaO
Basic )
CO2
(acidic )
3Mg + N2 ∆
→ Mg3N2 NaH +H2O NaOH + H2
Mg3N2 + 6H2O 3Mg(OH)2 + 2NH3- Order of solubility in water:
Hydrated ion of Rb+ is smallest among these four. LiF < NaF < KF < RbF

CO + H2 Steam Removal of e– is known as oxidation.


 , Catalyst
→ CO2 + H2

absorption in alkali
Chapter

Key Concepts

5d-series or III transition series:-

 The elements lying in the middle of the periodic table


between group 2 and group 13 are known as d-block
elements.
 These d-block elements are called transition elements
because they exhibit transitional behaviour between
s-block and p-block elements.
 Depending upon the subshell (3d, 4d, 5d) involved, 1. Metallic character:- All the transition elements are
transition elements are mainly classified into three metallic in nature and nearly all of them have simple
series. hcp, ccp or bcc lattices. Due to their greater effective
1. First transition series or 3d series. nuclear charge and the large number of valence
2. Second transition series or 4d series. electrons, the metallic bond is quite strong and
3. Third transition series or 5d series. hence they are hard, posses high densities and high
Outer electronic configuration of the transition enthalpies of atomization.
elements 2. Oxidation states:- Transition elements exhibit variable
3d-series or I transition series:- oxidation state due to the participation of ns as well as
(n –1) d electrons.
Except scadium, the most common oxidation state of
the first row (3d series) elements is +2 which arises
from the loss of two 4s electrons, which means
that after scandium, d-orbital become stable than
4d-series or II transition series:- s-orbital.
In the +2 and +3 oxidation states, bond formed are
generally ionic while in higher oxidation states the
bond formed are essentially convalent. For example in
MnO4–, CrO42–, etc. the bond formed between metal
and oxygen are covalent.
6.2

Sc Ti V Cr Mn Fe Co Ni Cu Zn (i) Due to their variable oxidation state, they form


+1 (+1) unstable intermediate compounds and provide
+2 +2 +2 (+2) (+2) (+2) (+2) (+2) (+2) a new path with lower activation energy for the
(+3) (+3) (+3) (+3) +3 (+3) (+3) (+3) reaction (intermediate compound formation
(+4) +4 +4 +4 +4 +4 +4 theory).
(+5) +5 +5 (ii) In some cases the finely divided metal or
(+6) +6 +6 their compounds provide a large surface area
(+7) for adsorption and the adsorbed reactants react
faster due to the closer contact (Adsorption
Oxidation states of the first row of transition metals (the
theory).
most common ones are in circles)
3. Complex formation (complexation):- Transition 8. Ionization energy:- The ionization energies of
metal ions form variety of complex due to the transition elements are higher than those of S-block
following reasons: elements but lower than p-block elements.
(i) Small size and high nuclear charge In a particular transition series, ionization
energy increases gradully as we move from left
(ii) Availability of vacant d-orbital of suitable energy, to right, and it is due to the increase in nuclear
which can accept lone pair of electrons donated charge.
by the molecule or ion (ligand).
4. Magnetic Properties:- Two types of magnetic
provide an indication of the energy needed to
behaviour are found in substances diamagnetism
raise the metal to a particular oxidation state in
and paramagnetism. Paramagnetic substances are
a compound. From the knowledge of values of
attracted by the magnetic field and weigh more while
ionization energies of the metal it is possible
the diamagnetic substances are slightly repelled by
to rationalize the relative stabilities of various
the magnetic field and weight less.
oxidation state.
As the transition metal ions generally contain one
or more unpaired electrons in them and hence their
stable than Pt(II) compounds, on the other hand
complexes are generally paramagnetic. Paramagnetic
Pt(IV) compounds are more stable than Ni(IV)
character increases with increase in number of
compounds. It is due to that sum of first four
unpaired electrons. Paramagnetism is expressed in
ionization energies is less for platinum whereas
terms of magnetic moment.
sum of the first two ionization energies is less for
µ = n(n 2) BM (Bohr magneton) nickel.
n – number of unpaired electrons 9. Coloured compounds:- Compounds of transition
More the magnetic moment, more will be the elements are usually coloured due to the promotion
paramagnetic character. of an electron from one d-orbital to another by the
5. Formation of Alloys:- As the transition elements absorption of visible light. It can be clearly explained
have similar atomic sizes hence in the crystal lattice, as follows:
one metal can be readily replaced by another metal In the transition elements which have partly filled
giving solid solution and smooth alloys. The alloys so d-orbitals, the transition of electron can take from
formed are hard and have often high melting point. one of the lower d-orbitals to some higher d-orbital
6. Interstitial compound:- Transition metal form within the same subshell. The energy required for this
number of interstitial compounds, in which they take transition falls in the visible region. So when white
up atoms of small size, e.g., H, C and N in the vacant light falls on these complexes they absorb a particular
spaces in the their lattices. The presence of these colour from the radiation for the promotion of electron
atoms results in decrease in ductility and malleability and the remaining colours are emitted. The colour of
of the metals but increases their tensile strength. the complex is due to this emitted radiation.
7. Catalytic properties:- Transition metals and their A few of the transition metal ions such as Cu+, Ag+,
compounds are known to act as a good catalyst due to Sc3+ are colourlesss. In these ions, the d-orbital are
the following reasons: either completely filled or empty.
6.3

becomes yellow and on cooling it becomes white


(this is due to change in the structure of lattice).
(A) Ferric Oxide, Fe2O3 2. It is reduced to Zn by the reaction of charcoal or
dry H2.
Preparation:

ZnO + H2 $ Zn + H2O
1. In lab, it can be prepared by heating ferrous
ZnO + C $ Zn + CO
sulphate or ferric hydroxide.
3. It dissolves readily in mineral acids forms the
2 FeSO4 ∆
→ Fe2O3 + SO2- + SO3-
corresponding salts and with alkalies, it forms
2Fe (OH)3 ∆
→ Fe2O3 + 3H2O zincates [Zn (OH)4]2– or ZnO22–.
ZnO + 2HCl $ ZnCl2 + H2O
Fe2(SO4)3 ∆ → Fe2O3 + 3SO3-
ZnO + 2NaOH + H2O $ Na2 [Zn(OH)4]
2. 4FeS2 + 11O2 $ 2Fe2O3 + 8SO2
ZnO + 2NaOH $ Na2ZnO2 + H2O
4FeS + 7O2 $ 2Fe2O3 + 4SO2
3. Hydrolysis of FeCl3 actually gives a red- (C) Ferric chloride (FeCl3)
brown gelatinous precipitute of the hydrous Preparation:
oxide Fe2O3(H2O)n which on heating at 200°C 1. Anhydrous ferric chloride is obtained by passing
gives red-brown Fe2O3 (which occurs as the dry chlorine gas over heated iron filling
mineral haematite). 2Fe + 3Cl2 ∆ → Fe2Cl6 or 2FeCl3
Properties:
(anhydrous)
1. It is reddish brown powder, insoluble in water but 2. Hydrated FeCl3.6H2O can be prepared by
soluble in acid. Amphoteric in nature and reacts dissolving iron in aqua regia or iron oxide in
with acids and alkalies. hydrochloride acid then the crystallization of the
Fe2O3 + 6HCl $ 2 FeCl3 + 3H2O solution.
Fe2O3 + Na2CO3 $ 2NaFeO2 + CO2 2Fe + 4HCl + Cl2 $ 2FeCl3 + 2H2
Sodium ferrite Fe2O3 + 6HCl $ 2FeCl3 + 3H2O
Fe2O3 + 2NaOH $ 2NaFeO2 + H2O Properties:

2. It liberates oxygen at 1300° C. 1. Anhydrous FeCl3 is deep red-black flaky crystals
6Fe2 O3 1300° C
→ 4Fe3O4 + O2 but hydrated FeCl3.6H2O is yellowish brown,
deliquescent solid, soluble in water, alcohol and
3. It is reduced to Fe as: ether.
Fe2O3 + 3C $ 2Fe + 3CO
Fe2O3 + 3H2 $ 2Fe + 3H2O 2. It dissociates on heating above 973 K first into
Fe2O3 + 3CO $ 2Fe + 3CO2 FeCl3 and then into FeCl2 and Cl2

Fe2Cl6 973 K
→ FeCl3  → FeCl2 + Cl2-
(B) Zinc Oxide (ZnO)
3. Its aqueous solution is acidic in nature due to
Preparation:
hydrolysis.
1. It is prepared by burning zinc metal in air or by FeCl3 + 3H2O $ Fe(OH)3 + 3HCl
heating the zinc carbonate, zinc nitrate or zinc
hydroxide. 4. Fe3+ solution gives deep blue ppt. of Prussian
2Zn + O2 $ 2ZnO blue with K4[Fe(CN)6], potassium ferrocyanide:

ZnCO3  → ZnO + CO2 4FeCl3+3K4[Fe(CN)6]$Fe4[Fe(CN)6]3+12KCl
2Zn(NO3)2 ∆ → 2ZnO + 4NO2 + O2          Prussian blue
Zn(OH)2  ∆
→ ZnO + H2O FeCl3 + K4[Fe(CN)6] $ KFe[Fe(CN)6] + 3KCl

2. 2ZnS + 3O2 $ 2ZnO + 2SO2 5. 2FeCl3 + H2S $ 2FeCl2 + 2HCl + S (oxidizing)


Properties: 6. Fe3+ solution gives blood red colour with SCN–
1. It is a white, light powder insoluble in water ions:
and known as philospher's wool. On heating it Fe3+ + SCN– $ [Fe(SCN)]2+
6.4

(D) Zinc Chloride, (ZnCl2.2H2O) 3. On heating, green FeSO4 7H2O gives a white
Preparation: anhydrous salt. On strong heating it forms Fe2O3,
SO2 and SO3.
1. ZnO + 2HCl $ ZnCl2 + H2O
ZnCO3+ 2HCl $ ZnCl2 + CO2 + H2O FeSO4.7H2O ∆
→ FeSO4 + 7H2O
White
Zn(OH)2 + 2HCl $ ZnCl2 + 2H2O
The solution on concentration and cooling give 2FeSO4 ∆
→ Fe2O3 + SO2- + SO3-
hydrated zinc chloride crystals ZnCl2.2H2O. Brown

2. Anhydrous ZnCl2 is obtained by heating zinc in 4.


FeSO4 + (NH4)2SO4 + 6H2O $ FeSO4.(NH4)2
the atmosphere of dry Cl2 or Dry HCl gas. SO4.6H2O
(Mohr salt)
Zn + Cl2 $ ZnCl2
Zn + 2HCl $ ZnCl2 + H2 5. Aqueous solution of FeSO4 is acidic due to
hydrolysis of Fe2+:
3. Anhydrous ZnCl2 can also be formed by distilling
zinc powder with mercuric chloride. Fe2+ + 2H2O    Fe(OH)2 + 2H+

Zn + HgCl2 $ ZnCl2 + Hg 6. Acidified MnO4– and Cr2O72– oxidize Fe2+ to
Properties:
Fe3+.

1. Anhydrous zinc chloride is white solid, MnO4– + 8H+ + 5Fe2+ $ 5Fe3+ + Mn2+ + 4H2O

deliquescent and soluble in water. It melts at 660° Cr2O72– + 14H+ + 6Fe2+$ 6Fe3+ + 2Cr3+ + 7H2O
and boils at 730°C.
(F) Copper sulphate(CuSO4.5H2O) or Blue vitriol
2. Hydrated ZnCl2 on heating from zinc
Preparation:
hydroxychloride or zinc oxychloride.
ZnCl2.2H2O $ Zn(OH) Cl + HCl + H2O 1. It is prepared by dissolving copper (II) oxide or
copper (II) carbonate in dil Sulphuric acid.
2ZnCl2.2H2O $ Zn2OCl2 + 2HCl + 3H2O
CuO + H2SO4 $ CuSO4 + H2O
Zinc oxychloride
CuCO3 + H2SO4 $ Cu SO4 + CO2 + H2O
3.
4ZnCl2 + 4Na2CO3 + 3H2O $ ZnCO3.3Zn(OH)2
+ 8NaCl + 3CO2 On evaporation, solution is concentrated, blue
crystal of CuSO4.5H2O separate out on cooling.
ZnCl2+2NaHCO3$ZnCO3+2NaCl+H2O+CO2
2. On a large scale, copper (II) sulphate is obtained
4. ZnCl2 + 4NH3 $ ZnCl2.4NH3 by passing air through a hot mixture of copper
(E) Ferrous sulphate (FeSO4.7H2O) or green and dil H2SO4
vitriol 2Cu + 4H+ + O2 $ 2Cu2+ + 2H2O
Preparation: 3. CuFeS2 + 4O2 $ CuSO4 + FeSO4
1. It is prepared by the action of dil. H2SO4 on iron. Properties:
Fe + H2SO4 $ FeSO4 + H2 1. On heating CuSO4 . 5H2O looses water molecules
This should be mode in a reducing atmosphere in as follows.
order to prevent the oxidation of Fe2+ into Fe3+. 373 K
CuSO4.5H2O → CuSO4.H2O
2.
FeCO3 + H2SO4 $ FeSO4 + H2O + CO2 432 K strong
→ CuSO4 heating
→ CuO + SO3
3. FeS + H2SO4 $ FeSO4 + H2S (kipp's apparatus)
Properties: 2. CuSO4 + 6NH4OH $[Cu(NH3 )4 ](OH)2 +
Schweitzer ’s reagent
1. It is pale green crystalline solid, soluble in water. (NH4)2SO4 + 4H2O
It turns brown due to oxidation into the ferric 3.
2CuSO4 + 4KI $ Cu2I2. + I2-+ 2K2SO4 (CuI is
compound by atmospheric oxygen. not formed)
2. It is an effloresccent substance, and in isomorphous 4. A mixture of copper sulphate and lime, under the
with Epson salt (MgSO4.7H2O) and white vitriol name of Bordeaux mixture is used as fungicide in
(ZnSO4.7H2O). agriculture.
6.5

5.
2CuSO4 + 2KSCN + SO2 + 2H2O $ 2CuSCN.
2AgNO3 + 2NaOH $ Ag2O. + 2NaNO3+ H2O
+ K2SO4 + 2H2SO4   Brown ppt.
(G) Zinc sulphate: ZnSO4.7H2O (white vitriol) 2AgNO3 + 2NH4OH$Ag2O. + 2NH4NO3+H2O
Ag2O + 4NH4OH $ 2[Ag(NH3)2]OH + 3H2O
Prepareation:
6. Ammonical AgNO3 is called Tollen's reagent and
1. It is prepared by the action of dil. H2SO4 on Zn
is used to identify reducing sugars (including
metal or its oxide or carbonate
aldehydes). It is called silver mirror test of
ZnCO3 + H2SO4 $ ZnSO4 + H2O + CO2 aldehydes and reducing sugar (like glucose,

2ZnS + 3.5O2  → ZnO + ZnSO4 + SO2 fructose).
ZnO + H2SO4 $ ZnSO4 + H2O 7. It dissolves in excess of KCN:
Properties: AgNO3 + KCN $ AgCN + KNO3
White ppt.
1. It is highly soluble in water. The solution is acidic
AgCN + KCN $ K[Ag(CN)2]
in nature due to hydrolysis.
Soluble potassium
ZnSO4 + 2H2O $ Zn(OH)2 + H2SO4
argentocyanide
2. ZnSO4.7H2O 100°
∆
C
→ ZnSO4.6H2O 280°
∆
C
→ (I) Silver (I) oxide (Ag2O)
ZnSO4 Preparation:
760° C
3. ZnSO4 → ZnO + SO3 2AgNO3 + 2NaOH$Ag2O. + 2NaNO3 + H2O
4. It is isomorphous with Epsom salt and green Properties:
vitriol. 1. It is brownish powder, insoluble in water,
(H) Silver Nitrate, AgNO3 (Lunar Caustic):- thermally unstable and soluble in aqueous
ammonia.
Preparation:
2. It decomposes to silver and oxygen.
1. It is prepared by dissolving the metal in dilute 2Ag2O $ 4Ag + O2
nitric acid and crystallizing the solution
3Ag + 4HNO3 $ 3AgNO3 + 2H2O + NO- (J) Silver thiosulphate (Ag2S2O3)
Preparation:
Properties:

1. With Na2S2O3,
1. It is a colourless crystalline solid, soluble in water
2AgNO3 + Na2S2O3$Ag2S2O3. + 2NaNO3
and alcohol; melting point 212°C.
white ppt.
2. On heating, it gives metallic silver and nitrogen
2AgBr + Na2S2O3 $ Ag2S2O3. + 2NaBr
dioxide.
∆ Properties:

2AgNO3 450°  C
→ 2Ag + 2NO2 + O2
1. Ag2S2O3 + 3Na2S2O3 2Na3[Ag(S2O3)2]
3. It reacts with iodine in two ways
sodium
5AgNO3 + 3I2 (excess) + 3H2O $ HIO3 + 5AgI argentothiosulphate
+ 5HNO3
(K) Potassium Dichromate (K2Cr2O7)
6AgNO3 (excess) + 3I2 + 3H2O $ AgIO3 + 5AgI Preparation:
+ 6HNO3
1. It is prepared from chromite ore (FeCr2O4)
4. It gives turbidity with tap water (Cl–) and turbidity
4FeCr2O4 + 16NaOH + 7O2(air) $
is soluble in NH4OH.
8Na2CrO4 + 2Fe2O3 + 8H2O
AgNO3 + Cl– $ AgCl. + NO3
  Sodium chromate
tap water turbidity
Or, 4FeCr2O4 + 8Na2CO3 + 7O2 $
AgCl + 2NH4OH $ Ag(NH3)2Cl + 2H2O
8Na2CrO4 + 2Fe2O3 + 8CO2
(soluble)
2Na2CrO4 + dil. H2SO4$Na2Cr2O7+Na2SO4+H2O
5. When treated with alkali, it gives precipitate of   Sodium dichromate
silver oxide, which dissolves in excess of NH4OH.
Na2Cr2O7 + 2KCl $ K2Cr2O7 + 2NaCl
6.6

Properties:
(L) Potassium Permanganate (KMnO4)
1. It is orange-red crystalline compound having Preparation:
melting point 670 K. 1. Potassium permanganate is prepared from mineral
2. It is moderately soluble in cold water but readily pyrolusite (MnO2).
soluble in hot water. 2. Steps involved are:
3. 4K2Cr2O7 ∆
→ 4K2CrO4 + 2Cr2O3 + 3O2- (a) 2MnO2+4KOH+O2 

→ 2K2 MnO4+2H2O
Potassium chromate chromic acid Pot. Manganate
acid acid
4. 2CrO24 – +
+ 2H 2HCrO 4− (Green mass)
alkali alkali
chromate
( yellow )
Hydrogen
chromate
or, 2MnO2 + 2K2CO3 + O2 ∆
→ 2K2 MnO4
+ 2CO2
Cr2 O2− 7 + H2O
dichromate
( orange )
(b) 2K2 MnO4 + Cl2 $ 2KCl + 2K MnO4
5. Action with HCl: or, 2K2 MnO4 + H2O + O3 $ 2KMnO4 +
K2Cr2O7 + 14HCl $ 2KCl+2CrCl3+7H2O+3Cl2-­ 2KOH + O2
Chlorine or, 2K2 MnO4 + 2CO2 $2K2CO3 + 2MnO2.
6. Chromyl chloride Test (This is the test of + 2KMnO4
chloride): Properties:

K2Cr2O7 + 6H2SO4 + 4NaCl $ 2KHSO4 + 1. It is a dark violet crystalline solid having a
4NaHSO4 + 2CrO2Cl2+ 3H2O metallic luster (meeting point 523 K). It is fairly
chromyl chloride soluble in water giving a purple solution.
4 N a O H   +   C r O 2 C l 2 $ Na 2 CrO 4 2. 2KMnO4 ∆
→ K2MnO4 + MnO2 + O2-
+2NaCl + 2H2O yellow solution
(sod. Chromate ) 3. Action of alkalies:
4KMnO4 + 4KOH $ 4K2MnO4 + 2H2O + O2-
7. Oxidising character: 4. Oxidizing character:
(a) Both Na2Cr2O7 and K2Cr2O7 are oxidizing
Potassium permanganate acts as an oxidizing
agents but K2Cr2O7 is preferred since it is
agent in neutral, alkaline and acidic solutions.
not hygroscopic and can be used as primary
standard. (a) In neutral medium MnO2 is formed.
(b) The dichromates act as powerful oxidising 2KMnO4 + 3H2S $ 2KOH + 2MnO2 + 3S
agent in acidic medium. + 2H2O
K2Cr2O7 + 4H2SO4 $ K2SO4 + Cr2(SO4)3 2KMnO4 + 3MnSO4 + 2H2O $ 5MnO2 +
K2SO4 + 2H2SO4
+ 4H2O + 3[O]
nascent oxygen 2KMnO4 + 3Na2S2O3 + H2O $ 2MnO2 +
3Na2SO4 + 2KOH + 3S
(c) Some examples are:
(b) In alkaline medium, MnO2 is formed.
K2Cr2O7 + 6KI + 7H2SO4 $ 4K2SO4 +
Cr2(SO4)3 + 7H2O + 3I2 2KMnO4 + KI + H2O $ 2MnO2 + 2KOH
+ KIO3
K2Cr2O7 + 7H2SO4 + 6FeSO4 $3Fe2(SO4)3
+ K2SO4 + Cr2(SO4)3 + 7H2O (c) In acidic medium, Mn2+ is formed.
2KMnO4 + 10 FeSO4 + 8H2SO4 $
K2Cr2O7 + 4H2SO4 + 3H2S $ K2SO4 +
2MnSO4 + K2SO4 + 5Fe2(SO4)3 + 8H2O
Cr2(SO4)3 + 3S + 7H2O
K2Cr2O7 + 3Na 2 SO3 + 4H2SO4 $ K2SO4 2KMnO4 + 8H2SO4 + 10 KI $ 2MnSO4 +
(sod. Sulphite ) 6K2SO4 + 8H2O + 5I2
+ Cr2(SO4)3 + 3Na 2 SO 4 + 4H2O 2KMnO4 + 3H2SO4 + 5H2S $ 2MnSO4 +
(sod. Sulphate )
K2SO4 + 8H2O + 5S
K2Cr2O7 + H2SO4 + 3SO2 $ K2SO4 +
Cr2(SO4)3 + H2O 2KMnO4 + 5SO2 + 2H2O $ K2SO4 +
2MnSO4 + 2H2SO4
6.7

2KMnO4 + 3H2SO4 + 5KNO2 $ K2SO4 + Outer Electronic configuration of Lanthanum and


2MnSO4 + 5KNO3 + 3H2O Lanthanides
  Atomic Element Electronic
2KMnO4 + 10HI + 3H2SO4 $ K2SO4 +
2MnSO4 + 8H2O + 5I2  Number Name Configuration
57 La 5d1 6s2
2KMnO4 + 5C2H2O4 + 3H2SO4 $ K2SO4  +
2MnSO4 + 10CO2 + 8H2O 58 Ce 4f1 5d1 6s2
59 Pr 4f3 6s2
60 Nd 4f4 6s2
61 Pm 4f5 6s2

The elements from atomic number 58 (cerium) to 71 62 Sm 4f6 6s2


(lutetium) are known as lanthanides as they follow the 63 Eu 4f7 6s2
element lanthanum (atomic number 57). These elements 64 Gd 4f7 5d1 6s2
are placed together at the bottom of the periodic table.
These elements are characterized by the filling up of 65 Tb 4f9 6s2
the antipenultimate 4f energy levels. The compound of 66 Dy 4f10 6s2
these elements show +III oxidation state and form ionic 67 Ho 4f11 6s2
compounds. Some of the elements also show +II and +IV
oxidation states. 68 Er 4f12 6s2
Many trivalent lanthanide ions are coloured both in the 69 Tm 4f13 6s2
solid state and in aqueous solution. The colour seems to 70 Yb 4f14 6s2
depend on the number of unpaired electrons. Elements 71 Lu 4f14 5d1 6s2
with nf electrons offer a similar colour to those with
(14-n) f electrons. The atomic and ionic sizes of lanthanides Outer Electronic configuration of Actinium and
progressively decreases from the first element to the last Actinoids
element. The contraction is about 20 pm and is known as   Atomic Element Electronic
the lanthanide contraction. This is due to the poor shielding  Number Name Configuration
effect of 4f electrons causing more and more attraction 89 Ac 6d1 7s2
between the nucleus and the outer electrons.
90 Th 6d2 7s2
The properties of an ion depends on its size and its charge.
91 Pa 5f2 6d1 7s2
Because of the very small decrease in ionic size, the
chemical properties of lanthanides are very similar. The 92 U 5f3 6d1 7s2
sizes of the last four elements of the lanthanide series 93 Np 5f4 6d1 7s2
become lower than that of the element Y of the preceding 94 Pu 5f6 7s2
trasition series. Also the elements which follows in the
third transition series are considerable smaller than 95 Am 5f7 7s2
the expected value. The pairs Zr-Hf, Nb-Ta and Mo-W 96 Cm 5f7 6d1 7s2
have almost identical sizes. The sizes of the third row of 97 Bk 5f9 7s2
transition elements are very similar to those of the second
row elements. 98 Cf 5f10 7s2

The elements from atomic numeber 90 (Thorium) to 103 99 Es 5f11 7s2


(Lawrencium) are known as actinides as they follow the 100 Fm 5f12 7s2
element actinium (atomic number 89). The actiinides also 101 Md 5f13 7s2
have an oxidation state of +III but this state is not always
102 No 514 7s2
the most stable state. All the actinides are very reactive
and show the phenomenon of radioactivity. Their melting 103 Lr 5f14 6d1 7s2
points are moderating high but are considerable low as
compared to those of transition elements. They also show
actinide contraction to the lanthanide contraction.
6.8

Solved Examples

1. The oxidation number is changed in which of the Zn2+ + 2NaOH $ Zn(OH)2 + 2Na+
following case- (ppt.)
(a) SO2 gas is passed into Cr2O72–/H+ (Amphoteric in nature)
(b) Aqueous solution of CrO42– is acidified Zn(OH)2 + 2NaOH $ Na2[Zn(OH)4]
             Soluble
(c) CrO2Cl2 is dissolved in NaOH
(d) Cr2O72– solution is made alkaline 6. Black coloured solid (A) KNO 3 + KOH
 ∆
→ green
Sol.(a)     +6 +4    +3  +6
coloured solution (B) CO
 2
→ (C) + (A)
(a) Cr2O72– + SO2 + 2H+ " 2Cr3++3SO42–+ H2O Pink
+6     +6 2+
(C) is decolorised by Fe . Identify (A), (B) and
(b) 2CrO42– + 2H+ " Cr2O72– + H2O (C). Explain the reaction.
+6 +6 Sol. (A) MnO2 (B) K2MnO4 (C) KMnO4
(c) CrO2Cl2 + 2NaOH " Na2CrO4
MnO2 + KOH + O2 (KNO3) ∆

+6 +6 –
K2MnO4 CO2
→ MnO4 + MnO2
(d) CrO72–+ 2OH– " 2CrO42– + H2O
(B)      (C)   (A)
2. What happen when FeSO4(X) is subjected to CO2 + H2O ) H2CO3 ) H+ + HCO3–
heating, compound R,S,T are obtained. R is red- 3MnO42– + 4H+ $ MnO2 + 2MnO4– + 2H2O
brown solid, S can be oxidized to (T). Identify
R,S,T. 7. Explain why mercury (I) ion exists as Hg22+ ion
Sol. 2FeSO4 ∆ white copper (I) ion exists as Cu+ ion.
→ Fe2O3 + SO2 + SO3
(X) (R) (S) (T) Sol. Hg(Z = 80) & 4f14 5d10 6s2; Hg≈ & 4f14 5d10 6s1
Red-Brown Hg+ has one electron in its valence 6s-orbital, due
3. Heating of Ag with dil. HNO3 give to this, Hg+ compounds should be paramagnetic
(a) NO (b) NO2 but actually they are diamgnetic. Hence, the
single filled 6s-orbitals of the two Hg≈ ions
(c) N2O (d) N2O3
overlap from a Hg–Hg covalent bond. Thus, Hg≈
Sol.(a) 2HNO3(dil.) " 2NO + H2O + 3[O] ions exist as dimeric species, i.e., Hg22+.
6Ag + 8HNO3 $ 6AgNO3 + 2NO + 4H2O
Cu(Z = 29) & 3d10 4s1
4. When excess of SnCl2 is added to HgCl2, the Cu≈ & 3d10
substance formed is –

Therefore, Cu≈ ion has no unpaired electron to
(a) Hg2Cl2 (b) Sn
form dimeric species. i.e., Cu22+ and hence, it
(c) Hg (d) Cl2 always exists as Cu≈ ion.
Sol.(c) SnCl2 reduces HgCl2 to Hg2Cl2 and finally to Hg- 8. Why hydrated copper sulphate is blue while
2HgCl2 + SnCl2 $ Hg2Cl2 + SnCl4 unhydrous copper sulphate is white?
Hg2Cl2 + SnCl2 $ 2Hg + SnCl4
Sol. In CuSO4. 5H2O, four water molecules are
present as ligand. In the presence of these ligands
5. A mixture of Mn2+ & Zn2+ can be separated by
d-orbitals are no longer degenerate in energy.
using an excess of-
Hence d-d transition takes place absorbing red
(a) NH4OH (b) NaOH wavelength. The complementary colour, viz,
(c) H2SO4 (d) HNO3 blue is reflected. In anhydrous CuSO4, d-orbitals
remain degenerate. Hence, no d-d transition can
Sol.(b) Mn2+ + 2NaOH $ Mn(OH)2 + 2Na+ occur. The white light is completely reflected
(ppt.) back. Hence, it looks white.
6.9

9. (a) Of the lanthanides, cerium (Z = 58) forms a coloured compound B.


tetrapositive ion, Ce4+ in aqueous solution. (b) The solution of B in boiling water on
Why? acidification with dilute H2SO4 gives a pink
(b) The +3 oxidation states of lanthanum (Z = coloured compound C.
57), gadolinium (Z = 64) and lutetium (Z = (c) The aqueous solution of A on treatement with
71) are especially stable. Why? NaOH and Br2 - water gives a compound D.
(c) Why do Zr and Hf or Nb and Ta exhibit (d) A solution of D in conc. HNO3 on treatment
similar properties ? with lead peroxide at boiling temperature
(d) Which out of the two, La(OH)3 and Lu(OH)3, produced a compound E which was of the
is more basic and why? same colour as that of C.
Sol. (a) Ce3+ having the configuration 4f15d06s0 (e) A solution of A in dilute HCl on treatment
can easily loose an electron to acquire the with a solution of barium chloride gave a
configuration 4f0 and form Ce4+. In fact, this white precipitate of compound F which was
is the only +4 state lanthanide which exists in insoluble in conc. HNO3 and conc. HCl.
solution.
Sol. A is MnSO4
(b) This is because they have empty, half-
filled and completely filled 4f subshells (a) MnSO4 + Na2CO3 + 2KNO3 " Na 2 MnO 4
(A) ( B) Green coloured
respectively.
+ 2KNO2
(c) Due to the consequence of lanthnoid
contraction, Hf (Z = 72) has size similar to (b) 3Na2MnO4 + 2H2SO4 " 2NaMnO 4 +
that of Zr (Z = 40). Hence, their properties are ( C ) Pink coloured

similar. For the same reason. Nb and Ta have MnO2 + 2Na2SO4 + 2H2O
similar size and hence similar properties.
(c) MnSO4 + 4NaOH + Br2 " MnO2 + Na2SO4
(d) La(OH)3 is more basic than Lu(OH)3. As ( D)
the size of the lanthanid ions decreases from + 2NaBr + 2H2O
La3+ to Lu3+, the covalent chracter of the
(d) 2MnO2 + 10HNO3 + 5PbO2 " 2HMnO 4
hydroxides increases (Fajan’s rules). Hence, ( E ) Pink coloured
the basic strength decreases from La(OH)3 to + 5Pb(NO3)2
Lu(OH)3.
10. Identify A to F. (e) MnSO4 + BaCl2 " BaSO4 + MnCl2
( F ) ( Inso lub le in conc. HNO3
(a) A powdered substance A on fusion with and conc. HCl )
(Na2CO3 + KNO3) mixture gives a green

Exercise
(a) Enthalpy of sublimation of the metal
(b) Ionization energy
1. Which of the following statements concerning
(c) Enthalpy of hydration of the metal ion
transition elements, is not true?
(d) All of these
(a) They are all metals.
3. Which of the following is likely to form white
(b) They easily form complexes.
salts?
(c) Compounds containing their ions are
coloured. (a) Cu2+ (b) Sc3+
(d) They show multiple oxidation states always (c) Ti3+ (d) Fe2+
differing by two units. 4. Brass is an alloy of
2. The stability of particular oxidation state of a (a) Silver and copper (b) Copper and zinc
metal in aqueous solution is determined by: (c) Copper and tin (d) Copper, zinc and tin
6.10

5. Zr and Hf have almost equal atomic and ionic 14. The metals which are present in insulin and
radii because: vitamin B12 respectively are:
(a) of diagonal relationship. (a) Zn, Co (b) Fe, Cr
(b) both are in the same group.
(c) Co, Fe (d) Zn, Fe
(c) of lanthanide contraction.
(d) they have same outermost shell. pH = X
15. CrO24− 

  2−
 Cr2 O7
pH = Y
6. Which of the following compounds is expected to
be coloured? The pH values of X and Y are respectively:

(a) Ag2SO4 (b) CuF2 (a) 4 and 5 (b) 4 and 8


(c) MgF2 (d) CuCl (c) 8 and 4 (d) 8 and 9

7. Stainless steel contains 16. In which of the following oxoanions, the oxidation
state of the central atom is not the same as that of
(a) Fe + Cr + Cu (b) Fe + C + Ni
its group number in the periodic table?
(c) Fe + Cr + Ni (d) Fe + Ni + Cu
(a) MnO4– (b) Cr2O72–
8. The catalytic activity of the transition metals and
(c) VO43– (d) FeO42–
their compounds is ascribed to
(a) Their chemical reactivity 17. Interstitial compounds are not formed by:
(b) Their magnetic behavior (a) Co (b) Ni
(c) Their unfilled d-orbitals (c) Fe (d) Ca
(d) Their ability to adopt multiple oxidation
18. Which compound does not exist?
states and their complexing ability.
(a) MnF6 (b) MnF4
9. In the reaction Zn + NaOH ∆
→ X, the product
X is: (c) MnO3F (d) MnO4–2
(a) Na2ZnO2 (b) 2NaZnO2 19. The incorrect match is:
(c) Zn (OH)2 (d) None of these (a) CrO5 peroxide
(b) Mn2O7 Acidic oxide
10. Which of the following is not correctly matched?
(c) CrO3 Amphoteric
(a) SiC – Covalent carbide
(d) FeO Basic oxide
(b) WC – Interstitial carbide
(c) Al4C3 – Ionic carbide 20. Solder is an alloy of:
(d) B4C – Molecular carbide (a) Pb + Sn (b) Mg + Al
11. Which of the following is not a property of (c) Cu + Sn (d) Al + Mn + Cu
intersitial compounds? 21. Most common oxidation states are matched below
(a) Neither ionic nor covalent with the elements. Which one is mismatched ?
(b) High chemically reactivity (a) Iron (+2, +3)
(c) Retain metallic conductivity
(b) Chromium (+1, +2)
(d) Non-stoichiometric compound
(c) Manganese (+2, +7)
12. K2MnO4 can be converted into KMnO4 by: (d) Titanium (+3, +4)
(a) Passing CO2 gas
22. Which of the following pair of ions has same
(b) By passing Cl2 value of “spin-only" magnetic moment:
(c) Electrolytic oxidation
(a) Cu+, Cu2+ (b) Co3+, Fe2+
(d) All of these
(c) Ti2+, Cu2+ (d) Sc2+, Zn+2
13. Which of the following metals of 3d series do not
show variable oxidation state? 23. CO2 and SO2 gas can be distinguish by:

(a) Sc, Ti (b) Ti, Cu (a) Slaked lime (b) Beryta water

(c) Sc, Zn (d) Co, Ni (c) Acidified KMnO4 (d) All of these
6.11

24. Acidified K2Cr2O7 can not oxidise: products obtained from it in the three conditions
(a) Green vitriol (b) Mohr’s salt are, respectively

(c) Ferric oxalate (d) Ferric sulphate (a) MnO42–, Mn3+ and Mn2+
(b) MnO2, MnO42– and Mn2+
25. In which of the following oxo-anion, all M–O
bond length are not identical? (c) MnO2, MnO2+ and Mn3+
(a) MnO4– (b) MnO4–2 (d) MnO2 , MnO2 and Mn3+
(c) Cr2O7–2 (d) CrO4–2 3.
Amongst TiF62–, CoF63–, Cu2Cl2 and NiCl42– (Atomic
numbers : Ti = 22, Co = 27, Cu = 29, Ni = 28) the
26. Which of the following is not a similarity between colourless species are
sulphur and chromium?
(a) TiF62– and Cu2Cl2
(a) Both exhibit hexacovalency
(b) Cu2Cl2 and NiCl42–
(b) Sulphate and chromate of Ba2+ are water

insoluble (c) TiF62– and CoF63–


(c) Trioxide (MO3) both are acidic

(d) CoF63– and NiCl42–
(d) Sulphate (SO42–) and chromate (CrO42–) have 4.
CrO3 dissolves in aqueous NaOH to give:
same colouration (a) CrO42– (b) Cr(OH)3
27. Copper (II) ions gives reddish brown precipitate (c) Cr2O7 2–
(d) Cr(OH)2
with potassium ferrcroyanide. The formula of the
5.
A compound of a metal ion Mx+ (Z = 24) has a spin
precipitate is:
only magnetic moment of 15 Bohr Magnetons.
(a) Cu[Fe(CN)6] (b) Cu2[Fe(CN)6] The number of unpaired electrons in the compound
(c) Cu3[Fe(CN)6] (d) Cu3[Fe(CN)6]2 are:
(a) 2 (b) 4
28. CeO2 is:
(a) A good oxidising agent (c) 5 (d) 3
(b) Diamagnetic in nature 6.
Which one of the following statement is not correct?
(c) Colourless compound (a) La(OH)3 is less basic than Lu(OH)3.
(d) All of these (b) In lanthanide series, ionic radius of Ln3+ ions
29. Which of the following show highest oxidation decreases.
state? (c) La is actually an element of transition series
rather than lanthanide series.
(a) Cl (b) Mn
(d) Atomic radii of Zr and Hf are same because of
(c) Np (d) All of these lanthanide contraction.
30. Which of the following ion has maximum
7.
Which of the following compounds has colour but
complex forming tendency?
no unpaired electrons?
(a) La+3 (b) Ce+3
(a) KMnO4 (b) K2MnO4
(c) Eu+3 (d) Lu+3
(c) MnSO4 (d) MnCl2

8.
Zn gives H2 gas with H2SO4 and HCl but not with
HNO3 because:
1.
Number of Cr—O bonds in dichromate ion (Cr2O72–) (a) Zn acts as an oxidising agent when react with
is: HNO3.
(a) 6 (b) 7 (b) HNO3 is weaker acid than H2SO4 and HCl.
(c) 8 (d) 4 (c) In electrochemical series Zn is above hydrogen.
2
Potassium permanganate acts as an oxidant in (d) NO3– ion is reduced in preference to hydronium
neutral, alkaline as well as acidic media. The final ion.
6.12

9.
Which of the following is incorrectly matched? (X) + K2CO3 + Air Heat
18.  → (Y).
Catalyst Process (Y) + Cl2 $ (Z) Pink.
(a) V2O5 Contact process Which of the following is correct?
(b) Cu2Cl2 Sandmeyer reaction (a) X = Black, MnO2, Y = Blue, K2CrO4, Z =
(c) Finely divided Fe Vegatable oil to ghee KMnO4.
(d) TiCl4 + Al(CH3)3 Zieglar Natta Catalyst (b) X = Green, Cr2O3, Y = Yellow, K2CrO4, Z =
10.
An inorganic molecule X on heating gives green K2Cr2O7.
colouration and evolve O2 gas. The X is: (c) X = Black, MnO2, Y = Green, K2MnO4, Z =
KMnO4.
(a) (NH4)2Cr2O7 (b) K2Cr2O7
(d) X = Black, Bi2O3, Y = Colourless, KBiO2, Z =
(c) RbCrO4 (d) CrO2Cl2 KBiO3.
11.
Consider the following reaction: 19.
When acidified KMnO4 is added to hot oxalic
2Cu2+ + X– $ Cu2X2(s) + X2 acid solution, the decolourization is slow in the
Then X– can be: beginning but becomes very rapid after some time.

This is because:
(a) F– (b) Cl–
(a) Mn+2 acts as autocatalyst
(c) Br– (d) I–
(b) CO2 is formed as the product
12. In which reaction no colour change will be observed? (c) Reaction is exothermic
(a) K2Cr2O7 CO  2
→ (d) MnO4– catalyses the reaction
SO2

(b) K 2
Cr O
2 7   → 20.
Which of the following statements are correct when
CO2
(c) Na2CrO4  → a mixture of NaCl and K2Cr2O7 is gently warmed

(d) Na2S 
Na 2 [ Fe ( CN )5 NO ]
→ with conc. H2SO4?
13.
Which of the following property first increases then (A) a deep red vapour is evolved.
decreases on moving from Sc to Zn? (B) the vapour when passed into NaOH solution
(a) Paramagnetism gives a yellow solution of Na2CrO4
(b) Heat of atomisation
(C) Chlorine gas is evolved
(c) Maximum oxidation state (D) chromyl chloride is formed
(d) All of these (a) A, B, D (b) A, B, C
14.
Coloured and paramagnetic oxoanion is: (c) B, C, D (d) all are correct
(a) MnO 4 –
(b) CrO4 –2
21.
Mercury (II) chloride solution on reaction with
(c) MnO4 –2
(d) Cr2O7 –2 gaseous ammonia forms:
15.
Product formed when Au react with aquaregia is: (a) Hg(NH2)Cl.HgO
(b) Hg(NH3)Cl2
(a) AuCl (b) AuCl3
(c) [Hg(NH3)4]Cl2
(c) Au(NO3)3 (d) HAuCl4

(d) [Hg(NH3)2]Cl
16.
When KMnO4 react with H2O2 in slightly alkaline
AgNO3 ∆
22. → (W) + (X) + O2
and acidic medium, the respective products obtained:
(a) K2MnO4 and Mn2+(aq) (X) + H2O $ HNO2 + HNO3
(b) MnO2 and MnO2
(W) + HNO3 $ Y + NO + H2O
(c) MnO2 and Mn2+(aq) (Y) + Na2S2O3 (excess) $ (Z) + NaNO3
(d) Mn2+ (aq) and MnO2 Identify (W) to (Z).

17.
K2Cr2O7 when reacts cold conc. H2SO4 gives red (a) W = Ag; X = N2O; Y = AgNO3; Z = Na2
crystal of:
[Ag(S2O3)2]
(a) CrO4–2 (b) CrO3 (b) W = Ag2O; X = NO; Y = AgNO3; Z = Na3
[Ag(S2O3)2]
(c) Cr2(SO4)3 (d) Cr2O3
6.13

(c) W = Ag; X = NO2; Y = AgNO3; Z = Na3



[Ag(S2O3)2]
(d) W = AgO ; X = N2; Y = AgNO3; Z = Na
One or more than one correct type

[Ag(S2O3)2]
1.
The metal oxide which decomposes on heating is/
23. Which of the following electronic configuration
are:
is associated with the highest stable oxidation
state? (a) ZnO (b) Al2O3
(a) [Ar] 3d1 4s2 (b) [Ar] 3d5 4s1 (c) Ag2O (d) HgO
(c) [Ar] 3d5 4s2 (d) [Ar] 3d6 4s2 2.
Which of the following acids attack(s) on copper
and silver?
24.
A white precipitate of AgCl dissolves in excess of:
(a) dilute HNO3 (b) dilute HCl
(I) NH3(aq) (II) Na2S2O3
(c) conc. H2SO4 (d) aqua regia

(III) NaCN
3.
Which statements are correct regarding copper
(a) III only (b) I, II, III
sulphate?
(c) I, II (d) I only (a) It reacts with NaOH and glucose to give Cu2O.
25.
Zinc (II) ion on reaction with NaOH first gives (b) It reacts with KCl to give Cu2O.
a white precipitate which dissolves in excess of (c) It gives CuO on strong heating in air.
NaOH due to the formation of:
(d) It reacts with KI to give brown colouration.
(a) ZnO (b) Zn(OH)2
4.
Pick out the correct statements (s):
(c) [Zn(OH)4] 2–
(d) [Zn(H2O)4]2+ (a) MnO2 dissolves in conc. HCl, but does not form
26.
Dilute nitric acid on reaction with silver liberates: Mn4+ ions.
(a) NO gas (b) NO2 (b) Decomposition of acidic KMnO4 is not
catalysed by sunlight.
(c) N2 gas (d) O2 gas
(c) MnO42– is strongly oxidising and stable only
27.
Acidified permanganate solution does not oxidize:
in very strong alkali. In dilute alkali, water or
(a) C2O42– (aq.) (b) NO2– (aq.) acidic solutions, it disproportionates.
(c) S2– (aq.) (d) F–(aq.) (d) KMnO4 does not act as oxidising agent in
28.
Which of the following characteristic is not the point alkaline medium.
of resemblance between lanthanoids and actinoids? 5.
The species that undergoes disproportionation in an
(a) Reducing property alkaline medium are:
(b) Oxidation state of +3 (a) Cl2 (b) MnO42–
(c) Trends of ionic radii for M+3 ions (c) NO2 (d) ClO4–
(d) Radioactivity 6.
Mercuric chloride is converted into mercury by:
29.
Which of the following statement is not correct? (a) Placing copper metal in aqueous solution of
(a) Lu+3 has the strongest tendency toward complex HgCl2.
formation among trivalent lanthanoid ions. (b) Treating aqueous solution of HgCl2 with excess
(b) Ce has maximum composition in misch metal. of stannous chloride.
(c) f-block elements can have electrons from f0 to (c) Treating aqueous solution of HgCl2 with PbCl4
f14. solution.
(d) Nd, Np and Nb all are f-block elements. (d) None of these.
30.
Which of the following lanthanoid has one electron 7.
Choose correct statement (s) regarding the following
in 6d subshell? reaction:
(a) La (b) Ce Cr O2 − + 3SO2 − + 8H + → 2Cr 3+ +
2 7(aq.) 3(aq.) (aq.)
(c) Gd (d) None of these 3SO24(−aq.) + 4H 2 O
6.14

(a) Cr2 O27 is an oxidising agent. 13.


Yellow ppt in the above observation is :
(a) Mercuric oxide
(b) SO32 is a reducing agent.
(b) Basic mercury (II) sulphite
(c) The oxidation number of per S-atom in
(c) Basic mercury (II) sulphate
SO32 is increased by two.
(d) Mercuric iodide
(d) The oxidation number of per Cr-atom in is
Passage # 2 (Q. 14 and 15)
Cr2 O72(aq.) decreased by three.
MnO2 is the most important oxide of maganese. It occurs
8.
Transition elements have greater tendency to form
naturally as the black coloured mineral pyrolusite. It is an
complexes because they have:
oxidising agent, and decomposes to Mn3O4 on heating to
(a) vacant d-orbitals 530°C. It is used in the preparation of potassium permanganate
(b) small size and in the production of Cl2 gas. Over half a million tonnes
(c) higher nuclear charge per year of MnO2 is used in dry batteries.
(d) variable oxidation states 14.
When MnO2 is fused with KOH in the presence at
9.
Which of the following ions give(s) coloured air, the product formed is:
aqueous solution? (a) purple colour KMnO4
(a) Ni2+ (b) Fe2+ (b) green colour K2MnO4
(c) Cu2+ (d) Cu+ (c) colourless MnO4–
10.
What are the characteristics of products obtained (d) purple colour K2MnO4
when green vitriol is strongly heated? 15.
In which of the following species, the colour is due
(a) Basic oxide (b) Neutral oxide to charge transfer.
(c) Acidic oxide (d) Reducing agent (I) [Mn(OH)4]2– (II) MnO42–
11.
Which of the following statements are correct when
(III) MnO2 (IV) KMnO4
a mixture of NaCl and K2Cr2O7 is gently warmed
(a) I, II, III (b) II, IV
with conc. H2SO4?
(c) I, II (d) Only IV
(a) Deep red vapours are liberated
(b) Deep red vapours dissolve in NaOH forming Passage # 3 (Q. 16 and 17)
a yellow solution. Iron forms iron halide salts by reacting the metal directly with
(c) Greenish yellow gas is liberated halogen. Fel3 does not exist. FeF3 is white solid inspite of five
(d) Deep red vapours dissolve in water forming unpaired electrons with d5 configuration. FeCl3 is soluble in
yellow solution water and is used as a mordant in dyeing industry.

PASSAGE-BASED QUESTIONS 16.


FeI3 does not exist because:
Passage # 1 (Q. 12 and 13) (a) of its large size.
(b) Fe3+ oxidises I– to I2.
∆

High
Temp.
 → (c) of low lattice energy.
(d) iodine is not highly electronegative enough to
(i) ‘D’ and ‘E’ are two acidic gas.
oxidise Fe to Fe3+.
(ii) ‘D’ is passed through HgCl2 solution to give yellow
precipitate. 17.
FeCl3 solution added to K4[Fe(CN)6] gives A while
with KSCN gives B. A and B respectively are:
(iii) ‘E’ is passed through water first and then H2S is
passed, white turbidity is obtained. (a) Fe3[Fe(CN)6]2, Fe(CNS)3
(iv) A is water soluble and addition of HgCl2 in it, white (b) Fe4[Fe(CN)6]3, KFe(CNS)3
ppt is obtained but white ppt does not turn into grey (c) Fe4[Fe(CN)6]3, K3[Fe(CNS)6]
on addition of excess solution of ‘A’. (d) Fe4[Fe(CN)6]3, K3[Fe(SCN)6]
12.
‘D’ and ‘E’ are respectively. Passage # 4 (Q. 18 and 19)
(a) SO2 and SO3 (b) SO3 and SO2 Pyrolusite ore on oxidation with KClO3/KNO3 in basic
(c) SO2 and CO2 (d) CO2 and CO medium produces dark green coloured compounds (A),
6.15

which on electrolysis produces a purpule coloured compound


(B). The purple coloured compound can be crystallisd to
Column I Column II
deep purple rhombic prisms. It shows different reactions
in different mediums. Excess of compound (B) on heating
with concentrated H2SO4 gives an explosive oil (C), which
on heating decomposes to give another compound (D) along
with oxygen.
18.
The nature of compound (C) is:
(a) basic (b) acidic
(c) neutral (d) amphoteric
19.
Identify (D)
(a) Mn2O7 (b) MnO2
(c) MnSO4 (d) Mn2O3

INTEGER VALUE TYPE QUESTIONS


20.
Sum of highest stable oxidation states of following
elements is:
Sc, Zn, Ti, Mn, Cr
1.
Match the reactions in column-I with nature of
21.
Determine total number of unpaired electrons in the reaction/type of the products in column-II
following ions.
Column-I Column-II
Ti3+, V3+, Cr3+, Cr2+, Mn3+, Fe3+, Fe2+, Co2+, Ni2+, Cu2+
∆ O2– " O2 + O22–
FeC2O4 
22. → products (A) (p) Redox reaction

Number of dimagnetic products = x (B) CrO2− +


4 +H "
(q) One of the products
has trigonal planar
Number of unpaired electrons in paramagnetic structure
product = y
(C) MnO4– + NO2– + (r) Dimeric bridged
Report your answer as (x + y). H+ " tetrahedral metal ion
NO3– + H2SO4 +
+
23. H
KMnO4  → Mnx (D) (s) (s) Disproportionation
R . A. Fe2+ "
OH −

KMnO4 R
. A.
→ Mny [IIT-2007]


2. Among the following, the coloured compound is:

K Cr O7 OH
 → Crz
2 2 (a) CuCl (b) K3[Cu(CN)4]

x + y + z is:
(c) CuF2 (d) [Cu(CH3CN)4]BF4
(here x, y and z are oxidation states)
[IIT-2008]
COLUMN MATCHING TYPE QUESTIONS The oxidation number of Mn in the product of
3.
24. alkaline oxidation fusion of MnO2 is:
Column I Column II [IIT-2009]
(A) Kipp’s apparatus waste (P) (NH4)2SO4.FeSO4.6H2O Reduction of the metal centre in aqueous
4.
(B) Green coloured (Q) Cu(OH)2. CuCO3 permanganate ion involves :
compound (a) 3 electrons in neutral medium
(C) Leave(s) brown residue (R) FeSO4 (b) 5 electrons in neutral medium
on heating (c) 3 electrons in alkaline medium
(D) Leave(s) black residue (S) CuCl2.2H2O (d) 5 electrons in acidic medium
on heating [IIT-2011]
6.16

The colour of light abosrbed by an aqueous


5. (d) La and Lu have partially filled d orbitals and
solution of CuSO4 is: no other partially filled orbitals.
(a) orange-red (b) blue-green [AIEEE-2007]

(c) yellow (d) violet 11.


The actinoids exhibit more number of oxidation
[IIT-2012] states in general than the lanthanoids. This is
because:
The correct statement(s) about Cr2+ and Mn3+ is/
6.
(a) The actinoids are more reactive than the
are:
lanthanoids.
[Atomic numbers of Cr = 24 and Mn = 25] (b) The 5f orbitals extend farther from the
(a) Cr2+ is a reducing agent nucleus than the 4f orbitals.
(b) Mn3+ is an oxdizing agent (c) The 5f orbitals are more buried than the 4f
(c) Both Cr2+ and Mn3+ exhibit d4 electronic orbitals
configuration (d) There is a similarity between 4f and 5f orbitals
(d) When Cr2+ is used as a reducing agent, in their angular part of the wave function.
the chromium ion attains d5 electronic [AIEEE-2007]
configuration. 12.
Larger number of oxidation states are exhibited
[JEE (Advanced) 2015] by the actinoids than those by the lanthanoids, the
Fe3+ is reduced to Fe2+ by using
7. main reason being:
(a) H2O2 in presence of NaOH (a) lesser energy difference between 5f and 6d
(b) Na2O2 in water than between 4f and 5d orbitals
(c) H2O2 in presence of H2SO4 (b) more energy difference between 5f and 6d
than between 4f and 5d orbitals
(d) Na2O2 in presence of H2SO4
(c) more reactive nature of the actinoids than the
[JEE (Advanced) 2015] lanthanoids
The “spin-only” magnetic moment [in units of
8. (d) 4f orbitals more diffusion than the 5f orbitals.
Bohr magneton, (µs) of Ni2+ in aqueous solution [AIEEE-2008]
would be (atomic number of Ni = 28)
13.
In context with transition elements, which of the
(a) 2.84 (b) 4.90
following statements is incorrect?
(c) 0 (d) 1.73
(a) In the highest oxidation states, the transition
[AIEEE-2006] metal show basic character and forms cationic
9.
Lanthanoid contraction is caused due to: complexes.
(a) the appreciable shielding on outer electrons (b) In the highest oxidation states, of the first five
by 4f electrons from the nuclear charge. transition elements (Sc to Mn), all the 4s and
(b) the appreciable shielding on outer electrons 3d electrons are used for bonding.
by 5f electrons from the nuclear charge. (c) Once the d5 configuration is exceeded, the
(c) the same effective nuclear charge from Ce to tendency to involve all the 3d electrons in
Lu. bonding decreases.
(d) the imperfect shielding on outer electrons by (d) In addition to the normal oxidation states, the
4f electrons from the nuclear charge. zero oxidation state is also shown by these
[AIEEE-2006] elements in complexes.
10.
Identify the incorrect statements among the [AIEEE-2009]
following: 14.
Knowing that the chemistry of lanthanoids (Ln) is
(a) The chemistry of various lanthanoids is very dominated by its +3 oxidation state, which of the
similar. following statements is incorrect?
(b) 4f and 5f orbitals are equally shielded. (a) The ionic sizes of Ln(III) decreases in general
(c) d-block elements show irregular and erratic with increasing atomic number.
chemical properties among themselves. (b) Ln(III) compounds are generally colourless.
6.17

(c) Ln(III) hydroxides are mainly basic in (d) Ferrous compouds are more easily hydrolysed
character. than the corresponding ferric compounds.
(d) Because of the large size of the Ln(III) ions the [AIEEE-2012]
bonding in its compounds is predominently Which of the following arrangements does not
18.
ionic character. represent the correct order of the property stated
[AIEEE-2009] against it?
In context of the lanthanoids, which of the
15. (a) V2+ < Cr2+ < Mn2+ < Fe2+; paramagnetic
following statement is not correct? behaviour
(b) Ni2+ < Co2+ < Fe2+ < Mn2+: ionic size
(a) There is a gradual decreases in the radii of the
members with increasing atomic number in (c) Co3+ < Fe3+ < Cr3+ < Sc3+: stability in aqueous
the series. solution
(d) Sc < Ti < Cr < Mn: number of oxidation
(b) All the member exhibit +3 oxidation state.
states
(c) Because of similar properties the separation [JEE-Main - 2014]
of lanthanoids is not easy.
Which series of reactions correctly represents
19.
(d) Availability of 4f electrons results in the chemical relations related to iron and its
formation of compounds in +4 state for all compound?
the members of the series.
[AIEEE-2011] (a) Fe dil

H 2 SO 4 H 2 SO 4 ,O2
→ FeSO4  →
The outer electron configuration of Lu (Atomic
16. Fe2(SO4)3 heat → Fe
O2 , heat dil H 2 SO 4
number : 71) is: (b) Fe  → FeO  → FeSO4
(a) 4f3 5d5 6s2 (b) 4f8 5d10 6s2
heat
 → Fe
heat , air
(c) 4f4 5d4 6s2 (d) 4f14 5d1 6s2 (c) Fe Cl 2 , heat
 → FeCl3  → FeCl2
[AIEEE-2011] →
Zn Fe
Iron exhibits +2 and +3 oxidation states. Which of
17. (d) Fe O
2 , heat
→ Fe3O4 CO , 600°C
 → FeO
the following statements about iron is incorrect? CO , 700°C
 → Fe
(a) Ferrous oxide is more basic in nature than the [JEE-Main - 2014]
ferric oxide.
The colour of KMnO4 is due to:
20.
(b) Ferrous compounds are relatively more ionic (a) M " L charge transfer transition
than the corresponding ferric compounds (b) d " d transition
(c) Ferrous compounds are less volatile than the (c) L " M charge transfer transition
corresponding ferric compounds. (d) s " s transition
[JEE-Main - 2015]

Answer Key

1. (d) 2. (d) 3. (b) 4. (b) 5. (c) 6. (b) 7. (c) 8. (d) 9. (a) 10. (d)
11. (b) 12. (d) 13. (c) 14. (a) 15. (b) 16. (d) 17. (d) 18. (a) 19. (c) 20. (a)
21. (b) 22. (b) 23. (c) 24. (d) 25. (c) 26. (d) 27. (b) 28. (d) 29. (d) 30. (d)

1. (c) 2. (b) 3. (a) 4. (a) 5. (d) 6. (a) 7. (a) 8. (d) 9. (c) 10. (b)
11. (d) 12. (a) 13. (d) 14. (c) 15. (d) 16. (c) 17. (b) 18. (c) 19. (a) 20. (a)
21. (a) 22. (c) 23. (c) 24. (b) 25. (c) 26. (a) 27. (d) 28. (d) 29. (d) 30. (d)
6.18

1. (c, d) 2. (a, c, d) 3. (a, c, d) 4. (a, c) 5. (a, b, c) 6. (a, b ) 7. (a, b, c, d) 8. (a, b, c)


9. (a,b,c) 10. (a,b,c,d) 11. (a, b, d) 12. (b) 13. (c) 14. (b) 15. (b) 16. (b)
17. (d) 18. (b) 19. (b) 20. (6) 21. (29) 22. (6) 23. (12)
   24. A " R; B" P, Q,R,S; C " P, R; D " Q
   25. A " S; B " Q, R; C " Q; D " P

    
1. A " p. s; B " r; C " p, q; D " p
2. (c) 3. (6) 4. (a, c, d) 5. (a) 6. (a, b, c) 7. (a, b) 8. (a) 9. (d) 10. (b)
11. (b) 12. (a) 13. (a) 14. (b) 15. (d) 16. (d) 17. (d) 18. (a) 19. (d) 20. (c)

Hints and Solutions

13. (c)
Sc and Zn show fixed oxidation state +3 and
+2 respectively.
1. (d) Unlike p-block elements, the various oxidation
14. (a)
Zn and Co are present in insulin and vitamin
states of d-block elements differ by one unit.
B12 respectively.
2. (d) M(s) " M+n(aq) + ne- pH < 7

15. (b) CrO42- ←  → Cr O 2-
The above change involves sublimation, pH > 7 2 7
ionization and hydration. X will be less than 7 and Y will be more than 7.
3. (b) Sc3+ ([Ar] 4s0 3d0) has no electrons in d-sub 16. (d)
+7
shell and hence, d-d transitions are not possible. MnO4- (VII group)
4. (b) Brass (Cu and Zn)
+6
5. (c) Due to lanthanide contraction, elements in 5d- Cr2O72- (VI group)
series have almost equal atomic and ionic radii
+5
with 4d- series elements.
VO43- (V group)
6. (b) CuF2 (Cu+2 = [Ar] 4s0 3d9)
+6
Due to d-d transitions, this compound is
coloured. FeO42- (VIII group)
7. (c) Stainless steel (Fe + Cr + Ni) 17. (d)
S-block elements (Ca) are not formed
8. (d) Transition metals and their compound show interstitial compounds.
catalytic activity because they can show 18. (a)
MnF6 does not exist.
variable oxidation state and they have tendency 19. (c)
Mn2O7 and CrO3 are acidic oxides.
to form complex.
20. (a)
Solder (Pb + Sn)
9. (a) Zn + 2NaOH " Na2ZnO2 + H2 -
21. (b)
Chromium (+3, +6)
10. (d) B4C is a covalent network carbide.
11. (b) Interstitial compound do not have high Co+3 = [Ar] 4s0 3d6 (4 unpaired e-)
22. (b)
chemical reactivity. Fe+2 = [Ar] 4s0 3d6 (4 unpaired e-)
12. (d) K2MnO4 can be converted into KMnO4 by 23. (c)
SO2 can decolorize acidified KMnO4 but CO2
passing Cl2 (oxidizing agent) or by electrolytic cannot decolorize acidified KMnO4.
oxidation or by disproportionation in acidic or 24. (d)
Fe2(SO4)3 [Ferric sulphate] does not behave as
in neutral medium. reducing agent.
6.19

25. (c) In Cr2O7-2, Six Cr-O bonds are identical while 9. (c) In process vegetable oil to ghee, the catalyst
other two Cr-O bonds are identical but all eight used is finely divided Ni.
Cr-O bonds are not identical. 10. (b) The X is K2Cr2O7.
26. (d) Sulphate (SO42-) and chromate (CrO42-) do not K2Cr2O7 ∆ → K2CrO4 + Cr2O3 + O2-
have same colour. 11. (d) X- ion is I-
27. (b) 2Cu+2 + K4[Fe(CN)6] "Cu2[Fe(CN)6]. + 4K+ 2+
2Cu + 4I- " Cu2I2. + I2
Reddish-brown 12. (a) CO2 does not react with K2Cr2O7.
28. (d) Ce+4 = [Xe] 6s0 5d0 (No unpaired e-) 13. (d) As we move from Sc to Zn, number of unpaired
Hence, CeO2 is colourless and diamagnetic in e- increases upto Cr and then decreases.
nature. The more common oxidation state of 14. (c) MnO4-2 is green coloured due to L " M charge
Ce is +3 hence, CeO2 acts as a good oxidizing transfer. Oxidation state of Mn+6.
agent.
Mn+6 = [Ar] 4s0 3d1 (1 unpaired e-)
29. (d) Cl, Mn and Np all can show +7 oxidation state.
15. (d)
3HCl+HN
 O3 " NOCl + 2H2O + 2[Cl]
30. (d) Order of complex forming tendency:
Aqura regia
La+3 < Ce+3 < Eu+3 < Lu+3
Au æ3[ææ
CI ]
Æ AuCl3 æHCI
æÆ HAuCl4
16. (c)
KMnO4 æSliqhtly
ææææ alkaline
Æ MnO2
+2
KMnO4 æAcidic ææ Æ Mn
1. (c) Total number of Cr-O bonds in dichromate ion 17. (b)
K2CrO7 + H2SO4 (Cold and Conc.) " CrO3
is 8. 18. (c)
MnO2 + K2CO3 + Air æHeat ææ
Æ K2MnO4
Acidic Mn+2 (X) (Y)
+7
Neutral
+4
K2MnO4 + Cl2 $ KMnO4
2. (b) KMnO MnO2
4
+6 (Z)
Basic MnO4
2–

19. (a)
When acidified KMnO4 reacts with oxalic
+4
acid then reaction is slow in the beginning
3. (a) TiF62-
but becomes very rapid because Mn+2 acts as
Ti+4 = [Ar] 4s0 3d0 (No unpaired e- hence, no autocatalyst.
d-d transition)
20. (a) NaCl + K2Cr2O7 + H2SO4(Conc) " NaHSO4 +
+1
Cu2Cl2 KHSO4 + H2O + CrO2Cl2-
Cu+ = [Ar] 4s0 3d10 (No unpaired e- hence, no deep-red
d-d transition) CrO2Cl2 + NaOH " Na2CrO4 + NaCl + H2O
4. (a)
CrO3 + 2NaOH " Na2CrO4 + H2O Yellow
acid base     salt 21. (a)
HgCl2 + NH3 " Hg(NH2)Cl. HgO
5. (d)
‘Spin only’ magnetic moment (ms) = 15 BM 22. (c) AgNO3 " Ag + NO2 + O2
µ s = n ( n + 2 ) = 15 (W) (X)

NO2 + H2O " HNO2 + HNO3
Here, n = 3 (unpaired e-)
HNO3 + Ag " AgNO3 + NO + H2O
M = [Ar] 4s1 3d5
AgNO3 + Na2S2O3 (excess) " Na3[Ag(S2O3)2]
M+3 = [Ar] 4s0 3d3 (3 unpaired e-)
       (Z)
6. (a) As atomic number increases, basic strength of 5 2
hydroxides of lanthanides decreases. 23. (c) Mn = [Ar] 3d 4s
7. (a) In KMnO4, oxidation state of Mn is +7. Mn shows +7 oxidation state.
24. (b) AgCl + NH3 " [Ag(NH3)2]Cl
Mn+7 = [Ar] 4s0 3d0 (No unpaired e-)
soluble
Colour in KMnO4 is due to L " M charge AgCl + Na2S2O3 " Na3[Ag(S2O3)2]
transfer. soluble
8. (d) Zn + HNO3 (conc.) " Zn(NO3)2 + NO2 + H2O AgCl + NaCN " Na[Ag(CN)2]
NO3- ion is reduced in preference to H+ ion. soluble
6.20

Zn+2 + NaOH " Zn(OH)2.


25. (c) 7. (a, b, c, d)
Zn(OH)2 + NaOH(excess) " Na2[Zn(OH)4] +6 +4 3+
soluble Cr2 O7−2 + 3SO32 − + 8H + → 2 Cr +
Oxidizing agent Reducing agent
26. (a)
HNO3(dil) + Ag " AgNO3 + NO + H2O +6
27. (d) C2O42-, NO2- and S2- can behave as reducing 3 SO24− + 4H 2 O
agent while F- ion cannot behave as reducing 8 (a, b, c)
agent.
Conditions required to form complexes are:
28. (d)
Actinoids are radioactive while lanthanoids are (a) Metal ion must have vacant orbitals.
not radioactive.
(b) Metal ion must have small size or high charge
29. (d) Nb is a d-block element. density or higher nuclear charge.
30. (d)
No lanthanoids have electron in 6d-subshell. 9 (a, b, c)
Ni+2 = [Ar] 4s03d8
Fe+2 = [Ar] 4s03d6
Cu+2 = [Ar] 4s03d9
1.

(c, d) 2Ag2O  → 4Ag + O2 Cu+ = [Ar] 4s03d10
∆ Conditions required for d-d transition is electronic
2HgO  → 2Hg + O2
configuration of central metal from d1 to d9 hence,
2. (a, c, d)
aqueous solution of Cu+ is colourless.
Cu + HNO3 (dil) $ Cu(NO3)2 + NO + H2O
10. (a, b, c, d)
Cu + H2SO4 (conc.) $ CuSO4 + SO2 + H2O
FeSO 4 .7H 2 O Strongly
 heated
→ Fe2O3 + SO2
Cu + Aqua regia (3HCl + HNO3) $ CuCl2
( green vitriol ) + SO3 + H2O
3. (a, c, d) Fe2O3 is basic oxide
+2 +1
Cu SO + NaOH + Glucos e → Cu O
4 2 H2O is neutral oxide
Oxidizing agent reducing agent red
SO2 and SO3 are acidic oxide
4. (a, c)
SO2 is a reducing agent
MnO2 + 4HCl " MnCl2 + Cl2 + 2H2O
11. (a, b, d)
Decomposition of acidic KMnO4 is catalysed by NaCl + K2Cr2O7 + H2SO4 (conc.) $ CrO2Cl2-­
sunlight.
(Deep red)
4MnO4– + 4H+ " 4MnO2 + 2H2O + 3O2 CrO2Cl2 + NaOH$Na2CrO4 + NaCl + H2O
3K2MnO4 + 2H2O " 2KMnO4 + MnO2 + 4KOH Yellow
MnO4– + 2H2O + 3e– " MnO2 + 4OH– CrO2Cl2 + H2O $ H2CrO4 + HCl
      Yellow
(KMnO also acts as oxidizing agent in alkaline
4
∆ ∆
medium). 12. (b) FeSO4.7H2O  → FeSO4  → Fe2O3
5. (a, b, c) (A) (B) (C)
Cl2 + OH– " Cl– + ClO3– + H2O + SO3 + SO2
(D) (E)
MnO42– + OH– " MnO4– + MnO2 + H2O SO3 + HgCl2 $ HgO.HgSO4.
NO2 + OH– " NO2– + NO3– + H2O (Basic yellow mercury (II) sulphate)
6. (a, b) SO2 + 2H2S $ 3S + 2H2O
(turbidity)
Cu + HgCl2 " CuCl2 + Hg
13. (c) SO3 + HgCl2 $ HgO. HgSO4.
SnCl2 + 2HgCl2 " Hg2Cl2 + SnCl4
14. (b) MnO2 + KOH + O2 $ K2MnO4 + H2O
Hg2Cl2 + SnCl2 " 2Hg + SnCl4
       green
6.21

(b) Cause of colour in MnO42– and KMnO4 is L "


15. 25.   (A " S ; B " Q, R ; C " Q ; D " P)
M charge transfer.
16. (b) I– is very good reducing agent. It oxidizes into
I2 and reduces Fe3+ into Fe2+.
17. (d) FeCl3 + K4 [Fe(CN)6] " Fe4[Fe(CN)6]3
FeCl3 + KSCN " K3[Fe(SCN)6]
electrolysis
18. (b) MnO42–  → MnO4– 1. (A " p, s ; B " r ; C " p, q ; D " p]
(A) (B) 2
2. (c) Cu F2
MnO4– + H2SO4 (conc.) ∆ → Mn2O7 +2
(A) (C) Cu = [ Ar ] 4s°3d 9
Mn2O7 is an acidic oxide. (due to d-d transition, CuF2 is coloured)
6
19. (b) Mn2O7 ∆ → MnO2 + O2- 3. MnO2 + KOH + O2 ∆
→ K 2 MnO 4 H 2 O
 (D) +7
20. (22) 4. (a, c, d) MnO −4 acidic
→ Mn +2
Element Higher stable oxidation state +4
MnO −4 neutral
→ MnO2
  Sc +3
+4
  Zn +2 MnO −4 alkaline
  → Mn O2
  Ti +4 5. (a) Colour of aqueous solution of CuSO4 is blue
  Mn +7 green. It absorbs orange-red colour.
  Cr +6 6. (a, b, c) Cr 2 + → Cr 3+
Oxidation
Re ducing agent
21. (29)
Element No. of unpaired e– Mn3+ Re
duction
→ Mn 2 +
Oxidizing agent
 Ti3+ 1
Cr2+ = [Ar] 4s03d4
 V3+ 2
Mn3+ = [Ar] 4s03d4
 Cr3+ 3 7. (a, b)
 Cr2+ 4 H2O2 in alkaline medium acts as reducing
 Mn3+ 4 agent, reduces Fe3+ to Fe2+.
 Fe3+ 5 8. (a) Ni2+ = [Ar] 4s0 3d8 (2 unpaired e–)
 Fe2+ 4 ms = 2(2 2) ª 2.84
 Co2+ 3 9. (d) Cause of lanthanoid contraction is the
 Ni2+ 2 imperfect shielding on outer electrons by 4f
 Cu2+ 1 electrons from the nuclear charge.
10. (b) 4f and 5f are not equally shielded.
22. FeC2 O 4 ∆
→ FeO + CO + CO2
( Paramagnetic)  11. (b) The 5f orbitals of actinoids extend farther from
diamagnetic
the nucleus than the 4f orbitals of lanthanoids.
Number of diamagnetic products (x) = 2 Hence, removal of e– from 5f-orbitals is easier
Number of unpaired e– in FeO (y) = 4 than 4f-orbitals.
x+y=6 12. (a) Antinoids show larger number of oxidation
H +
+2 states than lanthanoids. It is due to lesser
23.   KMnO4  → Mn
R . A. energy difference between 5f and 6d than 4f
KMnO4 OH
 −
→ Mn
+4 and 5d orbitals.
R . A.
13. (a) In the highest oxidation states, the transition
K2Cr2O7 OH 

→ Cr
+6
metal show acidic character and form cationic
x + y + z = 12 complexes.
24.  (A " R ; B " P, Q, R, S ; C " P, R ; D " Q) 14. (b) In general, due to presence of partially filled
f-orbitals, Ln(III) compounds are coloured.
6.22

15. (d) All the members of lanthanoids exhibit +3 18 (a) Ions Number of unpaired e–
oxidation state not + 4 oxidation state. V+2 3
16. (d) The outer electronic configuration of Lu is 4f14 Cr +2
4
5d1 6s2 Mn+2 5
17. (d) As the positive oxidation state increases, Fe +2
4
tendency of hydrolysis increases.
Order of paramagnetic behaviour :
Ferric salts (Fe+3) are more easily hydrolysed
V+2 < Fe+2 = Cr+2 < Mn+2
than the corresponding ferrous salts (Fe+2).
19. (d) CO acts as a reducing agent to reduce FeO into
Fe.
20. (c) Cause of colour of KMnO4 is L " M charge
transfer.
Chapter

Key Concepts
BF3 is hydrolysed as follows:-


Group 13 contains boron (B), aluminium (Al), gallium
(Ga), indium (In), and thallium (Tl). These elements have
outer electronic configuration (ns)2 (np)1, where n varies The fluorides of Al, Ga, In and Tl are ionic while the other
from 2 to 6. Boron is nonmetal while others are metals. halides are generally covalent and exist as dimer.
The atomic radii of Ga, In and Tl are smaller than expected
values due to d-block contraction. The atomic radius of Tl The trihalides of boron are electron-deficient compounds.
is a little larger than In due to lanthanide contraction. On Du to back bonding, the electron density on boron in
descending the group, +1 oxidation state becomes more increased. The tendency to form pp-pp bond is maximum
stable than +3 state due to the inert pair effect. is BF3 and falls rapidly on passing to BCl3 to BBr3. The
increasing order of acid strength follows the order BF3 <
The very high melting point of boron is due to its covalent BCl3 < BBr3.
network structure. In boron family, gallium has the lowest
melting point. The ionization energies do not follow the
expected trend of decreasing values on descending the
group. All elements burns in oxygen at high temperatures
forming M2O3. The reaction of aluminium with oxygen
(known as thermite reaction) is strongly exothermic.
Boron belongs to Group 13 of the periodic table. The chief
Aluminium is amphoteric. It dissolves in dilute minerals minerals of boron are borax (Na2[B4O5(OH)4].8H2O), i.e.,
acids and in aqueous sodium hydroxide. Na2B4O7. 10H2O, colemanite (Ca2[B3O4(OH)3]2.2H2O)
The acidic character of hydroxides decreases on i.e., Ca2B6O11.5H2O and kernite (Na2[B4O5(OH)4].2H2O)
descending the group. i.e., Na2B4O7.4H2O.
Boric acid is a very weak monobasic acid. It does not Boron is isolated by converting its mineral into boron
liberate hydrogen ion but accepts a hydroxyl ion. In the trioxide followed by its reduction with magnesium.
presence of cis-diol (glycerol, mannitol or sugars), boric
Na2(B4O5(OH)4).8H2O + 2HCl 2NaCl +
acid behaves as a strong acid and can be titrated with
borax 5H2O + 4H3BO3
NaOH in the presence of phenolphthalein indicator.
orthoboric acid
7.2

2H3BO3 B2O3 + 3H2O


B2O3 + Mg 2B + 3MgO
Crystalline boron is obtained by the reduction of boron
trichloride with zinc or dihydrogen at the high temperatures. Group 14 contains carbon (C), silicon (Si), germination
(Ge), tin (Sn) and lead (Pb). Their outer electronic
2BCl3 + 3Zn 1200
 K
→ 3ZnCl2 + 2B configuration is (ns)2 (np)2, where n varies from 2 to 6. The
2BCl3 + 3H2 1200
 K
→ 2B + 6HCl metallic character of elements increases on descending the
Boron is an extremely hard refractory solid of high melting group; C and Si are nonmetals, Ge is nonmetal but also has
point, low density and very low electrical conductivity. some metallic characterstics, and Sn and Pb are metals.
Boron is quite inert to chemical attack at ordinary The melting points decreases on descending the group,
temperature. It reacts with strong oxidising agents such as with the exception of Pb whose melting point is slightly
fluorine and concentrated HNO3 at room temperature. At higher than that of Sn. Carbon has extremely high melting
elevated temperatures, it combines with metals (forming point. This is due to the stronger C–C bonds in the network
borides) and nonmetals. of carbon atoms. The ionization energies decrease from C
to Si, but then change in an irregular way because of the
Diborane is B2H6. It has two coplanar BH2 groups and the
effects of filling d and f sub-shells.
remaining two hydrogen atoms lie centrally between BH2
groups in a plane perpenticular to the plane containing Carbon forms single, double and triple bonds with carbon
BH2 groups. itself and with other elements. The tendency to form
multiple bond by other elements is rare. However, silicon
In borax two boron atoms are in a trianglular geometry can form double bond due to back bonding in which the
and two boron atoms are in tetrahedral geometry. lone pair in p orbitals of an atom is extended to an empty
OH orbital of Si. One of the examples of back bonding is
– trsilylamine, N(SiH3)3.
H H B
H O O The chemical reactivity of elements decreases down the
B B HO–B O B–OH
group. The inert effect becomes increasingly effective on
H O O
H H B– descending the group.
OH The stability of +4 oxidation state decreases while that
2– of the +2 oxidation state increases on descending the
B2H6 [B4O5(OH)4] ion in borax
group.
Boric acid (H3BO3) is obtained by treating borax with
minerals acids. 4Sn + 10HNO3 4Sn(NO3)2 + NH4NO3 + 3H2O
3Pb + 8HNO3 3Pb(NO3)2 + 2NO + 4H2O
Na2[B4O5(OH)4].8H2O + 2H+ 4H3BO3 +
5H2O + 2Na+ C is not affected by alkalis, Si reacts forming silicates
while Sn and Pb form stannate [Sn(OH)6]2–, and plumbate
Boric acid is a white crystalline substance, soft and soapy [Pb(OH)6]–2, respectively.
to touch. It is moderately soluble in cold water. On heating
it decomposes as follows: All the elements of Group 14 from tetrahalides with
the exception of PbI4 which is not known. The stability
H3BO3 −375

H O
K
→ HBO2 of halides decreases down the group. CCl4 is stable
2
        Metaboric acid while other halides are hydrolysed. The hydrolysed. The
hydrolysis of SiCl4 produces SiO2 while SiF4 produces
4HBO2 −435

H O
K
→ H2B4O7 SiO2 as well as (SiF6)–2.
2
         
Tetraboric acid
The acidic nature of the dioxdes of carbon family decreases
H2B4O7 red
 heat
→ 2B2O3 down the group; CO2 and SiO2 are acidic, GeO2 is weakly
−H O 2
Boric oxide acidic and SnO2 and PbO2 are amphoteric.
Boric acid is a very weak monobasic acid. It does liberate Silicones are organosilicon polymers with general
hydrogen but accepts a hydroxyl ion, i.e., it behaves as a formula (R2SiO)n, where R may be methyl, ethyl or phenyl
Lewis acid. group.
7.3

Because of the directed lone pair of electrons on carbon,


the molecule forms carbonyls with a number of metal
in which the coordinate bond is formed through carbon
atom and not through oxygen atom. With nickel, it forms
tetracarbonyl which decomposes at higher temperature.
325 345 K
The burning of carbon in a limited supply of air or in a 
Ni + 4CO  
 [Ni(CO)4]
450 K
deficiency of oxygen produces carbon monoxide. A few
The poisonous nature of carbon monoxide is due to its
reactions producing carbon monoxide are as follows:
ability to form a bond with iron atom in the haemoglobin
Fe2O3 + 3C 2Fe + 3CO of blood. In the form of producer gas (CO + N2), water

COOH gas (CO + H2) or semiwater gas (mixture of producer and


concentrated H 2 SO 4
 → CO + CO2 + H2O water gases), is used as fuel.
COOH

concentrated H SO
HCOOH  2 4
→ CO + H2O
Carbon dioxide can be prepared by any of the following
CO2 + Zn heat
 → CO + ZnO reactions.
K4Fe(CN)6 + 6H2SO4 2K2SO4 + FeSO4 +
CaCO3 + 2HCl CaCl2 + H2O + CO2
concentrated 3(NH4)2SO4 + 6CO
2NaHCO3 heat
 → Na2CO3 + H2O + CO2
Carbon monoxide is an extremely poisonous gas. A
concentration of one in 800 volume of air will lead to MgCO3 heat
 → MgO + CO2
death in 30 minutes. It combines with haemoglobin of Solid carbon dioxide is known as dry ice and is used as a
the blood to give more stable carboxyhaemoglobin and refrigerant.
thus render it useless as an oxygen carrier. In air, it burns Carbon dioxide is an acidic oxide.
with a blue flame to give carbon dioxide. The gas readily
CO2 + H2O H2CO3
dissolves in ammonical or acidic solution of cuprous
chloride giving the additional product CuCl. CO. 2H2O. With reactive metals, it is reduced to CO.
Some the reactions shown by carbon monoxide are given 2Na + 2CO2 Na2CO3 + CO
below.
2Mg + CO2 2MgO + C
CO + NaOH pressure 
heat
→ HCOONa
sodium formate Carbon dioxide is absorbed by green plants in the presence
CO + Cl2 hv → COCl2 of sunlight and is ultimately transformed into starch and
cellulose in the chloroplast. This process is known as
+ Cu powder
CO + 2H2 ZnO

425 – 675 K
→ CH3OH photosynthesis.
− 1175 K Carbon dioxide is a linear molecule with carbon-oxygen
Fe2O3 + 3CO 875
 → 2Fe + 3CO2
ond equal to 115 pm, which is intermediate between those
I2O5 + 5CO I2 + 5CO2 calculated for carbon-oxygen double and triple bond. It is
The structure of carbon monoxide may be represented as thus cosidered to the resonance hybrid of the following
– + structures.
C O or C O + – – +

Carbon atom is considered to be sp hybridized. One sp O C O O C O O C O


orbital used to form a single bond with oxygen atom while Carbon in CO2 is sp hybridized. The two sp orbitals form
the other sp orbital which points away from the C–O bond two bonds wiith two oxygen atoms. The two p orbital not
contains a lone pair of electrons. The sideways overlap included in hybridization give rise two p bonds.
of singly filled 2p orbitals on carbon and oxygen atoms
produces a p bond. The second p bond is formed by the
overlap of doubly filled 2p orbitals on carbon and oxygen The are prepared by direct combination of metals with
atoms produces a p bond. The second p bond is formed by carbon at elevated temperature or indirectly, the heating of
the overlap of doubly filled 2p orbital on oxygen with the metallic oxide with carbon. The carbides may be classified
vacant 2p orbital on carbon. But once the bond is formed, into three groups, namely, ionic, covalent and interstitial.
it is not possible two distinguish the two p bonds. Ionic carbides are formed by metals of Group 1, 2 and 3.
7.4

These compounds, in general, occur as transparent crystals are shared with other tetrahedra resulting in a three-
and in the solid state they are nonconductors of electric dimensional lattice. The formula of such silicates is SiO2.
current. They give hydrocarbons when treated with water Silicones
or acids. On the basis of anions, these have been classified
Silicones are polymeric organosilicon compounds
as methanides (C4–), acetylides (C22–) and allylides. (C34–).
containing individual or cross-linked Si–O chains or
The examples are:
rings in which some of the oxygens of SiO4 tetrahedron
Be2C + 4H2O 2Be(OH)2 + CH4 are replaced by –OH, –CH3, –C2H5 groups. For example,
Al4C3 + 12H2O Al(OH)3 + 3CH4 dialkyldichlorosilane (R2SiCl2), which is produced by the
CaC2 + 2H2O Ca(OH)2 + C2H2 reaction
Al2C6 + 6H2O 2Al(OH)3 + 3C2H2 2RCl + Si(Cu) R2SiCl2
Mg2C3 + 4H2O 2Mg(OH)2 + CH3C∫CH reacts with water producing dialkyldihydrosilane. This, in
turn, may be dehydrated to give a linear polymer.
R2SiCl2 + 2H2O R2Si(OH)2 + 2HCl
Silicon belongs to Group 14 and is classified as metalloid. nR2Si(OH)2 − → (R2SiO)n
H O
It exists in two allotropic forms; the amorphous silicon 2

Silicone
and the crystalline or admantine silicon. Silicon does not
occur freely in nature. It occurs as silica (SiO2) or as Silicones have good thermal and oxidative stability.
silicates like feldspar, kaolinite, mica, etc. in rocks and These are excellent water repellants and chemically inert
clays. substances. Silicon rubber is not attacked by ozone. Liquid
silicones are used as excellent lubricants. These are mixed
Silicon is produced by the reduction of sand with coke in
with paints and enamels to increase the resistance to the
an electric arc furnace.
effects of high temperatures, sunlight and chemicals.
SiO2 + 2C Si + 2CO
Silicon is a hard solid having melting point 1793 K and
boiling point 3550 K. It reacts with fluorine at room
temperature to form SiF4. With other elements, it reacts at
elevated temperatures. Group 15 contains nitrogen (N), phosphorus (P), arsenic
Silicates and silica contains SiO44– tetrahedra differing in (As), antimony (Sb) and bismuth (Bi). Their outer
the way the tetrahedra are linked together as described in electronic configuration is (ns)2 (np)3, where n varies from
the following. 2 to 6. The metallic character of these elements increases
Orthosilicates on descending the group; N and P are nonmetals, As and
Sb are metalloids and Bi is a metal. The melting and
These contain individual discrete SiO44– tetrahedra. boiling points follows the order
Examples are phenacite (Be2SiO4) and zircon (ZrSiO4).
melting point N < P < As > Sb > Bi
Pyrosilicates
boiling point N < P < As < Sb > Bi
These contain discrete Si2O76– ions and are formed
when one oxygen atom of two SiO4 tetrahedra is shared. Phosphorus has two common allotropic forms; white and
Example is thorteveitite (Sc2Si2O7). red. White phosphorus is more reactive than red form due
to highly strained structure (P–P–P angle is 60°).
Chain and Cyclic Silicates
Black phosphorus is a highly polymerised form and is
In these silicates two oxygen atoms per tetrahedron are most stable.
shared.
Nitrogen form triple bond in dinitrogen because bond
Sheet Silicates enthalpy e(N∫N) is greater than three times bond enthalpy
These are formed by the sharing of three oxygen atoms by e(N–N). In phosphorus, the reverse is true, hence, it
each tetrahedron giving an infinite two-dimensional sheet involves single bonds.
of the empirical formula (Si2O5)n–2n The melting points of hydrides follow the order
Three-Dimensional Silicates NH3 > PH3 < AsH3 < SbH3.
In these silicates, all the four oxygen atom of a tetrahedron All the five elements of Group 15 form trihalides. Of these
7.5

nitrogen halides are least stable. All the trihalides with the Eyde method in which N2 and O2 are sparked together in
exception of NF3 hydrolyse in aqueous solution. an electric furnace. The nitric oxide formed is converted to
NCl3 + 3H2O NH3 + 3HOCl nitrogen dioxide when exposed to air. The dissolution of
NO2 in water gives nitric acid.
PCl3 + 3H2O H3PO3 + 3HCl
+O
AsCl3 + 3H2O H3AsO3 + 3HCl N2 + O2 spark
 → NO 
2
→ NO2 water
→ HNO3.

SbCl3 + H2O SbO+ + 2H+ + 3Cl– Phosphorus Trioxide


+ + – Burning of phosphorus in a limited supply of air gives
BiCl3 + H2O BiO + 2H + 3Cl
Nitrogen forms many oxides; N2O, NO, N2O3, NO2 and phosphorus trioxide. It is a colourless crystalline solid
N2O5. The most common oxides of phosphorus are P4O6 with odour of garlic. It exists as dimer both in naphthalene
and P4O10. Phosphorous trioxide is an acidic oxide and is and in the vapour phase.
anhydride of orthophosphorus acid (H3PO3). Phosphorus It dissolves in cold water giving phosphorous acid (H3PO3)
pentoxide in anhydride of phosphoric acid (H3PO4). whereas in hot water it gives phosphoric acid (H3PO4) and
Nitrogen forms two acids; nitrous acid (HNO2) and phosphine (PH3).
nitric acid (HNO3). Phosphorus forms two series of P4O6 + 6H2O 4H3PO3
oxoacids. These are phosphorous and phosphoric series.     Cold
In phosphorous series, the acids include pyrophosphorous P4O6 + 6H2O 3H3PO4 + PH3
acid (H4P2O5), orthophosphorous acid (H3PO3), hot
metaphosphorous acid (HPO2) and hypophosphorous In its structure, four P atoms lie at the corners of a
acid (H3PO2). The acids H4P2O5 and H3PO3 are reducing tetrahedron and six oxygen atoms along the edges. The
agents as they involve P–H bond. P–O bond distance is 165.6 pm which is shorter than the
In phosphoric series, the acids include orthophosphoric expected single bond distance (184 pm). This suggests
acid (H3PO4), pyrophosphoric acid (H4P2O7) and that there exists a considerable double bond characcter in
polymetaphosphoric acid, (HPO3)n. the P–O bonds because of the formation of a pp–dp bond
Oxides of Nitrogen: with oxygen donor.
O P
Name Structure Magnetic Nature Physical
property appearance O O
P O
1. Nitrous oxide, N N O Diamag- Neutral Colourless
N2O netic gas P P
O
2. Nitric oxide, N = O or N = O Para- Neutral Colourless P
NO magnetic gas    Structuure of P4O6
3. Dinitrogen O O Diamag- Acidic Blue solid Phosphorus Pentoxide
N—N netic
trioxide, N2O3 O
Burning of phosphorus in an excess of dried air or oxygen
4. Nitrogen O Para- Acidic Reddish
gives phosphorus pentoxide. It is a white solid which
dioxide, NO2
N
magnetic brown gas sublimes on heating. It is odourless when pure. It dissolves
O
in cold water giving metaphosphoric acid.
5. Dinitrogen O O Diamag- Acidic Colourless P4O10 + 2H2O 4HPO3
N—N
tetraoxide, netic solid           cold
O O
N2O4 With hot water, orthophophoric acid is formd.
6. Dinitrogen O O O Diamag- Acidic Colourless P4O10 + 6H2O 4H3PO4
N N
pentaoxide, netic solid
O O     hot
N2O5 O
O P
Oxoacid of Nitrogen
O O
Two oxacids of nitrogen are nitrous acid and nitric acid.
O P O
Nitrous acid is unstable except in dilute solution. Pure
nitric acid is a colourless liquid but on exposure to light it P P
O O
turns slightly brown because of slight decomposition into P
O
NO2 and O2. Nitric acid can be prepared by Brikeland-     Structure of P4O10
7.6

It is a strong dehydrating agent. For example, H2SO4 and Cyclictrimetaphosphoric acid, (HPO3)3 or H3P3O9
HNO3 are converted into corresponding anhydride.
O OH
2H2SO4 + P4O10 2SO3 + 4HPO3 P
4HNO3 + P4O10 2N2O5 + 4HPO3 O O

In the structure of P4O10, each P atom forms three bonds HO
to oxygen atoms and also an additional coordinate bond P P O
with an oxygen atom. Terminal coordinate P–O bond is O
O OH
143 pm which is much shorter than the expected bond
length of 162 pm. The shows the presence of considerable
pp-dp back bonding because of the leteral overlap of full
p orbitals on oxygen with empty d orbitals on phosphorus.
Oxoacids of Phosphorus :-

Hypophosphorus acid, H3PO2 Group 16 contains oxygen (O), sulphur (S), selenium
O (Se), tellurium (Te) and polonium (Po). Their electronic
P
configuration is (ns)2 (np)4, where n varies from 2 to 6.
H OH
H The metallic character of these elements increases on
descending the group; O and S are nonmetallic, Se and Te
Phosphosrus acid, H3PO3
are weaker nonmetallic and Po is metallic.
O
Electron affinity of sulphur is larger than that of oxygen,
P
H OH this due to the more repulsion experienced by the incoming
OH electron from the smaller, more compact electronic cloud
Pyrophosphorus acid, H4P2O5 of oxygen atom.

O O Oxygen is diatomic with the unpaired electrons. Sulphur


exists in two allotropic forms – rhombic and monoclinic
P P
sulphur. Rhombic sulphur is stable at room temperature
H O H
OH HO while monoclinic sulphur is stable above 369 K. Selenium
Hypophosphoric acid, H4P2O6 exists in six allotropic forms. Tellurium has only one
crystalline form and polonium has two allotropic forms
O O (cubic and rhombohedral).
P P
HO OH Ozone is another allotropic form of oxygen. It is very
OH HO reactive. It is formed in the upper layer of atmosphere
(about 20 km from the earth) by the action of ultraviolet
Phosphoric acid, H3PO4
radiation on oxygen. Ozone is a strong oxidizing agent.
O In organic chemistry, ozone is used to locate the carbon
P double and triple bonds. The ozone molecule is angular
HO OH
with bond angle about 117° and bond length 127.8 pm.
OH
Pyrophosphoric acid, H4P2O7 The melting and boiling point of hydrogen compounds

of elements of Group 16 follow the order H2O > H2S <
O O
H2Se < H2Te. The exceptional high values of H2O is due
P P to hydrogen bondings.
HO O OH
OH HO H2O2 is a strong oxidizing agent. With stronger oxidizing
Metaphosphoric acid, HPO3 or (HPO3)n agents such as KMnO4, KIO3, and O2, hydrogen peroxide
O O O acts as a reducing agent.

P P P Oxoacids of sulphur may be classified into four series.


~O O O O~
OH OH OH
7.7

Sulphurous Acid Series Peroxodisulphuric acid (H2S2O8)


Sulphurous acid (H2SO3) O O
HO

HO S O O S OH
S O
HO O O
Di-or pyrosulphurous acid (H2S2O5)
O O


HO S S OH
O Group 17 contains fluorine (F), chlorine (Cl), bromine
Dithionous acid (H2S2O4) (Br), iodine (I) and astatine (At). Their outer electronic
O O configuration is (ns)2(np)5, where n varies from 2 to 6. The
trends in this group are as follows.
HO S S OH
Covalent and ionic radii – increases down the group.
Sulphuric Acid Series
Electronegativity and ionization energy – decreases down

Sulphuric acid (H2SO4) the group.
O
Electron affiinity – Cl > F > Br > I

HO S OH Melting and boiling points – increases down the group.
O Bond enthalpy (X–X)– Cl2 > Br > F2 > I2
Thiosulphuric acid (H2S2O3) Oxidizing ability – decreases down the group.
S
Halogens are very reactive and do not occur in free

HO S OH state. Fluorine is most electronegative atom, there exists
hydrogen bondings in gaseous HF.
O
Di-or pyrosulphuric acid (H2S2O7) HF is a weak acid and HCl, HBr and HI behave as strong
O O acids. In the glacial acetic acid medium, the acid strength
follows the order HI > HBr > HCl > HF.

HO S O S OH
Halogens with the exception of F form a number of
O O oxoacids– hypohalous acids (HOX), halous acids (HXO2),
halic acid (HXO3) and perhalic acid (HXO4). The acid
Thionic Acid Series
strength follows the order HXO4 > HXO3 > HXO2 > HXO.

Dithionic acid (H2S2O6) Fluorine forms only hypofluorous acid (HOF).
O O Halogens also form interhalogen compounds AX, AX3,

HO S S OH AX5 and AX7. The compounds AX and AX3 are formed
where the electronegativity is not very large. The
O O compounds AX5 and AX7 are formed by large Br and I
Polythionic acid (H2Sn+2O6) atoms surrounded by small atom F. The molecule AX3 is
O O T-shaped with two lone pair of electrons at the equilateral
positions. The orientations of five pair of electrons around

HO S (S)n S OH the atom A is trigonal bipyramidal. Interhalogens are more
O O reactive than elemental halogens except fluorine.
The six pairs of electrons around atom A in AX5 acquire
Peroxoacid Series
octahedral orientation. The seven pairs of electrons around

Peroxomonosulphuric acid (H2SO5) atom A in AX7 acquire pentagonal bipyramidal orientation.
O
Halogens are nonmetallic and have high electron affinity.

H O O S OH The nonmetallic character decreases down the group and
iodine shows some metallic character. With metals, they
O form ionic compounds by accepting one electron and with
7.8

nonmetals covalent compounds are formed by sharing an are formed. Halogens react with metals and nonmetals to
electron. form halides, the reactivity decreases down the group.
All halogens exhibit-1 oxidation state. Except fluorine, rest All the halogens react with hydrogen to form hydrogen
of the halogens also exhibit +1, +3, +5 and +7 oxidation halides (HX). The reaction between flourine and hydrogen
states. Fluorine is the most electronegative and thus there is violent while that between iodine and hydrogen is
exists hydrogen bonding in HF with the result that it has very slow at room temperature. The acidic character of
exceptionally high melting and boiling points as compared hydrogen halides increases in the order, HF < HCl < HB2
to those of HCl, HBr and HI. < HI.
Because of high reactivity, halogens do not exist in the Oxoacids of Chlorine
free state. The chief ore of flurine are fluorspar (CaF2), Four oxoacids of chlorine are known. These are:
cryolite (Na3AlF6) and fluoroapatite (Ca2(PO4)F). Other
halogens mainly occur in seawater as salt. Some sea weeds Hypochlorous acid, HOCl
and sponges contain iodine as iodides. Chile slatpeter O

Cl H
(NaNO3) contains 0.02 – 1% iodide in the form of sodium
iodate. Chlorous acid, HClO2
Fluorine was obtained by electrolysis of KF dissolved Cl
in anhydrous HF. The products H2 and F2 are collected HO O
separately so as to avoid explosion caused by combination Chloric acid, HClO3
of these two gases.
Cl
Chlorine, in the laboratory, can be prepared by the O OH
following methods: O
1. Action of concentrated hydrochloric acid on Perchloric acid HClO4
manganese dioxide. O
2. Oxidation of HCl by strong oxidizing agents such Cl
as KMnO4 and K2Cr2O7. O OH
Bromine can be obtained by the oxidation of bromide O
with chlorine gas or manganese dioxide in the presence of
concentrated sulphuric acid. Iodine can also be obtained
by the oxidation of iodide.
Halogens are oxidizing agents, oxidizing power decreases The Group 18 contains helium (He), neon (Ne), argon
down the group. Flourine is the most oxidizing in nature (Ar), krypton (Kr), xenone (Xe) and radon (Rn). Their
and can oxidize water to oxygen and ozone. With dilute outer electronic configurations is (ns)2(np)6 with the
alkalis, fluorine forms oxygen difluoride (OF2) and with exception of the electronic configuration of He which is
hot concentrated alkaliis flouride and oxyen are formed. 1s2, all elements exist as monatomic gas.
The other halogens reacts with cold and dilute alkali Xenon forms a number of fluorides –XeF2, XeF4 and
solution to give hypohalites XeF6. The other compounds are XeO3, XeOF4, XeO2F2,
(XO–) and with hot and concentrated alkali, halates (XO3–) XeO4 and [XeO6]4–

Solved Examples

1.
Which species does not exist - (a) Metaboric acid (b) Pyroboric acid
3– 3–
(a) [BF6] (b) [AlF6] (c) Boron and water (d) Boric anhydride
3–
(c) [GaF6] (d) [InF6]3–
Sol.(a) [BF6]3– does not exist because boron does not Sol.(d) H3BO3 100°
 C
→ HBO2 160°
 C
→ H2B4O7 + H2O
have vacant d-subshells.
2B2O3 + H2O
2.
Orthoboric acid when heated to red hot gives
7.9

3.
Alumnium vessels should not be washed with Gas (A) burns with blue flame and is oxidised to
materials containing washing soda because - gas (B). Gas (B) turns lime water milky.
(a) Washing soda is expensive. 3 ,∆
Gas(A) + Cl2 (D) NH
 → (E)
(b) Washing soda is easily decomposed.
3 ,∆
(c) Washing soda reacts with aluminium to form (B) NH
 → (E)
soluble aluminate. Identify (A) to (E) and explain the reactions.
(d) Washing soda reacts with aluminium to form
CO OH
insoluble aluminium oxide.
Sol. ∆
→ CO + CO2 + H2O
Sol.(c) Na2CO3 + 2H2O 2NaOH + H2O + CO2 COO H
         (A) (B) (C)
O
2NaOH + 2Al + 2H2O 2NaAlO2 + 3H2. ||
NH 3 , ∆
CO + Cl2 COCl2  → NH2–C–NH2
soluble
(A)     (D) (E) Urea
4.
When Al is added to KOH solution -
(a) No action takes place (A) is CO (B) is CO2
(b) Oxygen gas is evolved (C) is H2O (D) is COCl2
(c) Water is produced (E) is NH2CONH2
(d) Hydrogen gas is evolved 9.
In P4O6 the number of oxygen atoms bonded to
each P atom is –
Sol.(d) 2Al + 2KOH + 2H2O 2KAlO2 + 3H2
(a) 1.5 (b) 2
5. Carbon forms carbon monoxide when burnt in - (c) 3 (d) 4
(a) Absence of air or oxygen
Sol.(c) Each P in P4O6 is bonded to 3 oxygen atoms.
(b) Excess of air or oxygen
O P
(c) Limited supply of air or oxygen O O
(d) Moist air P O
1 P P
Sol. (c)   C + O2 (limited ) ∆ → CO O O
2
6. CCl4 does not act as Lewis acid, while SiCl4 and 10. Which trihalides is not hydrolysed by water -
SnCl4 acts as Lewis acid as well as their aqueous (a) NF3 (b) NCl3
solution is acidic. Explain why? (c) PCl3 (d) AsCl3
Sol.
SiCl4 and SnCl4 are hydrolysed to form acidic Sol.(a) In the first stage of hydrolysis, an extra bond is
solution as well as they can act as Lewis acid formed by water molecule. While chlorine and
because they can increase their co-ordination the group 15 elements (except nitrogen) can
number greater than four due to availability of expand their octet by using vacant d-orbitals of
d-orbitals. the valence shell, F and N cannot. As a result, NF3
SnCl4 + 4H2O Sn(OH)4 + 4HCl is extremely stable.
SiCl4 + 4H2O Si(OH)4 + 4HCl NF3 + H2O (l) No hydrolysis
SiCl4 + 2Cl– (SiCl6)–2 NCl3 + 3H2O (l) NH3 + 3HOCl
PCl3 + 3H2O (l) H3PO3 + 3HCl
7.
PbCl4 exists while PbBr4 and PbI4 do not exist.
Explain why? 2AsCl3 + 3H2O 6HCl + As2O3

Pb4+ is an oxidising agent and readily changes


Sol. 11.
Derivatives of nitrogen (III) act as –
into Pb2+ (due to inert pair effect) while Br– and (a) Oxidizing agent only
I– ions are reducing agents. Thus, redox reaction (b) Reducing agent only
occurs indicating that PbBr4 and PbI4 are unstable (c) Both Oxidizing and Reducing agent
compounds. (d) Nitrating agent
H2C2O4 ∆
8. → gas(A) + gas(B) + liquid(C) Sol.(c) Deriatives of nitrogen (III) have both oxidizing
oxalic acid and reducing properties.
7.10

2 molecules are held by weak inter molecular


2Na+3NO2 + 2KI + 2H2SO4 I2 + 2 N O +
vander waal’s force of attraction.
   (Oxidant) 2K2SO4 + Na2SO4 + 2H2O
17.
O3 is a powerful oxidising agent. Write equations
3
2KMnO4 + 5Na N O + 3H2SO4 to represent oxidation of –
2
       (Reductant) (a) I¯ to I2 in acidic solutions.
5 (b) Sulphur to sulphuric acid in the presence of
2MnSO4 + 5Na N O + K2SO4 + 3H2O
3 moisture
12.
In the compound of type POX3, P atom show (c) [Fe(CN)6]4– to [Fe(CN)6]3– in basic solution.
multiple bonding of the type –
(a) O3 + 2I¯ + 2H+ O2 + I2 + H2O
(a) pp – dp (b) dp – dp (b) 3O3 + S + H2O H2SO4 + 3O2
(c) pp – pp (d) No multiple bonding (c) O3 + 2[Fe(CN)6] + H2O4–

Sol.(a) POX3 has sp3 hybridised P having vacant 2[Fe(CN)6]3– + 2OH¯ + O2


d-orbitals. p-orbital of O atom and d-orbitals of P 18.
Explain why sugar turns black on addition of
undergoes pp-dp bonding. concentric H2SO4.
13.
With the help of NH3, O2, Pt and H2O, write Sol.
Concentric H2SO4 removes water molecules from
equations for preparation of N2O from these sugar leaving behind black carbon because of its
substances. strong dehydrating nature.
4NH3(g) + 5O2(g) Pt.
Sol. → 4NO(g) + 6H2O(g) C12H22O11 
2 4 H SO
→ 12C + 11 H2O.
2NO(g) + O2(g) 2NO2(g) 19. The reaction of chlorine with CO in the presence
3NO2(g) + H2O(l) 2HNO3(aq.) + NO(g) of sunlight gives –
HNO3(g) + NH3(aq.) NH4NO3(aq.) (a) COCl2 (b) CO2Cl2
NH4NO3(aq.) 
→ NH4NO3(s)∆
(c) HOCl (d) H2Cl2O2
NH4NO3(s) ∆
→ N2O(g) + 2H2O (g) CO + Cl2 COCl2
14. Which of the following is a laboratory method of 20.
Excess of Cl2 reacts with ammonia, products
preparation of oxygen? formed are :
(a) heating of KClO3 (a) NH3 and N2 (b) NCl3 and HCl
(b) from liquid air (c) NCl3 and N2 (d) NH4Cl and N2
(c) by electrolysis of water NH3 + Cl2 NCl3 + 3HCl
(d) by Brin’s process      (excess)
Sol.(a) The other three methods are employed for large 21.
Bromine is added to cold dilute aqueous solution
production of O2 for industrial applications. of NaOH. The mixture is boiled. Which of the
Note: In Brin’s process BaO is heated to get BaO2
following statements is true?
at 500ºC which is decomposed at 800ºC to give (a) During the reaction bromine is present in
oxygen. four different oxidation states.
15.
The catalyst used during decomposition of H2O2 (b) The greatest difference between the various
is – oxidation states of bromine is 5.
(a) Ni (b) Fe (c) on acidification of the final mixture, bromine
(c) MnO2 (d) Pt is formed.
Sol. (c) 2H2O2 MnO 2 Catalyst
 → 2H2O + O2 (d) Disproportionation of bromine occurs during
∆ the reaction.
16. H2O is liquid while H2S is gas under given
conditions. Explain why? 6NaOH + 3Br2 5NaBr + NaBrO3 +
Sol.
H2O molecules have intermolecular H – bonding 3H2O (disproportionation)
due to greater electronegativity of oxygen hence 22.
Which reaction cannot be used for the preparation
H2O is liquid. H2S lacks H – bonding. H2S of the halogen acid?
7.11

(a) 2KBr + H2SO4 (conc.) K2SO4 + 2HBr I2 + I 3– I3–, I – behaves as a Lewis base.
(b) NaCl + H2SO4 (conc.) NaHSO4 + HCl 25.
The statement, which prompted Neil Barlett to
(c) NaHSO4 + NaCl Na2SO4 + HCl prepare the first noble gas compound was –
(d) CaF2 + H2SO4 (conc.) CaSO4 + 2HF (a) Xe-F bond has high bond energy
(b) F2 has exceptionally low bond energy
Sol. (a) HBr is strong reducing agent and will be oxidised
by H2SO4 an oxidant (c) PtF6 is a strong oxidant
(d) O2 molecule and Xe atom have very similar
23.
Which of the following reactions will give
ionization energies.
bleaching powder?
(a) CaCl2 + H2O (b) CaO + HCl Both Xe and O2 have comparable ionization
energies.
(c) Ca(OH)2 + Cl2 (d) ClO2 + Ca(OH)2
Xe Xe+ + e¯ ;
Ca(OH)2 + Cl2 CaOCl2 + H2O
lonization energy = 1,170 kJ/mol
Slaked lime Bleaching powder
O2 O2+ + e¯ ;
24.
I3¯ is formed when I2 combines with I¯. Which of
them is a Lewis base? Ionization energy = 1,175 kJ/mol

Exercise
5. Which of the following is covalent carbide?
(a) CaC2 (b) Al4C3
1. Amorphous boron is extracted from borax by (c) SiC (d) Be2C
following steps: 6. Product formed by catalytic oxidation of NH3 by
Borax (
x)
→ H3BO3 ∆
→ B2O3 (
Y)
→ air is:
Boron (a) N2 (b) NO
Then X and Y are: (c) N2O (d) N2O3
(a) H2SO4, Al (b) HCl, C 7. Which one of the following properties is not
(c) HCl, Fe (d) H2SO4, Na shown by NO?
(a) It combines with oxygen to from nitrogen
2.
In which of the following compounds B atoms are
dioxide.
in sp2 and sp3 hybridisation states?
(b) It’s bond order is 2.5.
(a) Borax (b) Diborane (c) It is diamagnetic in gaseous state.
(c) Borazole (d) All (d) It is a neutral oxide.
3. Which of the following statement is not correct 8. P4 + Cl2 " A
about CO ?
P4 + Excess Cl2 " B.
(a) Producer gas is the mixture of CO + N2
Hydrolysis products of A and B are respectively:
(b) CO formes a volatile compound with Nickel.
(a) H3PO2, H3PO3 (b) H3PO4, H3PO3
(c) CO is absorbed by ammonical solution of
Cu2Cl2 (c) H3PO3, H3PO4 (d) H3PO2, H3PO4
(d) CO and Cl2 form phosphine gas in presence 9. Holme’s signal can be given by using:
of sunlight. (a) CaC2 + CaCN2 (b) CaC2 + Ca3P2
Conc.
4. H2C2O4 H
SO
→ A + B. A is neutral and B is (c) CaC2 + CaCO3 (d) Ca3P2 + CaCN2
2 4

acidic. Gases A and B are absorbed by conc.: 10. Nitrogen cannot be obtained by heating:
(a) CuCl2, KOH (b) Cu2Cl2, KOH (a) Ba(N3)2 (b) Pb(NO3)2
(c) KOH, Cu2Cl2 (d) CuCl2, H2SO4 (c) (NH4)2Cr2O7 (d) NH4NO2(s)
7.12

11. Which of the following statements is wrong? (c) It can form compounds with other elements
(a) Single N–N bond is stronger than the single of its group.
P–P bond. (d) The pH of aqueous solution of its oxides is
(b) PH3 can act as a ligand in the formation less than 7.
of coordination compound with transition 20. Which of the following speices is not a
elements. pseudohalide?
(c) NO2 is paramagnetic in nature. (a) CNO– (b) RCOO–
(d) Covalency of nitrogen in N2O5 is four.
(c) CN– (d) N3–
12. The nitrogen oxide(s) that do(es) not contain(s)
21. Which interhalogen compound exists in dimeric
N–N bond(s) are:
form?
(a) N2O (b) N2O3
(a) BrF5 (b) IF7
(c) N2O4 (d) N2O5
(c) ICl (d) ICl3
13. Impure phosphine is combustible due to presence
22. Which of following halogen oxides is ionic?
of:
(a) I4O9 (b) I2O5
(a) P2H4 (b) N2
(c) BrO2 (d) ClO3
(c) PH5 (d) P2O5
23. Which of the following is not oxidised by MnO2?
14. Extra pure N2 can be obtained by heating: (a) F– (b) Cl–
(a) NH3 with CuO (b) NH4NO3 (c) Br– (d) I–
(c) (NH4)2 Cr2O7 (d) Ba(N3)2 24. Which pair of Halogen will not give
15. Ammonia can be dried by: disproportionation reaction with water?
(a) Conc. H2SO4 (b) P4O10 (a) F2, I2 (b) Cl2, Br2
(c) CaO (d) Anhydrous CaCl2 (c) Br2, I2 (d) Cl2, F2
16. Which metal sticks on glass in contact with 25. F2 being the strongest oxidising agent among
ozone? halogen is due to:
(a) Ag (b) Au (a) Highest hydration energy
(b) Low bond dissociation energy
(c) Pt (d) Hg
(c) High electron affinity
17. What is true about H2SnO6(polythionic acid)?
(d) All of these
(a) Total S-S bond = n
26. The state of hybridisation of Xe and Sb in the
(b) Number of S with ‘O’ state = n – 1
product, when XeF4 react with SbF5 respectively:
(c) Number of S with +5 state = n – 2
(a) sp2d2, sp3d (b) sp3d, sp3d2
(d) Number of dp-pp bond = 4
(c) sp3, sp3d2 (d) sp3d3, sp3d
18. In which of the following reactions, is there
a change in the oxdiation number of nitrogen 27. Which fluoride is isostructural with ICl2– ?
atoms? (a) XeF4 (b) XeF6
(a) 2NO2 N2O4 (c) XeF2 (d) XeF82–
(b) 2NO2 + H2O HNO2 + HNO3
28. Which of the following compound does not exist?
(c) NH3 + H2O NH4+ + OH–
(a) KrF2 (b) XeO2F2
(d) N2O5 + H2O 2HNO3
(c) XeO3 (d) NeF2
19. Atom ‘X’ belong to group VII A and is not present
in the second period. Choose incorrect statement 29. The hybrid states of C in diamond and graphite
regarding ‘X’: are respectively:
(a) The most acidic oxyacid of X is HXO3. (a) sp3, sp3 (b) sp2, sp2
(b) Its highest oxidation state is +7. (c) sp2, sp3 (d) sp3, sp2
7.13

30. The compound H2N—Hg—O—Hg—I is formed 3. B2H6 +2 2BH3L (L is a lewis base)


by the action of: (Lewis bases)
(a) NH3 and HgO in the presence of iodine L would be:
(b) NI3, HgO and H2O (a) NH3 (b) CH3NH2
(c) NH3 + K2HgI4 (c) (CH3)2NH (d) (CH3)3N
(d) KI + NH3 + Hg 4. Which of the following is an incorrect statement
31. The part of the skin which comes in contact with about boric acid:
nitric acid turns yellow because: (a) Polymerises due to H-bonding
(a) proteins are converted into xanthoproteins. (b) Gives green flame with ethyl alcohol
(b) water is removed by the acid. (c) Act as strong acid with trans-1, 2-diol
(c) skin gets burnt. (d) Weak, monobasic, lewis acid
(d) nitrocellulose is formed. 5. CO2 is gas, while SiO2 is a solid but both are:
32.
Which halide of boron is a weakest Lewis acid? (a) Covalent containing p-bond
(b) Molecules having pp–dp bonding
(a) BCl3 (b) BI3
(c) Acidic
(c) BF3 (d) BBr3
(d) Discrete molecules
33. Which of the following compounds of nitrogen is
6. If one mole P4 completely reacts with NaOH
coloured?
solution then the incorrect statement among the
(a) NO2 (b) NH3 following is:
(c) N2O (d) N2O4 (a) Products are NaH2PO2 and PH3
34. The number of lone pairs, and the number of (b) Total number of P–H bonds in products are 5
S—S bonds in S8 molecules are respectively: (c) It is a disproportination reaction
(a) 8, 8 (b) 16, 8 (d) None of the above
(c) 8, 16 (d) 8, 4 7. At 0°C, NO2 is :
35. Bond energy is highest for which of the following: (a) Paramagnetic, Coloured gas
(b) Diamagnetic, Coloured gas
(a) F2 (b) Cl2
(c) Diamagnetic, Colourless solid
(c) Br2 (d) I2 (d) Paramagnetic, Coloured solid
8. Ca3P2 + H2O A + B(gas)
B + HI
" C ∆→ B + KI + H2O
1. Which of the following is an incorrect statement B, C, D are:
for Boric acid: (a) P2H4, PH4I, KNO3 (b) PH3, PH4I, KOH
(a) It gives B(OCH3)3 when reacts with CH3– (c) H3PO2, I2, KOH (d) PH3, P2H4, KClO3
OH.
9. NH3 is not obtained by:
(b) Hydrogen bonding in H3BO3 gives it a
(a) Heating of NH4NO3 or NH4NO2
layered structure
(b) Heating of NH4Cl or (NH4)2CO3
(c) It is bronsted acid
(c) Heating of NH4NO3 with NaOH
(d) It can be prepared by reaction of borax with
mineral acid. (d) Reaction of AlN or Mg3N2 or CaCN2 with
H2O
2. Na2B4O7.10H2O ∆ → X + NaBO2 + H2O
∆ 10. Which of the following statements is wrong?
X + Cr2O3  → Y (green coloured).
(a) The stability of hydrides increase from NH3
The X and Y are: to BiH3 in group 15 of the periodic table.
(a) Na3BO3 and Cr(BO2)3 (b) Nitrogen cannot form NCl5.
(b) Na2B4O7 and Cr(BO2)3 (c) Single N–N bond is weaker than the single
(c) B2O3 and Cr(BO2)3 P–P bond.
(d) B2O3 and CrBO3 (d) N2O4 has two resonance structure.
7.14

11. (NH4)2Cr2O7 on heating liberates a gas. The same (a) I–Cl bond is stronger than Br–Br–bond
gas will be obtained by: (b) Ionisation energy of I < ionisation energy of
(a) Heating NH4NO2 Br
(b) Heating NH4NO3 (c) I-Cl is polar whereas Br2 is non polar
(c) Treating H2O2 with NaNO2 (d) Size of I > size of Br
(d) Treating Mg3N2 with H2O 19. When chlorine is passed slow over dry slaked like
12.
When Cl2 reacts with NH3 of low concentration Ca(OH)2 at room temperature, the main product
and of high concentration, then oxidised is:
products obtained from NH3 are ____ and _____ (a) CaCl2 only (b) CaOCl2
respectively: (c) Ca(ClO2)2 (d) Ca(OCl)2 only
(a) N2, NH2Cl (b) NCl3, N2 20. Which of the following is not correct?
(c) N2H4, N2 (d) N2, NH4Cl (a) Among halogens, radius ratio between iodine
13. The incorrect statement(s) about O3 is (are): and fluorine is maximum
(a) O–O bond lengths are equal. (b) All halogens have weak X–X bond than X–X'
bond in interhalogens
(b) Thermal decomposition of O3 is endothermic.
(c) Among interhalogen compounds maximum
(c) O3 is diamagnetic in nature. number of atoms are present in iodine
(d) O3 has a bent structure. fluroide.
14. Compounds A and B are treated with dilute (d) Interhalogen compounds are more reactive
HCl separately. The gases liberated are Y and Z than halogen compounds.
respectively. Y turns acidified dichromate paper 21. Which of the following order is not correct ?
green while Z turns lead acetate paper black. So,
(a) F– < Cl– < Br– < I– reducing nature
A and B compounds are respectively:
(b) F– > Cl– > Br– > I– hydration energy
(a) Na2SO3, Na2S
(c) Cl2 > F2 > Br2 > I2 bond dissociation energy
(b) NaCl, Na2CO3
(d) F2 > Cl2 > Br2 > I2 reactivity
(c) Na2S, Na2SO3
(d) Na2SO4, K2SO3 22.
What should be the correct statement with respect
to XeF5–?
15. When H2SO4 reacts with Cl2 gas then X is (a) Central atom Xe has sp3d2 hybridisation
produced. X is a good chlorinating agent and
(b) It is square planar
given H2SO4 after hydrolysis. Then [X] is:
(c) There are two non bonding electron pairs, one
(a) SOCl2 (b) SO2Cl2 above the plane and the other below the plane
(c) SCl2 (d) S2Cl2 (d) It is an odd electron species
16. Gaseous products formed when Zn react with dil. 23. XeF4 act as fluoride acceptor with:
H2SO4 and conc. H2SO4 respectively:
(a) PF5 (b) SbF5
(a) H2S, SO2 (b) H2 and SO2
(c) KF (d) All of these
(c) SO2 and H2 (d) SO3, H2S
24. What is not true for ozone?
17. Which of the following process is not feasible
spontaneously? (a) The two O—O bond lengths are not equal.
(b) O—O bond order is between 1 and 2.
(a) F2 + H2O " HF + O2
(c) O—O—O angle is approximately 117°.
(b) Cl2 + H2O " HCl + HOCl
(d) It is light blue gas with pungent odour.
(c) Br2 + H2O " HBr + HOBr
25. The formation of which of the substance is known
(d) I2 + H2O " HI + HOI
as tailing of mercury?
18. Molecular size of I-Cl and Br2 is nearly same but
(a) Hg2O (b) HgO
boiling point of I-Cl is about 40°C higher than
Br2. This might be due to: (c) Hg2O3 (d) Hg(NO3)2
7.15

26. Heating of which of the following nitrate produces (a) Xe > Kr > Ar > Ne > He
a gaseous substance which is used as anaesthetic (b) He > Ne > Ar > Kr > Xe
in dental surgery? (c) Xe > Ar > Kr > Ne > He
(a) NH4NO2 (b) Pb(NO3)2 (d) Xe > He > Kr > Ar > Ne
(c) NH4NO3 (d) NaNO3
34. XeF6 on complete hydrolysis gives:
27. Which allotropic form of phosphorus is good
conductor of electricity? (a) XeO4 (b) XeOF2
(a) Yellow phosphorus (c) XeOF4 (d) XeO3
(b) Red phosphorus 35. XeF6 on reaction with CsF gives:
(c) Black phosphorus (a) [XeF5]+ [CsF2]–
(d) None of these (b) XeF8
28.
The inertness of nitrogen is due to: (c) [XeF4]2+ [CsF3]2–
(a) Its intermediate electronegativity. (d) Cs+[XeF7]–
(b) High bond dissociation energy of nitrogen-
nitrogen bond.
(c) Stable configuration of N atom.
(d) Small atomic size. ONE OR MORE THAN ONE OPTIONS CORRECT
TYPE
29.
Which is not true for phosphorus?
1.
Select the correct statement(s)
(a) Phosphorus exists in different allotropic
forms. (a) Graphite is diamagnetic and diamond is
paramagnetic in nature.
(b) Black phosphorus has layer type structure.
(b) Graphite acts as a metallic conductor along
(c) White phosphorus is less reactive than red
the layers of carbon atoms.
phosphrous.
(c) Graphite is less denser than diamond.
(d) White phosphorus exists in tetrahedral
molecular solid. (d) C60 is called as Buckminster fullerene.
30. What is not true about N2O5? 2.
Borax bead test is given by:
(a) It is anhydride of HNO3. (a) An aluminium salt (b) A cobalt salt
(b) In solid stable it exists as NO2+ NO3–. (c) A copper (II) salt (d) A nickel salt
(c) It is structurally similar to P2O5. 3.
Which of the following species exists:
(d) It can be prepared by heating HNO3 over (a) [BF6]3– (b) [AlF6]3–
P2O5.
(c) [GaF6]3– (d) [lnF6]3–
31. Sulphur does not exist as S2 molecule because:
4.
A complex cross-linked polymer (silicone) is
(a) it is less electronegative.
formed by:
(b) it is not able to constitute pp-pp bond.
(a) Hydrolysis of (CH3)3SiCl
(c) it has ability to exhibit catenation.
(b) Hydrolysis of a mixture of (CH3)3SiCl and
(d) of tendency to show variable oxidation states. (CH3)2SiCl2
32. Among the oxo-acids of chlorine, the correct (c) Hydrolysis of CH3SiCl3
order of increasing acid strength is: (d) Hydrolysis of SiCl4
(a) HClO4 < HClO < HClO2 < HClO3 5.
White phosphorus be removed from red
(b) HClO3 < HClO2 < HClO4 < HClO phosphorus by:
(c) HClO4 > HClO3 > HClO2 > HClO (a) Sublimation under reduced pressure
(d) HClO4 < HClO3 < HClO2 < HClO (b) Dissolving in water
33. The ease of liquification of noble gases decreases (c) Dissolving in CS2
in the order (d) Heating with an alkali solution
7.16

6.
A gas is obtained on heating ammonium nitrate. 13.
Which of the following inter-halogen compounds
Which of the following statements are incorrect is/are possible?
about this gas? (a) ClF3 (b) IF5
(a) Causes laughter
(c) FCl3 (d) BrF5
(b) Brings tears to the eyes
(c) Is acidic in nature PASSAGE-BASED QUESTIONS
(d) Is basic in nature Passage # 1 (Q. 14 and 15)
7.
Which of the following represents correct Species having X—O—H linkage (X = non metal with
dissociation of nitrate salts on heating? positive oxidation state) are called oxy acids and parent
1 acid of a non-metal may exist in two form (a) –ic form of
(a) 2LiNO3 Li2O + 2NO2 + O parent oxy acid (b) -us form of parent oxy acid.
2 2
1 14.
Which of the following parent oxy acid does not
(b) Pb(NO3)2 PbO + 2NO2 + O2
2 have its pyro-oxy acid?
(c) NH4NO3 N2O + 2H2O
(a) H2SO3 (b) HNO3
(d) NH4NO2 N2 + 2H2O
(c) H3PO3 (d) H4SiO4
8.
Which of the following is/are correct regarding
15.
X—O—X bond (where X = central atom) is not
nitrogen family?
present in species.
(a) Nitrogen is restricted to a maximum
covalency of 4 as only four orbitals are (a) Cl2O7 (b) H2N2O2
available for bonding. (c) N2O5 (d) H2S2O7
(b) The single N–N bond is weaker than the
Passage # 2 (Q. 16 and 17)
single P–P bond.
The property of hydrides of p-block elements mostly
(c) The catenation tendency is weaker in nitrogen
depends on:
as compared to phosphorous.
(d) Nitrogen forms pp-pp bond as well as pp-dp (i) electronegativity difference between central
bonds. atom and hydrogen,

9.
P2O5 can dehydrate: (ii) size of central atom, and

(a) H2SO4 (b) HNO3 (iii) number of valence electrons in central atom.

(c) HClO4 (d) HPO3 Some undergo hydrolysis in which central atom is
less electronegative, react with OH– to give hydrogen.
10.
Which statements are correct about halogen? While acidic property of hydride in a period depends on
(a) They are all diatomic and form univalent ions. electronegativity of central atoms, i.e. more electronegative
(b) Halogen have the smallest atomic radii in the atom, more acidic is the hydride. In a group, acidic
there respective periods. property is proportional to size of the central atom. Some
(c) They are all diatomic and form diatomic ions. electron deficient hydrides behave as Lewis acid while
(d) They are all reducing agents. only one hydride of an element in p-block behaves as
Lewis base with lone pair of electrons. Hydrides in which
11.
In the reaction 2Br– + X2 Br2 + 2X–, X2 is/ central atom’s electronegativity is close to hydrogen has
are: no reaction with water.
(a) Cl2 (b) F2
16.
Which one is the weakest acid among the
(c) I2 (d) N2 following?
12
Among the following which reactions are possible (a) HF (b) HCl
(a) F2 + H2O HF + O2 (c) HBr (d) HI
(b) Cl2 + H2O HCl + HClO 17.
Which one is strongest base?
(c) Br2 + H2O HBr + HBrO (a) OH– (b) HS–
(d) I2 + H2O Hl + HIO (c) HSe– (d) HTe–
7.17

Passage # 3 (Q. 18 and 19) 23.


What is the number of oxygen atoms which are
An orange solid (A) on heating gives a green residue (B), shared between tetrahedrons in Si3O96– ?
a colourless gas (C) and water vapours. The dry gas (C) 24.
How many of the following properties increase
on passing over heated Mg gave a white solid (D), (D) on down the group for nitrogen family?
reaction with water gave a gas (E) which formed black (a) Atomic size
precipitate with mercurous nitrate solution.
(b) Acidic character of oxides
18.
Select the incorrect statement. (c) Boiling point of hydrides
(a) The central atom of the anion of solid (A) has
(d) Reducing power of hydrides
d3s hybridisation.
(e) Extent of pp-pp overlap
(b) The orange solid (A) is diamagnetic in nature.
(c) The anion of orange solid (A) is oxidising in (f) Metallic character
nature. 25.
Number of gaseous oxides among the following
(d) All metal oxygen bond lengths are equal in at room temperature is:
anion of solid (A). (a) N2O (b) NO
(c) N2O3 (d) NO2
19.
Which of the following is false for the gas (E)?
(e) N2O5 (f) P4O6
(a) It gives a deep blue colouration with CuSO4 (g) P4O10 (h) SO2
solution. (i) SO3

(b) It is oxidised to a colourless gas (neutral
26. The number of mixed anhydride among the
oxide) at 1200 K in presence of a catalyst Pt/
following are:
Rh in air.
Cl2O; ClO2; Cl2O6; Cl2O7; N2O5; NO2; N2O
(c) It gives the same gas (C) with potassium
permanganate solution. 27.
How many ion in following behave like
pseudohalide?
(d) It gives black precipitate with HgCl2.
CN–, SCN–, I3–, O–2, N3–, CNO–, S2O3–2, C2O4–2
INTEGER VALUE TYPE QUESTIONS
MATCH THE COLUMN TYPE QUESTIONS
20.
Consider a prototypical fullerence C60.
28.
Let, a = Number of 5-membered rings;
 b = Number of 6-membered rings; Column I Column II
 c = Number of p-bonds in C60. (A) Hypo phosphoric (P) All hydrogen
acid are ionizable in
Find the value of (3a – 2b + c)
water
21.
Central atom may exhibit sp3 hybridisation in
(B) Pyro phosphorus (Q) Lewis acid
how many of the following species:
acid
(a) CO2 (b) Graphite (C) Boric acid (R) Monobasic in
(c) Diamond (d) CO water
(e) H3BO3(aq) (f) Zeolites (Si-central) (D) Hypo phosphrous (S) sp3 hybridized
(g) Silicones (Si) (h) Chlorosilane (Si) acid central atom

(i) Borax (Boron) (j) Al2Cl6 29.


(k) B2H6
(l) SiO2(solid) Column I Column II
(m) H2CO3 (n) COCl2 Silicates Number of oxygen atoms
(o) CH4 (p) CCl4 shared per tetrahedron

22.
Which of the following compounds are amphoteric (A) Ortho silicate (P) 4
in nature? (B) Pyro silicate (Q) 1
PbO, PbO2, SnO, SnO2, Al2O3, ZnO, BeO, Ga2O3, (C) Cyclic silicate (R) 0
B2O3. (D) 3-D silicate (S) 2
7.18

30.
Match the reactions island in column-I with 6.
Amongst H2O, H2S, H2Se and H2Te, the one with
characteristic(s) type of reactions listed in the highest boiling point is:
column-II. (a) H2O because of hydrogen bonding
Column-I Column-II (b) H2O because of higher molecular weight
(A) Moist (P) Hydrolysis (c) H2S because of hydrogen bonding
PCl5 
→Air
(d) H2Se becuase of lower molecular weight
(B) P4 + NaOH(conc.) + H2O (Q) At least one of
[IIT-2000]
Warm
 → the products has
tetrahedral geometry 7. The number of P—O—P bonds in cyclic
(C) H3PO3 
→ 200° C (R) Disproportionation metaphosphoric acid is:
(a) zero (b) two
(D) P4O6 + H2O 200°
 C
→ (S) At least one of the
products has pp-dp (c) three (d) four
bonding [IIT-2000]
8. The number of S—S bonds in sulphur trioxide
trimer, (S3O9) is:
(a) three (b) two
(c) one (d) zero
[IIT-2001]
9. The set with correct order of acidic strength is:
1. KF combines with HF to form KHF2. The
(a) HClO < HClO2 < HClO3 < HClO4
compound contains the species:
(b) HClO4 < HClO3 < HClO2 < HClO
(a) K+, F– and H+ (b) K+, F– and HF
(c) HClO < HClO4 > HClO3 > HClO2
(c) K+ and [HF2]– (d) [KHF]+ and F–
(d) HClO4 < HClO2 < HClO3 < HClO
[IIT-1996] [IIT-2001]
2. The following acids have been arranged in the ASSERTION AND REASON TYPE QUESTIONS
order of decreasing acidic strength. Identify the (Q. 10 and 11)
correct order.
Read the following questions and answer as per
ClOH (I), BrOH (II), IOH (III) the direction given below:
(a) I > II > III (b) II > I > III (a) Statement I is correct; Statement II is correct
(c) III > II >I (d) I > III > II Statement II is the correct explanation of
[IIT-1996] Statement I.
(b) Statement I is correct; Statement II is correct
3. Which one of the following species is not a Statement II is not the correct explanation of
pseudo halide? Statement I.
(a) CNO– (b) RCOO– (c) Statement I is correct; Statement II is incor-
(c) OCN– (d) NNN– rect.
[IIT-1997] (d) Statement I is incorrect; Statement II is
correct.
4. In compounds of type ECl3, where E = B, P, As or
Bi, the angles Cl — E — Cl for different E are in 10. Statement I: Pb4+ compounds are stronger
the order: oxidzing agents than Sn2+ compounds.
(a) B > P = As = Bi (b) B > P > As > Bi Statement II: The higher oxidation states for the
group 14 elements are more stable for the heavier
(c) B < P = As = Bi (d) B < P < As < Bi
members of the group due to ‘inert pair effect’.
[IIT-1999]
[IIT-2008]
5. Ammonia can be dried by:
11. Statement I: Between SiCl4 and CCl4, only SiCl4
(a) conc. H2SO4 (b) P4O10 reacts with water.
(c) CaO (d) anhydrous CaCl2 Statement II: SiCl4 is ionic and CCl4 is covalent

[IIT-2000] [IIT-2001]
7.19

12. H3BO3 is: 20.


A pale blue liquid obtained by equimolar mixture
(a) monobasic acid and weak Lewis acid of two gases at –30°C is
(b) monobasic and weak Bronsted acid (a) N2O (b) N2O3
(c) monobasic and strong Lewis acid (c) N2O4 (d) N2O5
(d) tribasic and weak Lewis acid [IIT - 2005]
[IIT-2003]
21.
Which of the following allotropes of phosphorus
13. Me2SiCl2 on hydrolysis will produce: is thermodynamically most stable?
(a) (Me)2Si(OH)2 (a) Red (b) White
(b) (Me)2 Si = O (c) Black (d) Yellow
(c) [—O—(Me)2Si — O—]n [IIT-2005]
(d) Me2SiCl(OH) 22.
B(OH)3 + NaOH NaBO2 + Na[B(OH)4] + H2O
[IIT-2003] How can this reaction be made to proceed in the
14. For H3PO3 and H3PO4, the correct choice is: forward direction?
(a) H3PO3 is dibasic and reducing (a) Addition of cis 1, 2-diol
(b) H3PO3 is dibasic and non reducing (b) Addition of borax
(c) H3PO4 is triabsic and reducing (c) Addition of trans 1, 2-diol
(d) H3PO3 is tribasic and non reducing (d) Addition of Na2HPO4
[IIT-2003] [IIT-2006]
15. When I– is oxidised by KMnO4 in alkaline
medium, I– converts into:
(a) IO–3 (b) I2 Read the following questions and answer as per
the direction given below:
(c) IO–4 (d) IO–
(a) Statement I is correct; Statement II is correct.
[IIT-2004]
Statement II is the correct explanation of
16.
Which of the following has —O—O— linkage? Statement I.
(a) H2S2O6 (b) H2S2O8 (b) Statement I is correct; Statement II is correct.
(c) H2S2O3 (d) H2S4O6 Statement II is not the correct explanation of
Statement I.
(c) Statement I is correct; Statement II is
17. Name the structure of silicates in which three incorrect.
oxygen atoms of [SiO4]4– are shared:
(d) Statement I is incorrect; Statement II is
(a) pyrosilicate correct.
(b) sheet silicate 23. Statement I : Boron always forms covalent bond.
(c) linear chain silicate
Statement II : The small size of B3+ favours

(d) three-dimensional silicate formation of covalent bond.
[IIT-2005] [IIT-2007]

18.
Which of the following is not oxidised by O3? 24. Statement I : In water, orthoboric acid behaves
as a weak monobasic acid.
(a) KI (b) FeSO4
In water, orthoboric acid acts as a
(c) KMnO4 (d) K2MnO4 proton donor.
[IIT - 2005] [IIT-2007]
19.
Which gas is evolved when PbO2 is treated with 25.
The percentage of p-character in the orbitals
concentrated HNO3? forming P—P bonds in P4 is:
(a) NO2 (b) O2 (a) 25 (b) 33
(c) N2 (d) N2O (c) 50 (d) 75
[IIT - 2005] [IIT - 2007]
7.20

(c) moist O2
(d) O2 in the presence of aqueous NaOH
There are some deposits of nitrates and phosphates in [IIT - 2009]
earth’s crust. Nitrates are more soluble in water. Nitrates 30.
Extra pure N2 can be obtained by heating:
are difficult to reduce under the laboratory conditions but (a) NH3 with CuO (b) NH4NO3
microbes to it easily. Ammonia forms large number of
complexes with transition metal ions. Hybridisation easily (c) (NH4)2Cr2O7 (d) Ba(N3)2
explains the case of sigma donation capability of NH3 and [IIT - 2011]
PH3. Phosphine is a flammable gas and is prepared from 31.
The reaction of white phosphorus with aqueous
white phosphorus. NaOH gives phosphine along with another
[IIT-2008] phosphrous containing compound. The reaction
type, the oxidation states of phosphorus in
26.
Among the following, the correct statement is:
phosphine and the other product respectively are:
(a) Phosphates have no biological significance in (a) redox reaction, –3 and –5
humans.
(b) redox reaction, +3 and +5
(b) Between nitrates and phosphates, phosphates (c) disproportionation reaction, –3 and + 1
are less abundant in earth’s crust. (d) disproportionation reaction, –3 and + 3
(c) Between nitrates and phosphates, nitrates are
less abundant in earth’s crust. 32.
Which ordering of compounds is according to
(d) Oxidation of nitrates is possible in soil. the decreasing order of the oxidation state of
27.
Among the following, the correct statement is: nitrogen?
(a) Between NH3 and PH3, NH3 is a better (a) HNO3, NO, NH4Cl, N2
electron donor because the lone pair of (b) HNO3, NO, N2, NH4Cl
electrons occupies spherical ‘s’-orbital and is
(c) HNO3, NH4Cl, NO, N2
less directional.
(b) Between NH3 and PH3, PH3 is a better (d) NO, HNO3, NH4Cl, N2
electron donor because the lone pair of
electrons occupies sp3 orbital and is more 33.
The shape of XeO2F2 molecule is:
directional. (a) trigonal bipyramidal
(c) Between NH3 and PH3, NH3 is a better (b) square planar
electron donor because the lone pair of (c) tetrahedral
electrons occupies sp3 orbital and is more
(d) see-saw
directional.
[IIT-2012]
(d) Between NH3 and PH3, PH3 is a better
electron donor because the lone pair of
electrons occupies spherical s-orbital and is The reactions of Cl2 gas with cold-dilute and hot-
less directional. concentrated NaOH in water give sodium salts of two
28.
White phosphorous on reaction with NaOH gives (different) oxoacids of chlorine, P and Q, respectively. The
PH3 as one of the products. This is a: Cl2 gas reacts with SO2 gas in the presence of charcoal,
(a) dimerisation reaction to give a product R.R reacts with white phosphorus to
give a compound S. On hydrolysis, S gives an oxoacid of
(b) disproportination reaction
phosphorus T.
(c) condensation reaction
34.
P and Q respectively, are the sodium salts of:
(d) precipitation reaction
(a) hypochlorous and chloric acids
29. The reaction of P4 with X leads selectively to (b) hypochlorous and chlorous acids
P4O6. The X, is: (c) chloric and perchloric acids
(a) dry O2 (d) chloric and hypochlorous acids
(b) a mixture of O2 and N2 [JEE Advanced - 2013]
7.21

35.
R, S and T, respectively, are: 39.
Which of the following properties in not shown
(a) SO2Cl2, PCl5 and H3PO4 by NO?
(b) SO2Cl2, PCl3 and H3PO3 (a) It is paramagnetic in liquid state.
(c) SOCl2, PCl3 and H3PO2 (b) It is neutral oxide.
(d) SOCl2, PCl5 and H3PO4 (c) It combines with oxygen to form nitrogen
dioxide.
36.
The product formed in the reaction of SOCl2 with
(d) Its bond order is 2.5.
white phosphorus is:
[JEE Main - 2014]
(a) PCl3 (b) SO2Cl2
40.
Among the following oxacids, the correct
(c) SCl2 (d) POCl2
decreasing order of acidic strength is
[JEE Advanced - 2014] (a) HOCl > HClO2 > HClO3 > HClO4
37.
Concentrated nitric acid upon long standing, turns (b) HClO4 > HOCl > HClO2 > HClO3
yellow-brown due to the formation of:
(c) HClO4 > HClO3 > HClO2 > HOCl
(a) NO (b) NO2
(d) HClO2 > HClO4 > HClO3 > HOCl
(c) N2O (d) N2O4
[JEE-Main - 2014]
[JEE Advanced - 2014]
41.
Which among the following is the most reactive?
38.
Which of the following is the wrong statement?
(a) ONCl and ONO– are not isoelectronic (a) Cl2 (b) Br2

(b) O3 molecule is bent (c) I2 (d) ICl

(c) Ozone is violet-black in solid state [JEE-Main - 2015]


(d) Ozone is diamagnetic gas 42.
Which one has highest boiling point?
(a) He (b) Ne
[JEE Main - 2013]
(c) Kr (d) Xe
[JEE-Main - 2015]

Answer Key

1. (a) 2. (a) 3. (d) 4. (b) 5. (c) 6. (b) 7. (c) 8. (c) 9. (b) 10. (b)
11. (a) 12. (d) 13. (a) 14. (d) 15. (c) 16. (d) 17. (d) 18. (b) 19. (a) 20. (b)
21. (d) 22. (a) 23. (a) 24. (a) 25. (a) 26. (b) 27. (c) 28. (d) 29. (d) 30. (c)
31. (a) 32. (c) 33. (a) 34. (b) 35. (b)

1. (c) 2. (c) 3. (d) 4. (c) 5. (c) 6. (b) 7. (c) 8. (b) 9. (a) 10. (a)
11. (a) 12. (b) 13. (b) 14. (a) 15. (b) 16. (b) 17. (d) 18. (c) 19. (b) 20. (b)
21. (c) 22. (c) 23. (c) 24. (a) 25. (a) 26. (c) 27. (c) 28. (b) 29. (c) 30. (c)
31. (b) 32. (c) 33. (a) 34. (d) 35. (d)
7.22

1. (b, c, d) 2. (b, c, d) 3. (b, c, d) 4. (c) 5. (a, c, d) 6. (b, c, d) 7. (a, b, c, d) 8. (a, b, c)


9. (a, b, c) 10. (a, b) 11. (a, b) 12. (a, b, c) 13. (a, b, d) 14. (b) 15. (b) 16. (a)
17. (a) 18. (d) 19. (d) 20. (26) 21. (11) 22. (8) 23. (3) 24. (3)
25. (5) 26. (3) 27. (4)
28. A P, S; B S; C Q, R; D R,S
 29. A R; B Q; C S; D P
30. A P, Q, S; B P, Q, R, S; C Q, R, S; D P, Q, R, S

1. (c) 2. (a) 3. (b) 4. (b) 5. (c) 6. (a) 7. (c) 8. (d) 9. (a) 10. (c)
11. (c) 12. (a) 13. (c) 14. (a) 15. (a) 16. (b) 17. (b) 18. (c) 19. (b) 20. (b)
21. (c) 22. (a) 23. (a) 24. (a) 25. (d) 26. (c) 27. (c) 28. (b) 29. (b) 30. (d)
31. (c) 32. (b) 33. (d) 34. (a) 35. (a) 36. (a) 37. (b) 38. (*) 39. (a) 40. (c)
41. (d) 42. (d)
    (*)  No answer  

Hints and Solutions

P4 + excess Cl2 PCl5 (B)


PCl5 + H2O H3PO4 + HCl
1. (a) Borax H
SO 9. (b) Holme’s signal can be given by using (CaC2 +
→ H3BO3 ∆
→ B2O3 Al
2 4
→
Ca3P2)
B
( Boron )
10. (b) Pb(NO3)2 ∆
→ PbO + NO2 + O2 ­
2. (a) In borax, anion [B4O5(OH)4]–2 is present.
OH 11. (a) Single N–N bond is weaker than the single P–P
O O bond.
B O O O
  12. (d) N N
HO – B – O – B – OH O
O
B
O O 13. (a) Phosphine (PH3) is combustible due to presence
OH of P2H4.
In this, hybridisation of B is sp2 and sp3. 14. (d) Extra pure N2 can be obtained by heating metal
3. (d) CO + Cl2 h
ν
→ COCl2 azides Ba(N3)2.
( phosgene )
15. (c) Ammoia can be dried by anhydrous CaO.
4. (b) H 2 C2 O 4  conc. H 2 SO4
→ CO + CO 2 16. (d) 2Hg + O3 Hg2O + O2
( Oxalic acid ) (A) ( B)
17. (d) Polythionic acid, H2SnO6
CO is absorbed by ammonical cuprous chloride O O
while CO2 is absorbed by KOH.
HO—S—(s)n–2 — S — OH
5. (c) SiC is a covalent network solid.
6. (b) NH3 + O2 pt→ NO + H2O (ostwald process) O O

7. (c) NO is paramagnetic in gaseous state. Total S–S bond = n – 1


Number of S with zero oxidation state = n – 2
8. (c) P4 + Cl2 PCl3(A)
Number of S with + 5 oxidation state = 2
PCl3 + H2O H3PO3 + HCl
Number of dp – pp bond = 4
7.23
+4 +3 +5
18. (b) 2 NO 2 + H 2 O → HNO 2 + H NO3 3. (d) With large sized Lewis base like (CH3)3N,
19. (a) Element ‘X’ may be Cl, Br or I. symmetrical deavage of B2H6 takes place.
The most acidic oxyacid of X is HXO4. 4. (c) H3BO3 + NaOH  
 Na[B(OH)4] + NaBO2 +
H2O
20. (b) RCOO– is not a pseudohalide because it does not
have N-atom. This reaction can be shifted in forward direction
by adding cis-1, 2-diol not by trans-1,2-diol.
21. (d) ICl3 can form dimer I2Cl6.
22. (a) I4O9 is an ionic oxide. 5. (c) Both CO2 and SiO2 are acidic.
I4O9 is I(IO3)3 6. (b) P4 + 3NaOH + 3H2O 3NaH2PO2 + PH3
23. (a) F– ion does not behave as reducing agent. In products
total 9 P–H bonds are present.
24. (a) F2 acts as oxidizing agent with water. 7. (c) At 0°C, NO2 diamerises into N2O4(s) which is
colourless.
5F2 + 5H2O 10HF + O2 + O3
8. (b) Ca3P2 + H2O Ca (OH) 2 PH 3
I2 does
not react with water. (A) ( B)

25. (a) F2 is the strongest oxidizing agent due to PH3 + HI PH 4 I KOH


 → PH 3 + KI + H 2 O
(C) (D) ( B)
exceptionally high hydration energy of F–.
9 . (a) NH4NO3 ∆
→ N2O + 2H2O
26. (b) XeF4 + SbF5 [ XeF3 ]+ [SbF6 ]− NH4NO2 ∆
→ N2 + 2H2O
sp3 d sp3 d 2
10. (a) Order of stability of hydrides:-
27. (c) Both ICl2– and XeF2 are linear. NH3 > PH3 > AsH3 > SbH3 > BiH3
28. (d) No real compounds of He, Ne and Ar are known.
11. (a) (NH4)2Cr2O7 ∆
→ N2­- + Cr2O3 + H2O
29. (d) Hybridisation of C in diamond and graphite is sp3
and sp2 respectively. NH4NO2 ∆
→ N2­- + 2H2O

30. (c) K2HgI4 + KOH + NH3 H2N–Hg–O–Hg–I 12. (b) NH3 + Cl2(excess) NCl3 + HCl
31. (a) The part of the skin which comes in contact with NH3(excess) + Cl2 N2 + NH4Cl
HNO3 turns yellow because proteins of skin are 13. (b) Thermal decomposition of O3 is exothermic.
converted into xanthoproteins.
14. (a) Na 2 SO3 + HCl(dil ) → NaCl + SO2 ↑ + H 2 O
32. (c) Order of lewis acidic strength:- (A) (Y)

BF3 < BCl3 < BBr3 < BI3 Na 2 S + HCl(dil ) → NaCl + H 2 S
( B) (Z)
33. (a) NO2 is a reddish-brown gas. 15. (b) H2SO4 + Cl2 SO2Cl2
34. (b) S S S
S S (X)
S
S S
Total number of lone pairs = 16 SO2Cl2 + H2O H2SO4 + HCl
Total number of S–S bonds = 8 16. (b) Zn + H2SO4(dil) ZnSO4 + H2
35. (b) Order of bond energy is : Zn + H2SO4(conc.)
ZnSO4 + SO2 + H2O
Cl2 > Br2 > F2 > I2 17. (d) I2 + H2O HI + HOI

18. (c) I–Cl is polar and Br2 is non-polar.


19. (b) Cl2 + Ca(OH)2 CaOCl2 + H2O
1. (c) Boric acid is not a bronsted acid because it is not bleaching powder
a H+ donor. It is an OH– acceptor.
∆ 20. (b) Interhalogen compounds are more reactive than
→ B2 O3 + NaBO2 + H2O
2. (c) Na2B4O7 . 10H2O 
(X) halogen compounds because X–X' bond of
interhalogen is polar while X–X bond of halogen
→ Cr (BO 2 )3
B2O3 + Cr2O3  ∆
(Y) is non-polar.
7.24

21. (c) Correct order of bond dissociation energy:- CH3 CH3 CH3 CH3
| | | |
HO—Si—OH HO—Si—OH ~ Si—O—Si—O ~
Cl2 > Br2 > F2 > I2 | |
| |
OH OH O O
22. (c) Shape of XeF5– is pentagonal planar. Hybridisation | |
of Xe is sp3d3. ~ Si—O—Si—O ~
OH OH | |
| | CH3 CH3
23. (c) XeF4 acts as fluoride acceptor with KF. HO—Si—OH HO—Si—OH
| |
24. (a) Due to resonance both O—O bond lengths are CH3 CH3
Cross linked silicones
equal.
In white phosphorus P4 molecules are attached
25. (a) 2Hg + O3 Hg2O + O2 with weak vander waal’s force of attraction while
26. (c) NH4NO3 ∆
→ N2O­+ 2H2O red phosphorus is polymeric. White phosphorus is
N2O acts as anaesthesia. insoluble in water but soluble in organic solvents
like CS2. White phosphorus disproportionates in
27. (c) Black phosphorus has layery structure. It behaves alkali solution.
as conductor of electricity.
NH4NO3 ∆
→ N2O + 2H2O
28. (b) N2 is almost chemically inert. It is due to very
high bond dissociation energy of N/N. N2O is also called as laughing gas. N2O is a
neutral oxide. It does not bring tears to the eyes.
29. (c) Order of reactivity :
White P > Red P > Black P
All decompositions are correct.
30. (c) N2O5 has different structure from P2O5.
P2O5(P4O10) has cage like structure.
Nitrogen is restricted to a maximum covalency
31. (b) Third period elements can not form stable 3pp – of 4 because in its valence shell only 4 orbitals
3pp bond. are available for bonding. N–N bond is weaker
32. (c) Correct order of acidic strength is:- than P—P bond hence, catenation tendency of P
HClO < HClO2 < HClO2 < HClO4 is stronger than N.
33. (a) The ease of liquification µ intermolecular bonding N can not form p -d bonding.
order of case of liquification : 9. (a, b, c)
Xe > Kr > Ar > Ne > He
P2O5 (P4O10) is an acidic dehydrating agent. It
34. (d) XeF6 + 3H2O XeO3 + 6HF dehydrate acidic substances.
35. (d) XeF6   +  CsF Cs+[XeF7]– 10. (a, b)
Lewis acid    Lewis base
They (halogens), all are diatomic, form univalent
ions and are oxidizing agents.
LEVEL III 11. (a, b) F2 and Cl2 can oxidize Br¯ into Br2
12. (a, b, c)
1. (b, c, d)

I2 + H2O HI + HIO

Both graphite and diamond are diamagnetic in
nature. 13. (a, b, d)
F can not form 3 bonds. It can form only one
bond.
Co+3, Cu+2 and Ni+2 give borax bead test.
14. (b) To form pyro-oxy acid atleast two OH groups
must present in parent oxyacid.
[BF6]–3 does not exist because due to unavailability 15. (b) Structure of H2N2O2
of d-orbital B can not form more than 4 bonds.
OH
CH3 CH3 N
| | ||
Cl—Si—Cl + 3H2O HO—Si—OH + 3HCl
| | N
Cl OH
HO
7.25

16. (a) Order of acidic strength: Gaseous oxides are:


HF < HCl < HBr < HI N2O , NO , NO2 , SO2 , SO3
17. (a) Mixed anhydride are:

Order of acidic strength: ClO2 , Cl2O6 , NO2
4 3 5
H2O < H2S < H2Se < H2Te 2 Cl O2
H2 O H Cl O2 H Cl O3
Order of basic strength : 6 5 7
Cl2 O6
H2 O H ClO3 H Cl O 4
OH¯ > HS¯ > HSe¯ > HTe¯
4 3 5
2 N O2
H2 O H N O2 H N O3
(NH4)2Cr2O7 ∆
→ N2 + Cr2O3 + H2O Pseudohalides are:
(A) (C)   (B) CN¯ , SCN¯ , N3¯ , CNO¯
Mg + N2 Mg3N2 A P, S ; B S;C Q, R ; D R, S
A R;B Q;C S;D P
(D)
A P, Q, S ; B P, Q, R, S ; C Q, R, S ;
Mg3N2 + H2O Mg(OH)2 + NH3   D P, Q, R, S
(E)
Anion of (A) is Cr2O7 –2. In structure of dichromate

(Cr2O7–2) all metal-oxygen bond lengths are not


equal.

KHF2 K+ + HF2¯
Gas (E) is NH3 Order of acidic strength:
CuSO4 + NH3 [Cu(NH3)4] SO4 ClOH > BrOH > IOH
deep blue RCOO¯ is not a pseudo halide because it does not
NH3 + O2 → NO + H2O Pt / Rh contain any N-atom.
NH3 + KMnO4 N2 Order of bond angle:
BCl3 > PCl3 > AsCl3 > BiCl3
NH3 form
black precipitate with Hg2Cl2.
NH3 can be dried by anhydrous CaO.
In C60, Number of 5-membered rings (a) = 12
Order of biling point:
Number of 6-memberd rings (b) = 20 H2O > H2Te > H2Se > H2S
Number of -bonds in C60(c) = 30 Cyclic metaphosphoric acid, (HPO3)3
Value of (3a – 2b + c) = 36 – 40 + 30 = 26 O OH

Species having sp3 hybridization: P
O O
Diamond, H3BO3(eq) , Zeolites (Si-central).
O
Silicones (Si), chlorosilane (Si), Borax (boron), HO P P
Al2Cl6 , B2H6 , SiO2 , CH4 , CCl4 O OH
O
Ampoteric oxixdes are: It has three P—O—P bonds.
PbO , PbO2 , SnO , SnO2 , Al2O3 , ZnO , BeO , 8. (d)
O O
Ga2O3
S
Si3O96– is a cyclic trimer. In this , 3 oxygen atoms O O
are shared between tetrahedrons. O O
S S
Atomic size, reducing power of hydrides and O
O O
metallic character increase down the group for

It has zero S—S bonds
nitrogen family.
7.26

9. (a) Order of correct acidic strength: In P4, hybridisation of P is sp3.


HClO < HClO2 < HClO3 < HClO4 Due to greater solubility in water, nitrates are less
10. (c) The lower oxidation states for the group 14 abundant in earth’s crust.
elements are more stable for the heavier members Lone pair in N of NH3 is present in sp3-hybrid
of the group due to inert pair effect. orbital which is more directional.
Both SiCl4 and CCl4 are covalent. White phosphorus (P4) undergo disproportionation
H3BO3 is weak monobasic and Lewis acid. in alkaline medium.
Me2SiCl2 H 2O
→ linear silicones P4 + (mixture of O2 and N2) P4O6
H3PO3 is dibasic and reducing agent because it Due to addition of N2 , rate of reaction decreases.
can oxidize to +5 oxidation state. Extra pure N2 can be obtained by heating metallic
KMnO4 + KOH + KI K2MnO4 + KIO3 azides, Ba(N3)2.
o −3 +1
    + H2O P4 + NaOH + H2O PH3 + NaH 2 P O2
H2S2O6 H2S2O8
(disproportionation)
+5 +2 o −3
O O S O HNO3
|| || NO N2 N H 4 Cl
|| ||
HO—S—S—OH HO—S—O—O—S—OH
|| || || ||
O O O O

H2S2O3 H2S4O6
See - saw shape
O O
S || || P is NaOCl (Salt of HOCl, hypochlorous acid)
|| HO—S—S—S—S—OH
HO—S—OH || || Q is NaClO3 (Salt of HClO3, Chloric acid)
|| O O
O R is SO2Cl2
In sheet silicates, three oxygen atoms are shared S is PCl5
of [SiO4]4–. T is H3PO4
In KMnO4 , Mn is present in its highest oxidation P4 + SOCl2 PCl3
state +7.
Due to release of NO2 gas, upper layer of HNO3
PbO2 + HNO3 Pb(NO3)2 + H2O + O2 turns yellow.
N2O3 is mixture of NO and NO2. No answer because all statements are correct.
Order of stability: NO is diamagnetic in liquid state.
Black > Red > White phosphorus Order of acidic strength:
Cis-1, 2-diol consumes Na[B(OH)4] hence, this HClO < HClO2 < HClO3 < HClO4
reaction proceed in forward direction.
Interhalogen compound (ICl) is more reactive
Due to small size of B+3, Boron always forms than pure halogen.
covalent bond.
Order of boiling point:
In water, orthoboric acid donates only one H+.
He < Ne < Kr < Xe
H3BO3 + H2O [B(OH)4]– + H+
or
B(OH)3
Chapter

Key Concepts
Chlorine gas smell Hypochlorites (ClO-)
Qualitative analysis involves the detection of basic radicals
(cations) and acidic radicals (anions) of a salt or a mixture Bitter almond smell Cyanides
of salts. Physical appearance (Coloured substance):
Acid + Base " Salt + H2O + Heat of Neutralisation
Light pink Hydrated salt of Mn
HCl + NaOH" NaCl + H2O + Heat
Reddish Pink Hydrated salt of Co(II)

Red HgO, HgI2, Pb3O4
Na+ Cl- Orange – red Sb2S3, Some dichromates and
(Basic radical because (Acidic radical because it ferricyanides
it comes from base) comes from acid) Reddish brown Fe2O3
The systematic procedure for qualitative analysis is: Dark brown PbO2, Bi2S3, CdO, Ag2O, CuCrO4, SnS
Preliminary tests: Light yellow or brown Chromates, As2S3, As2S5, AgBr, AgI,
PbI2, CdS, SnS2, a few iodides and

" Physical appearance (Colour and smell) ferrocyanides.

" Dry heating test Green K2MnO4, Ni salts, hydrated ferrous

" Flame test salts, some Cu (II) Compound


" Borax bead test Dark green Salt of Cr(III)


" Charcoal cavity test Blue Hydrated CuSO4, anhydrous CoSO4
Black Sulphides of Ag+, Cu+, Cu+2, Fe+2, Ni+2,

" Cobalt nitrate test
Co+2, Hg+2, and Pb+2. MnO2, Fe3O4,
Wet tests for acidic radicals FeO, CuO, Co3O4, Ni2O3
Wet tests for basic radicals Physical appearance (Solution is coloured):
Physical appearance (Smell): Green or blue Ni+2, Fe+2, Cr+3 and Cu+2
Smell Inference Pink Co+2 and Mn+2
Ammonical Smell NH4+ Yellow CrO42-, Fe+3, [Fe (CN) 6]4-
Vinegar like Smell CH3COO- Orange Dichromates
Smell like that of rotten eggs S-2 Purple Permanganates
8.2

Dry heating: Acetic acid vapours (characteristic CH3COO-


Observation Inference vinegar like smell)
1. Substance decrepitates (Crackling noise) NaCl, KI, (iii) Coloured gas
Pb(NO3)2, NO2 (Reddish brown, turns ferrous NO2- or NO3-
Ba(NO3)2 sulphate solution brownish black)
2. Substance Melts (or, fuses) Alkali metal Br2 (Reddish brown turns starch paper Br-
salts or salt orange-red or yellow, turns starch
containing water iodide paper blue)
of crystallization
Cl2 (Greenish yellow, turns starch Cl-
3. Substance swells (due to loss of water of Alums, borates iodide paper blue, bleaches moist
crystallization) and Phosphates litmus paper, bleaches indigo solution)
4. The substance Sublimes and the colour of I2 (violet, turns starch paper blue) I-
sublimate is:-
a. White Hg2Cl2, NH4X, Flame test: The chlorides of the metals are more volatile
AlCl3, HgCl2, as compared to other salts and these are prepared by mixing
As2O3,Sb2O3 the compounds with a little concentrated HCl. On heating
b. Yellow As2S3 and HgI2 in a non- luminous Bunsen flame, they are volatilized and
(turns red when impart a characteristic colour to the flame.
rubbed with
glass rod) Metal Colour of flame
c. Blue black and violet vapours Iodides Li Crimson red
5. A residue (generally oxides) is left and Na Golden yellow
its colour is: K Violet/lilac
White (Cold) and Yellow (Hot) ZnO Rb Red violet
Reddish brown (Hot); Yellow (Cold) PbO Cs Blue violet
c. Black (Hot); Red (Cold) HgO, Pb3O4 Ca Brick red
Black (Hot); Red brown (Cold) Fe2O3 Sr Crimson red
Original salt Blue becomes White on Hydrated CuSO4 Ba Apple green
heating
6. Gas is evolved: Ammonium Borax bead test:
nitrate
Na2B4O7.10H2O ∆
→ Na2B4O7 ∆
→ 2NaBO2 + B2O3
(i) Colourless and odourless
CO2 (Turns lime water milky) CO3-2
C2O4-2   
Colourless transparent
O2 (Rekindles a glowing splinter) Nitrates,   glassy bead
permanganate,
On heating with a coloured salt, the glassy bead forms a
Dichromate,
chlorate coloured metaborate in oxidizing flame.
N2 Amonium Colour of bead
Nitrate
Metal Oxidising flame Reducing flame
(ii) Colourless gas with odour
Hot Cold Hot Cold
NH3 (Characteristic smell, turns NH4+
nesseler’s solution brown and turns red Cr Yellow Green Green Green
litmus blue) Mn Violet Amethyst Colourless Colourless
H2S (Smell of rotten eggs, turns lead S-2 or Hydrated (Amethyst)
acetate paper black) S-2 Fe Yellowish Yellow Green Green
SO2 (suffocating or irritating smell SO32- Brown
of burning sulphur, turns acidified
Co Blue Blue Blue Blue
K2Cr2O7 paper green)
Ni Violet Reddish- Grey Grey
HCl (Pungent smell, white fumes Hydrated Cl-
Brown
with ammonia, white ppt with AgNO3
Solution) Cu Green Blue Colourless Red opaque
8.3

Solubility rules: v NH4Cl + Ca2+, Sr2+,Ba2+ CaCO3


Ions Solubility Exceptions (NH4)2CO3 SrCO3 White
+ + + + BaCO3
NH4 , Li , Na , K , Soluble None
Rb+, Cs+ vi NH4Cl + Mg2+ Mg(NH4)PO4
- - NH4OH+ (White)
NO3 , CH3COO , Soluble None Na2 HPO4
ClO4-, ClO3-,
Zero NaOH or NH4+ NH3 Gas
MnO4-, HCO3-, Ca(OH)2
HSO3- (heat)
Cl-,Br-,I- Soluble Ag+, Pb+2, Hg2+2, Cu+
(a) From the filtrate of iind group, H2S gas is boiled
SO4-2 Soluble Hg+2, Pb+2, Sr+2, off and then one or two drop of concentrated
Ca+2, Ba+2 HNO3 is added and again boil so that if Fe2+ is
OH- Insoluble Li+, Na+, K+, Rb+, present, it will oxidize into Fe3+.
Cs+, Ca+2, Sr+2, Ba+2
-3 -2 -2
(b) iiird group radicals are precipitated as hydroxides
PO4 , S , CO3 , Insoluble NH4+, Li+, +
Na , K , +
and the addition of NH4Cl suppresses the
SO3-2 +
Rb , Cs+
ionization of NH4OH so that only iii group
radicals are precipitated as hydroxides because of
Classification of basic radicals:
their low solubility products.
Group Group Basic radical Precipitate and its
reagent colour
(c) Excess of NH4Cl should not be added, Mn2+ will
precipitate as MnO2.H2O.
Dil HCl Pb2+, Hg22+, Ag+ AgCl (white)
PbCl2 (white) (d) (NH4)2SO4 cannot be used in place of NH4Cl
Hg2Cl2 (white)
because SO42- will also give the precipitate of
2+ 2+ 3+ BaSO4, SrSO4 etc.
ii H2S in ii (a) Hg , Pb , Bi HgS
presence of Cu2+, Cd2+ PbS (e) In acidic medium, hydroxides do not precipitate.
dilute HCl (Copper group) CuS Black (f) In place of NH4OH, NaOH can’t be used because
Their sulphides are Bi2S3 in excess of it we get soluble complexes of Al3+
not soluble in YAS and Cr3+.
((NH4)2S2) as well as CdS (Yellow)
in colourless (NH4)2S (g) In ivth group radicals NH4OH increases the
ii (b) As3+,As5+,Sb3+, As2S3 ionization of H2S by removing H+ from H2S as
5+ 2+
Sb ,Sn ,Sn 4+ Yellow unionized water. (Ksp values of these sulphides
As2S5
are very high)
(Aresenic group).
Sb2S3
Their sulphides are Orange (h) In vth group radicals, (NH4)2CO3 should be added
soluble in YAS, also Sb2S5
in alkaline or neutral medium. In the absence
they are soluble in Sb2S5 (Brown) of ammonia or NH4+ ions, Mg2+ will also be
colourless (NH4)2S
SnS2 (Yellow) precipitated.
except SnS
iii NH4Cl + Fe3+, Al3+, Cr3+ Al(OH)3 Zero group (NH4+ ion):
NH4OH (Gelatinous Sodium hydroxide solution:
White)
NH4+ + OH- $ NH3 - + H2O
Fe (OH)3 (Reddish
Brown) (NH4Cl) (NaOH)
Cr(OH)3 (Green) Ammonia gas can be identified, if
2+ 2+ 2+ 2+
iv NH4Cl + Co , Ni , Zn , Mn ZnS (White) By its characteristics smell.
NH4OH + MnS (Buff or
H2S in warm By formation of white fumes of NH4Cl with HCl.
Pink) of flesh
solution colour NH3 + HCl $ NH4Cl- (white fumes)
(or, excess
(NH4)2S) CoS By its turning moistened red litmus paper blue or
NiS Black
turmeric paper brown.
8.4

By its ability to turn filter paper moistened with Group II


mercury (I) nitrate solution black. l group filtrate + dil.HCl + H2S
Hg2(NO3)2 + 2NH3 $ Hg(NH2)NO3 + Hg + NH4NO3
(Copper group) (Arsenic group)
HgS, PbS, Bi2S3, CuS, CdS As2S5 Yellow
Black As2S3
(Black) Yellow Sb2S3, Sb2S5
With Nessler’s reagent: [Insoluble in YAS (NH4)2S2 Orange
+ 2- - - and NaOH and (NH4)2S] SnS, SnS2
NH4 + 2[HgI4] + 4OH $HgO.Hg(NH2)I.+7I [II A]
or Brown Yellow
+ 3H2O [Soluble in YAS and NaOH and
NH3 Nessler’s reagent Brown ppt. (NH4)2S. SnS is not soluble in
(NH4)2S] [II B]
or
Group II (A)
Brown or Yellow colouration
HgS, PbS, CuS, Bi2S3, CdS
(Iodide of millon’s base) (Yellow)
Black
dissolve ppt. in 50%
Group I HNO3 and filter

dil.HCI
Insoluble (Hgs) Filtrate (Nitrates of
Pb+2, Bi+3, Cu+2 and Cd+2)
AgCl, Hg2Cl2, PbCl2 (White ppt.)
Dissolve in
Dissolve in hot water and filter
Aquaregia add dilute H2SO4 and
+SnCl2
little C2H5OH and filter
If ppt. doesn’t If ppt. dissolve Black grey
dissolve, add NH4OH (PbCl2) Ppt. (Hg2+ White ppt. filterate (nitrates and sulphates
and shake and filter present) (PbSO4) of Bi3+, Cu2+, Cd2+)
[soluble in
CH3CO ONH4] add NH4OH solution
KI K2CrO4 dil. H2SO4

PbI2 PbCrO4 PbSO4


Yellow Yellow White
(in excess (soluble in (soluble
KI, a soluble HNO3 are well in KI K2CrO4 Bi(OH)3 filtrate
complex in NaOH, Ammonium PbCrO4 white may contain
K2PbI4) insoluble in dil. Acetate) YELLOW Yellow [Cu(NH3)4]2+
CH3COOH) dil. HCl [Cd(NH3)4]2+
Ppt. get soluble [Ag(NH3)2]Cl BiCl3
Excess
of water
BiOCl (white turbidity)
dil.HNO3 KI K2CrO4
AgCl AgI Ag2CrO4
White Yellow Brick red
or red ppt.
If filtrate is blue add KCN
ppt. turns black In colour Cu
2+

Hg(NH2)Cl + Hg May be present


dissolve in aquaregia
and boil Add acetic acid and colourless colourless
K4[Fe(CN)6] solution of solution of
3- 2-
SnCl2 KI Cu turnings [Cu(CN)4] [Cd(CN)4]
HgI2 Black ppt. Cu2[Fe(CN)6] pass H2S pass H2S
Grey or Black Red or orange (soluble (Hg finely divided) Reddish-brown ppt.
ppt. (Hg) in excess KI to form Or, chocolate ppt. no black CdS
K2[HqI4] ppt. of CuS Yellow ppt.
(Cu2+ cofirms)
(Cd2+ cofirms)
8.5

Group II(B) Group IV


As2S3, As2S5, Sb2S3, Sb2S5, SnS, SnS2 III group filtrate + NH4OH (excess) and NH4Cl, then pass H2S gas
Yellow Orange Brown Yellow
Soluble in Soluble only CoS (black), MnS (light pink or buff pink or pale brown), ZnS (White), NiS (black)
YAS(NH4)2S2 in (NH4)2S2
and (NH4)2S not in (NH4)2S add dilute HCI and
shake and filter
(NH4)3AsS4 (NH4)3SbS4 (NH4)2SnS3

Residue Filtrate
(Black ppt. of CoS or NiS) (MnCIz + ZnCIz)
Boil to remove H2S
Add dilute HCI and fitter Dissolve in aqua regia and add Br2/H2O + NaOH
evaporate to dryness
May be As2S5, As2S3 Black Residue Filtrate
Yellow Blue Residue (MnO2) (Na2ZnO2)
and S (Yellow ppt.) Residue (CoCIz)
dissolve in warm NH3 (NiCIz)
or alkali hydroxides Dissolve in Conc. HNO3 and
then add PbO2 and heat
3- 3-
Turns green Turns pink
AsO3 AsO4 in water
Arsenite Arsenate MgCl2, NH4Cl and NH3 in water
purple solution
AgNO3 AgNO3 Mg(NH4) AsO4 of MnO4-
Ag3AsO3 Add excess of Solution+ NH4CNS (Solid)
Ag3AsO4 White
(Yellow) (Brownish-red)
Filtrate may contain dimethylglyoxime NH4OH+ +
soluble SbCl3 and SnCl4 in ammonical CH3COOH+ amyl alcohol
solution KNO 2 Solution
+ CH3COOH
Blue colour +K4 [Fe(CN)6]
in alcohol
Red/Rosy red ppt. Yellow ppt. of layer of
Of [Ni(dmg)2] K3 [Co(NO2)6]
(NH4)2 [Co(SCN)4] Light blue-white
ppt. of Zn2 [Fe(CN)6]

add Make proper solution Fe power or Zn dust In excess of reagent


Zn3K2 [Fe(CN6)2]
excess by adding H2O + NH4OH+ Acidify with acetic
water Oxalic acid acid and pass H2S gas
pass H2S gas Heat and then add
HgCl2
SbOCl Sb2S3 (Orange)
(white Black grey ppt. ZnS
turbidity) 4+
(Sn confirms) (white)

Group V
Group III Boil of H2S then add (NH4)2 CO3 (aq), NH4OH and NH4Cl(s)

II group Boil off H2S then add conc. HNO3 (1-2) drops + NH4Cl + NH4OH BaCO3, CaCO3, SrCO3 (white ppt.)
Filtrate
Al(OH)3 Fe(OH)3 Cr(OH)3
Gelatinous or, Reddish or, green Dissolve in CH3COOH
White brown
Add excess of NaOH and the solution is boiled
K2CrO4 (NH4)2 SO4 (NH4)4C2O4
with excess of Br2 water (or, H2O2 solution)
then filter
BaCrO4 (Yellow) SrSO4 (White) CaC2O4 (White)
(insoluble in CH3COOH)
Residue Filtrate
2-
Fe(OH)3 may contain CrO4 (Yellow)
-
and AlO2 or [Al(OH)4]
-
Group VI
Dissolve residue (Colourless)
in HCl
FeCl3
If colourless If solution
then add NH4Cl is yellow
and heat
K4[Fe(CN)6] KSCN
(Prussian blue) or, NH4SCN
Blue ppt. of Deep red or BaCl2 CH3COOH +
Fe4[Fe(CN)6]3 blood red colouration Gelatinous (CH3COO)2Pb
of Fe (SCN)3 White ppt.
of Al(OH)3
$ .
Yellow ppt. Yellow ppt.
of BaCrO4 of PbCrO4
8.6

Classification of acidic radicals (anions): CaCO3 + H2O + CO2 " Ca(HCO3)2


Acidic radicals can be broadly divided into two groups White ppt.      soluble
Note: Sulphur dioxide evolved from sulphites also turns
lime water milky.
Class A Class B Ca(OH)2 + SO2 " CaSO3 + H2O
(On treatment with acids (They are dependent upon
they produce volatile products) reactions in solution) White ppt.
However SO2 can be identified by its pungent odour of
(I) (II) (I) (II) burning sulphur.
(Gases evolved (Gases or (Precipitation (Oxidation
With dilute HCI vapours reactions and Sulphite, SO32-:
or dilute H2SO4 evolved reduction
with conc. H2SO4) So42- in solution) (i) The sulphite gives out sulphur dioxide gas, having
CO32- F- PO43- suffocating smell of burning sulphur.
HCO3- NO3- CrO42-
S2-
Cl Cr2O72-
- CaSO3 + H2SO4 " CaSO4 +H2O + SO2 -
SO3 Br-
2-
MnO4-
(ii) When acidified potassium dichromate paper is
S2O32- I-
NO2- C2O42- exposed to the gas, it attains green colour due to
CH3COO- BO33-, B4O72-, BO2-
the formation of chromic sulphate.
K2Cr2O7 + H2SO4 + 3SO2 " K2SO4 + Cr2(SO4)3
Preparation sodium carbonate extract:-
+ H2O
Salt(s) + Na2CO3 (s) + distilled water
1 : 3 or 4 The sulphite also gives white precipitate with
Heat and filter BaCl2, soluble in dil. HCl
Na2SO3 + BaCl2 " 2NaCl + BaSO3.
Residue Filtrate
(Carbonates of basic radicals) (Sodium salts of acidic radicals Sulphide, S-2:
[It is known as sodium
carbonate extract] (i) The sulphide salts form H2S which smells like
Used for identification rotten eggs.
of acidic radicals
Except: CO32- and HCO3-
Na2S + H2SO4 " NaSO4 +H2S-
BaCl2 + Na2CO3 $ BaCO3 . + 2NaCl (aq)

(ii) On exposure to this gas, the lead acetate paper
White
turns black due to the formation of lead sulphide.
Sodium carbonate extract is used when: Pb(CH3COO)2 + H2S " PbS. + 2CH3COOH
a. Salt is only partially soluble in water or insoluble black ppt.
in water.
(iii) The sulphides also turn sodium nitroprusside
b. Cations interfere with the tests for acidic radicals
solution violet (use sodium carbonate extract for
or the coloured salt solution may be too intense in
this test).
colour that the test results are not too clear.
Na2S + Na2[FeNO(CN)5] " Na4[Fe(NOS)(CN)5]
Carbonte, CO32-:
Sulphide of lead, calcium, nickel, cobalt,
(i) The carbonates are decomposed with the antimony and stannic are not decomposed with
effervescence of carbon dioxide gas. dilute H2SO4. Conc. HCl should be used for their
Na2CO3 + H2SO4 " Na2SO4 +CO2 test. However brisk evolution of H2S takes place
(ii) When this gas is passed through lime water, even by use of dilute H2SO4 if a pinch of zinc dust
it turns milky with the formation of calcium is added.
carbonate. Zn + H2SO4 " ZnSO4 + 2H
Ca(OH)2 + CO2 " CaCO3 + H2O HgS + 2H " Hg + H2S-
Lime water white ppt.
Nitrite, NO2–:
(iii) If the CO2, gas is passed in excess, the milky
solution becomes colourless due to the formation (i) The nitrites yield a colourless nitric oxide gas
of soluble calcium bicarbonate. which in contact with oxygen of the air becomes
8.7

brown due to the formation of nitrogen dioxide. white precipitate of silver chloride.
2KNO2 + H2SO4 " K2SO4 + 2HNO2 Ag(NH3)2Cl + 2HNO3 " AgCl + 2NH4NO3
Nitrous acid Chromyl chloride Test: When solid chloride is
3HNO " H2O + 2NO + HNO3 heated with conc. H2SO4 in presence of K2Cr2O7,
deep red vapours of chromyl chloride are evolved.
2NO + O2 " 2NO2
NaCl + H2SO4 " NaHSO4 + HCl
brown coloured gas
K2Cr2O7 + 2HCl " 2KHSO4 + 2CrO3 + H2O
(ii) On passing the gas through dilute FeSO4 solution,
brown colored complex salt is formed. CrO3 + 2HCl " CrO2Cl2 + H2O
Chromyl chloride
FeSO4.7H2O + NO " [Fe(H2O)5NO]SO4 + 2H2O
These vapours on passing through NaOH solution
Brown ring complex
form the yellow solution due to the formation of
(iii) When a mixture of iodide and nitrite is treated sodium chromate.
with dilute H2SO4, the iodides are decomposed
CrO2Cl2 + 4NaOH " Na2CrO4 + 2NaCl + 2H2O
giving violet vapours of iodine, which turns starch
iodide paper blue. Yellow colour

2NaNO2 + H2SO4 " Na2SO4 + 2HNO2 The yellow solution neutralized with acetic acid
gives a yellow precipitate of lead chromate with
2KI + H2SO4 " K2SO4 +2HI lead acetate.
2HNO2 + 2HI " 2H2O + I2 + 2NO Na2CrO4 + Pb(CH3COO)2"PbCrO4 + 2CH3COONa
            Violet
Yellow ppt.
           vapours
I2 + Starch " colour Note:
-
Chloride Cl This test is not given by the chloride of mercuric,
(i) Colourless pungent fumes of hydrogen chloride tin, silver, lead and antimony.
are evolved. The chromyl chloride test is always to be
NaCl + H2SO4 " NaHSO4 + HCl performed in a dry test tube otherwise the chromyl
chloride vapours will be hydrolysed in the test
(ii) Yellowish green chlorine gas with suffocating tube.
odour is evolved on addition of MnO2 to the
above reaction mixture. CrO2Cl2 + 2H2O " H2CrO4 + 2HCl
Bromides and iodides do not give this test.
NaCl + H2SO4 " NaHSO4 + HCl
MnO4 + 4HCl " MnCl2 + 2H2O + Cl2 Bromide, Br-:

(iii) The gas evolved forms white fumes of ammonium Reddish-brown fumes of bromine are formed.
chloride with NH4OH. NaBr + H2SO4 " NaHSO4 + HBr
NH4OH + HCl " NH4Cl + H2O 2HBr + H2SO4 " Br2 + 2H2O + SO2
(iv) The gas evolved or solution of chloride salt forms More reddish brown fumes of bromine are
a curdy precipitate of silver chloride with silver evolved when MnO2 is added.
nitrate solution.
2NaBr + MnO2 + 3H2SO4 " 2NaHSO4 + MnSO4
AgNO3 + HCl " AgCl.  + HNO3
+ 2H2O + Br2
Note:
The aqueous solution of bromide or sodium
The curdy precipitate of AgCl dissolves in carbonate extract gives pale yellow precipitate of
ammonium hydroxide forming a complex salt. silver bromide which partly dissolve in excess of
AgCl + 2NH4OH " Ag(NH3)2Cl + 2H2O NH4OH forming a soluble complex.
The solution having the silver complex on NaBr + AgNO3 " AgBr. + NaNO3
acidifying with dilute nitric acid gives again a Pale yellow ppt.
8.8

AgBr + 2NH4OH " Ag(NH3)2 Br + 2H2O Oxalate, C2O4-2:


Iodide, I-: A mixture of CO and CO2 is given off. The CO
Violet vapours of iodine are evolved. burns with blue flame.

2KI + H2SO4 " 2KHSO4 + 2HI Na2C2O4 + H2SO4 " Na2SO4 + H2C2O4

2HI + H2SO4 " I2 + SO2 + 2H2O H2C2O4 + H2SO4 " CO + CO2 + H2O + [H2SO4]

Violet vapours with starch produce blue colour. 2CO + O2 " 2CO2

I2 + starch " Blue colour Sulphate, SO4-2:

More violet vapours are evolved when MnO2 is Add conc. HNO3 to a small amount of substance
added. or take sodium carbonate extract and then add
BaCl2 solution. A white precipitate of BaSO4
2KI + MnO2 + 3H2SO4 " 2KHSO4 + MnSO4 + insoluble in conc. acid is obtained.
2H2O + I2
NaSO4 + BaCl2 " 2NaCl + BaSO4
Aqueous solution of the iodide or sodium White ppt.
carbonate extract gives yellow precipitate of
Agl with silver nitrate solution which does not Borate:
dissolve in NH4OH. To a small quantity of the substance (salt or
NaI + AgNO3 " AgI + NaNO3 mixture), add few multilitres of ethyl alcohol and
Yellow ppt. conc. H2SO4 and stir the contents with a glass
rod. Heat the test tube and bring the mouth of the
Nitrate, NO3– : test tube near the flame. The formation of green
Light brown fumes of nitrogen dioxide are edged flame indicates the presence of borate.
evolved. 2Na3BO3 + 3H2SO4 " 3Na2SO4 + 2H3BO3
NaNO3 + H2SO4 " NaHSO4 + HNO3 H3BO3 + 3C2H5OH " (C2H5)3BO3 + 3H2O
4 HNO3 " 2H2O + 4NO2 + O2 Ethyl borate

These fumes intensify when copper turnings are Phosphate:


added.
Add conc. HNO3 to a small amount of substance
Cu + 4HNO3 " Cu(NO3)2 + 2NO2 + 2H2O or take sodium carbonate extract, heat and then
Ring Test: An aqueous solution of salt is mixed add ammonium molybdate. A canary yellow
with freshly prepared FeSO4 solution and conc. precipitate of ammonium phosphomolybdate is
H2SO4 is poured in test tube from sides, a brown formed.
ring is formed on account of the formation of a Ca3(PO4)2 + 6HNO3 " 3Ca (NO3)2 + 2H3PO4
complex at the junction of two liquids.
H3PO4 + 12(NH4)2MoO4 + 21HNO3
NaNO3 + H2SO4 " NaHSO4 + HNO3 " (NH4)3PO4.12MoO3 + 21NH4NO3 + 12H2O
6FeSO4 + 2HNO3 + 3H2SO4 " 3Fe2(SO4)3   Canary yellow ppt.
+ 4H2O + 2NO
[Fe(H2O)6]SO4.H2O+NO"[Fe(H2O)5NO]SO4 + H2O

         Brown ring
8.9

Solved Examples
1. Chemical volcano is produced on heating:
2Cu2+ + 4SCN- " [Cu2(SCN)2]. + (SCN)2
K2Cr2O7 (NH4)2Cr2O7 White
ZnCr2O7 K2CrO4
In which of the following tests, Cl- ion is tested by
4.
Sol. (b) On heating (NH4)2Cr2O7, N2 is given out with observing the colour of a precipitate which does
Cr2O3 powder at higher rate giving a look artificial not contain Cl- ions at all:
volcano. Chromyl chloride test
2.
Which of the following ions forms(s) ppt. with Lassaigne’s test
KI:
Silver mirror test
(a) Mg2+ (b) Pb2+
2+
(c) Hg (d) Cu2+ Fehling solution test
Sol. (b, c, d) Sol. (a) CrO2Cl2 + 4NaOH " Na2CrO4 + 2NaCl + 2H2O
Pb2+ + 2I- " PbI2 Yellow
Yellow ppt.
Na2CrO4+Pb(CH3COO)2"PbCrO4+2CH3 COONa
Hg2+ + 2I- " Hgl2
Red ppt.
Yellow ppt.

2+ -
Cu + 2I " Cul2 " Cu2I2 + I2 5. When an inorganic mixture was treated with excess
White ppt.
of dil. H2SO4, effervescence were produced. The
3. Name one common reagent that can form solution was heated till effervescence ceased.
precipitate or react and differentiate the following After this a small pinch of MnO2 were added,
pairs: fresh effervescence were produced. Select the
correct statements:
Ag+ and Ba2+ Cu2+ and Pb2+
Mixture contains CO32- ions
Fe3+ and Cu2+ Co2+ and Cu2+
Mixture contains C2O42- ions
Sol. (a) K2Cr2O4
+ Mixture contains SO32- ions
2Ag + CrO42- " Ag2CrO4.
Red Mixture contains CO32- and C2O42-

Ba2+ + CrO42- " BaCrO4. Sol. (d) Effervescence on heating the mixture with dil.
Yellow H2SO4, it is due to decomposition of carbonate.

(b) K2CrO4 Na2CO3 + H2SO4 " Na2SO3 + CO2 + H2O

Cu2+ + CrO42- " CuCrO4. When evolution of CO2 occurs again after adding
Chocolate MnO2, it is due to decomposition of oxalate.

Pb2+ + CrO42- " PbCrO4. Na2C2O4 + 2H2SO4 + MnO2 " Na2SO4 + MnSO4
Yellow + 2H2O + 2CO2

(c) NH4SCN 6. Few drops of a salt solution are shaken with


chloroform, chlorine water. Chloroform layer
Fe3+ + 3SCN- " [Fe(SCN)3].
becomes violet. The solution contains:
red colour
2+ - F- ion (b) Cl- ion
2Cu + 4SCN " Cu2(SCN)2. + (SCN)2
    white (c) Br- ion (d) I- ion
(d) NH4SCN Sol. (d) 2KI + Cl2 " 2KCl + I2;
Co2+ + 4SCN- " [Co(SCN)4]2- I2 + CCl4 " violet solution
blue colour
8.10

7. Name one common reagent that can precipitate or white ppt. with NaOH which dissolves in excess
react and differentiate following pairs: of NaOH, Also its a solution produced light
I- and Cl- yellow ppt. with AgNO3, sparingly soluble in
I- and Br- NH4OH the probable salt is:
SO32- and SO42- Sol. (a) AlBr3 + 3NaOH " Al(OH)3 + 3NaBr
Sol. (a) AgNO3 White ppt.

Ag+ + I- " Agl Al(OH)3 + NaOH " NaAlO2 + 3NaBr


yellow Soluble

3AgNO3 + AlBr3 " 3AgBr + Al(NO3)3


Ag+ + CI- " AgCl-
Light yellow ppt.
white
The salt is AlBr3.
(b) Cl2 water + CHCl3:
11.
A compound (A) forms an unstable pale blue
2I- + Cl2 " I2 + 2Cl-
(violet in CHCl3 layer)
colour solution in water. The solution decolorizes
Br2 water and an acidified solution of KMnO4.
2Br- + Cl2 " Br2 + 2Cl- The possible compound (A) is:
(orange in CHCl3 layer)
HNO2 HNO3
(c) BaCl2 N2O5 None of these
BaCl2 + SO32- " BaSO3. + 2Cl- Sol. (a) HNO2 is pale blue in colour due to dissolution of
white ppt. soluble in conc. HCl
N2O3. In conc. solution it decomposes as:
BaCl2 + SO42- " BaSO4. + 2Cl- 2HNO2 " NO + NO2 + H2O
white ppt. insoluble in conc. HCl
and in dilute solution as:
8. During qualitative test of nitrate radical, a brown 3HNO2 " HNO3 + H2O + 2NO
ring is formed. The ring formed is due to the
Reaction with:
formation of:
FeSO4NO Br2 water-
(FeSO4)2NO HNO2 + H2O + Br2 " HNO3 + 2HBr
FeSO4(NO)2 KMnO4 + 3H2SO4 + 5HNO2 " K2SO4
[Fe(H2O)5NO]SO4 + 2MnSO4 + 3H2O + 5HNO3
Sol. (d) NaNO3 + H2SO4 " NaHSO4 + HNO3 12.(a) If CO2(g) under pressure is passed into
6FeSO4 + 2HNO3 + 2H2SO4 " 2Fe2(SO4)3 Na2CrO4(aq.), Na2Cr2O7(aq.) is formed. What is the
+ 2NO + 4H2O function of the CO2(g)?
FeSO4 + NO + 5H2O " [Fe(H2O)5NO]SO4 When zinc is added to acidic solution of Na2Cr2O7.
Nitrosyl ferrous sulphate What is the colour of the solution?
9. An inorganic compound gives a white ppt. with a In acidic solution silver (II) oxide first dissolves
solution of AgNO3, a white ppt. with dil. H2SO4 to produce Ag2+ (aq.) and then reduces to Ag+ by
and imparts green colour to flame. H2O which is oxidized to O2(g) . Explain?
The probable compound is: Sol. (a) CO2 makes the solution acidic that can convert
CuCl2 BaCl2
Na2CrO4 to Na2Cr2O7
Cu(NO3)2 PbCl2
CO2 + H2O " HCO3- + H+
Sol. (b) 2AgNO3 + BaCl2 " 2AgCl + Ba(NO3)2 2CrO42- + 2H+ " Cr2O72- + H2O
BaCl2 + H2SO4 " BaSO4 + 2HCl (b) Zn reduces Cr2O72- to Cr3+ (green) and then to
White ppt.
Cr2+ (blue). Over a long time, Cr2+ is oxdised
Barium imparts green colour to flame. to Cr3+ by atmospheric O2 and thus blue colour
10.
An inorganic salt in its a solution produced a changes to green.
8.11

Cr2O72- + 14H+ + 3Zn " 2Cr3+ + 7H2O + 3Zn2+ 14. Identify (A) and (B) based on following reactions:
Orange green (A) NaOH
∆
→ (B) HCl  → white fumes
Cr3+ + e- " Cr2+ (gas)
Green blue After (B) is expelled completely, resultant
alkaline solution again gives gas (B) on
Cr2+ " Cr3+ + e- heating with zinc.
Blue green
(A) ∆ → N2O + H2O
(Note: These reactions are spontaneous based on the Sol. By (iii) (A) gives N2O, H2O on heating hence (A)
standard reduction electrons potential values.) is NH4NO3

(c) AgO + 2H+ " Ag2+ + H2O NH4NO3  → N2O + 2H2O
4Ag2+ + 2H2O " 4Ag+ + 4H+ + O2 (i) NH4NO3 + NaOH ∆
→ NaNO3 + NH3 + H2O
13. Colourless salt (A), on heating with sodium (B)
hydroxide, give gas (B) that can also be obtained NH3 + HCl " NH4Cl + H2O
when Mg3N2 reacts with H2O. When reaction
of (A) with NaOH was complete, solution (B) white fumes
obtained on reaction with FeSO4 gave a brown (ii) 2NaOH + Zn ∆
→ Na2ZnO2 + 2H
coloured ring (C) between two layers. (A) on NaNO3 + 8H ∆
→ NaOH + NH3 + 2H2O
heating strongly, forms (D) and (E). (D) and (E)
(B)
reacting together forming a dibasic acid (F) that
exists as cis and trans isomers. Identify (A) to (F) (NH3 gas is obtained due to reduction of NO3- to
and explain reactions. NH3)
H O
Sol. (A) NaOH.∆
 → (B) ←
2
 Mg3N2 15.
A mineral popularly known as apatite is used to
prepare a fertilizer, which provides phosphorus
(A) has NH4+ ion and (B) is NH3
element to the soil.
Mg3N2 + 6H2O " 3Mg(OH)2 + 2NH3
The fertilizer is obtained by treating apatite
NH4+ + NaOH " Na+ + NH3 + H2O with H2SO4.
Resultant solution after NH3 has escaped When heated with silica and coke, it yields
completely. It contains Na+ and anion of (A) which white phosphorus and calcium silicate.
also gives Ring Test of NO3- thus, (A) is NH4NO3 Suggest formula for apatite and explain the
NH4NO3 + NaOH ∆
→ NH3- + NaNO3 + H2O chemical reactions (i) and (ii)
NaNO3 + H2SO4 " NH3- + NaHSO4 + HNO3 Sol.
Chemically apatite is Ca3(PO4)2
2HNO3 " H2O + 3NO + 3O When apatite is heated with H2SO4(s)
[Fe(H2O)6]SO4 + NO " [Fe(H2O)5NO]SO4 " Ca(H2PO4)2
+ H2O (super phosphate of lime)
(C)
          When apatite mineral is heated with silica (SiO2)
NH4NO3 ∆ → N2O + 2H2O and coke (C), white phosphorus is obtained as
(A) (D) (E) follows:
N2O + H2O $ H2N2O2 2Ca3(PO4)2 + 6SiO2 " 6CaSiO3 + 3P2O5
          (F)
HO HO (calcium silicate)
2P2O5 + 10C " P4 + 10CO
N N
(white phosphorus)
N N 16.
(A)+tap water"white turbidity soluble in aq. NH3
(A) ∆
→ residue (B) + (C) (oxides of N) + O2
OH HO aq. (A) gives brown ring on adding FeSO4 and
Trans cis conc. H2SO4. (C) is paramagnetic and forms
8.12

dimer of vapour density 46. Identify (A), (B) and ppt. with ZnCl2 or AlCl3 but ppt. dissolves in
(C) and explain reactions. excess of (A). What is (A) and explain reaction.
Sol.
(A)+tap water"white turbidity soluble in a NH3. Sol.
(A) turns red litmus blue & (A) is basic in nature.
Tap water has Cl- and turbidity is soluble in a NH3 imparts golden yellow colour in flame & (A)
hence, turbidity is of AgCl has Na+
has Ag+ gives H2 gas with Zn or Al " (A) is NaOH.
(A) also gives ring test of NO3- Explanation:
is AgNO3 2NaOH + Zn "Na2ZnO2 + H2-
2AgNO3 " 2Ag + 2NO2 + O2
2NaOH + 2H2O + 2Al " 2NaAlO2 + 3H2-
(B) (C)
ZnCl2 + 2NaOH " Zn(OH)2. + 2NaCl
NO2 is paramagnetic due to one unpaired electron White ppt.
and thus forms dimer by using unpaired electron.
2NO2 E N2O4 Zn(OH)2 + 2NaOH " Na2[Zn(OH)4] or Na2ZnO2
dimer (V.D. = 46) Sodium zincate

17. If Cu2+ and Cd2+ both are present, it is difficult to AlCl3 + 3NaOH " Al(OH)3. + 3NaCl
analyse. Outline a scheme to analyse in a mixture. White ppt.
2+ 2+
Sol.
KCN forms complex with Cu and Cd
Al(OH)3 + NaOH " Na[Al(OH)4] or NaAlO2
Cu2+ + KCN " K2[Cu(CN)4] " K3[Cu+1(CN)4] Sodium meta-aluminate
          Stable
20. What single reagent solution (including H2O)
Cd2+ + KCN " K2[Cd(CN)4] could be used to effect the separation of the
Unstable following of solids?
NaOH and Fe(OH)3
When H2S gas is passed unstable complex of
Cd2+ gives yellow ppt. Ni(OH)2 and Fe(OH)3
Cr2O3 and Fe(OH)3
[Cd(CN)4]2- " Cd2+ + 4CN-, Cd2+ + S2- " CdS.
yellow
MnS and CoS
AgCl and AgI
18. Identify (A), (B), (C) and (D) and explain
Sol. H2O can dissolve NaOH, Fe(OH)3 remains
reactions.
insoluble in water.
(A) (green coloured salt) + K2Cr2O7 +
NH4OH dissolve Ni(OH)2, Fe(OH)3 is insoluble
conc. H2SO4 ∆
→ (B)
Ni(OH)4 + 4NH4OH " [Ni(NH3)4]2+
(B) (reddish brown gas) + NaOH " (C) Soluble
(yellow coloured solution)
Excess of NaOH can dissolve Cr2O3 but Fe(OH)3
(C) + (CH3COO)2Pb " (D) (yellow ppt.)
remains insoluble.
(A) + NaOH + Br2 water ∆→ (C)

Cr2O3 + 3H2O " 2Cr(OH)3
(C) + (CH3COO)2Pb  → (D)
Cr(OH)3 + NaOH " Na[Cr(OH)4]
Sol.
(A): CrCl3 Soluble
(B): CrO2Cl2 [by chromyl-chloride test of Cl-]
(C): Na2CrO4 Dilute HCl dissolves MnS soluble while CoS
(D): PbCrO4 remains insoluble.

19.
(A), an important laboratory reagent, turns red MnS + 2HCl " MnCl2 + H2S-
litmus blue, imparts golden yellow colour in A NH3 would dissolve AgCl while AgI remains
flame and is a good precipitating agent. (A) reacts insoluble
with Zn or Al forming H2 gas. (A) gives white AgCl + 2NH3(aq.) " [Ag(NH3)2]Cl
Soluble
8.13

Exercise
Ba2+ and SO42- ions
Pb2+ and I- ions
Pb2+ and SO42- ions
1. Which of the following salt on heating with conc.
H2SO4 gives violet vapours? 10. Which of the following gives black precipitate on
passing H2S through it?
Iodide salt Nitrate salt
Acidified zinc nitrate solution
Sulphate salt Bromide salt
Ammonical barium chloride solution
2. Salts of which of the following metal are white? Magnesium nitrate solution
Zinc Cobalt Copper nitrate solution
Chromium Fe 11.
All ammonium salts liberate ammonia gas when:
3. A glassy bead formed by heating borax on a Heated with water
platinum wire loop is: Heated with caustic soda
Sodium tetraborate Heated with H2SO4
Sodium metaborate Heated with NaNO2
Sodium metaborate and boric anhydride 12. Addition of solution containing C2O42- ions to an
Boric anhydride and sodium tetraborate aqueous solution containing Ba2+, Sr2+ and Ca2+
4. An oxalate salt gives which of the following gas will precipitate.
in dry heating test: Ca+2 Ca+2 and Sr2+
CO + CO2 Only CO2 Ba+2 and Sr2+ All three
Only CO Oxalic acid vapours 13. Sodium sulphide react with sodium nitroprusside
5. The salts of which of the following elements are to form a purple coloured Compound. During the
generally dark green coloured? reaction the oxidation state of iron:
Chromium Copper(I) Changes from +2 to +3
Changes from +3 to +2
Barium Cobalt
Changes from +2 to +4
6. The chromyl chloride test is meant for which of
Remains unchaged
the following ion?
14. Which of the following sulphide is not soluble in
Cl- ions Both Cl- and Br- ions
dil HNO3?
I- ions Cl- and CrO42- ions
PbS HgS
7.
Which of the following gases turn lime water
ZnS Bi2S3
milky?
Cu ions will be reduced to Cu+ ion by addition
15. 2+
(a) SO2 (b) CO2
of an aqueous solution of:
(c) H2S (d) Both (a) and (b)
(a) KF (b) KCl
8. Yellow ammonium sulphide solution can be used
(c) KI (d) KOH
for the separation of which of the following pair
of species? 16. Precipitate of AgCl dissolves in liquid ammonia
CuS and PbS PbS and Bi2S3 due to the formation of:

Bi2S3 and CuS CdS and As2S3 [Ag(NH4)2]OH

9. Reddish-brown (chocolate) ppt. are formed by [Ag(NH4)2]Cl


mixing solutions containing respectively: [Ag(NH3)2]OH
Cu2+ and [Fe(CN)6]4- ions [Ag(NH3)2]Cl
8.14

17. On adding a solution of CrO42- ions to an aqueous 26.


Among the pair of species given below which
solution containing Ba2+, Sr2+ and Ca2+ ions. The react with each other on mixing their aqueous
precipitate obtained first of all will be: solutions to give yellow precipitate:
CaCrO4 SrCrO4 (I) KI and Silver nitrate
BaCrO4 A mixture of all the three (II) KI and Lead (II) nitrate
(III) KI and KBr
18.
Brown ring test is used to detect:
(IV) KI and I2
Iodide Nitrate
(a) I, II (b) II, III
Iron Bromide
(c) I, II, IV (d) Only I
19.
When sodium thiosulphate solution is shaken
with iodine, thiosulphate is changed to: 27. Which salt would give a colourless gas having
pungent smell with hot dil. H2SO4 and at the same
Sulphite ion
time it will decolourise bromine water?
Sulphate ion
Na2SO4 NaHSO4
Tetrathionate ion
Sulphide ion Na2SO3 Na2CO3

20.
Reaction of K2Cr2O7 with NaCl and conc. H2SO4 28.
Which of the following contains colourless gases
gives: which form white fumes on coming in contact
with each other?
CrCl3 CrOCl2
SO2, HCl Cl2, NH3
CrO2Cl2 Cr2O3
HCl, NH3 CO, Cl2
21. Which of the following gives blood red colour
with KCNS? 29. Red vapour obtained by heating a mixture of KCl
and potassium dichromate is passed through dil.
Cu2+ ­(b) Fe3+ Sodium hydroxide solution. The colour of the
3+
(c) Al (d) Zn2+ solution so obtained is:
22.
Which of the following imparts green colour to Bright red Yellow
the Bunsen’s flame? green Scarlet
B(OMe)3 (b) Na(OMe)
30. Which of the following reagent can help in the
(c) Al(OPr)3 (d) Sn(OH)2
separation of Cu2+ an Cd2+ ions in the solution?
23.
The aqueous solutions of which of the following H2S in acidic medium
pairs of salts will give yellow precipitate separately
H2S in alkaline medium
with aqueous solutions of barium bromide?
KCN solution
K2CrO4, AgNO3 AgNO3, K2SO4
K4 [Fe(CN)6] solution
K2CrO4, K2SO4 AgNO3 , Na2CO3
24.
An aqueous solution is prepared by dissolving a
mixture containing ZnCl2, CdCl2 and CuCl2. H2S
gas is now passed through the aqueous solution of 1. Fe(OH)3 can be separated from Al(OH)3 by
salt to form black ppt. The ppt. contains: addition of:
CdS, CuS CdS, CuS, ZnS (a) BaCl2 (b) Dil. HCl
(c) NaOH solution (d) NH4Cl and NH4OH
CuS, ZnS Only CuS
2. An aqueous solution of a substance, on treatment
25.
Which of the following compound will turn black
with dilute HCl, gives a white precipitate soluble
on adding NH4OH to it?
in hot water. When H2S is passed through the hot
Lead chloride acidic solution, a black precipitate is formed. The
Silver chloride substance is:
Mercurous chloride (a) Hg22+ salt (b) Cu2+ salt
Barium chloride (c) Ag+ salt (d) Pb2+ salt
8.15

Lead (a) BeCO3 (b) MgCO3


→ (B) acetate
 →
NH 4 CI
3. CrCl3 NH OH
→ (A) Na
H
2 O2
O
4
(C) ; compound (C) is:
2 (c) CaCO3 (d) BaCO3
(a) Na2CrO4 (b) Na2Cr2O7 14.
A + Na2CO3 $ B + C,
CO
(c) Cr(OH)3 (d) PbCrO4 A 
2
→ (Milky) C
4. 2Cu2+ + 5I- $ 2CuI. + [X] The chemical formula of A and B are respectively:
[X] + 2S2O32-$3[Y] + S4O62- ; X and Y are: (a) NaOH and Ca(OH)2
(a) I3- and I- (b) I2 and I3- (b) Ca(OH)2 and NaOH
(c) I2 and I- (d) I3- and I2 (c) NaOH and CaO
5. Which of the following reagents can separate a (d) CaO and Ca(OH)2
mixture of AgCl and AgI? 15. Which of the following salt on heating with
(a) KCN (b) Na2S2O3 concentrated H2SO4, coloured vapours do not
(c) HNO3 (d) NH3 evolve?

6. FeSO4 is used in the brown ring test for a nitrate. (a) NaBr (b) NaNO3
What is the oxidation state of Fe in the compound (c) CaF2 (d) KI
responsible for the brown colour of the ring? 16. A salt made of bi-bivalent ions X and Y each
(a) 0 (b) +1 of which is capable of decolourising acidified
(c) + 2 (d) + 3 KMnO4. The salt is likely to be:
7. In an alkaline solution, sodium nitroprusside (a) Ferric oxalate (b) Ferrous oxalate
gives a violet colour with: (c) Ferrous sulphate (d) Stannic chloride
(a) S2- (b) SO32-
17.
When concentrated H2SO4 is added to dry KNO3,
(c) SO42- (d) NO2- brown fumes are evolved. These fumes are due to:
8. Which of the following sulphides is white? (a) SO2 (b) SO2 + SO3
(a) CdS (b) PbS
(c) NO (d) NO2
(c) ZnS (d) SnS
18. Freshly prepared chlorine water is added to the
9. A white sublimate substance, that turns black on
aqueous solution of some halide salt containing
treatment with an NH3 solution can be:
some CS2. After shaking the contents, a violet
(a) Hg2Cl2 (b) HgCl2 colour appeared in CS2 layer. The halide ion in
(c) As2O3 (d) NH4Cl solution is:
10. Which of the following pairs of cations can be (a) Iodide (b) Bromide
separated by adding NH4Cl and NH4OH to the (c) Chloride (d) Iodide as well as bromide.
mixture and then passing H2S through it?
19.
For the confirmatory tests of acid radicals, sodium
(a) Fe3+, Al3+ (b) Cr3+, Ni2+ carbonate extract is prepared because:
(c) Al3+, Cr3+ (d) Fe3+, Cr3+ (a) All anions react with Na
11. Which of the following pairs of sulphides are (b) Na is more reactive
insoluble in dilute HCl? (c) Na2CO3 is water soluble
(a) CoS and NiS (b) CoS and MnS (d) Sodium salts of almost all anions are water
(c) NiS and MnS (d) NiS and ZnS soluble.

12. On heating, a salt gives a gas which turns lime 20.


In the precipitation of the radicals of iron group
water milky and an acidified dichromate solution in qualitative analysis, NH4Cl is added before
green. The salt may be: adding NH4OH. This causes:
(a) carbonate (b) sulphide (a) Decrease in the concentration of OH- ions
(c) sulphate (d) sulphite (b) Removal of PO43- ions
(c) Increase in the concentration of Cl- ions
13. Which of the following has the highest value of
Kp? (d) Increase in the concentration of NH4+ ions
8.16

21. The aqueous solution of which of the following 27. On addition of aqueous NaOH to a salt solution,
reagent will give Prussian blue coloured ppt. with a white gelatinous precipitate is formed, which
an aqueous solution containing iron (III) ions? dissolves in excess alkali. The salt solution
(a) Potassium thiocyanate contains:
(b) Potassium hexacyanoferrate (II) (a) Chromous ions
(c) Potassium pyroantimonate (b) Aluminium ions
(d) All of these (c) Barium ions
22. Aqueous solution of salt A gives yellow precipitate (d) Iron ions
with aqueous solution of K2CrO4. which of the
28. Dimethyl glyoxime in a suitable solvent was
following series of cation may be present in A?
refluxed for 10 minutes with pure pieces of nickel
(a) Pb2+, Ag+ (b) Pb2+, Ba2+ sheet, it will result in:
+
(c) Ag , Cu2+ (d) Hg2+, Ag+ (a) Red precipitate
23. The reagent that can distinguish between silver (b) Blue precipitate
and lead salt is: (c) Yellow precipitate
(a) H2S gas (d) No precipitate
(b) Hot dilute HCl solution 29. A metal X on heating in nitrogen gas gives Y.
(c) NH4Cl (solid) + NH4OH (solution) Y on treatment with H2O gives a colourless gas
(d) NH4Cl (solid) + (NH4)2 CO3 (solution) which when passed through CuSO4 solution gives
a blue colour. Y is:
24. A yellow turbidity, sometimes appears on passing
(a) Mg(NO3)2 (b) Mg3N2
H2S gas even in the absence of the second group
radicals. Explain why? (c) NH3 (d) MgO
(a) Sulphur is present in the mixture as an 30. A light green coloured salt (X) does not react
impurity with dilute and conc. H2SO4. Its aqueous solution
(b) The fourth group radicals are precipitated as becomes dark brown when sodium nitrite solution
sulphides is added to it. X can be:
(c) The H2S is oxidized by some acidic radical (a) Some salt of Ni (b) Some salt of copper
present in solution (c) FeSO4 (d) Unpredictable
(d) The third group radicals are precipitated
25. Colourless salt (A)

Conc. H2SO4
Brown fumes ONE OR MORE THAN ONE CORRECT TYPE
Cu Blue + metal C 1.
Which of the following salts release reddish
A solution brown gas when heated in a dry test tube?
HCI
White ppt. soluble in NH4OH (a) LiNO3 (b) KNO3
The salt A can be: (c) Pb(NO3)2 (d) AgNO3
(a) Cu(NO3)2 (b) AgBr 2.
When Borax is heated it forms a colourless glassy
(c) AgNO3 (d) Pb(NO3)2 bead because of formation of :
26. Al3+, Cr3+, Fe3+ are grouped together for (a) B2H6 (b) NaBO2
qualitative analysis because: (c) B2O3 (d) Na2B4O7
(a) Their carbonates are insoluble in ammonia
3.
Which of the following metal chloride will give
(b) Their hydroxides are insoluble in ammonia chromyl chloride test ?
(c) Their sulphides are insoluble in acid
(a) NaCl (b) KCl
(d) They belong to same group of periodic table
(c) AgCl (d) SbCl3
8.17

4.
Which of the following statement(s) is/are correct (c) Fe3+ gives red colour with potassium
with respect to bromide ions ? thiocyanate
(a) KBr on heating with MnO2 and concentrated (d) Fe2+ gives brown colour with potassium
H2SO4 liberates Br2 and SO2 gases. thiocyanate
(b) KBr on heating with concentrated H2SO4 11. Which of the following sulphates are soluble in
liberates Br2 and SO2 gases. water ?
(c) KBr forms HBr with concentrated H3PO4. (a) CuSO4 (b) PbSO4
(d) KBr(s) liberates Br2 on gentile warming with (c) Ag 2 SO4 (d) BaSO4
concentrated H2SO4 and K2Cr2O7(s). 12.
Which of the following substances on being
5.
KI solution is the reagent for : heated will give a gas that turns lime water milky?
(a) Hg2+ (b) Pb2+ (a) Na2CO3 (b) ZnCO3
(c) ZnSO3 (d) MgCO3
(c) Ag+ (d) Cu2+
13.
A yellow precipitate is obtained when :
6.
Which of the following cations form(s) black
precipitate(s) with H2S(g)? (a) lead acetate solution is treated with K2CrO4
(b) Pb(NO3)2 solution is treated with K2CrO4
(a) Cu2+ (b) Sb3+
(c) AgNO3 solution treated with KI
(c) Pb2+ (d) Bi3+
(d) H2S is passed through a solution of CdSO4
7.
Which of the following is/are correct for
14.
Which of the following species will be
potassium ferrocyanide?
decomposed on acidification?
(a) It gives a brown precipitate with Cu2+ ions.
(a) [Ag(NH3)2]+ (b) [Cu(NH3)4]2+
(b) It gives a white preciptate of mixed salt with
Ca2+ ions. (c) [Zn(OH)4]2– (d) [Pb(OH)4]2–
(c) It in excess gives a bluish white/white PASSAGE BASED QUESTIONS
precipitate with Zn2+ Passage # 1 (Q. 15 to 17)
(d) It develops a deep red colouration with Fe3+.
A colourless inorganic compound (A) imparts a green
8.
The following can be used to regulate the colour to the flame. Its solution gives a white ppt. (B) with
concentration of OH– ions for the scheme of basic H2SO4. When heated with K2Cr2O7 and conc. H2SO4, a
radical analysis (III group). brown red vapour/gas (C) is formed. The gas/vapour when
(a) NH4NO3 (b) NH4Cl passed through aqueous NaOH solution, it turns into a
yellow solution (D) which forms yellow precipitate (E)
(c) (NH4)2SO4 (d) (NH4)2CO3
with CH3COOH and (CH2COO)2Pb
9.
Which of the following statement(s) is/are
15.
The colourless inorganic compound (A) is:
correct?
(a) Nickel salts give rosy red precipitate with (a) Ba(NO3)2 (b) BaCl2
dimethyl glyoxime in excess of NH4OH. (c) CuCl2 (d) CrBr3
(b) Fe(III) salts give red colour with potassium 16.
The liberated gas vapour (C) is:
sulphocyanide
(a) Br2 (b) NO2
(c) In nitroprusside, the iron and NO exist as
Fe(III) and NO. (c) CrO2Cl2 (d) Cl2
(d) Mn(II) salts give white precipitate with NaOH 17.
The yellow ppt. formed when (D) reacts with
which turns brown on adding Br2 water. CH3COOH and (CH2COO)2 Pb is:
10.
Which statement(s) is/are correct with reference (a) PbI2 (b) PbCrO4
to the ferrous and ferric ions? (c) BaCrO4 (d) AgBr
(a) Fe2+ gives brown colour with potassium Passage # 2 (Q. 18 to 20)
ferricyanide + Air H SO
Black solid KOH
  → (A)  2 4
→ (B) + (C)
(b) Fe2+ gives blue colour with potassium ∆
ferricyanide (green) (purple)
8.18

(i) KI on reaction with alkaline solution of (B) 23.


Which of the following is the composition of the
changes into a compound (D). brown precipitate (D)?
(ii) The colour of the compound (B) disappears on (a) HgI2 (b) Hg(NH2)I
treatment with the acidic solution of FeSO4 (c) HgO (d) HgO.Hg(NH2)I
(iii) With cold conc. H2SO4 compound (B) gives (E),
Passage # 4 (Q. 24 to 26)
which being explosive decomposes to yield (F)
and oxygen. (i) A white solid mixture of two salts containing a
common cation is insoluble in water. It dissolved
18.
Nature of compound (E) is: in dilute HCl producing some gases (with
(a) Acidic oxide effervescence) that turns an acidified dichromate
(b) Basic oxide solution green. After the gases are passed through
(c) Amphoteric oxide the acidified dichromate solution, the emerging
(d) Neutral oxide gas turns baryta water milky.
(ii) On treatment with dilute HNO3, the white solid
19.
Colour of the solution obtained, when ferrous
gives a solution which does not directly give a
sulphate reacts with acidic solution of (B):
precipitate with a BaCl2 solution but gives a white
(a) Colourless (b) Pink precipitate when warmed with H2O2 and then
(c) Green (d) Yellow treated with BaCl2 solution.
20.
Which of the following options is correct ? (iii) The solution of the mixture in dilute HCl, when
treated with NH4Cl, NH4OH and an Na2HPO4
(a) (C) and (F) are same compounds having same
solution, gives a white precipitate.
colour.
(b) (C) and (F) are different compounds having 24.
The gases evolved in (i) are:
same colour. (a) CO2 and HCl (b) SO2 and CO2
(c) Compound (B) forms similar compound (E) (c) SO2 and H2S (d) NH3 and CO2
with hot and conc. H2SO4.
25.
The white precipitate obtained in (ii) indicates the
(d) Compound (A) does not give same type of presence of a:
reaction in acidic and neutral medium.
(a) carbonate (b) sulphide
Passage # 3 (Q. 21 to 23) (c) sulphite (d) chloride
When a crystalline compound X is heated with K2Cr2O7 26.
The white precipitate obtained in (iii) consists of:
and concentrated H2SO4, a reddish brown gas A is
evolved. On passing A into caustic soda, a yellow solution (a) Ba3(PO4)2 (b) Sr3(PO4)2
of B is formed. A yellow precipitate of C is obtained when (c) Ca3(PO4)2 (d) MgNH4PO4.6H2O
a solution of B is neutralised with acetic acid and then
INTEGER VALUE TYPE QUESTIONS
treated with a lead acetate solution. When X is heated
with NaOH, a colourless gas is evolved which, when 27.
How many compounds liberate NH3 on heating
passed into a solution of K2[HgI4], gives a reddish brown from the following?
precipitate of D. (a) (NH4)2 SO4, (b) (NH4)2 CO3,
21.
Compound (X) is: (c) NH4Cl, (d) NH4NO3,
(a) NH4Br (b) NH4Cl (e) (NH4)2 Cr2O7
(c) NH4NO2 (d) NH4NO3 28.
How many of the following reactions give yellow
22.
If the solution B is colourless, which of the ppt.
following ions would not be present in the solid (a) NaBr + AgNO3 $
X? (b) NaI + AgNO3 $
(a) Cl– (b) Br– (c) NaBr + Pb(NO3)2 $
(c) NO3– (d) NO2 –
(d) NaI + Pb(NO3)2 $
8.19

(e) Na2S + Cd(CH2COO)2 $ 34.


(f) K2CrO4 + Pb(CH3COO)2 $ Column I Column II
(g) K2CrO4 + (CH3COO)2Ba $ 3+
(A) Bi give(s) black (a) H2S (saturated
(h) K2CrO4 + AgNO3 $ precipitate with solution in water)
(i) NaBr + Cl2 water (excess) $ (B) Cu2+ give(s) black (b) Potassium
An aqueous solution contains Hg22+, Hg2+, Pb2+,
29. precipitate with thiocyanate solution
Ag+, Bi3+ and Cd2+. Out of these, how many ions (C) Zn2+ give(s) white (c) Potassium iodide
will produce white precipitate with dilute HCl? precipitate with solution
30.
aq.CuSO4 decolourizes on addition of KCN due (D) Ag+ give(s) white (d) Potassium
to formation of complex (A). In complex "A". precipitate with ferrocyanide solution
(i) Number of d-orbital in hybridisation is/are "a"
(ii) Geometry of complex (b) :
(iii) Coordination number of Cu is "c". then find 8a +
7b + 5c.
(1) represents linear geometry 1.
[X] + H2SO4 $ [Y] a colourless gas with
(2) represents tetrahedral geometry irritating smell; [Y] + K2Cr2O7 + H2SO4 $
(3) represents octahedral geometry green solution. [X] and [Y] is:
(4) represents square planer geometry (a) SO 2–, SO 3 2 (b) Cl–, HCl
2–
(c) S , H2S (d) CO32–, CO2
31.
How many of the following pairs of ions can be
[IIT-2003]
separated by H2S in dilute HCl?
2.
A dilute aqueous solution of a sodium salt froms
(a) Bi3+ and Sn4+, (b) Al3+ and Hg2+,
white precipitate with MgCl2, only after boiling.
(c) Cd2+ and Zn2+, (d) Fe3+ and Cu2+, The anion of the sodium salt is:
(e) As3+ and Sb3+ (a) HCO3– (b) CO32–
Match the column type questions (c) NO3– (d) SO42–
[IIT-2004]
32.
Match the reagent which are used in qualitatiive
analysis of gtiven anions : 3.
The species present in solution when CO2 is
dissolved in water are:
Column I Column II (a) CO2, H2CO3, HCO3–, CO32–
2–
(A) AgNO3 solution (a) CO3 (b) HCO3–, CO32–
(B) BaCl2 solution (b) Cl– (c) CO32–, HCO3–
(C) PbNO3 solution (c) S2– (d) CO2, H2CO3
(D) Acidified KMnO4 solution (d) NO2 – [IIT-2006]
4.
A white precipitate is obtained when a solution
33. is diluted with H2O and boiled. On addition of
excess NH4Cl/ NH4OH, the volume of precipitate
Column I Column II
decreases leaving behind a white gelatinous
(A) White turbidity (a) IO3– + SO2 + starch $ precipitate. Identify the precipitate which
dissolves in ammonia solution or NH4Cl.
(B) Rotten egg smell (b) SO2 + MnO4– $
(a) Al(OH)3 (b) Zn(OH)2
(C) Colourless solution (c) Zn + NaOH + SO2 $
(c) Mg(OH)2 (d) Ca(OH)2
(D) Blue color (d) CO2 + Ca(OH)2 $
[IIT-2006]
8.20

5. In blue solution of copper sulphate excess of 10.


The compound Y is:
KCN is added then solution becomes colourless (a) MgCl2 (b) FeCl2
due to the formation of: (c) FeCl3 (d) ZnCl2
(a) [Cu(CN)4]2– 11.
The compound Z is:
(b) Cu2+ get reduced to form [Cu(CN)4]3– (a) Mg2[Fe(CN)6] (b) Fe[Fe(CN)6]
(c) Cu(CN)2 (c) Fe4[Fe(CN6]3 (d) K2Zn3[Fe(CN)6]2
(d) CuCN The equilibrium, 2CuI E Cu0 + CII in aqueous
12.
[IIT-2006] medium at 25°C shifts towards the left in the
6.
MgSO4 + NH4OH + Na2HPO4 $ white presence of:
crystalline precipitate. The formula of crystalline (a) NO3– (b) Cl–
precipitate is: (c) SCN– (d) CN–
(a) MgCl2. MgSO4 (b) MgSO4 [IIT-2011]
(c) Mg(NH4)PO4 (d) Mg(PO4)2
Passage # 2 (Q. 13 to 15)
[IIT-2006]
When a metal rod M is dipped into an aqueous colourless
7.
A solution of a metal ion when treated with KI
concentrated solution of compound N the solution turns
gives a red precipitate which dissolves in excess
light blue. Addition of aqueous NaCl to the blue solution
KI to give a colourless solution. Moreover, the
gives a white precipitate O. Addition of aqueous NH3
solution of metal ion on treatment with a solution
dissolves O and give an intense blue solution.
of cobalt (II) thiocyanate gives rise to a deep blue
[IIT-2011]
crystalline precipitate. The metal ion is:
13. The metal rod M is:
(a) Pb2+ (b) Hg2+ (a) Fe (b) Cu
(c) Cu2+ (d) Co2+ (c) Ni (d) Co
[IIT-2007] 14. The compound N is:
8.
A solution of colourless salt H on boiling with (a) AgNO3 (b) Zn(NO3)2
excess NaOH produces a non-flammable gas. (c) Al(NO3)3 (d) Pb(NO3)2
The gas evolution ceases after some time. Upon
15. The final solution contains:
addition of Zn dust to the same solution, the gas
(a) [Pb(NH3)4]2+ and [CoCl4]2–
evolution restarts. The colourless salts(s) H is
(are): (b) [Al(NH3)4]3+ and [Cu(NH3)4]2+
(a) NH4NO3 (b) NH4NO2 (c) [Ag(NH3)2]+ and [Cu(NH3)4]2+
(d) [Ag(NH3)2]+ and [Ni(NH3)6]2+
(c) NH4Cl (d) (NH4)2SO4
[IIT-2008] Passing H2S gas into a mixture of Mn2+, Ni2+,
16.
Passage # 1 (Q. 9 to 11) Cu2+ and Hg2+ ions in an acidified aqueous
solution precipitates:
p-Amino-N, N-dimethylaniline is added to a strongly
acidic solution of X. The resulting solution is treated with a (a) CuS and HgS (b) MnS and CuS
few drops of aqueous solution of Y to yield blue coloration (c) MnS and NiS (d) NiS and HgS
due to the formation of methylene blue. Treatment of [IIT-2011]
the aqueous solution of Y with the reagent potassium 17.
For the given aqueous reaction which of the
hexacyanoferrate(II) leads to the formation of an intense statement(s) is (are) true?
blue precipitate. The precipitate dissolves on excess excess KI + K3[Fe(CN)6]
dilute H2SO4
brownish-yellow solution
additon of the reagent. Similarly, treatement of the solution ZnSO4
of Y with the solution of potassium hexacyanoferrate(III)
(white precipitate + brownish – yellow filtrate)
leads to a brown colouration due to the formation of Z.
[IIT-2009] Na2S2O3

9. The compound X is: colourless solution


(a) NaNO3 (b) NaCl (a) The first reaction is a redox reaction
(c) Na2SO4 (d) Na2S (b) White precipitate is Zn3[Fe(CN)6]2
8.21

(c) Addition of filerate to starch solution gives Bi2S3 and SnS2, the total number of BLACK
blue colour coloured sulphide is [JEE Advanced - 2014]
(d) White precipitates is soluble in NaOH 23. The pair(s) of ions where BOTH the ions are
solution precipitated upon passing H2S gas in presence of
[IIT-2012] dilute HCl, is (are):
18.
Concentrated nitric acid, upon long standing, (a) Ba2+, Zn2+ (b) Bi3+, Fe3+
turns yellow-brown due to the formation of: 2+ 2+
(c) Cu , Pb (d) Hg2+, Bi3+
(a) NO (b) NO2
[JEE Advanced - 2015]
(c) N2O (d) N2O4
24. Which one of the following statement is correct?
[JEE Advanced - 2013]
(a) From a mixed precipitate of AgCl and AgI,
19.
Upon treatment with ammoniacal H2S, the metal ammonia solution dissolves only AgCl.
ion that precipitates as a sulphide is: (b) Ferric ions gave a deep green precipitate on
(a) Fe(III) (b) Al(III) adding potassium ferrocyanide solution
(c) Mg(II) (d) Zn(II) (c) On boiling a solution having K+, Ca2+
[JEE Advanced - 2013] and HCO3– ions we get a precipitate of
Passage # 3 (Q. 20 and 21) K2Ca(CO3)2.
An aqueous solution of a mixture of two inorganic salts, (d) Manganese salts give a violet borax bead test
when treated with dilute HCl, gave a precipitate (P) and a in the reducing flame
filtrate (Q). The precipitate P was found to dissolve in hot [AIEEE - 2013]
water. The filtrate (Q) remained unchanged, when treated 25.
A red solid is insoluble in water. However it
with H2S in a dilute mineral acid medium. However, it becomes soluble if some KI is added to water.
gave a precipitate (R) with H2S in an ammonical medium. Heating the red solid in a test tube results in
The precipitate (R) gave a coloured solution (s), when liberation of some violet coloured fumes and
treated with H2O2 in an aqueous NaOH medium. droplets of a metal appear on the cooler parts of
[JEE Advanced - 2013] the test tube. The red solid is
20.
The precipitate P contains: (a) (NH4)2Cr2O7 (b) HgI2
(a) Pb2+ (b) Hg22+ (c) HgO (d) Pb3O4
+
(c) Ag (d) Hg2+ [AIEEE - 2003]

21.
The coloured solution S contains: 26.
Which of the following compounds is not
coloured yellow?
(a) Fe2(SO4)3 (b) CuSO4
(a) Zn2[Fe(CN)6] (b) K3[Co(NO2)6]
(c) ZnSO4 (d) Na2CrO4
(c) (NH4)3[As(Mo3O10)4] (d) BaCrO4
22.
Among PbS, CuS, HgS, MnS, Ag2S, NiS, CoS, [JEE Main - 2015]

Answer Key

1. (a) 2. (a) 3. (c) 4. (a) 5. (a) 6. (a) 7. (d) 8. (d) 9. (a) 10. (d)
11. (b) 12. (d) 13. (d) 14. (b) 15. (c) 16. (d) 17. (c) 18. (b) 19. (c) 20. (c)
21. (b) 22. (a) 23. (a) 24. (d) 25. (c) 26. (a) 27. (c) 28. (c) 29. (b) 30. (c)

1. (c) 2. (d) 3. (d) 4. (a) 5. (d) 6. (b) 7. (a) 8. (c) 9. (a) 10. (b)
11. (a) 12. (d) 13. (a) 14. (b) 15. (c) 16. (b) 17. (d) 18. (a) 19. (d) 20. (a)
21. (b) 22. (b) 23. (b) 24. (c) 25. (c) 26. (b) 27. (b) 28. (d) 29. (b) 30. (c)
8.22

1. (a, c, d) 2. (b, c) 3. (a,b) 4. (b,c,d )   5. (a, b, c, d)  6. (a, c, d)  7. (a, b, c)  8. (a, b)
  9. (a, b, d) 10. (b,c) 11. (a, c) 12. (b, c, d) 13. (a,b, c, d) 14. (a, b, c, d) 15. (b) 16. (c)
17. (b) 18. (a) 19. (d) 20. (a) 21. (b) 22. (a) 23. (d) 24. (b)
25. (c) 26. (d) 27. (3) 28. (7) 29. (3) 30. (34 ) 31. (3)
32. A " a,b,c,d; B " a; C " a,b,c; D " b,c,d
33. A " d; B " c; C " b; D " a
34. A " a,c; B " a,b; C " a,d; D " b,d

1. (a) 2. (a) 3. (a) 4. (b) 5. (b) 6. (c) 7. (b) 8. (a, b) 9. (d) 10. (c)
11. (b) 12. (b, c, d) 13. (b) 14. (a) 15. (c) 16. (a) 17. (a, c, d) 18. (b) 19. (d) 20. (a)
21. (d) 22. (7) 23. (c, d) 24. (a) 25. (b) 26. (a)

Hints and Solutions


2
13. (d) Na2S + Na2 [ Fe (CN)5NO] " Na4
2
(a) I- ions are oxidized by H2SO4 to violet coloured Sodium nitroprusside
[ Fe (CN)5(NOS)]
I2. There is no change in oxidation state of Fe.
(a) Due to electronic configuration of Zn+2 is [Ar] 14. (b) HgS is not soluble in dil. HNO3. HgS is soluble
4s03d10, salts of zinc are white (colourless). in aqua regia.
(c) Na2 B4 O7. 10H2O Strong
 heating
→ 15. (c) I- ion acts as good reducing agent.
2Cu2+ + 4I- " Cu2I2 +I2
2NaBO2 B2 O3 + 10H2O 16. (d) AgCl + NH3 " [Ag(NH3)2]Cl

galssy bead Soluble complex

(a) Dry heating of oxalate slats give CO and CO2.
17. (c) BaCrO4 is precipitated first.
(a) Chromium salts are in general green in colour.
(a) Chromyl chloride test is applied for the 18. (b) Brown ring test is used to detect nitrate ion.
detection of Cl- ion. 19. (c) I2 + S2O3-2 " I- + S4O6-2
(d) Both CO2 and SO2 turn limewater (Ca(OH)2) 20. (c) NaCl + K2Cr2O7 + H2SO4 " NaHSO4 +
milky. KHSO4 + H2O + CrO2Cl2
Ca(OH)2 + CO2 " CaCO3. + H2O CrO2Cl2 is chromyl chloride.
Ca(OH)2 + SO2 " CaSO3. + H2O 21. (b) FeCl3 + KCNS " Fe(SCN)Cl2 + KCl
White Blood red

8. (d)
As2S3 is soluble in YAS (yellow ammonium 22. (a)
H3BO3 + 3MeOH " B(OMe)3 + 3H2O
sulphide) whereas CdS is not. Methyl borate, B(OMe)3 burns with green
flame.
9. (a) 2Cu2+ + [Fe(CN)6]4- " Cu2 [Fe(CN)6]
Ba+2 + K2CrO4 " BaCrO4. + 2K+
23. (a)
       Chocolate ppt. Yellow
10. (d) Cu(NO3)2 + H2S " CuS. + 2HNO3
Black AgNO3 + Br " AgBr. + NO3-
-

yellow
11. (b) NH4 + NaOH " NH3 + H2O + Na+
+

12. (d) All the three ions will precipitate as their All three Zn+2, Cd+2 and Cu+2 form precipitate
24. (d)
respective oxalates. with H2S but ZnS is white and
8.23

CdS is yellow in colour. The only black (c) ZnS is white is colour.
precipitate is CuS. +1 0 +2
9. (a) Hg2 CI 2 + NH3 → 
Hg + Hg ( NH 2) Cl + HCl

25. (c) Hg2Cl2 + NH4OH " Hg + Hg(NH2) Cl + HCl
Black
+ H2O
+3 +3 +3
black 10. (b) Fe , Al and Cr are III group basic radicals
26. (a)
KI + AgNO3 " AgI. + KNO3 while Ni+2 is IV group basic radical.
yellow 11. (a) CoS and NiS, both are insoluble in dilute HCl
2KI + Pb(NO3)2 " PbI2. + 2KNO3 while ZnS and MnS are soluble in dilute HCl.
yellow 12. (d) CaSO3 ∆
→ CaO + SO2-
27. (c) Na2SO3 + H2SO4 (dil) " Na2SO4 + H2O SO2 gas turns lime water milky and an acidified
+ SO2- dichromate solution green.
SO2 is a colourless gas having pungent smell 13. (a) Order of thermal stability:
and it will decolourise bromine water. BeCO3 < MgCO3 < CaCO3 < BaCO3
28. (c) NH3 + HCl " NH4Cl-
(white fumes) BeCO3 easily decomposes into BeO + CO2

14. (b) Ca(OH)2 + Na2CO3 " 2NaOH + CaCO3
29. (b) K2Cr2O7 + KCl + H2SO4 " CrO2Cl2-
(Red) (A) (B) (C)

CrO2Cl2 + NaOH " Na2CrO4 Ca(OH)2 CO


 2
→ CaCO3
(Yellow) (C)
30. (c)
Both form colourless complex with KCN but 15. (c) NaBr + H2SO4(conc.) " Br2- (reddish-brown)
complex of Cd+2 unstable. Complex of Cd+2 NaNO3 + H2SO4(conc.) " NO2- (reddish-brown)
form yellow precipitate of CdS with H2S.
CaF2 + H2SO4(conc.) " HF (colourless)
KI + H2SO4(conc.) " I2- (violet)
16. (b) Ferrous oxalate consists of Fe2+ and C2O42- each
(c) Al(OH)3 dissolves in excess NaOH while of which is capable of reducing MnO4- to Mn2+
Fe(OH)3 does not dissolves in NaOH. ions.
(d) Pb2+ + HCl(dil) " PbCl2. (soluble in hot water) 17. (d) KNO3 + H2SO4(conc.) " KHSO4 + HNO3
white 4HNO3 " 2H2O + 4NO2 + O2
NH 4 Cl + NH 4 OH
(d) CrCl3 → Cr(OH)3
18. (a) Cl2 oxidizes I- ions to I2 which dissolves in CS2 to
(A)
Na 2 O2 + H 2 O
give violet colour.
Cr(OH)3 → Na2CrO4
19. (d) All sodium salts (except NaHCO3) are soluble in
(A) (B)
Pb ( CH 3 COO )2
water.
Na2CrO4  → PbCrO4
20. (a) NH4Cl " NH4+ + Cl-
(B) (C)
NH4OH E NH4+ + OH-
(a) 2Cu2+ + 5I- " 2CuI + I3-
NH4Cl controls the concentration of OH- ions
(X) by suppressing the ionization of NH4OH due to
I3- + 2S2O32- " 3I- + S4O62- common ion effect.
(Y) 21. (b) Fe+3 + K4[Fe(CN)6] " Fe4[Fe(CN)6]3 + K+
(Prussian blue)
(d) AgCl forms soluble complex with NH3 while AgI
2+ 2+
does not form soluble complex with NH3. 22. (b) Both Pb and Ba gives yellow precipitate with
1 1 aqueous solution of K2CrO4.
(b) Brown ring complex is [ Fe (H2O)5 NO ]SO4
23 (b) Hot HCl will produce precipitate of AgCl with
(a) S2- + Na2[Fe(CN)5 NO] " Na4[Fe(CN)5(NOS)] Ag+ only. PbCl2 will not precipitate because it is
Violet soluble in hot solution.
8.24

24. (c) Radicals such as NO3- oxidize H2S to S which 5. (a, b, c, d)


appears as turbidity. Hg2+ + 2I– $ HgI2.
25. (c) A is AgNO3 Red
conc. H2SO4 Pb2+ + 2I– $ PbI2.
Ag2 SO4 + 2NO2 + H2O
  Yellow
Cu + –
AgNO3
Cu(NO3 )2 + 2 (Ag) Ag + I $ AgI.
Blue (C)
HCl
  Yellow
AgCl + HNO 3 2+ –
White
2Cu + 4I $ Cu2I2. + I2
26. (b) The hydroxides of these cations are insoluble in   White
ammonical solution. 6. (a, c, d)
27. (b) Al+3 + 3OH- " Al(OH)3 (white gelatinous Cus, PbS, Bi2S3 $ black
precipitate) Sb2S3 $ Orange
Al(OH)3 + NaOH " NaAlO2 + H2O 7. (a, b, c)
Fe3+ + K4[Fe(CN)6] $ Fe4[Fe(CN)6]3
28. (d) DMG gives rose red precipitate with Ni+2 but not
Prussian blue
with Ni metal.
H O 8. (a, b)
29. (b) 3Mg + N2 " Mg3N2 
2
→ Mg(OH)2 + NH3
(NH4)2SO4 and (NH4)2CO3 can not be used to
4

(X) (Y) regulate the concentration of OH– ions for III


CuSO

group basic radicals. Ba+2(V group) can form


precipitate BaSO4 and BaCO3.

[Cu(NH3)4]SO4 9. (a, b, d)
30. (c) FeSO4 solution gives black-brown colour with In nitroprusside, the iron and NO exist as
NaNO2 Fe(II) and NO+.
10. (b, c)
Fe+2 + K3[Fe(CN)6] $ Fe3 [Fe(CN)6]2
(which finally converts
1. (a, c, d) into Fe4[Fe(CN)6]3)
1 Fe+3 + KSCN $ Fe(SCN)3
2LiNO3 ∆
→ Li2O + 2NO2 + O2
2   blood red
2KNO3 ∆→ 2KNO2 + O2 11. (a, c)
2Pb(NO3) ∆→ 2PbO + 4NO2 + O2 CuSO4 and Ag2SO4 are water soluble.
∆ 1 12. (b, c, d)
2AgNO3  → Ag2O + 2NO2 + O2
2 CO2 and SO2 both gas can turn lime water milky.
2. (b, c) ZnCO3 ∆ → ZnO + CO2-
Na2B4O7.10H2O N N O 2NaBO2 B2 O3
 ZnSO3 ∆ → ZnO + SO2-
glassy bead MgCO3 ∆ → MgO + CO2-
+ 10H2O 13. (a, b, c, d)
3. (a, b) PbCrO4, AgI, CdS are yellow precipitate.
NaCl and KCl give chromyl chloride test. 14. (a, b, c, d)
4. (b, c, d) All complex will decompose on acidification.
MnO2 + H2SO4(conc.) + KBr ∆ → KHSO4 + 15. (b) Compound (A) imparts a green colour to the
MnSO4 + H2O + Br2 flame. It implies that cation of compound is
Ba+2. This compound gives chromyl chloride
KBr + H2SO4(conc.) ∆→ SO2 + Br2 + K2SO4 + test. It implies that anion of the compound is
H2O Cl–.

KBr + H3PO4(conc.)  → KH2PO4 + HBr 16. (c) BaCl2 + K2Cr2O7 + H2SO4 $ Ba(HSO4)2
K2Cr2O7 + H2SO4(conc.) + KBr " Cr2(SO4)3 + (A)
+ KHSO4 + H2O + CrO2Cl2 -
Br2 + H2O + K2SO4         (C)
8.25

17. (b) Yellow precipitate (E) is PbCrO4. 29. Hg2Cl2, PbCl2, AgCl (all are white precipitate)
+ Air H SO sp3
MnO2 KOH
18. (a) ∆
 → K 2 MnO 4 
2 4
→ 30. CuSO4(aq) + KCN(excess) " K3[ Cu (CN)4]
(A) (Tetrahedral)
KMnO 4 + MnO2
( B) (C) a=0;b=2;c=4
KMnO4+H2SO4 (cold and conc.)$Mn2O7 (E) 8a + 7b + 5c = 34
Mn2O7 is an acidic oxide
31. (c)
19. (d) KMnO4 + H2SO4 + FeSO4 $ Al3+ and Hg2+
MnSO4 + Fe2(SO4)3 + H2O + K2SO4 (group III) (group II)
    Colourless Yellow Cd and Zn2+
2+

20. (a) (C) and (F) both are MnO2 (group II) (group IV)
21. (b)
Compound (X) gives chromyl chloride test and Fe and Cu2+
3+

it also reacts with nesseler’s reagent hence, (X) (group III) (group II)
is NH4Cl. Bi and Sn4+
3+

22. (a) If Br– is present in (X) then Br2 vapours are (group II) (group II)
evolved which forms colourless NaBr + As and Sb3+
3+
NaOBr/NaBrO3 with NaOH. (group II) (group II)
If NO3– or NO2– is present in (X) then NO2 gas
is evolved which forms NaNO3 and NaNO2 32. (A " a,b,c,d; B " a; C " a,b,c; D " b,c,d)
with NaOH. 33. (A " d; B " c; C " b; D " a)
23. (d) NH4Cl + NaOH " NH3-
(X) 34. (A " a,c; B " a,b; C " a,d ; D " b,d)
NH3 + K2HgI4 + OH– " HgO.Hg(NH2)I.
(D)
24. (b) White solid consists of MgCO3 and MgSO3.
With dilute HCl, this solid produces CO2 and
SO2 gases.
1. (a) SO32– + H2SO4 $ SO2
25. (c) White precipitate in (ii) is BaSO4. [X] [Y]
It indicates presence of a sulphite. SO2 + K2Cr2O7 + H2SO4 $ Cr2(SO4)2 + K2SO4
26. (d) MgCl2 + NH4OH+Na2HPO4 " Mg(NH4) (green)   + H2O
PO4.6H2O.
         (white) 2. (a) NaHCO3 + MgCl2 ∆
→ MgCO3.
(white)
27. (NH4)2SO4 ∆
→ NH3 + H2SO4
3. (a) CO2 + H2O $ H2CO3
(NH4)2CO3 ∆ → NH3 + CO2 + H2O (H+, HCO3–, CO32–)

NH4Cl  → NH3 + HCl
4. (b) Zn(OH)2 + NH3(solution) $ [Zn(NH3)4]2+
(NH4)2Cr2O74 ∆→ N2 + Cr2O3 + H2O Soluble complex

NH4NO3  → N2O + H2O +2 +1
Cu SO4 + KCN(excess) $ K3[ Cu (CN)4]
5. (b)
28. NaBr + AgNO3 " AgBr (yellow)
NaI + AgNO3 " AgI (yellow) 6. (c)
MgSO4 + NH4OH + Na2HPO4 $
Mg(NH4)PO4
NaI + Pb(NO3)2 " PbI2 (yellow)
            White
Na2S + Cd(CH3COO)2 " CdS (yellow)
Hg2+ + KI $ HgI2.
7. (b)
K2CrO4 + Pb(CH3COO)2 " PbCrO4 (yellow) red
K2CrO4 + (CH3COO)2Ba " BaCrO4 (yellow) HgI2 + KI(excess) $ K2[HgI4]
   Soluble
NaBr + Cl2 water (excess) " BrCl (yellow)
8.26

8. (a,b) NH4NO3 + NaOH $ NH3 + NaNO3 17. (a, c, d)


+3 2
NaNO3 + Zn dust $ NH3 + Na2ZnO2 K3[ Fe (CN)6] + KI(excess) " K4[ Fe (CN)6] + KI3
NH4NO2 + NaOH $ NH3 + NaNO2 (Brownish
NaNO2 + Zn dust $ NH3 + Na2ZnO2 yellow
solution)
9. (d) p-Amino-N, N-dimethylaniline + Na 2 S +
FeCl3 $ Methylene blue (X) K4[Fe(CN)6] + ZnSO4 "K2Zn3 [Fe(CN)6]2
(Y)
or K2 Zn[Fe(CN)6]
FeCl3 + K4[Fe(CN)6] $ Fe4[Fe(CN)6]3 white ppt.
(Y) blue I3– + 2Na2S2O3 " Na2S4O6 + 2NaI + I2
FeCl3 + K4[Fe(CN)6] $ Fe[Fe(CN)6] (Brownish yellow Clear    (Turns starch
(Y) (Z) filtrate) solution  solution blue)

10. (c) FeCl3 K2Zn[Fe(CN)6] reacts with NaOH as,


11. (b) Fe[Fe(CN)6 K2Zn[Fe(CN)6]+NaOH $[Zn(OH)4]2–
Soluble
12. (b, c, d) + [Fe(CN)6]4–
With Cl–, SCN–, CN– ions the more stable 4HNO3 hυ
18. (b) → 2H2O + 4NO2-+ O2-
oxidation state of Cu is +1. 19. (d) +2
Zn is IV group radical.
13. (b) Cu + AgNO3 " Ag + Cu(NO3)2 20. (a)
PbCl2 is soluble in hot water.
(M) (N) light blue 21. (d)
Q is group III radical
AgNO3 + NaCl " AgCl.
Cr(OH)3 + H2O2 + NaOH $ Na2CrO4 (s)
White (Q)    yellow solution
(O) 22. PbS, CuS, HgS, Ag2S, NiS, CoS (Black)
AgCl + NH3 " [Ag(NH3)2]+ MnS (buff or pink)
Cu(NO3)2 + NH3 " [Cu(NH3)4]2+ SnS2(yellow coloured)
14 (a) AgNO3 Bi2S3(brown/black coloured)
15. (c) [Ag(NH3)2]+ and [Cu(NH3)4]2+ 23. (c, d)
16. (a) CuS and HgS Cu2+, Pb2+, Hg2+ and Bi3+, all are group II
Cu2+ and Hg2+ are II group radicals. radicals.
24 (a) AgI does not dissolve in NH3.
25. (b) HgI2 + 2KI " K2[HgI4]
Soluble
26. (a) Zn2[Fe(CN)6] is white in colour.

You might also like